You are on page 1of 357

Chuyn

Bt ng thc hin i
V Quc B Cn-Phm Th Hng

ii

Mc lc
Li ni u

1 Tm ti mt s k thut gii ton


1.1 i lng (a b)(b c)(c a) . . . . . . . . . .
1.2 Nhng kiu li gii c bit bng AM-GM . . . .
1.3 K thut pqr . . . . . . . . . . . . . . . . . . . .
1.3.1 Li ni u . . . . . . . . . . . . . . . . .
1.3.2 Nhng ng thc cn nh . . . . . . . . .
1.3.3 Bt ng thc Schur . . . . . . . . . . . .
1.3.4 i lng (a b)2 (b c)2 (c a)2 . . . . .
1.3.5 Lm mnh hn na . . . . . . . . . . . .
1.3.6 pqr hon v . . . . . . . . . . . . . . . . .
1.4 The CYH techniques . . . . . . . . . . . . . . . .
1.4.1 Li ni u . . . . . . . . . . . . . . . . .
1.4.2 Bt ng thc Cauchy Schwarz v Holder.
1.4.3 Mt s k thut cn ch . . . . . . . . .
1.5 The Hyberbolic functional technique . . . . . . .
1.5.1 Li ni u . . . . . . . . . . . . . . . . .
1.5.2 Mt s v d m u . . . . . . . . . . . .
1.5.3 t vn . . . . . . . . . . . . . . . . .
1.5.4 Gii quyt vn . . . . . . . . . . . . .
1.5.5 Mt s m rng . . . . . . . . . . . . . . .
1.6 Cc dng tng bnh phng . . . . . . . . . . . .
1.7 Hm li, hm bc nht . . . . . . . . . . . . . . .
1.8 Quy np . . . . . . . . . . . . . . . . . . . . . . .
2 Sng to bt ng thc

.
.
.
.
.
.
.
.
.
.
.
.
.
.
.
.
.
.
.
.
.
.

.
.
.
.
.
.
.
.
.
.
.
.
.
.
.
.
.
.
.
.
.
.

.
.
.
.
.
.
.
.
.
.
.
.
.
.
.
.
.
.
.
.
.
.

.
.
.
.
.
.
.
.
.
.
.
.
.
.
.
.
.
.
.
.
.
.

.
.
.
.
.
.
.
.
.
.
.
.
.
.
.
.
.
.
.
.
.
.

.
.
.
.
.
.
.
.
.
.
.
.
.
.
.
.
.
.
.
.
.
.

.
.
.
.
.
.
.
.
.
.
.
.
.
.
.
.
.
.
.
.
.
.

.
.
.
.
.
.
.
.
.
.
.
.
.
.
.
.
.
.
.
.
.
.

.
.
.
.
.
.
.
.
.
.
.
.
.
.
.
.
.
.
.
.
.
.

.
.
.
.
.
.
.
.
.
.
.
.
.
.
.
.
.
.
.
.
.
.

.
.
.
.
.
.
.
.
.
.
.
.
.
.
.
.
.
.
.
.
.
.

.
.
.
.
.
.
.
.
.
.
.
.
.
.
.
.
.
.
.
.
.
.

1
1
12
22
22
23
23
28
42
55
70
70
70
72
143
143
143
146
152
164
179
186
196
201

A Mt s bt ng thc thng dng


343
A.1 Bt ng thc trung bnh cng-trung bnh nhn-trung bnh iu ha
(AM-GM-HM) . . . . . . . . . . . . . . . . . . . . . . . . . . . . . . . 343
iii

iv

MC LC
A.2
A.3
A.4
A.5
A.6

Bt
Bt
Bt
Bt
Bt

ng
ng
ng
ng
ng

thc
thc
thc
thc
thc

AM-GM suy rng . . . . . .


trung bnh ly tha . . . . .
trung bnh ly tha suy rng
Bernoulli . . . . . . . . . . .
Cauchy Schwarz . . . . . . .

A.7 Bt ng thc Holder . . .


A.8 Bt ng thc Minkowski .
A.9 Bt ng thc Chebyshev .
A.10 Khai trin Abel . . . . . . .
A.11 Bt ng thc Maclaurin .
A.12 Bt ng thc Schur . . . .
A.13 Hm li, hm lm . . . . .
A.14 Bt ng thc Jensen . . .
A.15 Tng, tch hon v-i xng

.
.
.
.
.
.
.
.
.

.
.
.
.
.
.
.
.
.

.
.
.
.
.
.
.
.
.

.
.
.
.
.
.
.
.
.

.
.
.
.
.
.
.
.
.

.
.
.
.
.
.
.
.
.

.
.
.
.
.
.
.
.
.

.
.
.
.
.
.
.
.
.

.
.
.
.
.
.
.
.
.

.
.
.
.
.

.
.
.
.
.

.
.
.
.
.

.
.
.
.
.

.
.
.
.
.

.
.
.
.
.

.
.
.
.
.

.
.
.
.
.

.
.
.
.
.

.
.
.
.
.

.
.
.
.
.

.
.
.
.
.

.
.
.
.
.

.
.
.
.
.

.
.
.
.
.

343
343
344
344
344

.
.
.
.
.
.
.
.
.

.
.
.
.
.
.
.
.
.

.
.
.
.
.
.
.
.
.

.
.
.
.
.
.
.
.
.

.
.
.
.
.
.
.
.
.

.
.
.
.
.
.
.
.
.

.
.
.
.
.
.
.
.
.

.
.
.
.
.
.
.
.
.

.
.
.
.
.
.
.
.
.

.
.
.
.
.
.
.
.
.

.
.
.
.
.
.
.
.
.

.
.
.
.
.
.
.
.
.

.
.
.
.
.
.
.
.
.

.
.
.
.
.
.
.
.
.

.
.
.
.
.
.
.
.
.

344
345
345
345
345
346
346
346
346

Li ni u
Bt ng thc l mt trong nhng vn hay v kh nht ca chng trnh ton ph
thng bi n c mt trn hu khp cc lnh vc ca ton hc v n i hi chng ta
phi c mt vn kin thc tng i vng vng trn tt c cc lnh vc. Mi ngi
chng ta, c bit l cc bn yu ton, d t d nhiu th cng tng au u trc
mt bt ng thc kh v cng tng c c mt cm gic t ho phn khch m
mnh chng minh c bt ng thc . Nhm kch hot nim say m bt ng
thc trong cc bn, chng ti thc hin quyn sch Chuyn bt ng thc hin
i.
Sch gm 2 chng. Chng I chng ti xin c gii thiu n cc bn nhng k
thut (xin ch gi l k thut) m chng ti tm ti tch ly c trong sut thi gian
hc tp ca mnh. Do tt c cc k thut m chng ti cp y u c mi lin
h khng kht vi nhau (ci ny b tr ci kia v ngc li) nn chng ti xin c
php trnh by theo kiu tng bi chuyn nh, mi chuyn l mt k thut.
Tuy nhin, lnh vc bt ng thc hin nay rt pht trin (pht trin nht ca ton
hc s cp hin nay), cho nn chng ti khng th cp ht cc k thut (phng
php) c, cc k thut (phng php) tng xut hin cc sch, chng ti s
khng nhc li y, cc bn c th tm c chng da vo cc ti liu m chng ti
t phn ti liu tham kho. V cc k thut m chng ti s gii thiu trong sch,
hu ht chng l nhng k thut mnh v c dng gii nhng bi ton kh (n
rt kh) nn i khi (vic gii cc bi ton kh) th c th gp phi nhng tnh ton,
bin i phc tp, y l iu khng th trnh khi. Nhng cc bn hy yn tm, v
cc bi ton xut hin trong cc k thi hc gii (quc gia, olypimpic 30/4, thm ch
thi ton quc t) thng ch l nhng bi rt n gin, bnh thng nn vic s dng
cc k thut ny rt nh nhng v n gin. Chng hn nh bi ton thi IMO 2006
sau
Bi ton 0.1 Tm hng s nh nht sao cho bt ng thc sau ng vi cc s thc
a; b; c
ab(a2 b2 ) + bc(b2 c2 ) + ca(c2 a2 )
k(a2 + b2 + c2 )2 :
Li gii ca p n l mt li gii rt di v phc tp (s dng bt ng thc AMGM), i hi ngi lm phi rt kho lo, nhng vi li gii bng k thut nh
v

vi

LI NI U

gi cc bt ng thc hon v, chng ta ch nhn c mt li gii ngn gn 1/3 so


vi li gii gc ban u.
Chng II ca sch l tuyn tp nhng bi ton m chng ti (theo quan nim ca
bn thn) l hay v rt kh. Chng ti ch yu tuyn chn nhng bi bt ng thc
cha cn hoc nhng bi khng mu mc v chng ta khng th dng nhng bin
i thng thng gii chng v nh th th mi thc y chng ta sng to c.
Trong chng ny, phn ln chng ti u gii bng cch s dng bt ng thc
Cauchy Schwarz-Holder (CYH techniques) v bt ng thc Schur (bc 3, bc 4).
Thc t l i vi mt s bi ton th khng ch c mt li gii duy nht m cn c
nhiu li gii khc na, nhng y chng ti chn li gii bng cc bt ng thc
trn, v chng ti mun cc bn ha nhp vo quan im ca chng ti l Ci n
gin nht l ci mnh nht! Trong chng ny, c mt s bi ton kh, li gii m
chng ti tm c rt phc tp, chng ti rt mong cc bn s suy ngh v chng v
tm c mt li gii n gin hn.
Chng ti thc hin quyn sch ny vi mong mun cung cp thm cho cc bn thm
mt ngun bi tp (kh) v bt ng thc c th luyn tp thm k nng gii ton
ca mnh. Mc d rt c gng nhng khng c iu g l tuyt i c, nn kh
trnh khi nhng thiu st, sai lm. Mong cc bn thng cm v gp cho chng ti
c th quyn sch c th c chnh sa v hon thin hn. Xin chn thnh cm
n.
Xin gi tng quyn sch ny n ngi con gi ti yu qu nht, bn Phm Th Hng,
hc sinh chuyn ton K34, trng THPT Chuyn Phan Bi Chu, thnh ph Vinh,
tnh Ngh An.

V Quc B Cn
SV lp YY0647A1, trng HYD Cn Th
S nh C65 khu dn c Ph An, phng Ph Th, qun Ci Rng, tp. Cn Th
E-mail: can_hang2007@yahoo.com

Chng 1

Tm ti mt s k thut gii
ton
1.1

i lng (a

b)(b

c)(c

a)

Vi nhng bt ng thc hon v vng quanh, vic x l chng kh hn cc bt ng


thc i xng rt nhiu. Tuy nhin, mt im ng ch cc dng bt ng thc
ny, chng ta c th bin i chng thnh dng "bn i xng" nh sau
t f (a; b; c) chnh l biu thc hon v vng quanh bi, ta c th vit li f (a; b; c)
nh sau
1
1
f (a; b; c) = [f (a; b; c) + f (c; b; a)] + [f (a; b; c) f (c; b; a)]
2
2
Khi , c mt im ng ch l f (a; b; c) + f (c; b; a) l mt biu thc i xng
theo a; b; c v f (a; b; c) f (c; b; a), ta c th tch ra mt i lng kh c bit l
(a b)(b c)(c a): T , vic nh gi bi ton tr nn n gin hn nhiu.
Sau y l mt vi v d
V d 1.1 Cho cc s dng a; b; c: Chng minh rng
ab
bc
ca
+ 2
+ 2
3a2 + b2
3b + c2
3c + a2

3
:
4
(Dng c Lm)

Li gii. Bt ng thc tng ng vi


X (a
cyc

b)(3a b)
3a2 + b2
1

CHNG 1. TM TI MT S K THUT GII TON


,

(a

b)

cyc

X (a
cyc

X a2

b2
a2 + b2
cyc

a+b
a2 + b2

2(3a b)
3a2 + b2
2

b) (3a2 2ab + 3b2 )


(a2 + b2 )(3a2 + b2 )

Y a2

b2
a2 + b2
cyc

S dng bt ng thc AM-GM, ta c


X (a
cyc

b) (3a2 2ab + 3b2 )


(a2 + b2 )(3a2 + b2 )

Nn ta ch cn chng minh
v
uY
u (a
3
3t
cyc

v
uY
u (a
3
3t
cyc

b) (3a2 2ab + 3b2 )


(a2 + b2 )(3a2 + b2 )

b) (3a2 2ab + 3b2 )


(a2 + b2 )(3a2 + b2 )

Y a2

b2
a2 + b2
cyc

2
b) (3a2 2ab + 3b2 ) Y (a2 b2 )3
(a2 + b2 )(3a2 + b2 )
(a2 + b2 )3
cyc
cyc
Y
Y
, 27 (3a2 2ab + 3b2 )(a2 + b2 )2
(a b)(a + b)3 (3a2 + b2 )

Y (a
, 27

cyc

cyc

Bt ng thc ny c chng minh nu ta chng minh c bt ng thc sau vi


mi x; y > 0
3(3x2

2xy + 3y 2 )(x2 + y 2 )2

jx

yj (x + y)3 (3x2 + y 2 )

Theo bt ng thc Cauchy Schwarz, ta c


x2 + y 2

1
(x + y)2
2

Nn ta ch cn chng minh
3(3x2

2xy + 3y 2 )(x2 + y 2 )

2 x2

y 2 (3x2 + y 2 )

Bt ng thc ny hin nhin ng do


x2 + y 2

x2

y2

v
3(3x2

2xy + 3y 2 )

2(3x2 + y 2 ) = 3x2

Bt ng thc c chng minh xong.


ng thc xy ra khi v ch khi a = b = c:

6xy + 7y 2 = 3(x

y)2 + 4y 2

0:

1.1. I LNG (A

B)(B

C)(C

A)

V d 1.2 Cho a; b; c l di ba cnh ca mt tam gic nhn. Chng minh rng

a2

b3
c3
a3
+ 2
+ 2
2
2
+b
b +c
c + a2

a2
b2
c2
+
+
:
a+b b+c c+a
(V Quc B Cn)

Li gii. Trc ht, ta hy ch rng


X b3
cyc

a3
a2 + b2

X (b

a)(a2 + ab + b2 ) X
(a
=
a2 + b2
cyc

cyc

X ab(b

b) +

cyc

a)(a2 + c2 )(b2 + c2 )

ab(b

a2

a)

b2 )

cyc

(a2 + b2 )(b2 + c2 )(c2 + a2 )


!
!
P 2 2
P
P
a b
ab(b a) + abc c3 (a
cyc

cyc

(a2 + b2 )(b2 + c2 )(c2 + a2 )

(a

b)(b

c)(c

a)

a2 b2 + abc

cyc

X a2

X
b2
=
(a
a+b
cyc

cyc

(a2 + b2 )(b2 + c2 )(c2 + a2 )

cyc

b)

cyc

b) = 0

T , ta c th vit li bt ng thc nh sau


X a3 + b3
cyc

X
cyc

a2

b2

X a2 + b2
cyc

ab(a b)2
(a + b)(a2 + b2 )

X b3

a+b

(a

cyc

b)(b

a2

c)(c

a3 X a2 b2
+
+ b2
a+b
cyc
a)

2 2

a b + abc

cyc

(a2 + b2 )(b2 + c2 )(c2 + a2 )

cyc

S dng bt ng thc AM-GM, ta c


X
cyc

ab(a b)2
(a + b)(a2 + b2 )

33

a2 b2 c2 (a b)2 (b c)2 (c a)2


(a + b)(b + c)(c + a)(a2 + b2 )(b2 + c2 )(c2 + a2 )

CHNG 1. TM TI MT S K THUT GII TON

Ta cn chng minh
s
33

a2 b2 c2 (a b)2 (b c)2 (c a)2


(a + b)(b + c)(c + a)(a2 + b2 )(b2 + c2 )(c2 + a2 )
(a

b)(b

c)(c

a)

a2 b2 + abc

cyc

cyc

(a2 + b2 )(b2 + c2 )(c2 + a2 )


,

27a2 b2 c2 (a b)2 (b c)2 (c a)2


(a + b)(b + c)(c + a)(a2 + b2 )(b2 + c2 )(c2 + a2 )
(a

b) (b

c) (c

a)

2 2

a b + abc

cyc

cyc

(a2 + b2 )3 (b2 + c2 )3 (c2 + a2 )3

!3

,27a2 b2 c2 (a2 + b2 )2 (b2 + c2 )2 (c2 + a2 )2


2

(a

b )(b

c )(c

a )

2 2

a b + abc

cyc

cyc

!3

Do a; b; c l di 3 cnh ca mt tam gic nhn nn ta d dng chng minh c


a2 b2 c2

(a2

b2 )(b2

c2 )(c2

Ngoi ra, ta cng c


2

(a + b )(b + c )(c + a ) =

cyc

8
9

X
cyc

!
2

a2 )

2 2

a b

cyc

X
cyc

2 2

a b

a2 b2 c2
!

v
!3
u
X
8u
t3
2
2
a b
9
cyc
v
P 13
u 0P 2 2
a b + abc a
u
u
8 t @ cyc
cyc
A
3
9
2
2

2 2

2 2

2 2

) (a + b ) (b + c ) (c + a )

8
27

X
cyc

2 2

a b + abc

X
cyc

!3

Nhn tng ng v vi v cc bt ng thc ny, ta thu c bt ng thc trn.


ng thc xy ra khi v ch khi a = b = c hoc a = b; c = 0 v cc hon v.

1.1. I LNG (A

B)(B

C)(C

A)

V d 1.3 Cho cc s khng m a; b; c; khng c 2 s no cng bng 0: Chng minh


rng
p
3(a2 + b2 + c2 )
b3
c3
a3
+ 2
+ 2
:
2
2
2
2
a +b
b +c
c +a
2
(V Quc B Cn)
Li gii. Vit li bt ng thc nh sau
s X
X a3 + b3
a+b
3
a2
2 + b2
a
2
cyc
cyc
,

X (a
cyc

(a

b)(b

cyc

c)(c

cyc

X b3
cyc

(a b)2
r P
P
3 a2 +
a
cyc

a)

cyc

a2 b2 + abc

a nn ta ch cn chng minh c

b)2 (a + b)
2(a2 + b2 )

X (a
cyc

(a

b)2
P
+
2 a

(a

b)(b

c)(c

a)

a2 b2 + abc

cyc

b)2

cyc

1
a+b+c

a+b
a2 + b2
b)(b

c)(c

a)

2 2

a b + abc

cyc

b)2

cyc

(a2 + b2 )(b2 + c2 )(c2 + a2 )


(a

cyc

(a2 + b2 )(b2 + c2 )(c2 + a2 )

cyc

cyc

2(a

cyc

(a2 + b2 )(b2 + c2 )(c2 + a2 )

a3
+ b2

a2

cyc

X (a
cyc

cyc

a+

cyc

+
r P
Do 3 a2

b)2 (a + b)
2(a2 + b2 )

2ab + ac + bc
a2 + b2
2(a

b)(b

c)(c

a)

cyc

a2 b2 + abc

cyc

(a2 + b2 )(b2 + c2 )(c2 + a2 )

cyc

CHNG 1. TM TI MT S K THUT GII TON

S dng bt ng thc AM-GM, ta c


X

b)2

2ab + ac + bc
a2 + b2

(a

b)2 (b

(a

cyc

33

c)2 (c

a)2 (2ab + ac + bc)(2bc + ab + ac)(2ac + bc + ba)


(a2 + b2 )(b2 + c2 )(c2 + a2 )

Ta phi chng minh


s

33

(a

2(a

b)2 (b

c)2 (c

b)(b

a)2 (2ab + ac + bc)(2bc + ab + ac)(2ac + bc + ba)


(a2 + b2 )(b2 + c2 )(c2 + a2 )
!
!
P
P 2 2
P
a)
a
a b + abc a

c)(c

cyc

cyc

cyc

(a2 + b2 )(b2 + c2 )(c2 + a2 )


"

#"
#
Y
Y
2
2 2
, 27
(2ab + ac + bc)
(a + b )
cyc

"

cyc

Y
8
(a
cyc

b)

X
cyc

!3

cyc

Y
(a2 + b2 )2
cyc

64
81

!2

a2

cyc

X
cyc

!3

!3

ab

cyc

a b + abc

cyc

V
Y
(2ab + ac + bc)

2 2

a2 b2

cyc

!2

nn ta ch cn chng minh c
16
3

X
cyc

!3

ab

X
cyc

"

!2

Y
(a
cyc

X
cyc

b)

2 2

a b

!2

X
cyc

!3

X
cyc

2 2

a b + abc

X
cyc

!3

1.1. I LNG (A
By gi, ch rng
!2
X
2 2
8
a b

cyc

= 8

B)(B

a b

cyc

= A

!2

ab

cyc

2 2

!2

2 2

a b

C)(C

a b + abc

2 2

a b + 2abc

+ 12abc

a2 b2

!2

"

cyc

cyc

!3

cyc

X
cyc

2 2

cyc

cyc

abc

A)

2 2

a b + abc

cyc

!3

cyc

trong
A=5

a2 b2

cyc

!2

cyc

X
cyc

+ 3a2 b2 c2

cyc

!2

Ta cn phi chng minh


X

ab

cyc

cyc

Y
(a

b)

cyc

X
cyc

!3

Chun ha cho a + b + c = 1: t q = ab + bc + ca; r = abc th ta c


p
(a b)2 (b c)2 (c a)2
(a b)(b c)(c a)
p
=
q 2 4q 3 + 2(9q 2)r 27r2

Ta phi chng minh

Nu 9q
2q(1

2 th
p

2q)2

Do

2q)2

2(1
Nu 9q

p
q2

2q)2

2q(1

2 th

p
q2

4q 3

q2

4q 3 + 2(9q
p
1

+ 2(9q

4q =

2)r

4q 3 + 2(9q

27r2

2)r
p
1

27r2

4q

1
2

4
(1
27
r
4
(1
27

2)r

27r2

h
q 2(1

2q)2

1
+ [2(1
4

3q)3
3q)3

4q

4q)2 + 1]

1
(27r
27

9q + 2)2

CHNG 1. TM TI MT S K THUT GII TON


)2q(1

2q)2

q2

2q(1

2q)2

2q(1

2q)2

4q 3 + 2(9q 2)r 27r2


r
p
4
2
(1 3q)3 = 2q(1 2q)2
(1 3q) 3(1 3q)
27
9
2
8
46
(1 3q) =
(9q 2)(81q 2 63q + 13) +
> 0:
9
729
729

Bt ng thc c chng minh xong. ng thc xy ra khi v ch khi a = b = c:


V d 1.4 Cho cc s dng a; b; c tha mn
gic. Xc nh hng s k nh nht sao cho
a
b
c
+
+
2
2
b+c
c+a
a + b2

1 1 1
a; b; c

l di 3 cnh ca mt tam

c
a b
+ +
b
c a

a
b
c
+ 2+ 2
2
b
c
a

(V Quc B Cn)
Li gii. Cho a = b = c, khi bt ng thc tr thnh
3
a+1

9k
a

a
1
=
3(a + 1)
3

,k

1
3(a + 1)

1
Cho a ! +1, ta c k
3 . Ta s chng minh y chnh l gi tr m ta cn tm,
tc l
a
b
c
1 a b
c
a
b
c
+
+
+ +
+ 2+ 2
b + c2
c + a2
a + b2
3 b
c a
b2
c
a
X b
X a
X a2 X a
+
+
3
,
3
2
b
bc cyc a
b + c2
cyc
cyc
cyc

Do

cyc

a
b+c2

cyc

a
c2

nn ta ch cn chng minh c
X a2
cyc

b3

X a
X b
+
bc cyc a2
cyc

X a2
cyc

b3

X a
bc
cyc

X a
c2
cyc

X b
a2
cyc

t x = a1 ; y = 1b ; z = 1c , khi x; y; z l di 3 cnh ca mt tam gic. Bt ng


thc tr thnh
X y 3 X yz
X x2
+
2
0
2
x
x
y
cyc
cyc
cyc

1.1. I LNG (A

B)(B
2y 2
x

y3
+y
x2

cyc

X
cyc

X (x

A)

X yz
cyc

x+2

cyc

2(x

X x2

X y2

cyc

y)(y

cyc

z)(z

x)

cyc

xyz
2(x

y) (2y + zx)
2x2 y

cyc

y
z
+
x2
2xy

y)2

(x

C)(C

y)(y

z)(z

x)

cyc

xyz

S dng bt ng thc AM-GM, ta c


X (x

y) (2y + zx)
2x2 y

cyc

rQ
(x
33

y)2

cyc

(2x2 + yz)

cyc

2xyz

Ta cn chng minh
rQ
(x
33

y)2

cyc

(2x2 + yz)

cyc

2(x

y)(y

x)

cyc

2xyz

Y
, 27 (2x2 + yz)

z)(z
xyz

64(x

y)(y

z)(z

x)

cyc

X
cyc

!3

chng minh bt ng thc ny, trc ht ta s chng minh


Y
9 (2x2 + yz)
cyc

X
cyc

!3

X
cyc

xy

Do tnh thun nht, ta c th chun ha cho x+y+z = 1. t q = xy+yz+zx; r = xyz,


khi ta c 31 q 14 v
Y
(2x2 + yz) = 27r2 + 2(1

9q)r + 4q 3

cyc

Bt ng thc tr thnh
243r2 + 18(1

9q)r + 36q 3

10

CHNG 1. TM TI MT S K THUT GII TON


5q 1 1
,
18

y l mt hm lm theo r v vi ch rng r
243r2 + 18(1

9q)r + 36q 3

5q 1
18

243
=

1
(16q
4

ta c

1)(1

+ (1

8q)(5q

3q)2

1) + 36q 3

Tip theo, s dng bt ng thc trn, ta ch cn chng minh


!
!
X
X
3
x
xy
64(x y)(y z)(z x)
cyc

cyc

t x = m + n; y = n + p; z = p + m (m; n; p > 0), bt ng thc ny tng ng


vi
!
!
X
X
X
2
3
m
m +3
mn
32(m n)(n p)(m p)
cyc

cyc

cyc

T y, gi s p = minfm; n; pg, v t m = p + u; n = p + v (u; v


X
m = 3p + u + v u + v

0), ta c

cyc

m2 + 3

cyc

mn = 12p2 + 8(u + v)p + u2 + 3uv + v 2

u2 + 3uv + v 2

cyc

(m

n)(n

p)(m

p) = uv(u

v)

Nn ta ch cn chng minh
3(u + v)(u2 + 3uv + v 2 )
, 3u3
, 3u u

32uv(u

20u2 v + 44uv 2 + 3v 3
10
v
3

v)
0

32 2
uv + 3v 3
3

0:

hin nhin ng. Vy ta c pcm.


V d 1.5 Cho cc s khng m a; b; c; khng c 2 s no ng thi bng 0: Chng
minh rng
(a

b)(13a + 5b) (b
+
a2 + b2

c)(13b + 5c) (c
+
b2 + c2

a)(13c + 5a)
c2 + a2

0:

(V Quc B Cn)
1 y

chnh l bt ng thc Schur bc 3

1.1. I LNG (A

B)(B

C)(C

A)

11

Li gii. Bt ng thc tng ng vi


X 4(a

b)2 + 9(a2
a2 + b2

cyc

,4
,4

X (a
cyc

a2

X (a

b)2
a2 + b2

cyc

b)
+ b2

b2 )
X b2
cyc

a2
a2 + b2

9(a
b )(b2 c2 )(c2 a2 )
2
(a + b2 )(b2 + c2 )(c2 + a2 )

Theo bt ng thc AM-GM,


4

X (a
cyc

b)2
2
a + b2

12 3

(a b)2 (b c)2 (c a)2


(a2 + b2 )(b2 + c2 )(c2 + a2 )

Ta cn chng minh
s
(a b)2 (b c)2 (c a)2
43 2
(a + b2 )(b2 + c2 )(c2 + a2 )

3(a2 b2 )(b2 c2 )(c2 a2 )


(a2 + b2 )(b2 + c2 )(c2 + a2 )

Bt ng thc ny l h qu ca bt ng thc sau vi mi x > y


4(x2 + y 2 )2

Nu x

6y th
x4

Nu x

, x4

3(x2

y 2 )(x + y)2

6x3 y + 8x2 y 2 + 6xy 3 + 7y 4

6x3 y + 8x2 y 2 + 6xy 3 + 7y 4 = x3 (x

6y) + 8x2 y 2 + 6xy 3 + 7y 4

3y)2 + xy 2 (6y

0:

6y; ta c
x4

6x3 y + 8x2 y 2 + 6xy 3 + 7y 4 = x2 (x

x) + 7y 4

Vy ta c pcm. ng thc xy ra khi a = b = c:


V d 1.6 Cho cc s khng m a; b; c; khng c 2 s no ng thi bng 0: Chng
minh rng
ab
bc
ca
3
:
+ 2
+ 2
2
2
2
2
a + 4b
b + 4c
c + 4a
5
V d 1.7 Cho cc s khng m a; b; c; khng c 2 s no ng thi bng 0: Chng
minh rng
(a b)(3a b)
(b c)(3b c)
(c a)(3c a)
+ 2
+ 2
3a2 + 2ab + 3b2
3b + 2bc + 3c2
3c + 2ca + 3a2

0:
(Thomas Mildorf)

12

1.2

CHNG 1. TM TI MT S K THUT GII TON

Nhng kiu li gii c bit bng AM-GM

V d 1.8 Cho cc s khng m a; b; c tha a + b + c = 3: Chng minh rng


r
r
r
3
a3
b3
c3
+
+
:
2
2
2
2
2
2
a + 3b
b + 3c
c + 3a
2
(Phan Thnh Vit)
Li gii. S dng bt ng thc AM-GM, ta c
r
X
X
a3
a2
p
=
6
a2 + 3b2
4a(a + b + c) 3(a2 + 3b2 )
cyc
cyc
6

X
cyc

X
cyc

a2
4a(a + b + c) + 3(a2 + 3b2 )
a2
7a2 + 9b2 + 4ab + 4ca

Mt khc, theo bt ng thc Cauchy Schwarz th


!"
#
X
X
a2
2
2
2
(c + 2a) (7a + 9b + 4ab + 4ca)
7a2 + 9b2 + 4ab + 4ca
cyc
cyc
"
#2
!2
X
X
X
2
a(c + 2a) = 2
a +
ab
cyc

cyc

cyc

Nn ta ch cn chng minh c
8 2

a2 +

cyc

X
cyc

a4 +

X
cyc

!2

ab

a2 b2 + 3

cyc

(c + 2a)2 (7a2 + 9b2 + 4ab + 4ca)

cyc

X
cyc

a3 b

ab3

cyc

Gi s a = min fa; b; cg ; t b = a + x; c = a + y (x; y


thnh
6(x2

xy + y 2 )a2 + (4x3 + 9x2 y

Ta c
4x3 + 9x2 y

2abc

0) th bt ng thc tr

9xy 2 + 4y 3 )a + x4 + 3x3 y + x2 y 2

9
9xy 2 + 4y 3 = 4x3 + y(2x
4

cyc

7
y)2 + y 3
4

3xy 3 + y 4

1.2. NHNG KIU LI GII C BIT BNG AM-GM


x4 + 3x3 y + x2 y 2

3xy 3 + y 4 =

13

3
x2 + xy
2

y2

3
+ x2 y 2
4

0:

Nn bt ng thc trn hin nhin ng. Vy ta c pcm. ng thc xy ra khi va


ch khi a = b = c = 1:
V d 1.9 Cho cc s khng m a; b; c; khng c 2 s no ng thi bng 0: Chng
minh rng
5(a + b + c)

p
p
p
4a2 + bc + 4b2 + ca + 4c2 + ab :

(V Quc B Cn)
Li gii. Khng mt tnh tng qut, ta c th gi s a
thc AM-GM, ta c
p
2 4a2 + bc

p
2 4b2 + ca

)2

Xp
cyc

c, khi theo bt ng

2a + c +

c(b 2a)
4a2 + bc
= 4a + c +
2a + c
2a + c

2b + c +

4b2 + ca
c(a 2b)
= 4b + c +
2b + c
2b + c

p
2 4c2 + ab

4a2 + bc

b+

c 2(ab + 4c2 )
+
2
2b + c

5
2(ab + 4c2 )
4a + 5b + c +
+c
2
2b + c

b 2a a 2b
+
2a + c
2b + c

Ta cn chng minh
5
a+ c
2
,

2(ab + 4c2 )
+c
2b + c

c(2a + 10b 11c)


2(2b + c)
,

2a + 10b 11c
2(2b + c)
,

b 2a a 2b
+
2a + c
2b + c
b 2a a 2b
+
2a + c
2b + c
b 2a a 2b
+
2a + c
2b + c

14b 11c 2a b
+
2(2b + c)
2a + c

0:

hin nhin ng v a b c:
ng thc xy ra khi v ch khi a = b; c = 0 hoc cc hon v tng ng.

14

CHNG 1. TM TI MT S K THUT GII TON

V d 1.10 Cho cc s khng m a; b; c; khng c 2 s no ng thi bng 0: Chng


minh rng
p
p
p
a a2 + 3bc + b b2 + 3ca + c c2 + 3ab 2(ab + bc + ca):

(Vasile Cirtoaje)

Li gii. S dng bt ng thc AM-GM, ta c


X a(b + c)(a2 + 3bc)
X p
p
a a2 + 3bc =
2
cyc (b + c) a + 3bc
cyc

X a(b + c)(a2 + 3bc)


a2 + 3bc + (b + c)2
cyc

Do , ta ch cn chng minh c

,2
,

X a(b + c)(a2 + 3bc)


a2 + 3bc + (b + c)2
cyc

X a(b + c)(a2

b2 c2 + bc)
s + 5bc

cyc

,
,
,

X a(b + c)(a2 + 3bc)


a2 + 3bc + (b + c)2
cyc

X a3 (b + c)

X ab(a2

X ca(c2

cyc

cyc

cyc

b2 )
s + 5bc

X ab(a2

b2 )
s + 5bc

, 5abc

cyc

a(b + c)

cyc

(s = a2 + b2 + c2 )

a(b3 + c3 )
s + 5bc

cyc

ab

cyc

a2 )
s + 5bc

X ab(a2

b2 )
s + 5ca

X (a b)(a2 b2 )
(s + 5bc)(s + 5ca)
cyc

0:

hin nhin ng.


ng thc xy ra khi v ch khi a = b = c hoc a = b; c = 0 hoc cc hon v tng
ng.
Nhn xt 1 Chng ta cng c mt cch khc gii bi ton ny nh sau
Vit li bt ng thc nh sau
X p
X
X
a
a2 + 3bc a
2
ab
a2
cyc

cyc

cyc

1.2. NHNG KIU LI GII C BIT BNG AM-GM


, 3abc

X
cyc

a+

1
a2 + 3bc

ab

cyc

15
a2

cyc

S dng bt ng thc Cauchy Schwarz, ta c


X
cyc

a+

1
a2 + 3bc

a+

cyc

9
Pp

v
u
u
P
a + t3

a2 + 3bc

cyc

cyc

3
!2 = P a
P
cyc
a

9
v
u
u
P
a + t4

cyc

9
P

cyc

a2 + 3

cyc

bc

cyc

Vy nn ta ch cn chng minh c
9abc
P
a
cyc

ab

cyc

a3 + 3abc

cyc

a2

cyc

ab(a + b):

cyc

y chnh l bt ng thc Schur bc 3 nn ta c pcm.

V d 1.11 Cho cc s khng m a; b; c, khng c 2 sn o ng thi bng 0: Chng


minh rng
b+c
c+a
a+b
p
+p
+p
3:
3ab + c2
3bc + a2
3ca + b2
(Michael Rozenberg)
Li gii. S dng bt ng thc AM-GM, ta c
X
cyc

a+b
3ab + c2

X
cyc

X
cyc

Ta cn chng minh

X
cyc

6(a + b)(a + b + c)
p
2(a + b + c) 3 3ab + c2
12(a + b)(a + b + c)
4(a + b + c)2 + 9(3ab + c2 )

12(a + b)(a + b + c)
4(a + b + c)2 + 9(3ab + c2 )

16

CHNG 1. TM TI MT S K THUT GII TON


,

X 8a2 + 8b2

cyc

X (a

11ab + 4c(a + b)
4s2 + 27ab + 9c2

c)(16a

cyc

X
cyc

(b

13c2

11b + 13c) + (b c)(16b


4s2 + 27ab + 9c2

(s = a + b + c)
11a + 13c)

16c 11a + 13b


16b 11a + 13c
2
2
4s + 27ab + 9c
4s2 + 27ca + 9b2
X
,
x(b c)2 (4s2 + 27bc + 9a2 ) 0

c)

cyc

trong
x = 99a2

150(b + c)a + 48b2 + 87bc + 48c2 + 4s2

v y; z tng t.
t t = b+c
2 , ta c
x = 99a2
= 99a2
99a2

150(b + c)a + 48b2 + 87bc + 48c2 + 4s2


9
300at + 4(a + 2t)2 + 183t2 + (b c)2
4
300at + 4(a + 2t)2 + 183t2

= 103a2 284at + 199t2


1
=
[(103a 142t)2 + 333t2 ]
103

Tng t, ta c y; z 0.
Bt ng thc c chng minh. ng thc xy ra khi v ch khi a = b = c.
V d 1.12 Cho cc s khng m a; b; c tha a + b + c = 1: Chng minh rng
r
r
r
a
b
c
+
+
2:
2a2 + bc
2b2 + ca
2c2 + ab
(V Quc B Cn)
Li gii. S dng bt ng thc AM-GM, ta c
Xr
X
a
a
p
=
2
2a + bc
a(a + b + c) (2a2 + bc)
cyc
cyc
X
a
2
a(a + b + c) + 2a2 + bc
cyc
X
a
= 2
2 + ab + bc + ca
3a
cyc

1.2. NHNG KIU LI GII C BIT BNG AM-GM

17

t q = ab + bc + ca; r = abc, ta cn chng minh


X
cyc

,
, 9abc

a(3b2 + q)(3c2 + q)

a2 (b + c) + q 2

cyc

, 9qr + 3q(q

q2

(3a2 + q)(3b2 + q)(3c2 + q)

27a2 b2 c2 + 9q

cyc

3r) + q 2

, [q 2
Ch rng r

cyc

ab + 3q

cyc

a
3a2 + q

a2 b2 + 3q 2

cyc

27r2 + 9q(q 2

4q 3 + 2(9q

2)r

2r) + 3q 2 (1

27r2 ] + 4r

a2 + q 3

cyc

2q) + q 3

0 v
4q 3 + 2(9q

27r2 = (a

2)r

b)2 (b

c)2 (c

a)2

0:

Nn bt ng thc cho hin nhin ng. ng thc xy ra khi v ch khi (a; b; c) =


1 1
2; 2; 0 :
Nhn xt 2 T bi ton ny, ta suy ra kt qu rt kh sau, hin ch mi nhn c
mt li gii ca chng ti trn mathlinks
r
r
r
p
a
b
c
2:
+
+
2
a2 + bc
b2 + ca
c2 + ab

V d 1.13 Cho cc s khng m a; b; c tha mn a + b + c = 1: Chng minh rng


p
p
p
a + b2 + b + c2 + c + a2 2:

(Phan Thnh Nam)

Li gii. Ta c

p
p
p
a + b2 + b + c2 + c + a2
X p
,
a + b2 b
1
cyc

X
cyc

a
a + b2 + b

18

CHNG 1. TM TI MT S K THUT GII TON

Theo bt ng thc AM-GM,


X
X
a
p
=
a + b2 + b
cyc
cyc

2(a+b) a+b2
2(a+b)

2a2

cyc

Do , ch cn chng minh c
X
cyc

,4

,4

a4 b2 + 3

cyc

a3 b2 c

19

4 2

a b

a b c

cyc

a2 b3 c + 16

3a b c

cyc

1
X

a4 bc

12a2 b2 c2

3 2

2 2 2

a b c

a b c

+ 15

a bc

cyc

cyc

2 3

cyc

cyc

ng thc xy ra khi v ch khi a = b = c =


v tng ng.

cyc

1
3

3a2 b2 c2 :

cyc

hoc a = 1; b = c = 0 hoc cc hon

V d 1.14 Cho cc s khng m a; b; c; khng c 2 s no ng thi bng 0: Chng


minh rng
1
1
1
4
p
+p
+p
:
a+b+c
4a2 + bc
4b2 + ca
4c2 + ab
(Phm Kim Hng)
Li gii. S dng bt ng thc AM-GM, ta c
X
cyc

1
4a2 + bc

X
cyc

X
cyc

X
cyc

3
2a

3
2a
1
2b

+ 12 b + 12 c
p
+ 12 c 4a2 + bc

3
2a

+ 12 b + 12 c

4a2 + bc +

3
2a

+ 12 b + 12 c

3a + b + c
16a2 + 4bc + (3a + b + c)2

a b c

cyc

Bng cch s dng bt ng thc AM-GM, ta thy ngay c


X
X
X
X
X
X
a4 b2
a2 b3 c;
a3 b2 c a2 b2 c2 ;
a4 bc
a2 b3 c;
a4 bc
cyc

cyc

cyc

+b

a(a + b)
+ 5ab + 4b2 + ca

+3

2 2 2

a bc

+b

cyc

2 3

(a+b)2 +a+b2
cyc
2(a+b)

a(a + b)
2a2 + 5ab + 4b2 + ca

cyc

cyc

1.2. NHNG KIU LI GII C BIT BNG AM-GM

19

Cui cng, ta ch cn chng minh


X
cyc

,
,

X
cyc

X
cyc

,2
,

1
a+b+c

3a + b + c
16a2 + 4bc + (3a + b + c)2

(3a + b + c)(a + b + c)
16a2 + 4bc + (3a + b + c)2

3(3a + b + c)(a + b + c)
16a2 + 4bc + (3a + b + c)2

X b2 + c2 8a2 + 3a(b + c)
16a2 + 4bc + (3a + b + c)2
cyc

X (c + 4a)(c
cyc

16a2

a) (b + 4a)(a b)
+ 4bc + (3a + b + c)2

(c + 4a)(c a)
16a2 + 4bc + (3a + b + c)2

(b + 4a)(a b)
16a2 + 4bc + (3a + b + c)2

16b2

(a + 4b)(a b)
+ 4ca + (3b + c + a)2

(b + 4a)(a b)
16a2 + 4bc + (3a + b + c)2

cyc

cyc

,7

(a

b)2 (7a2 + 7b2

cyc

[16a2

cyc

cyc

(a b)2 (7a2 + 7b2 c2 + 34ab 6ca 6bc)


+ 4bc + (3a + b + c)2 ][16b2 + 4ca + (3b + c + a)2 ]
c2 + 34ab

6ca

6bc)[16c2 + 4ab + (3c + a + b)2 ]

cyc

,
Gi s a

Sc (a

b)2

cyc

c; khi d thy Sb ; Sc
a2 [16b2 + ca + (3b + c + a)2 ]

0. Ta c
b2 [16a2 + bc + (3a + b + c)2 ]

(7b2 + 7c2 a2 + 34bc 6ca 6ab) + (7c2 + 7a2


= 6(a b)2 + 28c(a + b) 0
) a2 Sb + b2 Sa

b2 + 34ca

6ab

6bc)

20

CHNG 1. TM TI MT S K THUT GII TON

Suy ra
X

Sc (a

b)2

c)2 + Sb (c

Sa (b

a)2

Sa (b

cyc

(b

c)2
b2

(a2 Sb + b2 Sa )

c)2 +

a2
Sb (b
b2

c)2

0:

Bt ng thc c chng minh. ng thc xy ra khi v ch khi a = b; c = 0 hoc


cc hon v tng ng.
V d 1.15 Cho cc s khng m a; b; c tha mn a + b + c = 1: Chng minh rng
r
r
r
p
(b c)2
(c a)2
(a b)2
a+
+ b+
+ c+
3:
8
8
8
(V Quc B Cn)
Li gii. S dng bt ng thc AM-GM, ta c
r
p
p
X
16a + 2(b
1 X 3(a + 1)
(b c)2
p
=
a+
8
a+1
4 3 cyc
cyc

c)2

1 X 3(a + 1)2 + 16a + 2(b c)2


p
a+1
8 3 cyc
"
#
X 16a + 2(b c)2
1
p 12 +
a+1
8 3
cyc
"
#
X 8a + (b c)2
1
p 6+
a+1
4 3
cyc

Ta cn chng minh
X 8a + (b

c)2

a+1

[8a + (b

X
, 2 a3
cyc

Do

c)2 ](b + 1)(c + 1)

ab(a + b) +

cyc

bc(b

cyc

X
cyc

ab(a + b)

6
6(a + 1)(b + 1)(c + 1)
X
c)2 + 4 ab + 18abc
cyc

6abc

1.2. NHNG KIU LI GII C BIT BNG AM-GM


Nn ta ch cn chng minh c
X
X
2 a3 + 12abc +
bc(b
cyc

X
c)2 + 4 ab

cyc

X
X
, 2 a3 + 12abc +
bc(b
cyc

X
c) + 4 ab

X
X
, 2 a3 + 12abc +
bc(b
X

ab(a + b)

c)2

ab(a + b)

6abc

cyc

a (b + c) + 6

cyc

X
2 a2
a2

cyc

cyc

2
X

6abc

cyc

!2

cyc

cyc

,2
,2

c)2

cyc

cyc

cyc

X
X
, 2 a3 + 12abc +
bc(b
cyc

cyc

cyc

21

ab(a

cyc

b)2

cyc

cyc

2 2

a b

cyc

ab(a

b)2

cyc

X
8abc a:
cyc

Bt ng thc cui hin nhin ng theo bt ng thc AM-GM nn ta c pcm.


ng thc xy ra khi v ch khi a = b = c = 31 :
V d 1.16 Cho cc s khng m a; b; c; khng c 2 s no ng thi bng 0: Chng
minh rng
a
b
c
3
p
:
+p
+p
2
2
2
2
ab + 3c
bc + 3a
ca + 3b
(Vasile Cirtoaje)
Li gii. S dng bt ng thc AM-GM, ta c
X
cyc

a
ab + 3c2

=
=

Ta cn chng minh

X
a(c + a)
a(c + a)
p
2
2 + ab + 3c2
2
(c
+
a)
cyc (c + a) ab + 3c
cyc
X
a(c + a)
2
2 + ab + 2ca + 4c2
a
cyc

X
2

a(c + a)
2 + ab + 2ca + 4c2
a
cyc

3
2

22

CHNG 1. TM TI MT S K THUT GII TON

Bng bin i tng ng, ta thy bt ng thc ny tng ng vi


X
X
X
X
X
4 ab5 + 12 a2 b4 + 16 a4 b2 24 a3 b3 + 18 a4 bc
cyc

cyc

cyc

X
+39 a2 b3 c

X
17 a3 b2 c

cyc

Ta c

144a2 b2 c2

24

cyc

cyc

cyc

X
a3 b3 = 12 a2 b2 (a

cyc

17

cyc

S dng bt ng thc AM-GM, ta c


X
4 ab5

b)2

cyc

Nn ta ch cn chng minh c
X
X
X
X
4 ab5 + 4 a4 b2 + 18 a4 bc + 39 a2 b3 c
cyc

cyc

cyc

X
X
12 a2 b4 + 12 a4 b2
cyc

cyc

a3 b2 c + 144a2 b2 c2

cyc

12a2 b2 c2

cyc

X
4 a4 b2

12a2 b2 c2

cyc

a4 bc

3a2 b2 c2

cyc

X
17 a4 bc
cyc

39

a2 b3 c

17

a3 b2 c

cyc

117a2 b2 c2 :

cyc

Cng tng ng v vi v cc bt ng thc trn, ta suy ra pcm. ng thc xy ra


khi v ch khi a = b = c:

1.3
1.3.1

K thut pqr
Li ni u

K thut pqr l mt trong nhng k thut hay, hu ch v hiu qu nht i vi bt


ng thc 3 bin. Phn ln cc bi ton trong sch, chng ti u chn k thut pqr
gii, v vy, vic h thng li mt s kin thc cn bit v chng l khng th thiu
c. Bn s thy c nhng iu ngc nhin, l lm khi m khng ch c nhng bi
ton i xng mi c th c gii quyt theo pqr m thm ch c nhng bi ton
dng hon v vng quanh, ta cng c th dng n gii trong khi nhng phng
php, nhng k thut khc li khng kh nng thc hin iu ny. Sau khi xin
c bt u bi vit ca chng ti "K thut pqr".

1.3. K THUT P QR

1.3.2

23

Nhng ng thc cn nh

Vi 3 bin bt k a; b; c; ta t p = a+b+c; q = ab+bc+ca; r = abc (p2


Khi , chng ta c nhng ng thc sau
a2 + b2 + c2
a3 + b3 + c3
ab(a + b) + bc(b + c) + ca(c + a)
(a + b)(b + c)(c + a)
a4 + b4 + c4
a2 b2 + b2 c2 + c2 a2
a3 (b + c) + b3 (c + a) + c3 (a + b)
a3 (b2 + c2 ) + b3 (c2 + a2 ) + c3 (a2 + b2 )
a4 (b + c) + b4 (c + a) + c4 (a + b)
a5 + b5 + c5

=
=
=
=
=
=
=
=
=
=

p2
p3
pq
pq
p4
q2
p2 q
pq 2
qp3
p5

3q; q 2

2q
3pq + 3r
3r
r
4p2 q + 2q 2 + 4pr
2pr
2q 2 pr
(2p2 + q)r
3pq 2 + (5q p2 )r
5p3 q + 5pq 2 + 5(p2

3pr):

q)r

Cn rt nhiu nhng ng thc khc na, cc bn hy t xy dng cho mnh thm


nh, chng s rt c ng dng v sau.

1.3.3

Bt ng thc Schur

nh l 1.1 (Bt ng thc Schur) Cho cc s khng m a; b; c: Khi , vi mi


r > 0; ta c bt ng thc sau
ar (a

b)(a

c) + br (b

c)(b

a) + cr (c

a)(c

b)

ng thc xy ra khi v ch khi a = b = c hoc a = b; c = 0 hoc cc hon v tng


ng.
Chng minh. Do tnh i xng, gi s a b c: Khi , ta vit bt ng thc li
nh sau
(a b)[ar (a c) br (b c)] + cr (a c)(b c) 0
Ta c
a

0;

ar

br

Nn bt ng thc ng. Bt ng thc Schur c chng minh.


Chng ta c 2 trng hp c bit thng hay c ng dng gii ton l r = 1
v r = 2: Khi , chng ta c nhng bt ng thc tng ng l
H qu 1.1 (Bt ng thc Schur bc 3) Cho cc s khng m a; b; c: Khi ,
bt ng thc sau ng
a3 + b3 + c3 + 3abc

ab(a + b) + bc(b + c) + ca(c + a)

24

CHNG 1. TM TI MT S K THUT GII TON


, abc

(a + b

c)(b + c

a)(c + a

b):

ng thc xy ra khi v ch khi a = b = c hoc a = b; c = 0 hoc cc hon v tng


ng.
H qu 1.2 (Bt ng thc Schur bc 4) Cho cc s khng m a; b; c: Khi ,
bt ng thc sau ng
a4 + b4 + c4 + abc(a + b + c)

a3 (b + c) + b3 (c + a) + c3 (a + b):

ng thc xy ra khi v ch khi a = b = c hoc a = b; c = 0 hoc cc hon v tng


ng.
Dng pqr tng ng ca 2 bt ng thc trn l

9
(4q

p2 )

p(4q

p2 )(p2
6p

q)

Nhng do 4q p2 c th khng dng m r th lun lun khng m nn chng ta hay


dng c 2 bt ng thc trn dng sau (s rt hiu qu)
r

max 0;

max 0;

p2 )

p(4q

(4q

9
p2 )(p2
6p

q)

i khi bn s gp phi trng hp gi thit bi ton l a; b; c l di 3 cnh ca


mt tam gic (khi ta c 4q p2 ), khi ta thy a + b c; b + c a; c + a b l
nhng s khng m, vy nn theo bt ng thc Schur, ta c
X
(b + c a)[(b + c a) (c + a b)][(b + c a) (a + b c)] 0
cyc

(b + c

,r
Tng t, ta c

a)(a

b)(a

c)

cyc

X
cyc

(b + c

p(5q p2 )
18
a)2 (a

b)(a

c)

1.3. K THUT P QR

25
p4

,r

7p2 q + 13q 2
9p

Vy chng ta c cc nh gi
p(5q p2 ) p4
;
18

min

7p2 q + 13q 2
9p

p2 )(p2
6p

(4q

max 0;

q) p(4q
;

p2 )
9

Chng ta thng dng bt ng thc Schur gii bt ng thc trong trng hp


bt ng thc c nhng ng thc ti cc im a = b = c hoc a = b; c = 0 hoc
trong trng hp a; b; c l di 3 cnh tam gic th l a = 2; b = c = 1:
V d 1.17 Cho cc s khng m a; b; c tha mn ab + bc + ca = 3: Chng minh rng
a3 + b3 + c3 + 7abc

10:
(Vasile Cirtoaje)

Li gii. Bt ng thc tng ng vi


10r + p3
Nu p

10

2 3 th ta c
p3

p
Nu 2 3

9p

9p

10

3p

10

p
6 3

10 > 0

3 th theo bt ng thc Schur bc 3, ta c


r

p2 )

p(12
9

Do
10r + p3

9p

10

30

p2

10(p(12
9

M
3p

30

p2 )

+ p3

p
2 3

9p

10 =

1
(p
9

p
3 2 3 = 18

3)(30

p2

3p)

p
6 3 > 0:

Nn bt ng thc cn chng minh ng. ng thc xy ra khi v ch khi a = b =


c = 1:
V d 1.18 Cho cc s dng a; b; c tha mn a + b + c = 3: Chng minh rng
abc +

12
ab + bc + ca

5:
(Vasile Cirtoaje)

26

CHNG 1. TM TI MT S K THUT GII TON

Li gii. Bt ng thc tng ng vi


r+

12
q

S dng bt ng thc Schur bc 3, ta c


4q

9
3

Do
r+

12
q

4q

9
3

12
q

3)2

4(q

5=

0:

3q

Bt ng thc c chng minh. ng thc xy ra khi v ch khi a = b = c = 1:


V d 1.19 Cho cc s khng m a; b; c; khng c 2 s no ng thi bng 0 tha
mn a2 + b2 + c2 = 1: Chng minh

b2

a3
+ 2
bc + c2
c

b3
+ 2
ca + a2
a

c3
ab + b2

2:
(V Quc B Cn)

Li gii. Bt ng thc tng ng


X a3 (b + c)
cyc

,
,
Ta c

b3 + c3

X a3 (b + c)
+b+c
b3 + c3
cyc

a3

cyc

X
cyc

a2

X
cyc

a2

a+

P
9 a3
cyc
P
P
2 a2
ab
cyc

cyc

9
a2

X
cyc

a+

cyc

cyc

Nn ta ch cn chng minh

cyc

1
ab + b2

1
ab + b2

ab

cyc

a+

1.3. K THUT P QR

27

t p = a + b + c; q = ab + bc + ca; r = abc ) q =
1 p
2
2
Nu p

1 p
2
2

p
7p2 + 6 2p

5 + 27r
=

Bt ng thc tng ng

5 + 27r
p(4q p2 )
9

2 theo bt dng thc Schur, ta c r


p
7p2 + 6 2p

p2 1
2 :

p(p2 2)
;
9

do

p
1 p
2 p 7p2 + 6 2p
2
p
1
p
2 (5 p2 ) 0
2

5 + 3p(p2

2)

p
Nu 2 p th bt ng thc trn hin nhin ng.
Vy ta c pcm. ng thc xy ra khia = b = p12 ; c = 0 v cc hon v tng ng.
V d 1.20 Cho cc s khng m a; b; c; khng c 2 s no ng thi bng 0 tha
mn a2 + b2 + c2 = 3: Chng minh rng
p

2
b+c

2
c+a

2
a+b

(Phm Kim Hng)


Li gii. Ch l cc biu thc trong cc du ngoc u dng v
2+

b+c

2+

c+a

2+

p p
2 2
2+1

a+b

Nn
p p
2+1
2 2

Y
cyc

2
p
b+c

"

1
=

T gi thit ta suy ra p2

2+

b+c

cyc

(4

#"

Y
cyc

2
p
b+c

b c)(4 c a)(4 a
p
(a + b)(b + c)(c + a)

b)

2q = 3: Do theo bt ng thc Schur bc 3, ta c


r

p2 )

p(4q
9

p(p2 6)
9

28

CHNG 1. TM TI MT S K THUT GII TON

Khi , ta c
(4

b c)(4 c a)(4 a
p
(a + b)(b + c)(c + a)

b)

=
=

4(4

p)2 + (4 p)q + r
p
pq r

(4

p)(p2 8p + 29) + 2r
p
2[p(p2 3) 2r]

(4

p)(p2 8p + 29) + 2p(p9


r h
i
2
2 p(p2 3) 2p(p9 6)

6)

561p + 108p2 7p3


p
= f (p)
3 2p(7p2 15)
p
p
3; 3 nn f (p) f (3) = 2 2; do
D thy f (p) l hm nghch bin trn
1044

p p
2 2
2+1

Y
cyc

Y
cyc

2
b+c

p
1

2
b+c

p
2 2

1
p

Vy ta c pcm. ng thc xy ra khi a = b = c = 1:

1.3.4

i lng (a

b)2 (b

c)2 (c

a)2

i vi nhng bt ng thc rt cht v ng thc xy ra ti nhng im khng c


bit nh bt ng thc Schur (chng hn ng thc xy ra ti a = 3; b = 2; c = 2) th
vic s dng bt ng thc Schur gii chng l iu hin nhin khng thc hin
c, do chng ta cn tm mt nh gi khc ph hp hn v hiu qu hn
gii chng. i lng
P = (a b)2 (b c)2 (c a)2 0
l i lng trung gian khc m chng ta chn y. Ti sao ta li chn n? V hu
ht cc bt ng thc i xng u xy ra ng thc khi c t nht 2 bin bng nhau
m biu thc P xy ra du ng thc, ta cng ch cn a = b hoc b = c hoc c = a
l , cho nn ta c th thy P rt cht. V vy, ta s khai thc xem P c ng dng
g khng? Khai trin ra theo pqr ta c
P = p2 q 2

4q 3 + 2p(9q

2p2 )r

27r2

Ta hy xem y l mt tam thc bc 2 theo r, khi gii ra ta c nghim


p
p
p(9q 2p2 ) 2(p2 3q) p2 3q
p(9q 2p2 ) + 2(p2 3q) p2
r
27
27

3q

1.3. K THUT P QR

29

n y, c l cc bn vn cha thy c g ngoi s cng knh ca bt ng thc


trn. ng vi nn lng bn , bit u s c mt php mu no y. V thc s l
nh vy, ta hy t
p
p
p 2 p2 3q
p + p2 3q
; v0 =
u0 =
p3
p3
2
p
p
3q
p + 2 p2 3q
u1 =
; v0 =
3
3
Khi , ta thu c mt iu c bit l
8
< 2u0 + v0 = 2u1 + v1 = p
u2 + 2u0 v0 = u21 + 2u1 v1 = q
: 02
u0 v0 r u21 v1

Ngoi ra, trong trng hp a; b; c l cc s khng m, ta thy u0 ; u1 ; v1 l nhng s


khng m v v0 0 nu 4q p2 v v0 0 nu p2 4q:
Nh vy, ta thu c mt kt qu ht sc c bit sau khi chng minh mt bt ng
thc. Khi a bt ng thc v dng pqr c dng f (r) 0 th
1) Nu f (r) l hm ng bin, khi ta ch cn chng minh
f (u20 v0 )

tc l ta ch cn xt n trong trng hp c 2 bin bng nhau l .


Nu bt ng thc yu cu chng minh vi cc s khng m th ta ch cn chng
minh
f max 0; u20 v0
0
tc l ta ch cn chng minh n ng trong trng hp c 2 bin bng nhau v trong
trng hp p2 4q th f (0) 0:
2) Nu f (r) l hm nghch bin, khi ta ch cn chng minh
f (u21 v1 )

tc l ta ch cn xt n trong trng hp c 2 bin bng nhau l .


3) Nu f (r) l hm lm (f 00 (x) 0), khi ta ch cn chng minh
min f (u21 v1 ); f (u20 v0 )

tc l ta ch cn xt n trong trng hp c 2 bin bng nhau l .


Nu bt ng thc yu cu chng minh vi cc s khng m th ta ch cn chng
minh
min f (u21 v1 ); f max 0; u20 v0
0
tc l ta ch cn chng minh n ng trong trng hp c 2 bin bng nhau v trong
trng hp p2 4q th f (0) 0:

30

CHNG 1. TM TI MT S K THUT GII TON

V d 1.21 Cho cc s dng a; b; c: Chng minh rng


r
b2
c2
3 5 a5 + b5 + c5
a2
+
+
:
b+c c+a a+b
2
3
(Michael Rozenberg)
Li gii. S dng bt ng thc Cauchy Schwarz, ta c
X a2
b+c
cyc

cyc

"

a2
+ b + c 2a
b+c
#2

X (b + c

a)2

b+c

cyc

a)2

(b + c

cyc

a)2

(b + c)(b + c

cyc

Chun ha cho a + b + c = 1 v t q = ab + bc + ca; r = abc: Ta c


X
X
X
X
X
(b + c a)2 =
(1 2a)2 = 3
4 a + 4 a2 = 3
cyc

cyc

(b + c)(b + c

a)2

cyc

cyc

(1

cyc

2a)2 = 3

a)(1

cyc

=
X

a5

cyc

a3

cyc

X
a + 8 a2

cyc

2 + 8(1
X

a2

cyc

2q)

4(1

cyc

= (1 3q + 3r)(1 2q) (q 2
= 5(1 q)r + 1 5q + 5q 2

cyc

3q + 3r) = 2(1

8q

cyc

a2 b2

cyc

2r) + qr

2q

X
4 a3
cyc

6r)

X
+ abc ab
cyc

Ta cn chng minh
(3 8q)2
1 2q 6r
, f (r) = 81(1

2q

6r)5 [(5

5(1

q)r + 1
3

5q + 5q 2

5q + 5q 2 ]

5q)r + 1

(3

8q)10

Ta c
f 0 (r) = 405(1
Nu 1

2q

6r)4 [36(q

1)r + (1

4q)(7q

5)]

4q th ta c
36(q

1)r + (1

4q)(7q

5)

(1

4q)(7q

5)

1.3. K THUT P QR
Nu 4q
36(q

31
(4q 1)(1 q)
;
6

1 th theo bt ng thc Schur bc 4, r


1)r + (1

Vy nn f 0 (r)
thc

4q)(7q

5)

6(q 1)(4q
= (1 3q)(2q

ta c

1)(1 q) + (1 4q)(7q
1)(4q 1) 0

5)

0; tc f (r) nghch bin, t ta suy ra chng minh bt ng


"

(b + c

a)

cyc

(b + c)(b + c

#2
a)2

cyc

3
2

r
5

a5 + b5 + c5
3

ta ch cn xt n trong 2 trng hp sau l : b = 0; c = 1 hoc b = c = 1:


Trng hp 1. b = c = 1; khi bt ng thc tr thnh
r
5
(4 4a + 3a2 )2
5 a + 2
3
2
3
4 4a + 2a + a
3
, g(a) =

(4 4a + 3a2 )10
(a5 + 2)(4 4a + 2a2 + a3 )5

Ta c
g 0 (a) =

10(a

81

1)3 (7a5 + 5a4 + 16)(4 4a + 3a2 )9


(a5 + 2)2 (4 4a + 2a2 + a3 )6
g 0 (a) = 0 , a = 1

T y, ta d dng kim tra c


g(a)

g(1) = 81

Trng hp 2. b = 0; c = 1; khi bt ng thc tr thnh


r
5
(3a2 2a + 3)2
5 a + 1
3
2
(a + 1)(a + 1)
3
, h(a) =
Ta c
h0 (a) =

(a2

(3a2 2a + 3)10
+ 1)5 (a + 1)5 (a5 + 1)

81

5(a 1)k(a)(3a2 2a + 3)9


(a2 + 1)6 (a + 1)5 (a5 + 1)2

vi
k(a) = 7a6

4a5 + 7a4

12a3 + 7a2

4a + 7

32

CHNG 1. TM TI MT S K THUT GII TON

Ta s chng minh
k(a) > 0 , 7 a3 +
t t = a +

1
a

1
a3

4 a2 +

1
a2

+7 a+

1
a

12 > 0

2 th bt ng thc tr thnh
7(t3

3t)

4(t2

2) + 7t

12 > 0

, 7t3 > 4t2 + 14t + 4


14 4
4
, 3 + 2 + <7
t
t
t
Ta c

4
14 4
+ 2 +
t3
t
t

4
14 4
+ 2 + =6<7
23
2
2

T y, ta c
h0 (a) = 0 , a = 1
V t
h(a)

h(1) = 512 > 81:

Bt ng thc c chng minh xong. ng thc xy ra khi v ch khi a = b = c:


V d 1.22 Cho tam gic ABC: Chng minh rng
p
la lb + lb lc + lc la 3 3S:
(Walther Janous)
Li gii. y l mt bi ton rt kh trn tp ch Crux Mathematicorum, hin nay
theo chng ti c bit vn cha c c mt li gii no cho n. Li gii sau (kh
phc tp), chng ti tm c sau mt thi gian di c gng tm li gii cho n
t x = p a; y = p b; z = p c; ta c
p
p
2 bcp(p a)
2 x(x + y + z)(x + y)(x + z)
la =
=
b+c
2x + y + z
p
p
S = p(p a)(p b)(p c) = xyz(x + y + z)
Do , bt ng thc tng ng
!
p
P
xy
x (x + y) xy(z + x)(z + y)
X
cyc
4
(2x + y + z)(2y + z + x)
cyc

s
X
3 3xyz
x
cyc

1.3. K THUT P QR

,4

33
r

(2x + y + z)

cyc

p Q
3 3 (2x + y + z)
cyc
v
!
!
u
u P
Q
t
x
(x + y)

y+z
x

cyc

,8

X
Xp
x(y + z) + 4
cyc

cyc

cyc

p Q
3 3 (2x + y + z)
cyc
v
!
!
u
u P
Q
t
x
(x + y)

(y + z)3
x

cyc

S dng bt ng thc Holder, ta c


"

X
cyc

(y + z)3
x

X
cyc

#2 "

x(y + z)

cyc

(y + z)3
x

cyc

"

(y + z)

cyc

#3

v
u"
#3
u P
u
2
(y + z)
u
u cyc
u P
t
x(y + z)3
cyc

Li c
Xp
x(y + z)

cyc

v"
#2 s
u
u Xp
X
Xp
t
=
x(y + z) = 2
xy(z + x)(z + y)
xy + 2
cyc

cyc

cyc

s X
s X
Xp
X p
p
xy + 2
xy + 2
z xy
2
xy (z + xy) = 4
cyc

cyc

s X
p
xy + 6 3 x2 y 2 z 2
4
cyc

cyc

v
u X
18xyz
u4
xy + P
t
x
cyc

cyc

cyc

Nn ta ch cn chng minh
v
u"
#3
u P
u
2
(y + z)
v
u
u X
u cyc
18xyz
u
u
8t4
xy + P + 4t P
x
x(y + z)3
cyc
cyc

cyc

p Q
3 3 (2x + y + z)
cyc
v
!
!
u
u P
Q
t
x
(x + y)
cyc

cyc

34

CHNG 1. TM TI MT S K THUT GII TON

Chun ha cho x + y + z = 1; t xy + yz + zx = q; r = xyz; bt ng thc tr thnh


p
p
p
(9r + 2q)(q r)
(1 q) (1 q)(q r)
3 6
p
+
f (r) =
r+q+2
16
(r + q + 2) 5r + q 2q 2
Ta c

3q 2 + 14q (36 + 25q)r


p
f (r) =
2(r + q + 2)2 (9r + 2q)(q r)
0

Ta chng minh
p
2 (1
0
f (r) 0 ,

p
(1

q)3 [6q + 2q 2 3q 3 + (q 2 + 7q)r 5r2 ]


p
(r + q + 2)2 (q r)(5r + q 2q 2 )3

q)3 [6q + 2q 2 3q 3 + (q 2 + 7q)r


p
(5r + q 2q 2 )3

5r2 ]

3q 2 + 14q (36 + 25q)r


p
(9r + 2q)

Ta d dng chng minh

5r + q

2q 2

1
2
>p
9r + 2q
9r + 2q

do ta ch cn chng minh
2(1

q)[6q + 2q 2 3q 3 + (q 2 + 7q)r
5r + q 2q 2

, g(r) = 5(34 + 25q)r2

5r2 ]

3q 2 + 14q

10q(2 + 6q + 5q 2 )r + 12q

(36 + 25q)r

10q 2 + 19q 3 + 6q 4

Ta c
0
g

5q(408

60q + 276q 2 + 399q 3

150q 4

125q 5 )

nn hin nhin g(r) 0: Do f 0 (r) 0 nn f (r) nghch bin, nh vy ta ch cn


xt bt ng thc trong trng hp c 2 bin bng nhau l . Cho y = z = 1; bt
ng thc tr thnh
s
r
p
3
18x
[2(x + 1)2 + 4]
3 3(2x + 2)(x + 3)2
p
+4
8 4(2x + 1) +
x+2
2(x + 1)3 + 8x
2(x + 2)(x + 1)2
r

r
p
2(4x2 + 19x + 4)
(x2 + 2x + 3)3
3 6(x + 3)2
p
,8
+8
x+2
x3 + 3x2 + 7x + 1
x+2
s
p
p
2(4x2 + 19x + 4)
(x2 + 2x + 3)3 (x + 2)
3 6
, h(x) =
+
(x + 3)2
(x + 3)4 (x3 + 3x2 + 7x + 1)
8

1.3. K THUT P QR

35

Ta c
h0 (x)

=
=

p
(x2 + 2x + 3)(x2 1)(7x3 + 37x2 + 103x + 105)
(x 1)(8x + 41)
p
p
+
(x + 3)3 8x2 + 38x + 8
2(x + 3)3 (x + 2)(x3 + 3x2 + 7x + 1)3
"p
#
(x2 + 2x + 3)(x + 1)(7x3 + 37x2 + 103x + 105)
2(8x + 41)
x 1
p
p
2(x + 3)3
8x2 + 38x + 8
(x + 2)(x3 + 3x2 + 7x + 1)3

Ta chng minh
p
(x2 + 2x + 3)(x + 1)(7x3 + 37x2 + 103x + 105)
p
(x + 2)(x3 + 3x2 + 7x + 1)3

2(8x + 41)
8x2 + 38x + 8

D thy

p
(x2 + 2x + 3)

(x + 2)(x3 + 3x2 + 7x + 1)

p
8
2 2
p
> p
2
2
8x + 38x + 8
3 8x + 38x + 8

Nn ta ch cn chng minh
4(x + 1)(7x3 + 37x2 + 103x + 105)
3(x3 + 3x2 + 7x + 1)
, 4x4

19x3 + 23x2

53x + 297

8x + 41
0:

Bt ng thc cui hin nhin ng, do


h0 (x) = 0 , x = 1: T y bng cch lp
p
bng bin thin, ta thy h(x) h(1) = 3 8 6 : Vy ta c pcm. ng thc xy ra khi
v ch khi x = y = z; tc ABC u.
Khai thc thm na (vi a; b; c khng m), ta c
p
27r
p(9q 2p2 ) + 2(p2 3q) p2 3q
p
2(p2 3q) p2 32 q p p2 3q
2
= p(9q 2p ) +
p p2 32 q
h
i
2
(p2 3q) p2 32 q + p2 (p2 3q)
p(9q 2p2 ) +
p p2 32 q
=

27q 2 (p2 q)
2p(2p2 3q)

V ta thu c
r

q 2 (p2
2p(2p2

q)
3q)

36

CHNG 1. TM TI MT S K THUT GII TON

C th thy c bt ng thc ny cht hn bt ng thc bit sau (m ta vn


hay dng)
q2
r
3p
Tng t, ta cng c
27r

p(9q

2p2 )

2(p2

p
3q) p2

3q

p
3q)(p
2q)p p2 3q
= p(9q 2p )
p(p2 2q)
2(p2 3q) (p2 2q)2 + p2 (p2
p(9q 2p2 )
p(p2 2q)
2
4
2
2
(4q p )(4p
10p q + 3q )
=
p(p2 2q)
2

2(p

3q)

V ta cng thu c
r

max 0;

(4q

p2 )(4p4
p(p2

10p2 q + 3q 2 )
2q)

C th thy bt ng thc ny cht hn 2 bt ng thc Schur bc 3 v bc 4.


V d 1.23 Cho cc s dng a; b; c tha mn a + b + c = 1: Chng minh rng
p
p
p
a(a + bc)
b(b + ca)
c(c + ab)
+
+
b + ca
c + ab
a + bc

1
p
:
2 abc

Li gii. S dng bt ng thc Cauchy Schwarz, ta c


"

X
cyc

a(a + bc)
b + ca

#2

"

#2
p
a(a + b)(a + c)
=
(b + c)(b + a)
cyc
"
#2
p
X
a(a + c)
p
=
(b + c) a + b
cyc
"
#
!
X
Xa+c
a
(a + b)(b + c)
b+c
cyc
cyc
P 2 P
!
a +
ab
Xa+c
cyc
cyc
=
(a + b)(b + c)(c + a) cyc b + c
X

1.3. K THUT P QR

37

Li c
Xa+c
cyc

b+c

X1

X b
b X 1
=
b+c
b + c cyc b + c
cyc
cyc
!2
P
a
X 1
cyc
P 2 P
b+c
a +
ab
cyc
cyc

Nn ta ch cn chng minh c
P

6
6X 1
6
6
(a + b)(b + c)(c + a) 6 cyc b + c
4
a2 +

cyc

ab

cyc

1
q

q
r

1+q
q r

cyc

!2

7
7
P 2 P 7
7
a +
ab 7
5
cyc
cyc
a

cyc

1
1

1
4abc

1
4r

4(1 q 2 )
q r
,
4
q r
r
4(1 q 2 ) q
,
3
q r
r
Ta c
4(1
q

q2 )
r

q
r

4(1
q

q2 )
q 2 (1 q)
2(2 3q)
2

q
q 2 (1 q)
2(2 3q)

8(1 q )(2 3q) 2(2 3q)


q(4 7q + q 2 )
q(1 q)
2(2 3q)(3 5q + 4q 2 )
=
(1 q)(4 7q + q 2 )
q(1 3q)(5 7q)
= 3
3:
(1 q)(4 7q + q 2 )

Bt ng thc c chng minh xong. ng thc xy ra khi v ch khi a = b = c = 31 :


V d 1.24 Cho cc s dng a; b; c: Chng minh rng
r
r
r
2
2
2
2
2
2
a+b+c
3 b + c
3 c + a
3 a + b
p
+
+
:
3
2
2
2
a + bc
b + ca
c + ab
abc
(Phm Hu c)

38

CHNG 1. TM TI MT S K THUT GII TON

Li gii. S dng bt ng thc Holder, ta c


X
cyc

r
3

b2 + c2
a2 + bc

!3

"

(b2 + c2 )

cyc

cyc

1
a2 + bc

=6

cyc

Ta cn chng minh

cyc

X
cyc

!3

cyc

1
2
a + bc

a2

!3

cyc

a2

X
cyc

cyc

+6

X
cyc

cyc

a2

!3

cyc

abc

abc
+ bc

a2

a3
+ bc

cyc

S dng bt ng thc Cauchy Schwarz, ta c

X
cyc

a3
a2 + bc

a2

cyc

!2

a(a2 + bc)

cyc

a2

cyc

= P

!2

a3 + 3abc

cyc

Vy nn ta ch cn chng minh c
P

!3

cyc

a2

cyc

6
+P

cyc

a3

!2

+ 3abc

p2

2q

6(p2 2q)2
p3 3pq + 6r

6p

Chun ha cho p = 1; khi bt ng thc tr thnh


1
1

2q

6(1 2q)2
1 3q + 6r

cyc

cyc

p3

a2

X
cyc

1
a2 + bc

1.3. K THUT P QR

39

Ta c
1
1

2q

6(1 2q)2
1 3q + 6r

1
1
=
=

2q

2q)2

6(1
1

3q +

3q 2 (1 q)
2 3q
2

1
6(1 2q) (2 3q)
+
1 2q 2 9q + 12q 2 3q 3
14 99q + 264q 2 315q 3 + 144q 4
(1 2q)(2 9q + 12q 2 3q 3 )

Li c
14

99q + 264q 2

315q 3 + 144q 4
, (2

6(1

9q + 12q 2 )(1

9q + 12q 2

2q)(2
3q)2

3q 3 )

0:

Bt ng thc cui hin nhin ng. Do ta c pcm. ng thc xy ra khi v ch


khi a = b = c:
V d 1.25 Cho cc s dng a; b; c tha mn a + b + c = 1: Chng minh rng
1 1 1
+ + + 48(ab + bc + ca)
a b
c

25:

Li gii 1. Bt ng thc tng ng vi


q
+ 48q
r

25

y l mt hm nghch bin theo r nn ta ch cn chng minh bt ng thc trong


1
trng hp c 2 bin bng nhau l , gi s a = b ) a
2a; khi bt
2; c = 1
ng thc tr thnh
2 1
+ + 48(a2 + 2ac) 25
a c
,

2
1
+
+ 48[a2 + 2a(1
a 1 2a
,

2(1

3a)2 (1 4a)2
a(1 2a)

2a)]

25

0:

Hin nhin ng. Vy ta c pcm. ng thc xy ra khi v ch khi a = b = c =


hoc a = 12 ; b = c = 14 hoc cc hon v tng ng.

1
3

40

CHNG 1. TM TI MT S K THUT GII TON

Li gii 2. Ta c
27r

9q
=

p
3q) 1

2 + 2(1

9q

2+

2(1

3q) 1
1
h
3q) 1

(1
9q
=

3q
12
5 q
12
5 q

2+

2
12
5 q
12
5 q

+1

3q
3q

27q 2 (7 16q)
5(5 12q)

)r

q 2 (7
5(5

16q)
12q)

Do
q
+ 48q
r

25

5(5
q(7

12q)
+ 48q
16q)

25 =

(1

3q)(5 16q)2
q(7 16q)

0:

Bt ng thc c chng minh.


V d 1.26 Cho cc s khng m a; b; c: Chng minh rng
s

(a2 + b2 )(a2 + c2 )
+
(a + b)(a + c)

(b2 + c2 )(b2 + a2 )
+
(b + c)(b + a)

(c2 + a2 )(c2 + b2 )
(c + a)(c + b)

a + b + c:

(V Quc B Cn)
Li gii. t a = x2 ; b = y 2 ; c = z 2 (x; y; z
c

0): S dng bt ng thc Holder, ta

(x4 + y 4 )(x + y)2

)VT =

X
cyc

x4 + y 4
x2 + y 2

(x2 + y 2 )3

(x2 + y 2 )2
(x + y)2

(x4 + y 4 )(x4 + z 4 )
(x2 + y 2 )(x2 + z 2 )

X (x2 + y 2 )(x2 + z 2 )
cyc

(x + y)(x + z)

1.3. K THUT P QR

41

Do
X (x2 + y 2 )(x2 + z 2 )

VT

(x + y)(x + z)

cyc

(x + y)(x + z)

cyc

X [(x + y)2

cyc

X xy(z + x)

(x + y)(x + z)

cyc

X zx(x + y)
+

x+y

"

z+x

cyc

X xy(x + y + 2z)
cyc

x+y

cyc

2xy][(x + z)2
(x + y)(x + z)

2xz]
X

4x2 yz
(x + y)(x + z)
cyc

4x2 yz
(x + y)(x + z)
cyc

x
=
(x + y)(x + z) 2
xy + 4xyz
(x
+
y)(x
+ z)
cyc
cyc
cyc
!
P 2 P
4xyz
x + xy
X
X
cyc
cyc
=
x2 +
xy
(x
+
y)(y
+
z)(z
+ x)
cyc
cyc
Ta cn chng minh
X

4xyz
xy

cyc

x2 +

cyc

xy

cyc

(x + y)(y + z)(z + x)

1
x
+
y
cyc

Chun ha cho p = 1; bt ng thc tr thnh


q(q

r)

, q2

4r(1
(4

Ta c
q2

(4

3q)r

q2

(4

3q)

q)

3q)r
q 2 (1 q)
q 3 (1
=
2(2 3q)
2(2

3q)
3q)

0:

Bt ng thc c chng minh xong. ng thc xy ra khi v ch khi a = b = c


hoc a > 0; b = c ! 0 hoc cc hon v tng ng.
V d 1.27 Cho cc s dng a; b; c tha mn a + b + c = 3: Chng minh rng
8

1 1 1
+ +
a b
c

+9

10(a2 + b2 + c2 ):
(Vasile Cirtoaje)

Li gii. Bt ng thc tng ng vi


8q
+ 20q
r

81

42

CHNG 1. TM TI MT S K THUT GII TON

y l mt hm nghch bin theo r nn ta ch cn xt bt ng thc trong trng


hp c 2 bin bng nhau l . Gi s a = b ) a 32 v c = 3 2a; khi bt ng
thc tr thnh
16
8
+
+ 9 20a2 + 10(3 2a)2
a
3 2a
3(2a

1)2 (10a2 25a + 16)


a(3 2a)

0:

Hin nhin ng. ng thc xy ra khi v ch khi a = b = 21 ; c = 2 hoc cc hon v


tng ng.

1.3.5

Lm mnh hn na

i vi cc bi ton thng thng, chng ta c th lm theo cch trn gii, nhng


i vi nhng bi ton c cha cn thc, ly tha tng qut,... R rng cc cch trn
l bt kh thi. Do , chng ta cn lm mnh k thut ca chng ta hn na lm
sao n c th gii quyt c cc dng ton . Chng ta xut pht t b sau y
B 1.1 Cho cc s khng m a; b; c tha a b c; khng c 2 s no ng thi
bng 0; ta c nh a + b + c = p; abc = r: Khi tn ti 2 s khng m a0 a1 sao
cho a 2 [a0 ; a1 ]. Ngoi ra, nu a = a0 th b = c v nu a = a1 th a = c.
Chng minh. Theo bt ng thc AM-GM, p3
27r. T gi thit, ta c b + c =
p a; bc = ar nn theo nh l Viet, b; c l cc nghim ca phng trnh f (x) =
x2 (p a)x + ar = 0. Do c b a nn ta phi c
8
< f 0
f (a) 0
: b+c
p a
2 = 2

8
<(p
, a2
:p
3

Ta c g 0 (a) = (p

a)(p

4r
a)2
a
(p a)a +
a 0

8
<g(a) = a(p
, h(a) = 2a3
:p
a 0
3

3a)

g(0) =

r
a

a)2 4r
pa2 + r

0
0

0 nn g ng bin. Mt khc,

4r

0;

p
4(p3 27r)
=
3
27

1.3. K THUT P QR

43

nn tn ti duy nht a0 2 [0; p3 ] sao cho g(a0 ) = 0, nhng do g(a)


Tng t, h0 (a) = 2p(3a p) 0 nn h nghch bin. Mt khc
h(a0 ) = 2a30

pa20 + r = 2a30

pa20 +

3a0 )2

a0 (p

0;

0 nn a

27r p3
p
=
3
27

a0 .

nn tn ti duy nht a1 2 [a0 ; p3 ] sao cho h(a1 ) = 0, nhng h(a) 0 nn a a1 . Mt


khc, nu a = a0 th b = c v nu a = a1 th a = c. B c chng minh.
nh l 1.2 Cho cc s khng m a; b; c; khng c 2 s no ng thi bng 0; ta c
nh a + b + c = p; abc = r: Khi vi mi hm f kh vi trn [0; +1) tha mn
k(x) = f 0 x1 l hm li th
P (a; b; c) = f (a) + f (b) + f (c)
t gi tr ln nht v gi tr nh nht (nu c) khi c 2 bin bng nhau.
Chng minh. Khng mt tnh tng qut, gi s a b
c
8
p
p a
(p a)2 4r
<
a
b = b(a) =
p 2 2 4r
p
a+
(p
a)
:c = c(a) =
a

c, t b trn, ta suy ra

T , ta c

1
0

b (a)

=
=

c (a)

=
=

a p+ a2r2
(p a)2

4r
a

2
2b
2(c

2bc
a

b)

1+ p

b(a
a(c

(p a)2

2c
c(b
=
b)
a(c

2(c

(b+c)+ 2bc
a
p a 2b

(b+c)+ 2bc
a
c b

c)
b)

a p+ a2r2

2
2bc
a

4r
a

1+

(b+c)+ 2bc
a
p a 2b

1+

(b+c)+ 2bc
a
c b

a)
b)

Suy ra
0

Pa0 (a; b; c) = (f (a) + f (b(a)) + f (c(a)))


= f 0 (a) + b0 (a) f 0 (b(a)) + c0 (a) f 0 (c(a))
b(a c) 0
c(b a) 0
f (b) +
f (c)
a(c b)
a(c b)
b)f 0 (a) + b(a c)f 0 (b) + c(b a)f 0 (c)
a(c b)
0
0
b)(f (a) f (b)) + c(b a)(f 0 (c) f 0 (b))
a(c b)

= f 0 (a) +
=
=

a(c
a(c

44

CHNG 1. TM TI MT S K THUT GII TON

Do k(x) = f 0

1
x

l hm li nn
1
a

f0

f0
1
a

, f0

, f0

f0
1
c

k0

1
b
1
a

1
b

1
c

1
b

+ k0
1
b

1
a
1
b

k0

1
b

f0

1
b

+ k0
1
b

f0

1
c

f0

1
b

1
b

1
b
1
c

1
b

Do
a(c

b)[f 0 (a)

f 0 (b)] + c(b

a)[f 0 (c)
b)k 0

a(c
= k0
Vy nn Pa0 (a; b; c)
dng suy ra pcm.

1
b

f 0 (b)]
1
a

1
b

a(c

b)

1
b
1
a

+ c(b
1
b

+ c(b

a)k 0
a)

1
b
1
c

1
c
1
b

1
b
=0

0; tc P (a; b; c) = Pa (a; b; c) l hm ng bin, v t y, ta d

H qu 1.3 Cho cc s khng m a; b; c; khng c 2 s no ng thi bng 0; ta c


nh a + b + c = p; abc = r: Khi vi mi hm f kh vi trn [0; +1) tha mn
k(x) = f 0 x1 l hm lm th
P (a; b; c) = f (a) + f (b) + f (c)
t gi tr ln nht v gi tr nh nht (nu c) khi c 2 bin bng nhau.
B 1.2 Cho cc s khng m a; b; c tha a b c; khng c 2 s no ng thi
bng 0; ta c nh a + b + c = p; ab + bc + ca = q. Khi tn ti 2 s khng m a0 a1
sao cho a 2 [a0 ; a1 ]. Ngoi ra, nu a = a0 th b = c hoc a = 0, v nu a = a1 th
a = b.
Chng minh. Theo bt ng thc AM-GM, p2
3q. T gi thit trn, ta c
b + c = p a; bc = q a(p a) nn theo nh l Viet, b; c l cc nghim ca phng
trnh f (x) = x2 (p a)x + q a(p a) = 0. Do c b a nn
8
< f 0
f (a) 0
: b+c
p a
a
2 = 2

1.3. K THUT P QR

45

8
<(p
, a2
:p
3

a)2 q a(p a)
(p a)a + q a(p
a 0

a)

8
<g(a) = 3a2 + 2ap + p2 4q 0
, h(a) = 3a2 2pa + q 0
:p
a 0
3
p
p
8
p 2 p2 3q
p+2 p2 3q
>
a
<
3 p
3 p
p 2 p2 3q
p+2 p2 3q
,
_a
>
3
3
:ap
a
0
3
(
)
p
p
p 2 p2 3q
p + 2 p2
, max 0;
a
3
3
t a0 = max 0;

p 2

p2 3q
3

, a1 =

th a = b v nu a = a0 th khi p2
chng minh.

p+2

p2 3q
,
3

th ta c a0

4q th b = c, khi p2

3q

a1 v nu a = a1

4q th a = 0. B c

nh l 1.3 Cho cc s khng m a; b; c; khng c 2 s no ng thi bng 0; ta c


nh a + b + c = p; ab + bc + ca = q. Khi vi mi hm f kh vi trn [0; +1) tha
mn k(x) = f 0 (x) l hm li th
P (a; b; c) = f (a) + f (b) + f (c)
t gi tr ln nht (nu c) khi c 2 bin bng nhau v gi tr nh nht (nu c) khi
c 2 bin bng nhau hoc c mt bin bng 0.
Chng minh. Khng mt tnh tng qut, gi s a b c, khi , t b trn ta
c
8
p
p a
(p a)2 +4a(p a) 4q
<
b = b(a) =
2
p
:c = c(a) = p a+ (p a)2 +4a(p a) 4q
2

Do

1
0

b (a) =

c (a) =

p 3a
(p a)2 +4a(p a) 4q

2
1+ p

p 3a
(p a)2 +4a(p a) 4q

b+c 2a
p a 2b

2
1+

b+c 2a
p a 2b

b+c 2a
c b

2
1+

b+c 2a
c b

a
c

c
b

b
c

a
b

46

CHNG 1. TM TI MT S K THUT GII TON

Suy ra

Pa0 (a; b; c) = (f (a) + f (b(a)) + f (c(a)))


= f 0 (a) + b0 (a) f 0 (b(a)) + c0 (a) f 0 (c(a))
b a 0
a c 0
f (b) +
f (c)
= f 0 (a) +
c b
c b
0
0
(c b)f (a) + (a c)f (b) + (b a)f 0 (c)
=
c b
(c b)(f 0 (a) f 0 (b)) + (b a)(f 0 (c) f 0 (b))
=
c b
Do k(x) = f 0 (x) l hm li nn
f 0 (a)

f 0 (b)

k 0 (b)(a

b);

f 0 (c)

f 0 (b)

k 0 (b)(c

b)

V do
(c

b)(f 0 (a)

Vy nn Pa0 (a; b; c)
dng suy ra pcm.

f 0 (b)) + (b

a)(f 0 (c) f 0 (b))


k 0 (b)[(c b)(a b) + (b

a)(c

b)] = 0

0; tc P (a; b; c) = Pa (a; b; c) l hm ng bin v t y, ta d

H qu 1.4 Cho cc s khng m a; b; c; khng c 2 s no ng thi bng 0; ta c


nh a + b + c = p; ab + bc + ca = q. Khi vi mi hm f kh vi trn [0; +1) tha
mn k(x) = f 0 (x) l hm lm th
P (a; b; c) = f (a) + f (b) + f (c)
t gi tr ln nht (nu c) khi c 2 bin bng nhau hoc c mt bin bng 0 v gi
tr nh nht (nu c) khi c 2 bin bng nhau.
V d 1.28 Cho cc s khng m a; b; c; k tha mn khng c 2 s no ng thi
bng 0: Tm gi tr nh nht ca biu thc
P (a; b; c) = (ab + bc + ca)k

1
1
1
+
+
2k
2k
(a + b)
(b + c)
(c + a)2k

(Phm Sinh Tn)


Li gii. C nh a + b + c = p; ab + bc + ca = q, xt hm s k(x) =
c
k 00 (x) =

4k(k + 1)(2k + 1)
(p x)2k+3

Do , theo nh l trn, ta ch cn xt cc trng hp sau l

1
(p x)2k

. Ta

1.3. K THUT P QR

47

Trng hp 1. b = c; khi biu thc P c vit li l


P (a; b) = (2ab + b2 )k

1
2
+
(a + b)2k
(2b)2k

Chun ha cho b = 1, xt hm s
f (a) = (2a + 1)k
Ta c

2
1
+
2k
(a + 1)
(2)2k

k(2a + 1)k 1 [(a + 1)2k+1 22k+1 a]


22k 1 (a + 1)2k+1
2
a=1
0
2k+1
2k+1
4
a 6= 1
f (a) = 0 , (a + 1)
=2
a,
ap 1
g(a) = 2k+1
a
f 0 (a) =

2=0

T y, ta c th d dng chng minh c f 0 (x) = 0 c 2 nghim dng l 1 v


6= 1; t , bng cch lp bng bin thin, ta c th thy
f (a)

min ff (0); f (1); f ( )g = min

3k+1
1 (2 + 1)k+1
;
2
+
;
22k
22k
22k

Trng hp 2. a = 0; khi biu thc P c vit li l


P (a; b; c) = (bc)k

1
1
1
+ 2k + 2k
(b + c)2k
b
c

do tnh i xng v thun nht nn ta c th gi s 0


h(c) = (c)k
Ta c
h0 (c) =

1
1
+ 1 + 2k
(1 + c)2k
c

1) = c2k (c

2
c=1
,4 0 c<1
k(c) = (2k + 1) ln(c + 1) + ln(1
k 0 (c) =

2 kck+2

b = 1; xt hm s

k (c + 1)k+1 (c2k 1) c2k (c


(c + 1)2k ck+1
h0k+1 (c2k

1)

22k )

(k + 1)c2k+1 + (k + 1)c
c(1 c2 )(1 c2k )

1)

2k ln c
k

c(1

ln(1
2l(c)
c2 )(1

c) = 0

c2k )

48

CHNG 1. TM TI MT S K THUT GII TON


l0 (c) = (k + 1) 2kc2k+1

(2k + 1)c2k + 1

Suy ra l(c) ng bin, m


l(0) =

0;

0 8c 2 [0; 1) ) k 0 (c)

l(1) = 0 ) l(c)

0 8c 2 [0; 1)

Nn k(c) nghch bin vi mi c 2 [0; 1), suy ra k(c) c ti a 1 nghim thuc [0; 1).
Do h0 (c) c ti a 2 nghim l 1 (lun l nghim) v (ty trng hp m tn ti
hay khng). Bng cch lp bng bin thin, ta c th thy h(c) maxfh( ); h(1)g.
T y, ta d dng i n kt lun ca bi ton.
Nhn xt 3 Mt s kt qu c
? Vi k = 1, ta c bt ng thc Iran 1996
(ab + bc + ca)

1
1
1
+
+
(a + b)2
(b + c)2
(c + a)2

9
4

? Vi k = 12 ; ab + bc + ca = 1; ta c
1
1
1
+
+
(a + b) b + c c + a

5
2

? Vi k = 14 ; ab + bc + ca = 1; ta c
p

1
1
1
+p
+p
c+a
a+b
b+c

1
2+ p :
2

V d 1.29 Cho cc s khng m a; b; c; k tha mn a + b + c = 1. Tm gi tr ln


nht ca biu thc
P (a; b; c) = (ab)k + (bc)k + (ca)k :
(Vasile Cirtoaje)
Li gii. Ta c

1
1
1
+ k + k
k
a
b
c

P (a; b; c) = (abc)k

Nn nu ta c nh abc = r th P (a; b; c) s c dng nh l ca ta, xt hm s


f (x) = x1k ; ta c
f 0 (x) =

k
) k(x) = f 0
xk+1

1
x

kxk+1 ) k 002 (k + 1)xk

Do f (x) l hm s tha mn iu kin ca h qu nh l , nn P (a; b; c) s t


gi tr ln nht (nu c) khi c 2 bin bng nhau. Cho b = c; ta c
P (a; b; c) = 2ak bk + b2k =

2ak bk + b2k
2xk + 1
=
= g(x)
2k
(a + 2b)
(x + 1)2k

x=

a
b

1.3. K THUT P QR

49

Ta c
g 0 (x) =

2k(xk 2xk 1 + 1)
(x + 2)2k+1

D thy g 0 (x) c ti a 2 nghim l 1 (lun l nghim) v


bng cch lp bng bin thin, ta c
g(x)

max fg(0); g(1)g = max

1
32k 1

( 2 [0; 1]) nn t y
1
4k

Bi ton c gii quyt.


V d 1.30 Cho cc s khng m a; b; c tha mn a + b + c = 1: Chng minh rng
b2
c2
a2
+
+
5a + 1 5b + 1 5c + 1

1
p
:
8 3(ab + bc + ca)

Li gii. C nh ab + bc + ca = q, xt hm s f (x) =
k(x) = f 0 (x) =

x(5x + 2)
) k 00 (x) =
(5x + 1)2

x2
5x+1 ;

(Bi Vit Anh)

ta c

30
<0
(5x + 1)4

Do , ta ch cn xt bi ton trong 2 trng hp sau l


Trng hp 1. Nu b = c ) a = 1 2b v b 21 . Bt ng thc tng ng vi
2b2
(1 2b)2
+
5b + 1 2(3 5b)
,

(1

1
p
8 3b(2

3b)

3b)2 (256b4 128b3 71b2 + 18b + 9)


192b(2 3b)(3 5b)2 (5b + 1)2

Bt ng thc cui hin nhin ng do b 21 .


Trng hp 2. Nu a = 0 ) c = 1 b. Bt ng thc tng ng vi
b2
(1
+
5b + 1
6
,

(1

b)2
5b

1
p
8 3b(1

b)

2b)2 (1 + 4b 4b2 )(527 + 1278b 1278b2 ) + 49


3072b(2 3b)(6 5b)2 (5b + 1)2

Bt ng thc cui hin nhin ng do 0 < b


a = b = c = 31 .

1. ng thc xy ra khi v ch khi

50

CHNG 1. TM TI MT S K THUT GII TON

V d 1.31 Cho cc s khng m a; b; c; khng c 2 s no ng thi bng 0: Chng


minh rng vi k = ln 3ln 3ln 2 , ta c
a2
2
b + bc + c2

1=k

b2
2
c + ca + a2

1=k

1=k

c2
2
a + ab + b2

2:
(Phm Sinh Tn)

Li gii. S dng bt ng thc Holder, ta c


# "
#
"
1=k k X
X
a2
a(b2 + bc + c2 )
2 + bc + c2
b
cyc
cyc

Ta cn chng minh
!k+1
X 3
a k+1

2k+1

cyc

"

cyc

a(b2 + bc + c2 ) = 2k+1

cyc

3
k+1

cyc

!k+1

ab

cyc

C nh a + b + c = p; ab + bc + ca = q v xt hm s f (x) = x k+1 ; ta c
k(x) = f 0 (x) =

2 k
3(2k 1)(k 2)
3
x k+1 ) k 00 (x) =
x
k+1
(k + 1)3

3k
k+1

>0

Do , ta ch cn xt bi ton trong 2 trng hp sau


Trng hp 1. a = 0 , chun ha cho b c = 1, khi bt ng thc tng ng
vi
k+1
3
a k+1 + 1
2k b(b + 1)
k+1

, g(t) =
Ta c
g 0 (t) =

t3 + 1
tk+1 (tk+1 + 1)

(t =

(k + 1)(t3 + 1)k (tk+4 3tk+1


tk+2 (tk+1 + 1)2

g 0 (t) = 0 , tk+4
, t3

2k

1 + t3 t

k+1
2

3tk+1
+ 2tk+1 t

k+1

1)

2t3 + 1)

2t3 + 1 = 0
5

k
2

1 =0,t=1

k
T y, bng cch lp bng bin thin, ta d thy g(t) g(1)
p = 2 8t 1.
Trng hp 2. b = c; chun ha cho b = c = 1 v t x = k+1 a, khi bt ng thc
tr thnh
(x3 + 2)k+1
h(x) = k+1
2k
(x
+ 2)(2xk+1 + 1)

1.3. K THUT P QR

51

Ta c
h0 (x) =

2(k + 1)x2 (x3 + 2)k (x2k+2 5x2k 1 + 5xk+1


(xk+1 + 2)2 (2xk+1 + 1)2
x=0
x2k+2

h0 (x) = 0 ,
,

x=0
u(x) =

u0 (x) =

5x2k

+ 5xk+1

x2k+2 5x2k 1 +5xk+1 +3


5xk 2
2k+2

2k 1

(k + 4)x

(k + 1)x
5xk

u0 (x) = 0 , v(x) = (k + 4)x2k+2


0

v 0 (x)
(k + 1)xk

k+1

2(k + 4)xk+1
0
= 2xk0

> 2((k +
2
3

trong

< x0 =

2(k 2)(2k 1)
(k+1)(k+4)
0

1=0

+ 3)

+3=0

(x > 0)

+ 15x

3k + 6

1
1

+ 15xk+1
k 2

4(2k

1)x

4(2k

1)xk0

((k + 1)x30

1)x30

k+1

(k + 1)x2k

v (x) = (k + 1)(2(k + 4)x


D thy

5xk

5xk

2(2k

2(2k

3k + 6 = 0

+ 15)

+ 15

1)) + 15

1)) + 15 > 0

< 1.

Suy ra u (x) ng bin, li c u (0) = 3(2 k) < 0; u0 (1) = 3(7 2k) > 0 nn n
c duy nht mt nghim x1 2 (0; 1) do u(x) ng bin trn [x1 ; +1) v nghch
bin trn (0; x1 ]; do u(1) = 0 nn u(x1 ) < 0 m lim u(x) = +1 nn u(x) c ng 2
x!0

nghim l x2 2 (0; x1 ) v 1. T y, bng cch lp bng bin thin, ta c


h(x)

minfh(0); h(1)g = 2k :

Bt ng thc c chng minh. ng thc xy ra khi v ch khi a = b = c hoc


a = b; c = 0 hoc cc hon v tng ng.
V d 1.32 Cho cc s khng m a; b; c; khng c 2 s no ng thi bng 0: Chng
minh rng
k
k
k
a
b
c
3
+
+
min 2; k :
b+c
c+a
a+b
2
(Vasile Cirtoaje)
Li gii. Nu k

1; ta c
1 X ak
3 cyc (b + c)k

1X a
3 cyc b + c

!k

1
2k

52

CHNG 1. TM TI MT S K THUT GII TON


)

Nu k

X
cyc

ak
(b + c)k

3
2k

1; s dng bt ng thc Holder, ta c


#"
#k
"
X
X ak
a(b + c)
(b + c)k
cyc
cyc

X
cyc

ak
(b + c)k

2k
k+1

cyc

"

Ta cn chng minh
P

!k

!k+1

2k
k+1

cyc

#k =

2k

min 2;

3
2k

a(b + c)

!k+1

2k
k+1

cyc

!k+1

cyc

2k
k+1

cyc

2k

!k+1
!k

ab

cyc

ab

cyc

2k

C nh a + b + c = p; ab + bc + ca = q v xt hm s f (x) = x k+1 ; ta c
f 0 (x) =

2k 11+kk
2k kk+11
4k(1 k)
x
x
x
) k(x) =
) k 00 (x) =
k+1
k+1
(k + 1)3

k+3
k+1

>0

Do , ta ch cn xt bi ton trong 2 trng hp sau l


Trng hp 1. a = 0; ta cn chng minh
2k

2k

k+1

b k+1 + c k+1
2k bk ck

min 2;

3
2k

Bt ng thc ny hin nhin ng theo bt ng thc AM-GM.


1
Trng hp 2. b = c; chun ha cho b = c = 1 v t t = a k+1 ; ta cn chng minh
(t2k + 2)k+1
2k (2tk+1 + 1)k
, g(t) =
Ta c
g 0 (t) =

2kt2k

(t2k + 2)k+1
(2tk+1 + 1)k

min 2;

3
2k

min 2k+1 ; 3

(k + 1)(t2k + 2)k (tk+1


(2tk+1 + 1)k+1

2t1

+ 1)

1.3. K THUT P QR

53

g 0 (t) = 0 , tk+1

2t1

+ 1 = 0 , h(t) =

h0 (t) =

2ktk+1 + k
t2 k

tk+1 + 1
t1 k

2=0

T y d thy h0 (t) c ti a mt nghim thuc (0; 1], suy ra c ti a 2 nghim


thuc (0; 1], trong lun c mt nghim l 1. Bng cch cch lp bng bin thin,
d thy
g(t) min fg(0); g(1)g = min 2k+1 ; 3 :
Bi ton c gii quyt xong.
V d 1.33 Cho cc s khng m a; b; c tha mn a + b + c + abc = 1: Chng minh
rng
(2 + abc)(1 + 2abc)
ab + bc + ca
:
7 abc
Li gii. Ging nh cc bi trc, bi ny ta cng ch cn xt a = b l . Khi ,
ta c c = 11+a2a2 ) a 12 ; bt ng thc tr thnh
(2 + a2 c)(1 + 2a2 c)
7 a2 c

a2 + 2ac

,
, a(1

a(1

a)(2 + a)
a2 + 1

(2 + 3a2 2a3 )(1 a)(4a2 + a + 1)


(2a3 + 6a2 + 7)(a2 + 1)

a)(2 + a)(2a3 + 6a2 + 7)


, 2(a3 + 3a

(2 + 3a2
1)2

2a3 )(4a2 + a + 1)

0:

Vy ta c pcm.
V d 1.34 Cho cc s dng x; y; z tha mn xyz = 8: Chng minh rng
y2
z2
+p
+p
(x3 + 1)(y 3 + 1)
(y 3 + 1)(z 3 + 1)
(z 3 + 1)(x3 + 1)

x2

4
:
3

(APMO 2005)

q
p
p
Li gii. t x = 2 3 ab ; y = 2 3 ac ; z = 2 3 cb ; bt ng thc tr thnh
X
cyc

b1=6

a7=6

(8a + b)(8c + a)

1
3

54

CHNG 1. TM TI MT S K THUT GII TON

S dng bt ng thc Holder, ta c


"
#6 "
#3
X
X
a7=6
p
(8a + b)(8c + a)
1=6
(8a + b)(8c + a)
cyc b
cyc
Do ta ch cn chng minh
!10
X
6
4=5
3
a

cyc

!"

ab

cyc

ab

cyc

ab

cyc

4=5

cyc

!10

#3

(8a + b)(8c + a)

cyc

a2 + 73

cyc

!3

ab

cyc

Do
X
cyc

ab

a + 73

cyc

X
cyc

!3

ab

ab

cyc

9
!

ab

cyc

a + 72

cyc

X
cyc

a +8

cyc

X
cyc

!3

ab

!3

ab

nn ta ch cn chng minh
X

4=5

cyc

!10

ab

cyc

a +8

cyc

X
cyc

!3

ab

n y, s dng kt qu ca ta, d dng suy ra c ta ch cn xt cc trng hp


a = 0 hoc b = c.
Trng hp 1. a = 0; gi s b c = 1; bt ng thc tr thnh
(b4=5 + 1)10
, f (b) =
f 0 (b)

=
=

b(b2 + 8b + 1)3

(b4=5 + 1)10
b(b2 + 8b + 1)3

(b4=5 + 1)9 (b14=5

7b2 + 32b9=5 32b + 7b4=5


b2 (b2 + 8b + 1)4

(b4=5 + 1)9 (b2=5 1)m(b)


b2 (b2 + 8b + 1)4

1)

trong
m(b) = b12=5 + b2

6b8=5 + 32b7=5

6b6=5 + 32b

6b4=5 + b2=5 + 1 > 0

1.3. K THUT P QR

55
) f (b)

f (1) =

128
>1
125

Trng hp 2. b = c; gi s b = c = 1; bt ng thc tr thnh


(a4=5 + 2)10
, g(a) =
g 0 (a)

=
=

2(a4=5 + 2)9 (a3

(2a + 1)(a2 + 16a + 10)3

(a4=5 + 2)10
(2a + 1)(a2 + 16a + 10)3

14a11=5 + 65a2 134a6=5 + 110a


a1=5 (2a + 1)2 (a2 + 16a + 10)4

68a1=5 + 40)

2(a4=5 + 2)9 (a1=5 1)h(a1=5 )


a1=5 (2a + 1)2 (a2 + 16a + 10)4

Trong
h(x)

= x14 + x13 + x12 + x11 13x10 + 52x9 + 52x8 + 52x7 + 52x6


+28x3 + 28x2 + 28x 40

82x5 + 28x4

D thy h(x) ng bin, v h(0) h(1) < 0 nn tn ti duy nht nghim x0 2 (0; 1)
ca h(x), suy ra g 0 (a) c ng 2 nghim l 1 v x50 2 (0; 1). T y, bng cch lp
bng bin thin d thy
g(a)

min fg(0); g(1)g = min

128
;1
125

= 1:

Bt ng thc c chng minh xong. ng thc xy ra khi a = b = c , x = y =


z = 1:
Nhn xt 4 y l bi ton thi Olympic ton Chu -Thi Bnh Dng 2005 (APMO).
1
Cch gii p n rt hay v p mt nh s dng kt qu px13 +1
x2 +2 . Hin nay,
ngoi li gii p n v li gii ca chng ti ra cha c mt li gii no khc cho
bt ng thc ny.

1.3.6

pqr hon v

Vi cc kin thc b sung trn, ta gii c kh nhiu cc bi ton dng i xng.


Nhng cn cc dng hon v th sao? K thut ny liu c dng c cho n khng?
Cu tr li l c. iu kh khn ln nht khi gp phi cc dng ny l ta khng
bit lm sao biu din cc biu thc dng hon v sang pqr: C mt cch lm rt
th v chuyn cc dng ny sang pqr l dng tam thc bc 2. Chng ta c kt qu

56

CHNG 1. TM TI MT S K THUT GII TON

c bn sau (nh l o ca nh l Viet): Vi 2 s thc u; v tha u + v = S; uv = P


(S 2 4P ) th u; v l nghim ca phng trnh bc 2: X 2 SX + P = 0: Da trn
c s ny, ta c th d dng biu din cc biu thc hon v vng quanh cho 3 bin
a; b; c theo p; q; r. V sau khi biu din v dng ny, ta ch vic xt mt hm mt bin
theo r (hoc q) khi c nh p = const. Nh vy, c th ni bn cht ca k thut
ny chng qua ch l tam thc bc 2 v kho st hm s.
V d 1.35 Biu din a2 b + b2 c + c2 a; ab2 + bc2 + ca2 theo p; q; r:
x = a2 b + b2 c + c2 a
. Khi , ta c
y = ab2 + bc2 + ca2

Li gii. t

x+y =

ab(a + b) =

cyc

xy

a b

cyc

= q

a3 b3 +

cyc

ab

cyc
2

3pqr + 3r + r(p

ab

cyc

cyc

3abc = pq

3r

a4 bc + 3a2 b2 c2

cyc

3pq + 3r) + 3r2 = 9r2 + p(p2

6q)r + q 3

Vy nn x; y l cc nghim ca phng trnh X 2 (pq 3r)X+9r2 +p(p2 6q)r+q 3 = 0:


Gii phng trnh ny, ta c
8
p
pq 3r+ p2 q 2 4q 3 +2p(9q 2p2 )r 27r 2
<
X1 =
2
p
: X = pq 3r p2 q2 4q3 +2p(9q 2p2 )r 27r2
2

Cc nghim ny lun tn ti v ta lun c p2 q 2 4q 3 + 2p(9q p2 )r 27r2 0 (bi


x = X1 ; y = X2
v n bng (a b)2 (b c)2 (c a)2 !). Do
: Ty theo trng hp
x = X2 ; y = X1
m ta c th la chn p s, chng hn nh trong trng hp a b c th x y
nn ta phi c x = X1 ; y = X2 :
V d 1.36 Biu din a3 b + b3 c + c3 a; ab3 + bc3 + ca3 theo p; q; r:
Li gii. t

x+y =

X
cyc

x = a3 b + b3 c + c3 a
: Khi , ta c
y = ab3 + bc3 + ca3
2

ab(a + b ) =

X
cyc

X
cyc

ab

abc

X
cyc

a = (p2

2q)q

pr

1.3. K THUT P QR

xy

a b

cyc

X
cyc

+abc
=

a2 b2
"

57

!2

ab

cyc

7p2 r2 + p(p4 + q 2

a4 b4 + abc

cyc

2a2 b2 c2

cyc

X
2

a2 + a2 b2 c2

cyc

a5 + a2 b2 c2

cyc

cyc

X
cyc

ab

2 2

a b

cyc

5p2 q)r + q 4

ab

cyc

cyc

+ abc

ab

cyc

Do x; y l cc nghim ca phng trnh


X2

[(p2

2q)q

pr]X + 7p2 r2 + p(p4 + q 2

Gii phng trnh ny, ta c


8
p2 q 2q 2
<
X1 =
: X = p2 q 2q2
2

Do , ta c

pr+p

5p2 q)r + q 4 = 0

p2 q 2 4q 3 +2p(9q 2p2 )r 27r 2


2
p2 q 2 4q 3 +2p(9q 2p2 )r 27r 2
2

pr p

x = X1 ; y = X2
:
x = X2 ; y = X1

V d 1.37 Biu din a4 b + b4 c + c4 a; ab4 + bc4 + ca4 theo p; q; r:


Li gii. Thc hin tng t nh trn, ta d dng tm c
8
p
(5q p2 )r+pq(p2 3q) (p2 q) p2 q 2 4q 3 +2p(9q
< 4
a b + b4 c + c4 a =
2
p
: ab4 + bc4 + ca4 = (5q p2 )r+pq(p2 3q) (p2 q) p2 q2 4q3 +2p(9q

2p2 )r 27r 2
2p2 )r 27r 2

V d 1.38 Biu din a3 b2 + b3 c2 + c3 a2 ; a2 b3 + b2 c3 + c2 a3 theo p; q; r:


Li gii. Thc hin tng t nh trn, ta d dng tm c
8
p
pq 2 (2p2 +q)r q p2 q 2 4q 3 +2p(9q
< 3 2
a b + b3 c2 + c3 a2 =
2
p
: a2 b3 + b2 c3 + c2 a3 = pq2 (2p2 +q)r q p2 q2 4q3 +2p(9q

2p2 )r 27r 2
2p2 )r 27r 2

V d 1.39 Cho cc s khng m a; b; c tha mn a + b + c = 1: Tm gi tr ln nht


ca biu thc
P = a2 b + b2 c + c2 a + abc:
(Vasile Cirtoaje)

58

CHNG 1. TM TI MT S K THUT GII TON

Li gii. Gi s a
2

c; ta c

3r +

= a b + b c + c a + abc =
p
q r + q 2 4q 3 + 2(9q
=
2

Ta c
0

f (r) =

9q

q2

f (r) = 0 ) r = r0 =
2)

p
3q) 7(1

(1

7(9q

2)r

4q 3 + 2(9q

3q); ta c f 0 (r)

+r

27r2

2)r
27r2

2)r

p
(1 3q) 7(1
189

2)

27r2

= f (r)

4q 3 + 2(9q

Nu 7(9q

4q 3 + 2(9q
2

27r2

2)r

27r
p
2 q2

p
q2

3q)

0; suy ra

2
p
1 x2
p
+ x(1 4 x )
q + q 1 4q
(1 + x)(1 x2 )
4
f (r)
f (0) =
=
=
x= 1
2
2
8
(3x 1)2 (3x + 5)
4
4
=
+
216
27
27
p
Nu 7(9q 2) (1 3q) 7(1 3q); bng cch lp bng bin thin, ta c

f (r)

f (r0 ) =
=

r0 +

r0 + (9q 2
h2
2 7(9q

=
=
Do 7(9q

5q
3(1

2)

p
q2

4q 3 + 2(9q
2

27r0 )
2)

= 5q

(1

14r0
p
3q) 7(1

27
p
2 7t3

2)r0

27r02

1
i
3q)

9q + 1 + 2(1

p
p
t2 ) + 1 + 2 7t3
3t2 + 4
t= 1
=
27
27
p
p
7(1 3t2 ) ) t
(1 3q) 7(1 3q) ) t3
p
2 7t3

Tm li, ta c max P =

4
27

4q

p
3q) 7(1
27

3q
3
p
;
2 7

p
t2 2 7t 3 + 4
3t2 + 4
4
=
27
27
27
2
a = b = c = 31
6 a = 2; b = 1; c = 0
3
3
t c khi 6
4 a = 1 ; b = 0; c = 2 :
3
3
a = 0; b = 32 ; c = 13

do

3q)

1.3. K THUT P QR

59

Nhn xt 5 Chng ta c mt vi im cn ch
7(9q 2) (1 3q)
189

Tht ra, khi gii phng trnh f 0 (r) = 0 ta c n 2 nghim l


nhng cc nghim phi tha mn iu kin l r 0 v 9q 2 p27r
7(9q 2) (1 3q) 7(1
so li vi h iu kin ny th ch c nghim r0 =
189
p
7(9q 2) (1 3q) 7(1 3q)

7(1 3q)

0: Nhng khi
3q)

tha khi

Do ta phi xt 2 trng hp nh li giiptrn.


7(9q 2) (1 3q) 7(1 3q)
Trong trng hp nghim r0 =
tha th chc hn cc bn cng
189
rt ngi khi thay vo biu thc ban u, bi l ton l cn thc (cn trong cn),
tnh
p ton rt phc tp. Nhng chng ta c mt mo nh y l 9q 2 27r0 =
2
4q 3 + 2(9q 2)r0 27r02 , do khi thay r0 vo biu thc f (r); ta hy thay
pq
q 2 4q 3 + 2(9q 2)r0 27r02 bi 9q 2 27r0 ri hy thay trc tip gi tr ca r0
vo, tnh ton s tr nn n gin rt nhiu!
V d 1.40 Cho cc s thc a; b; c: Tm gi tr ln nht ca biu thc
P =

ab(a2

b2 ) + bc(b2 c2 ) + ca(c2
(a2 + b2 + c2 )2

a2 )

:
(IMO 2006)

Li gii. Chun ha cho p = 1; khi , ta c


p
q 2 4q 3 + 2(9q 2)r 27r2
j(a b)(b c)(c a)j
=
P =
(1 2q)2
(1 2q)2
q
q
3
p
2
4(1 3q)3
27 r 9q27 2 + 4(1 273q)
2(1 3q) 3(1 3q)
27
=
=
= f (q)
(1 2q)2
(1 2q)2
9(1 2q)2
Ta c
0

f (q) =

p
(6q + 1) 3(1 3q)
9(1 2q)3

f 0 (q) = 0 ,

q=
q=

1
3

1
6

Bng cch lp bng bin thin, ta thy


f (q)
8
1
>
< a= 3
1
b= 3+
Mt khc, cho
>
: c= 1
3

2
p2
2
2

1
6

th P =

p
9 2
=
8q
32
p
9 2
32

1
:
3

nn max P =

p
9 2
32 :

60

CHNG 1. TM TI MT S K THUT GII TON

Nhn xt 6 Bi ton ny l bi ton trong thi ton quc t nm 2006, cch gii
trn ngn gn hn cch gii p n rt nhiu.

V d 1.41 Cho cc s khng m a; b; c; d: Chng minh rng


(a

b)(a

c)(a d)(b c)(b


(a + b + c + d)6

d)(c

1
:
1728

d)

(V Quc B Cn)
Li gii. Khng mt tnh tng qut, gi s d = min fa; b; c; dg ; t a
y; c d = z (x; y; z 0); khi ta c
(a

b)(a

c)(a d)(b c)(b


(a + b + c + d)6

Chun ha cho p = 1 ) r
xyz j(x

y)(x z)(y
(x + y + z)6

1
27 ;

d)(c

d)

d = x; b

xyz(x y)(x z)(y z)


(x + y + z + 3d)6
xyz j(x y)(x z)(y z)j
(x + y + z + 3d)6
xyz j(x y)(x z)(y z)j
(x + y + z)6

ta c

z)j

=r

p
q2

4q 3 + 18qr

27r2 = r

4r

Ta li c

p
f (q)

f 0 (q) = 2(q

6q 2 + 9r)
p
1 + 216r + 1
0
f (q) = 0 , q =
12
Bng cch lp bng bin thin, ta thy
p
1 + 216r + 1
(216r + 1)3=2
f (q) f
=
27r2
12
216

5
1
r+
2
216

Do
r

q2

4q 3 + 18qr

27r2

4r

(216r + 1)3=2
216

27r2

5
1
r+
2
216

t t2 = 216r + 1 1 ) r = t2161 , t y ta c th thy


r
p
(t2 1)(3 t) (t + 1)(3
(216r + 1)3=2
5
1
2
p
r
27r
r+
=
216
2
216
5184 3

t)

= h(t)

d=

1.3. K THUT P QR

61

Ta c
0

h (t) =

p
t) (t + 1)(3
p
1296 3

t(2

t)

h0 (t) = 0 , t = 2
nn bng cch lp bng bin thin, ta thy
h(t)

h(2) =

1
:
1728

8
a = 2t cos2 18
>
>
<
b = 1 sin 18 t
v
T y ta c pcm. ng thc xy ra khi v ch khi
c
> = sin 18 2 sin 18 + 1 t
>
:
d=0
cc hon v tng ng.
V d 1.42 Cho cc s thc a; b; c. Chng minh rng
(a2 + b2 + c2 )2

3(a3 b + b3 c + c3 a):
(Vasile Cirtoaje)

Li gii. Chun ha cho p = 1; khi ta ch cn xt bt ng thc trong trng


hp a b c l , suy ra
p
X
q 2q 2 r + q 2 4q 3 + 2(9q 2)r 27r2
= f (r)
a3 b =
2
cyc
Ta c
0

f (r) =

9q

27r
p
2 q2

f 0 (r) = 0 , r = r0 =
Lp bng bin thin, ta c f (r)
f (r0 )

=
=
=
=

2q 2

2q 2

5q

q2

1 + 9q

q2

4q 3 + 2(9q

4q 3 + 2(9q

7(9q

2)

2)r

27r2
p
(1 3q) 7(1
189

27r2

2)r

3q)

f (r0 ) 8r: Mt khc, ta li c


p
r0 + q 2 4q 3 + 2(9q 2)r0 27r02
2
r0 + 9q 2 27r0
= 5q q 2 1 14r0
2h
i
p
2 7(9q 2) (1 3q) 7(1 3q)
1
p 27
27q 2 + 2(1 3q) 7(1 3q)
27

62

CHNG 1. TM TI MT S K THUT GII TON

Ta cn chng minh
(1 2q)2
p
, 1 + 9q 27q 2 + 2(1 3q) 7(1 3q) 9(1 2q)2
i
h
p
0
, (1 3q) 8 21q 2 7(1 3q)
3f (r0 )

(1

21q)2 28(1 3q)]


p
21q + 2 7(1 3q)

3q)[(8
8

3q)(2 7q)2
p
21q + 2 7(1 3q)

9(1
8

0:

Bt ng thc cui hin nhin ng nn ta c pcm.


V d 1.43 Cho cc s dng a; b; c tha mn a + b + c = 1 v ab + bc + ca = q
(1 3q): Tm gi tr nh nht ca biu thc
P =

b2
c2
a2
+
+ :
b
c
a
(V Quc B Cn)

Li gii. Khng mt tnh tng qut, ta ch cn xt a b c. Ta c


P 3
p
ab
2
2
2
q 2q 2 r
q 2 4q 3 + 2(9q 2)r 27r2
a
b
c
cyc
+
+
=
=
= f (r)
b
c
a
abc
2r
p
q 2 4q 3 + (9q 2)r + (2q 2 1) q 2 4q 3 + 2(9q 2)r 27r2
0
p
f (r) =
2r2 q 2 4q 3 + 2(9q 2)r 27r2
i
h
p
q 3 9q 2 2q + (1 3q) (1 2q)(1 3q)
f 0 (r) = 0 ) r = r0 =
27q 4 27q 3 + 27q 2 9q + 1
T y, bng cch lp bng bin thin, d thy f (r) f (r0 ) 8r, li c
p
q 2q 2 r0
q 2 4q 3 + 2(9q 2)r0 27r02
f (r0 ) =
2r0
=
=

q
(2q

2q 2
2

r0

q 2 4q 3 +(9q 2)r0
1 2q 2

2r0
9q + 1)r0 + q + 2q 3
2r0 (1 2q 2 )

T y, d dng i n kt lun bi ton.

3q 2 + 4q 4

1.3. K THUT P QR

63

Nhn xt 7 Chng ta c mt vi im kh th v, khng ch cho bi ny ni ring


m cn cho tt c cc bi khc ni chung. Xin phn tch r hn bi ny, cc
bi khc, ta c th ly tng tng t. Sau khi thay xong biu thc f (r0 ) =
(2q 2 9q+1)r0 +q+2q 3 3q 2 +4q 4
= g(r0 ), chng ta thy c g y? g(r0 ) l mt hm
2r0 (1 2q 2 )
n iu theo r0 ; c th l n nghch bin, iu ny c ngha rt ln, cc bn t
hn cn nh kt qu sau trong bt ng thc ba bin (xem bi vit trc)
p
p
p(9q 2p2 ) 2(p2 3q) p2 3q
p(9q 2p2 ) + 2(p2 3q) p2 3q
r
27
27
p+
,

p
2 p2

p2

3q
27

p
p2

3q
r

3q
27

p
p + 2 p2

3q

y, ta cng s c
p+

p
p2

p
2 p2

3q

p
27

p
p2

3q
r0

3q
27

p
p + 2 p2

3q

v ta c
q2

4q 3 + (9q

) [q 2

q
1) q 2

2)r0 +(2q 2

4q 3 + (9q

2)r0 ]2 = (1

4q 3 + 2(9q

2q 2 )2 [q 2

27r02 = 0

2)r0

4q 3 + 2(9q

2)r0

27r02 ]

Do , ta phi c
q2
p+
,

p
p2

3q

p
27

4q 3 + 2(9q

p
2 p2

2)r0

3q

27r02
p

r0

p
p2

0
2

3q
27

p
p + 2 p2

3q

Nh vy, chng ta s c

f (r)

f (r0 )

B p
g@

p2

3q
27

p
p + 2 p2

3q C
A

Ci li ca kt qu cui ny ch i vi nhng bt ng thc khng cht lm


m dng c mt kt qu khng
! l l f (r0 ) gii th qu l bt tin, nhng vi
p 2
p
2
p
p 3q
p+2 p2 3q
g
th mi vic s tr nn n gin hn rt nhiu, chng
27
ta s khng phi tnh ton vi cn thc (v nu ta t 3q = p2

x2 (x

0) th

64

CHNG 1. TM TI MT S K THUT GII TON


p

p2 3q

p+2

p2 3q

27

l mt biu thc khng cha cn). R hn, chng ta

ly v d n gin sau
a2
b2
c2
+
+
+a+b+c
b
c
a

6(a2 + b2 + c2 )
a+b+c

Bt ng thc ny tng ng
a2
b2
c2
+
+
5
b
c
a
n 2
Nh vy, nu ta chng minh f (r0 ) = min ab +

12q:
b2
c

c2
a

12q th bi ton c

gii quyt xong. V y l bt ng thc 1 bin theo nn ta tin chc l s lm c


nu c mt k thut tnh ton tt, nhng vi mt bi ton khng qu cht nh th ny
th vic dng f (r0 ) th c v l hi qu tay, chng ta hy th dng
0

p
p2

B p
g@

27

x2 ) 1

t 3q = 1
g

3q

p
p + 2 p2

x
2

3q

27
g

(1

1
3q C
A=g

3q

p
1+2 1

3q

27

0, khi ta c
!
p
1 + 2 1 3q

x) (1 + 2x)
27

=
5

7 + 7x + 32x2 + 16x3 + 5x4 + 28x5 8x6


(1 x)(1 + 2x)(7 + 4x2 2x4 )

6x7

12q = 1 + 4x2

Nn ta ch cn chng minh
7 + 7x + 32x2 + 16x3 + 5x4 + 28x5 8x6
(1 x)(1 + 2x)(7 + 4x2 2x4 )

6x7

1 + 4x2

Rt n gin, bng bin i tng ng, bn c th bt ng thc trn tng ng


vi
x2 (14 16x + 55x2 + 14x3 + 28x4 + 2x5 16x6 ) 0:
Nhng bt ng thc ny li hin nhin ng do 1

0:

1.3. K THUT P QR

65

V d 1.44 Cho cc s dng a; b; c: Chng minh rng


r
a2
b2
c2
a4 + b4 + c4
:
+
+
3
b
c
a
a2 + b2 + c2
(Nguyn Vn Thch)
Li gii. Theo bi trn, ta c
X a2
cyc

7 + 7x + 32x2 + 16x3 + 5x4 + 28x5 8x6


(1 x)(1 + 2x)(7 + 4x2 2x4 )

6x7

Li c
r

a4 + b4 + c4
a2 + b2 + c2

3
=

4r + 2q 2 4q + 1
1 2q
4
27 (1

x)2 (1 + 2x) + 2q 2
1 2q

4q + 1

1 + 12x2 + 8x3 + 6x4


1 + 2x2

Do ta ch cn chng minh
7 + 7x + 32x2 + 16x3 + 5x4 + 28x5 8x6
(1 x)(1 + 2x)(7 + 4x2 2x4 )

6x7

1 + 12x2 + 8x3 + 6x4


1 + 2x2

Ch l
3 + 13x2 7x4
7 + 7x + 32x2 + 16x3 + 5x4 + 28x5 8x6 6x7
2
4
(1 x)(1 + 2x)(7 + 4x
2x )
3(1 x2 )
x2 (14 62x + 63x2 + 99x3 + 101x4 + 40x5 25x6 14x7 )
=
0
3(1 x2 )(1 + 2x)(7 + 4x2 2x4 )
)

7 + 7x + 32x2 + 16x3 + 5x4 + 28x5 8x6


(1 x)(1 + 2x)(7 + 4x2 2x4 )

6x7

3 + 13x2 7x4
3(1 x2 )

Nn ta ch cn chng minh
3 + 13x2 7x4
3(1 x2 )
,

x2 (6

1 + 12x2 + 8x3 + 6x4


1 + 2x2

72x + 436x2 + 144x3 + 72x4 72x5


9(1 x2 )2 (1 + 2x2 )

Bt ng thc cui hin nhin ng do 1


ra khi a = b = c:

369x6 + 98x8 )

0: Vy ta c pcm. ng thc xy

66

CHNG 1. TM TI MT S K THUT GII TON

Nhn xt 8 Bt ng thc
a2
b2
c2
+
+
b
c
a

3 + 13x2 7x4
3(1 x2 )

C dng tng ng l
P
P
P 2 2
3 x4 + 13 x3 (y + z)
x y
2
Xx
cyc
cyc
cyc
!
!
y
P
P
cyc
3
x
xy
cyc

xyz

cyc

8x; y; z

cyc

Chng ta c th dng kt qu ny chng minh kt qu sau (hin vn unsolved trn


mathlinks)
r
6
6
6
b2
c2
a2
6 a + b + c
+
+
3
:
b
c
a
3
(V Quc B Cn)

V d 1.45 Cho cc s dng a; b; c: Chng minh rng


p
a b
c
3
+ + + 3 4
b
c a

ab + bc + ca
a2 + b2 + c2

p
3
3 4 + 1:

Li gii. R rng ta ch cn xt bt ng thc trong trng hp a


Chun ha cho p = 1; khi , ta c
p
q 3r
q 2 4q 3 + 2(9q 2)r 27r2
c
a b
+ + =
b
c a
2r

q
ab + bc + ca
=
2
2
2
a +b +c
1 2q
Nn bt ng thc tng ng vi
p
q 3r
q 2 4q 3 + 2(9q
2r
Xt hm s
f (r) =
Ta c
0

f (r) =

q2

3r

2)r

p
q2

27r2

p
334 2 q
+
1 2q

4q 3 + 2(9q
2r

2)r

p
3
3 4+1

27r2

p
4q 3 + (9q 2)r q q 2 4q 3 + 2(9q 2)r
p
2r2 q 2 4q 3 + 2(9q 2)r 27r2

27r2

c l .

1.3. K THUT P QR

67

f 0 (r) = 0 , r = r0 =

h
q 2 9q 2

Lp bng bin thin, ta c th thy f (r)


f (r0 )

=
=
=

2q + (1
27q 2

p
3q) q(1

9q + 1

f (r0 ) 8r > 0. Ta li c

p
q2

q
4q 3 + 2(9q 2)r0 27r02
=
2r0
3
2q + (1 6q)r0
2q 2
1
=
+
6
qr0
r0
q
1
2(27q 2 9q + 1)
p
+
6
9q 2 2q + (1 3q) q(1 3q) q

3r0

i
3q)

3r0

q 2 4q 3 +(9q 2)r0
q

2r0

Nh vy, hon tt yu cu ca bi ton, ta ch cn chng minh c


p
p
334 2 q
3
f (r0 ) +
3 4+1
1 2q
p
p
334 2 q
1
2(27q 2 9q + 1)
3
p
+ +
3 4+7
,
2
1 2q
9q
2q + (1 3q) q(1 3q) q
Bng khai trin trc tip, ta thy
2(27q 2
9q 2

p
p
334 2 q
1
3
3 4
+ +
1 2q
3q) q
(1 3q)A
h
i
p
2q) 9q 2 2q + (1 3q) q(1 3q)

9q + 1)
p
3q) q(1

2q + (1
=

q(1

p
p
Vi A = [3(k+6)q 2 (k+11)q +1] q(1 3q) q 2 [2k+9 9(k+2)q]; k = 3 3 4 2:
Do 2k + 9 9(k + 2)q nn nu 3(k + 6)q 2 (k + 11)q + 1 0 th hin nhin A 0
nn bt ng thc ca ta ng. Ngc li, nu 3(k + 6)q 2 (k + 11)q + 1 0 th ta
c
p
A 0 , q 2 [2k + 9 9(k + 2)q]
[3(k + 6)q 2 (k + 11)q + 1] q(1 3q)
, q 4 [2k + 9 9(k + 2)q]2 [3(k + 6)q 2 (k + 11)q + 1]2 q(1 3q)
p
p
p 2
p
9 3 16 + 6 3 4 + 4 q(27q 2 9q + 1) 20q 4 3 16
8q + 2 3 16
,
800

Do 3(k + 6)q 2

(k + 11)q + 1
q

0 nn
k + 11

k 2 + 10k + 49
6(k + 6)

68

CHNG 1. TM TI MT S K THUT GII TON

Suy ra
8q + 2

p
3

16
=
=
=

Do k >

5
2

p
k 2 + 10k + 49
3
16
+2
3(k + 6)
p
4 k + 11
k 2 + 10k + 49
12
+2
3(k + 6)
k+2
p
2
2
2 5k + 32k 28 2(k + 2) k + 10k + 49
3(k + 2)(k + 6)
3k(7k 2 + 46k 152)
p
3(k + 2) 5k 2 + 32k 28 + 2(k + 2) k 2 + 10k + 49
4 k + 11

nn
7k 2 + 46k

152 > 7

5
2

5
2

+ 46

152 =

27
> 0 ) A > 0:
4

Bi ton c gii quyt xong.


ng thc xy ra khi v ch khi a = b = c hoc
8
q
pp
p
p
p
p
3
3
>
3
3
3
4
1
2
1
>
+3
4 + 8 2 11 cos 3 arccos 17 203 4
> a= 3+ 2
3
>
<
c=t
q
>
p p
p
p
>
p
p
3
3
>
3
3
2
1
2 3
>
2 3 sin 3 arccos 27+27 202 27 4
t
: b= 3 4+ 3 3 4

t
2

v cc hon v tng ng.

p
V d 1.46 Cho cc s dng a; b; c: Chng minh rng vi k = 3 3 2
a b
c
+ + +k
b
c a

3; ta c

3(k + 3)(a2 + b2 + c2 )
:
(a + b + c)2
(V Quc B Cn, Bch Ngc Thnh Cng)

Li gii. Chun ha cho p = 1; theo kt qu trn, ta c


Xa
cyc

2(27q 2
9q 2

2q + (1

Ta phi chng minh


2(27q 2
9q 2

2q + (1

9q + 1)
p
3q) q(1

3q)

9q + 1)
p
3q) q(1
+

1
q

6+k

3q)

1
q

3(k + 3)(1

2q):

1.3. K THUT P QR

69

Ta c
VT

VP =
q

Trong
A = [6(k + 3)q 2
Do 4k + 21
15)q + 1
li 2k+15
A

(1

9q 2

3q)A
p
2q + (1 3q) q(1

p
(2k + 15)q + 1] q(1

18(k + 3)q nn nu ta c q

i
3q)

3q) + q 2 [4k + 21

2k+15

4k2 +36k+153
12(k+3)

0 th A
p

18(k + 3)q]:

, 6(k + 3)q 2

(2k +

0 nn bt ng thc hin nhin ng, trong trng hp ngc


4k2 +36k+153
q 13 : Ta c
12(k+3)
p
0 , q 2 [4k + 21 18(k + 3)q]
[6(k + 3)q 2 (2k + 15)q + 1] q(1 3q)
, q 3 [4k + 21 18(k + 3)q]2 [6(k + 3)q 2 (2k + 15)q + 1]2 (1 3q)
p
p
p 2
3
4(27q 2 9q + 1) 12q + 1 3 4 12q 2 3 4
,
0
12

Do q

2k+15

p
4k2 +36k+153
12(k+3)

nn
12q + 1

p
3

4 > 0:

Vy bt ng thc cn chng minh ng.


V d 1.47 Cho cc s khng m a; b; c; khng c 2 s no ng thi bng 0: Chng
minh rng
r
a + 2c b + 2a
c + 2b
5(a2 + b2 + c2 )
+
+
+ 4:
a + 2b
b + 2c
c + 2a
ab + bc + ca
(V Quc B Cn)
V d 1.48 Cho cc s
khng
m a; b; c; khng c 2 s no ng thi bng 0: Chng
p
p
3(3 3 3+ 3 9+1)
minh rng vi k =
th
8
ab2 + bc2 + ca2
a2 + b2 + c2
+k 2
ab + bc + ca
a b + b2 c + c2 a

k + 1:
(Bch Ngc Thnh Cng)

V d 1.49 Cho cc s khng m a; b; c; khng c 2 s no ng thi bng 0 tha


mn a + b + c = 1: Tm gi tr ln nht v gi tr nh nht ca biu thc
a b
b c
c a
P (a; b; c) = p
+p
+p
:
c+a
a+b
b+c
(Phm Kim Hng)

70

CHNG 1. TM TI MT S K THUT GII TON

Mt iu hn ch ca k thut ny l mc d rt mnh nhng n i hi chng ta


phi tnh ton nhiu. Nhng cc bn , cc bi ton y chng ti a ra u l
nhng bi ton rt kh, ng thc ca chng hu ht xy ra ti nhng im lch
nhau. iu ny rt kh cho chng ta tm c 1 li gii p mt cho n ngoi nhng
li gii nh th ny. Tuy nhin, chng ta c th thy mt iu l nu dng k thut
ny gii nhng bi ton thi quc gia, quc t th chng ta li thu c nhng li
gii gn p v rt nh nhng bi l cc bi ton y u l nhng bi ton "rt lng".
V vy, chng ti vit v k thut ny vi mong mun thit lp cho chng ta mt k
thut, mt phng php chng ta c th gii c nhng bi ton y khi "chm
trn" chng trong cc k thi.

1.4
1.4.1

The CYH techniques


Li ni u

Ngay t khi cn hc mi trng THCS, chng ta c lm quen vi bt ng


thc Cauchy Schwarz v khi bc sang THPT, chng ta c lm quen thm vi bt
ng thc Holder, c 2 bt ng thc ny u rt thng c s dng, ngay c
trong cc k thi hc sinh gii quc gia, quc t. C th ni chng v bt ng thc
trung bnh cng-trung bnh nhn (AM-GM) l nhng bt ng thc c in thng
dng nht hin nay, nhng vic s dng chng nh th no l hiu qu? Bi vit nh
ny, chng ti xin c chia s vi cc bn mt vi k thut thng dng, mong nhn
c kin ng gp ca cc bn.

1.4.2

Bt ng thc Cauchy Schwarz v Holder.

Trc khi bt u bi vit, chng ta hy nhc li vi nt v bt ng thc Cauchy


Schwarz v Holder.
nh l 1.4 (Bt ng thc Cauchy Schwarz) Vi mi s thc (a1 ; a2 ; :::; an )
v (b1 ; b2 ; :::; bn ); ta c
(a1 b1 + a2 b2 +

+ an bn )2

(a21 + a22 +

+ a2n )(b21 + b22 +

+ b2n ):

ng thc xy ra khi v ch khi ai : aj = bi : bj 8i; j 2 f1; 2; :::; ng :


Chng minh bt ng thc trn c rt nhiu cch nhng cch ngn gn nht l s
dng ng thc Lagrange
(a21 + a22 +

+ a2n )(b21 + b22 +

+ b2n )

(a1 b1 + a2 b2 +

+ an bn )2 =

X
i6=j

(ai bj

aj bi )2

1.4. THE CYH TECHNIQUES

71

H qu 1.5 Vi mi s thc (a1 ; a2 ; :::; an ) v (b1 ; b2 ; :::; bn ); bi > 0 8i = 1; 2; :::; n;


ta c
a21
a2
+ 2+
b1
b2

a2n
bn

(b1 + b2 +

+ bn )

(a1 + a2 +

+ an )2

ng thc xy ra khi v ch khi ai : aj = bi : bj 8i; j 2 f1; 2; :::; ng :


nh l 1.5 (Bt ng thc Holder) Cho cc s dng xij (i = 1; m; j = 1; n):
Khi vi mi ! 1 ; :::; ! n 0 tha ! 1 +
+ ! n = 1; ta c
0
1! j
!
n
m
n
m
Y
X
Y
X
!
j
@
xij :
xij A
i=1

i=1

j=1

j=1

Chng minh bt ng thc ny bng cch dng bt ng thc AM-GM tng qut
nh sau
m
P
Gi s
xij = 1 8i = 1; n (ta lun c th gi s c iu ny! Ti sao?), khi
j=1

bt ng thc trn tr thnh

m
n
X
Y

j=1

!
xijj

i=1

S dng bt ng thc AM-GM, ta c


m
n
X
Y
j=1

i=1

xijj

m
n
X
X
j=1

! j xij

i=1

n
X
i=1

0
1
m
X
@
! j xij A
j=1

0
1
n
m
n
X
X
X
xij A =
! j = 1:
!j @
=
i=1

j=1

j=1

Bt ng thc Holder c chng minh.


Mt trng hp c bit thng gp ca bt ng thc Holder l khi n = 3; ta c
(a3 +b3 +c3 )(m3 +n3 +p3 )(x3 +y 3 +z 3 )
(

a
b
c
m = n = p
b
c
a
x = y = z

ng thc xy ra khi v ch khi

(a3 + b3 + c3 )(m3 + n3 + p3 )2
ng thc xy ra khi v ch khi

(amx+bny+cpz)3 8a; b; c; m; n; p; x; y; z

a
m

: V khi (m; n; p)

(x; y; z) th

(am2 + bn2 + cp2 )3 8a; b; c; m; n; p


b
n

= pc :

72

CHNG 1. TM TI MT S K THUT GII TON

1.4.3

Mt s k thut cn ch

Tham s ha
y l k thut c bn nht trong cc k thut c trnh by y, cc bn hy
xem xt k n trc khi sang phn khc.
V d 1.50 Cho cc s khng m x; y; z tha mn 2x + 3y + z = 1: Tm gi tr nh
nht ca biu thc
P = x3 + y 3 + z 3 :
Li gii. S dng bt ng thc Holder, ta c
(x3 + y 3 + z 3 )(a3 + b3 + c3 )(m3 + n3 + p3 )
) P = x3 + y 3 + z 3

(a3

(xam + ybn + zcp)3 8a; b; c; m; n; p

(xam + ybn + zcp)3


+ b3 + c3 )(m3 + n3 + p3 )

Ta hy chn a; b; c; m; n; p sao cho gi thit 2x + 3y + z = 1 c tn dng trit ,


cp
bn
t theo l t nhin ta c th chn a; b; c; m; n; p tha am
2 = 3 = 1 = 1: Hn na,
do ta cn tm min P nn ng thc bt ng thc ny phi xy ra, tc l
y
x
z
a = b = c
y
x
z
m = n = p
2x+3y+z
z
1
= 3y
3b = c = 2a+3b+c = 2a+3b+c
2ax = 3by = cz
8
a
< x = 2a+3b+c
b
y = 2a+3b+c : T phng trnh th 2 suy ra
T phng trnh th nht suy ra
:
c
z = 2a+3b+c

2x
2a

2a2
3b2
c2
=
=
, 2a2 = 3b2 = c2
2a + 3b + c
2a + 3b + c
2a + 3b + c
p
p
T y, ta chn c a = p12 ; b = p13 ; c = 1 ) m = 2 2; n = 3 3; p = 1, t theo
2ax = 3by = cz ,

trn, ta c

P
p1
2

p1
3

(2x + 3y + z)3
h p 3
3
p
+1
2 2 + 3 3

36
p
p
36 + 4 3 + 9 2 1 + 81 3 + 16 2
p

i
+1

1.4. THE CYH TECHNIQUES


8
< x=
y=
ng thc xy ra khi
:
z=
min P =

73

a
2a+3b+c
b
2a+3b+c
c
2a+3b+c

8
1
< a = p2
b = p13 : Vy nn
vi
:
c=1

36
p
p :
36 + 4 3 + 9 2 1 + 81 3 + 16 2
p

V d 1.51 Cho cc s khng m x; y tha mn x3 + y 3 = 1: Tm gi tr ln nht


ca biu thc
P = x + 2y:
Li gii. S dng bt ng thc Holder, ta c
(x3 + y 3 )(a3 + b3 )(m3 + n3 ) (xam + ybn)3 8a; b; m; n
p
3
) xam + ybn
(x3 + y 3 )(a3 + b3 )(m3 + n3 )

L t nhin, do yu cu ca bi ton nn ta phi chn a; b; m; n sao cho biu thc


bn
xam + ybn P; tc l cc s a; b; m; n phi tha am
1 = 2 = 1: Ngoi ra, cng nh
v d trn, ta cn tm gi tr ln nht ca P nn bt buc ng thc bt ng thc
trn phi xy ra, tc l
y
x
a = b
y
x
m = n
,

x+2y
= 2y
2b = a+2b =
ax = 2by
8
a
< x = a+2b
b
,
y = a+2b
:
ax = 2by
8
a
>
< x = a+2b
b
y = a+2b
,
>
: a2 = 2b2
a+2b
a+2b
8
a
< x = a+2b
,
y= b
: 2 a+2b
a = 2b2
8
a
x = a+2b
>
>
<
b
y = a+2b
p
)
>
a= 2
>
:
b=1
x
a

1
a+2b

74

CHNG 1. TM TI MT S K THUT GII TON


8
a
x = a+2b
>
>
>
b
>
y = a+2b
>
>
p
<
a= 2
)
b=1
>
>
>
1
>
>
> m = p2
:
n=2

Do theo trn, ta c
P = x + 2y

p
3

(x3 + y 3 )(a3 + b3 )(m3 + n3 ) =

s
3

p
2 2+1

1
p +8
2 2

ng thc lun xy ra nn
max P =

s
3

p
2 2+1

1
p +8 :
2 2

i vi nhng bt ng thc m khng c ng thc xy ra th ta chn tham s l


nhng s m ng thc ca bt ng thc Cauchy Schwarz hoc Holder gii l
ln cn bng ca bt ng thc ban u.
V d 1.52 Cho cc s dng a1 ; a2 ; :::; an : Chng minh rng
1
2
+
+
a1
a1 + a2

n
a1 + a2 +

+ an

1
1
+
+
a1
a2

<2

1
an

Li gii. S dng bt ng thc Cauchy Schwarz, 8k = 1; n; bi > 0 8i = 1; n; ta c


b21
b2
+ 2 +
a1
a2
)

k
a1 + a2 +

b2k
ak

+ ak
)

n
X

k=1

(a1 + a2 +
k
(b1 + b2 +
k
a1 + a2 +

+ ak )

(b1 + b2 +
b21
b2
+ 2 +
a1
a2

+ bk )2

+ ak

+ bk )2

b2k
ak

n
X
ci
a
i=1 i

Vi
ck =

kb2k
(k + 1)b2k
+
+
2
(b1 + b2 +
+ bk )
(b1 + b2 +
+ bk+1 )2

nb2k
(b1 + b2 +
+ bk+1 )2

1.4. THE CYH TECHNIQUES

75

Chn bk = k; ta c
ck

k3

k
P

k 2 (k + 1)
k+1
P

i=1

4k 2

4k 2

< 4k 2
=

4k 2

4k 2

k2 n
n
P
i

i=1

i=1

1
1
1
+
+
+
k(k + 1)2
(k + 1)(k + 2)2
n(n + 1)2
1
1
1
1
+
+
k(k + 1)
n(n + 1) (k + 1)2
(n + 1)2
1 1
1 1
1
1
1
1
+
+
+
+
2 k2
(k + 1)2
2 n2
(n + 1)2
(k + 1)2
(n + 1)2
1
1
1
1
1
1
+
+
+
+ 2
2k 2
2(n + 1)2
(k + 1)2
n
(k + 1)2
(n + 1)2
1
1
< 2 8k = 1; n
2k 2
2(n + 1)2

Nn t y hin nhin ta c bt ng thc cn chng minh ng.


V d 1.53 Cho cc s thc a1 ; a2 ; :::; an : Chng minh rng
4(a21 + a22 +

+ a2n )

a21 +

a1 + a2
2

a1 + a2 +
n

+ an

Li gii. S dng bt ng thc Cauchy Schwarz, 8k = 1; n; bi > 0 8i = 1; n ta c


a21
a2
+ 2+
b1
b2

a1 + a2 +
k

a2k
bk

+ ak

n
X

k=1

(b1 + b2 +

+ bk )

b 1 + b2 +
k2

a1 + a2 +
k

+ ak

+ bk

(a1 + a2 +

a21
a2
+ 2+
b1
b2
n
X

+ ak )2

a2k
bk

ck a2k

k=1

Vi
ck =

b 1 + b2 +
+ bk
b 1 + b2 +
+ bk+1
+
+
k 2 bk
(k + 1)2 bk

b 1 + b2 +
+ bn
n2 bk

76

CHNG 1. TM TI MT S K THUT GII TON

Chn bk =
ck

=
=

k + 1; ta c

b1 + b 2 +
+ bk
b1 + b2 +
+ bk+1
b1 + b 2 +
+ bn
+
+
+
k 2 bk
(k + 1)2 bk
n 2 bk
1
1
1
1
p
+
+
+ 3=2
p
(k + 1)3=2
n
k
k 1 k 3=2
2 0
1
0
13
1
1
1
1
1
42 @ q
A+
A5
p
q
q
+ 2 @q
p
1
1
1
1
k
k 1
k 2
k+ 2
n 2
n+ 2
1
0
2
1
1
A< q
@q 1
p
q
p
p
p
k
k 1
k 12
n + 12
k 12
k
k 1
p p
p
2 2
k+ k 1
p
4
2k 1

Nn bt ng thc hin nhin ng.


0; x3 + y 3 = 1: Chng minh rng
r
5
p
p
p
6
5
x+2 y
1+2 2 :

V d 1.54 Cho cc s x; y

V d 1.55 Cho cc s a; b; c
biu thc

0; a2 + b2 + c2 = 1: Tm gi tr ln nh nht ca
P = a3 + 3b3 + 2c3 :

Hng dn. Dng bt ng thc Holder


(a3 + 3b3 + 2c3 )2 (m3 + n3 + p3 )

V d 1.56 Cho cc s a; b; c

p
p
3
3
a2 m + b2 n 9 + c2 p 4

0; a + b + c = 3: Tm gi tr nh nht ca biu thc


P = a4 + 2b4 + 4c4 :

Hng dn. Dng bt ng thc Holder


(a4 + 2b4 + 3c4 )3 (m4 + n4 + p4 )

p
p
4
4
a3 m + b3 n 8 + c3 p 27

1.4. THE CYH TECHNIQUES

77

V d 1.57 Cho cc s thc x1 ; x2 ; :::; xn : Chng minh rng


x21 + (x1 + x2 )2 +

+ xn )2

+ (x1 + x2 +

4 sin2

(x21 + x22 +

+ x2n ):

2(2n+1)

Hng dn. Dng bt ng thc Cauchy Schwarz,


n
k
X
X

k=1

i=1

xi

!2

n
X

k=1

"

k
X

ci

i=1

k
X
x2
i

ci

i=1

!#

n
X

k=1

"

Sk

k
X
x2
i

i=1

ci

!#

Chn cc s ci sao cho


S1 + S2 +
c1

+ Sn

) ci = sin

S2 +

i
2n + 1

+ Sn
c2
sin

Sn
=k
cn

(i 1)
:
2n + 1

V d 1.58 Cho cc s a; b; c
r
P =

0; a + b + c = 1: Tm gi tr nh nht ca biu thc


r
r
1
1
1
a2 + 2 + b2 + 2 + c2 + 2 :
b
c
a

Hng dn. D on ng thc


Cauchy Schwarz,
s
1
a2 + 2
b
s
1
b2 + 2
c
s
1
c2 + 2
a

xy ra ti a = b = c = 31 : S dng bt ng thc

(m2 + n2 )

ma +

n
b

(m2 + n2 )

mb +

n
c

(m2 + n2 )

mc +

n
:
a

"S khng m"


i vi cc bt ng thc dng ab11 + ab22 + + abnn
k m ta cha chc ai 0 8i = 1; n
hay khng, ch bit rng bi 0 8i = 1; n thi th vic thm vo l mt k thut cn

78

CHNG 1. TM TI MT S K THUT GII TON

thit. tng n gin nh sau, cng cc tham s mi vo tuong ?ng


ai + mi bi 0 8i = 1; n; khi a bt ng thc v dng
n
X
a0

i=1

k+

bi

n
X

ai
bi

sao cho

mi

i=1

Vi a0i ; bi
0 8i = 1; n: T y ta c th s dng bt ng thc Cauchy SchwarzHolder mt cch t nhin m khng phi lo ngi na.
V d 1.59 Cho cc s thc a; b; c; d tha mn a2 + b2 + c2 + d2 = 1: Chng minh
rng
1
1
1
16
1
+
+
+
:
1 ab 1 bc 1 cd 1 da
3
(V Quc B Cn)
Li gii. Trc ht, ta hy vit bt ng thc v dng
X

1
ab

X k(1

ab)
ab

cyc

,
V 1 = a2 + b2 + c2 + d2

cyc

a2 + b2

k(1

ab)

16
3
1

4k

16
3

2ab, suy ra
1

1
2

k 1

1=

k
2

Ta cn chn k > 0 sao cho k(1 ab) 1 0. T trn, ta ch cn chn k 2, v vi


k = 2, bt ng thc k(1 ab) 1 0 c ng thc ti a = b = p12 ; c = d = 0. Do
ta hy th s dng Cauchy Schwarz vi k = 2. Bt ng thc tr thnh
X1
cyc

2ab
ab

8
3

S dng bt ng thc Cauchy Schwarz, ta c

X1
cyc

2ab
1 ab

"

2ab)

cyc

(1

cyc

(1

#2

2ab)(1

ab)

4[2 (a + c)(b + d)]2


3(a + c)(b + d) + 2(a2 + c2 )(b2 + d2 )

1.4. THE CYH TECHNIQUES

79

Do , ta ch cn chng minh c
3[2

(a + c)(b + d)]2

, 3(a + c)2 (b + d)2


, 3[1

3(a + c)(b + d) + 2(a2 + c2 )(b2 + d2 )]

2[4

6(a + c)(b + d) + 4
2

(a + c)(b + d)] + 1

4(a2 + c2 )(b2 + d2 )

4(a + c )(b + d )

S dng bt ng thc AM-GM, ta c


4(a2 + c2 )(b2 + d2 )

(a2 + b2 + c2 + d2 )2 = 1:

Bt ng thc c chng minh. ng thc xy ra khi v ch khi a = b = c = d =

1
2.

V d 1.60 Cho cc s thc a; b; c: Chng minh rng


a2 bc
b2 ca
c2 ab
+
+
a2 + 2b2 + 3c2
b2 + 2c2 + 3a2
c2 + 2a2 + 3b2

0:
(Nguyn Anh Tun)

Li gii. Vit li bt ng thc nh sau


X

4(a2 bc)
a2 + 2b2 + 3c2

cyc

,
,2

X
cyc

a2

X
cyc

4(a2 bc)
+1
a2 + 2b2 + 3c2

X
(b c)2
5a2 + c2
+
2
2
2
+ 2b + 3c
a + 2b2 + 3c2
cyc

V
2

X
cyc

a2

(b c)2
+ 2b2 + 3c2

Nn ta ch cn chng minh c
X
cyc

a2

5a2 + c2
+ 2b2 + 3c2

Nhng bt ng thc ny hin nhin ng theo bt ng thc Cauchy Schwarz


!
!
"
#2
X
X
X
5a2 + c2
2
2
2
2
2
2
2
(5a + c )(a + 2b + 3c )
(5a + c )
a2 + 2b2 + 3c2
cyc
cyc
cyc
!2
X
2
= 36
a
cyc

80

CHNG 1. TM TI MT S K THUT GII TON

v
12

X
cyc

a2

!2

(5a2 + c2 )(a2 + 2b2 + 3c2 ) = 4

cyc

a4

cyc

a2 b2

0:

cyc

ng thc xy ra khi v ch khi a = b = c:


V d 1.61 Cho cc s khng m a; b; c tha a + b + c = 3: Chng minh rng
5

3ab 5 3bc 5 3ca


+
+
1+c
1+a
1+b

ab + bc + ca:
(Vasile Cirtoaje)

Li gii. Bt ng thc tng ng vi


X 9(5

3ab)
1+c

cyc

X 9(5

,5

cyc

ab

cyc

3ab) + 7ab(1 + c)
1+c

cyc

X9

16

ab

cyc

X 1
4ab
+ 7abc
1+c
1+c
cyc

16

ab

cyc

S dng bt dng thc AM-GM, ta c


9 = (a + b + c)2

(a + b)2

4ab

Do , s dng bt ng thc Cauchy Schwarz, ta c


!2
"
#2
P
P
27 4 ab
(9 4ab)
X 9 4ab
cyc
cyc
P
P
=
1+c
(9 4ab)(1 + c)
54 4 ab 12abc
cyc
cyc

cyc

X
cyc

1
1+c

cyc

Ta cn chng minh
5 27
54

4
P

cyc

9
3
=
(1 + c)
2

cyc

ab

!2

ab

12abc

21
abc
2

16

X
cyc

ab

1.4. THE CYH TECHNIQUES

81
n
max 0; (4q

t q = ab+bc+ca; r = abc, theo bt ng thc Schur bc 4, ta c r


Bt ng thc tr thnh
5(27 4q)2
21
+ r
54 4q 12r
2
Nu 9

4q th

21
5(27 4q)2
+ r
54 4q 12r
2
Nu 4q

16q

9)(9 q)
18

5(27
54

16q

4q)2
4q

16q =

9(9

4q)(45 4q)
2(27 2q)

9 th
5(27 4q)2
21
+ r
54 4q 12r
2

16q

21(4q 9)(9
5(27 4q)2
+
2
36
54 4q 3 (4q 9)(9 q)
(4q 9)(3 q)(36 7q)
=
0:
12(6 q)

q)

16q

ng thc xy ra khi v ch khi a = b = c = 1 hoc a = b = 32 ; c = 0 hoc cc hon


v tng ng.
V d 1.62 Cho a; b; c l di 3 cnh ca mt tam gic. Chng minh rng
b(b c)
c(c a)
a(a b)
+
+
a2 + 2bc b2 + 2ca c2 + 2ab

0:
(V Quc B Cn)

Li gii. Bt ng thc tng ng


X a(a b)
+1
a2 + 2bc
cyc
,

X 2a2
cyc

a2

ab + 2bc
+ 2bc

Do a; b; c l di 3 cnh ca mt tam gic nn c


2a2

ab + 2bc

2a2

ab + 2b(b

3
b

a) = 2(a

a, t
b)2 + ab

o
.

82

CHNG 1. TM TI MT S K THUT GII TON

Suy ra, theo bt ng thc Cauchy Schwarz, ta c

X 2a2

ab + 2bc
2
a + 2bc

cyc

"

(2a

#2

ab + 2bc)

cyc

(2a2

ab + 2bc)(a2 + 2bc)

cyc

Ta cn chng minh
"

(2a

#2

cyc

,7

a3 b + 4

cyc

ab + 2bc)
X

ab3

(2a2

ab + 2bc)(a2 + 2bc)

cyc

cyc

a4 + 3

cyc

a2 b2 + 6

cyc

a2 bc

cyc

Li doa; b; c l di 3 cnh ca mt tam gic nn tn ti x; y; z > 0 sao cho


a = y + z; b = z + x; c = x + y. Bt ng thc tr thnh
X
X
X
X
X
2
x4 + 2
xy(x2 + y 2 ) + 3
xy 3 6
x2 y 2 + 3
x2 yz
cyc

cyc

cyc

cyc

cyc

Nhng bt ng thc ny hin nhin ng theo bt ng thc AM-GM v


X
X
X
X
X
2
x4 + 2
xy(x2 + y 2 ) 6
x2 y 2 ; 3
xy 3 3
x2 yz:
cyc

cyc

cyc

cyc

cyc

ng thc xy ra khi v ch khi a = b = c:


V d 1.63 Cho cc s khng m a; b; c; khng c 2 s no ng thi bng 0: Chng
minh rng
2a2 bc
2b2 ca
2c2 ab
+
+
3:
b2 bc + c2
c2 ca + a2
a2 ab + b2
(Vasile Cirtoaje)
Li gii. Vit li bt ng thc nh sau
X
cyc

2a2 bc
+1
b2 bc + c2
X 2a2 + (b
cyc

b2

c)2
bc + c2

1.4. THE CYH TECHNIQUES

83

S dng bt ng thc Cauchy Schwarz, ta c

X 2a2 + (b
cyc

c)2
bc + c2

b2

"

#2

[2a + (b

c) ]

cyc

[2a2 + (b

c)2 ](b2

bc + c2 )

cyc

Ta cn chng minh
"

[2a + (b

c) ]

cyc

,2

a4 + 2abc

cyc

,2

#2

a2 (a

c)2 ](b2

bc + c2 )

cyc

a+

cyc

[2a2 + (b

ab(a2 + b2 )

cyc

b)(a

c) + 3

cyc

a2 b2

cyc

ab(a

b)2

0:

cyc

Bt ng thc cui hin nhin ng theo bt ng thc Schur bc 4. ng thc xy


ra khi v ch khi a = b = c hoc a = b; c = 0 hoc cc hon v tng ng.
V d 1.64 Cho cc s khng m a; b; c; tt c khng ng thi bng 0: Chng minh
rng
3a2 bc
3b2 ca
3c2 ab
+
+
2a2 + b2 + c2
2b2 + c2 + a2
2c2 + a2 + b2

3
:
2
(Vasile Cirtoaje)

Li gii. Vit li bt ng thc nh sau


X

cyc

,3

X
cyc

2(3a2 bc)
2a2 + b2 + c2

X 3b2 + 2bc + 3c2


cyc

2a2 + b2 + c2

X
(b c)2
bc
+8
2
2
2 + b2 + c2
+b +c
2a
cyc

2a2

84

CHNG 1. TM TI MT S K THUT GII TON

Nu (a b)2 + (b c)2 + (c a)2 = 0, bt ng thc l hin nhin. Nu (a


(b c)2 + (c a)2 > 0; khi theo bt ng thc Cauchy Schwarz, ta c
!2
P
2
(b c)
X (b c)2
cyc
P
2a2 + b2 + c2
(b c)2 (2a2 + b2 + c2 )
cyc
cyc

4
= "

a2

cyc

c)2

(b

cyc

!2

ab

cyc

a2

cyc

a2

cyc

ab

cyc

a2 (b

a2

!2

ab

cyc

a2

cyc

c)2

cyc

cyc

b)2 +

b2 c2

cyc

a2 bc

cyc

X
cyc

bc
2a2 + b2 + c2

!2

bc

cyc

bc(2a2 + b2 + c2 )

cyc

bc

cyc

bc

cyc

!2

a2

cyc

a2 bc

cyc

Ta cn chng minh
6

cyc

cyc

a2

cyc

ab

!2

ab

cyc

a2

cyc

b2 c2

cyc

8
P

+
a2 bc

cyc

bc

cyc

bc

cyc

!2

a2

cyc

Do tnh thun nht, ta c th chun ha cho a + b + c = 1. t q =

cyc

cyc

khi bt ng thc tr thnh


(1

3(1 3q)2
3q)(1 2q) + q 2
,

(1

3r

4q 2
q(1 2q) + r

(3r + q 4q 2 )2
5q + 7q 2 3r)(q 2q 2 + r)

0:

6
a2 bc

bc; r = abc,

1.4. THE CYH TECHNIQUES

85

hin nhin ng. ng thc xy ra khi v ch khi a = b = c hoc


a
b
c
1 = 0 = 0 hoc cc hon v tng ng.

a
1

b
1

c
0

hoc

V d 1.65 Cho cc s dng a; b; c: Chng minh rng

a2 bc
b2 ca
c2 ab
+p
+p
a2 + 2b2 + 3c2
b2 + 2c2 + 3a2
c2 + 2a2 + 3b2

0:

(Nguyn Anh Tun)

Li gii. S dng bt ng thc Cauchy Schwarz, ta c


X
cyc

"

8(a2

bc)

p
+b+c
6(a2 + 2b2 + 3c2 )

X 8(a2
cyc

X 8(a2
cyc

p
bc) + (b + c) 6(a2 + 2b2 + 3c2 )
p
6(a2 + 2b2 + 3c2 )

bc) + (b + c)(a + 2b + 3c)


p
6(a2 + 2b2 + 3c2 )

X 8a2 + c2 + ab + bc + ca + 2(b
p
=
6(a2 + 2b2 + 3c2 )
cyc

c)2

X 8a2 + c2 + ab + bc + ca
p
6(a2 + 2b2 + 3c2 )
cyc

Nn ta ch cn chng minh c
X 8a2 + c2 + ab + bc + ca
p
a2 + 2b2 + 3c2
cyc

p X
2 6
a
cyc

S dng bt ng thc Holder, ta c


X 8a2 + c2 + ab + bc + ca
p
a2 + 2b2 + 3c2
cyc

!2 "

(8a + c + ab + bc + ca)(a + 2b + 3c )

cyc

27 3

X
cyc

a +

X
cyc

!3

ab

86

CHNG 1. TM TI MT S K THUT GII TON

Do tnh thun nht, ta c th chun ha cho a+b+c = 1. t q = ab+bc+ca; r = abc,


khi theo bt ng thc Schur bc 3, ta c r 4q9 1 . T y, ta c
X
(8a2 + c2 + ab + bc + ca)(a2 + 2b2 + 3c2 )
cyc

= 6

ab

cyc

= 6

v
3

cyc

ab

cyc

= 53q 2

a2

cyc

38q + 11

a2 +

cyc

(8a2 + c2 )(a2 + 2b2 + 3c2 )

cyc

+ 11

a2

cyc

42r

ab = 3

!2

+ 21

a2 b2

cyc

5q

cyc

Ta phi chng minh


5q)3

27(3

911q + 1601q 2

, 155
Ta c
155

24(53q 2

911q + 1601q 2

38q + 11

42r)

1125q 3 + 336r

1125q 3 + 336r

155

911q + 1601q 2

1
(1
3

3q)(1125q 2

4q

1125q 3 + 336

1226q + 353)

1
9

0:

ng thc xy ra khi v ch khi a = b = c:


V d 1.66 Cho a; b; c l di 3 cnh ca mt tam gic. Chng minh rng
a(b + c)
b(c + a)
c(a + b)
+
+
a2 + 2bc b2 + 2ca c2 + 2ab

2:
(V Quc B Cn)

Li gii. Bt ng thc tng ng vi


X

a(b + c)
a2 + 2bc

X a(a

b) + (2b
a2 + 2bc

cyc

cyc

1
a)c

1.4. THE CYH TECHNIQUES

87

Ta s ch ra rng
a(a
Tht vy, nu 2b
a(a
Nu a

b) + (2b

a)c

a, ta c

b) + (2b

a)c

a(a

b) + (2b

a)(b

b)2

a) = 2(a

2b, ta c
a(a

b) + (2b

a)c

a(a

a)(a + b) = 2b2

b) + (2b

T y, s dng bt ng thc Cauchy Schwarz, ta c


#"
"
#
X a(a b) + (2b a)c X
2
[a(a b) + (2b a)c](a + 2bc)
a2 + 2bc
cyc
cyc
"
#2
X
[a(a b) + (2b a)c]
cyc

Ta cn chng minh
"
X
[a(a b) + (2b

#2

a)c]

cyc

[a(a

b) + (2b

a)c](a2 + 2bc)

cyc

b)2

ab(a

0:

cyc

hin nhin ng. ng thc xy ra khi v ch khi a = b = c hoc a = b; c = 0 hoc


cc hon v tng ng.
V d 1.67 Cho a; b; c l di 3 cnh ca mt tam gic. Chng minh rng
b
c
ab + bc + ca
a
+
+
+
b + c c + a a + b a2 + b2 + c2

5
:
2
(Phm Kim Hng)

Li gii. Bt ng thc tng ng vi


X

cyc

ab
1 cyc
+P 2
2
a

a
b+c

cyc

Xb+c
cyc

b+c

cyc

!2

cyc

a2

88

CHNG 1. TM TI MT S K THUT GII TON

S dng bt ng thc Cauchy Schwarz, ta c


!
!
"
#
!
X
Xb+c a
X
Xb+c a
2
2
a
=
(b + c)(b + c a)
b+c
b+c
cyc
cyc
cyc
cyc
"
#2
!2
X
X
(b + c a) =
a :
cyc

cyc

ng thc xy ra khi v ch khi a = b = c hoc a = b; c = 0 hoc cc hon v tng


ng.
V d 1.68 Cho cc s khng m a; b; c; d; khng c 3 s no ng thi bng 0: Chng
minh rng
ab
bc
cd
da
+
+
+
a+b+c b+c+d c+d+a d+a+b

1
(a + b + c + d):
3
(Park Doo Sung)

Li gii. Bt ng thc tng ng vi


X

4ab
a+b+c

a+b

cyc

X ac + bc + (a

b)2

a+b+c

cyc

2X
a
3 cyc
2X
a
3 cyc

S dng bt ng thc Cauchy Schwarz, ta c


"
#2
P
[ac + bc + (a b)2 ]
X ac + bc + (a b)2
cyc
P
a+b+c
[ac + bc + (a b)2 ](a + b + c)
cyc
cyc

Ta c
X

[ac + bc + (a

cyc

[ac + bc + (a

b)2 ] =

a2 + (a + c)2 + (b + d)2

(a + c)(b + d)

cyc

b)2 ](a + b + c) = 2

cyc

a3 +

cyc

a2 b + 3ac(a + c) + 3bd(b + d)

cyc

a + 3ac(a + c) + 3bd(b + d)

cyc

= 3(a + c)(a2 + c2 ) + 3(b + d)(b2 + d2 )

1.4. THE CYH TECHNIQUES

89

Nn ta ch cn chng minh c
"

#2

a2 + (a + c)2 + (b + d)2

(a + c)(b + d)

cyc

(a + c)(a2 + c2 ) + (b + d)(b2 + d2 )

Khng mt tnh tng qut, ta c th gi s b + d

cyc

a + c. t

2x = a + c
2y = b + d
(a + c)2
= 2x2
2

t = a2 + c2
A = b2 + d2 + (a + c)2 + (b + d)2

(a + c)(b + d) = b2 + d2 + 4x2 + 4y 2

B = (b + d)(b2 + d2 ) = 2y(b2 + d2 )
Bt ng thc tr thnh
f (t) =

(t + A)2
2xt + B

Ta c
f 0 (t) =
v
xt + B

2(2x + 2y)

2(t + A)(xt + B xA)


(2xt + B)2

xA = xt + 2y(b2 + d2 )
3

x(b2 + d2 + 4x2 + 4y 2

2x + 2y(b + d )
2

= (b + d )(2y
2

2y (2y
= 2(y
Nn f 0 (t)

x) + 4x y

x) + 4x y

x)(2y

x(b + d + 4x + 4y

2x

2x
2

xy + x )

4xy

4xy

4xy)
2

4xy)

0. Do f (t) ng bin, v v th ta ch cn chng minh c


f (2x2 ) =

Ta c
g 0 (bd) =

(3x2

(3x2

2xy + 4y 2 bd)2
x3 ybd + 2y 3

4(x + y) = g(bd)

2xy + 4y 2 bd)[ybd x(2x2


(x3 ybd + 2y 3 )2

g 0 (bd) = 0 , bd =

x(2x2

3xy + 2y 2 )]

3xy + 2y 2 )
y

4xy

90
Do x

CHNG 1. TM TI MT S K THUT GII TON


y nn

x(2x2 3xy+2y 2 )
y

g(bd)

y 2 , khi ta d dng kim tra c


x(2x2

3xy + 2y 2 )
y

x)3

4(y
y2

0:

Bt ng thc c chng minh. ng thc xy ra khi v ch khi a = b = c = d:


1
2

V d 1.69 Cho cc s a1 ; a2 ; a3

tha a1 a2 a3 = 1: Chng minh rng

1
1
1
+
+
a1 + 2a2
a2 + 2a3
a3 + 2a1
Li gii. t a1 = xy ; a2 = xz ; a3 =

y
z

1:

vi x; y; z > 0 v do a1 ; a2 ; a3

8
<2x
2y
:
2z

1
2

nn

y
z
x

Bt ng thc cn chng minh tr thnh


X
cyc

8
<2x
Do 2y
:
2z

x2

X 2x2

cyc

xy
+ 2yz

2
3

cyc

x2

xy
x2 + 2yz

3xy + 4yz
+ 2yz

y
z nn
x
2x2

3xy + 4yz

2x2

3xy + 2xy = x(2x

Do , s dng bt ng thc Cauchy Schwarz, ta c


!"
X 2x2 3xy + 4yz
X
(x2 + 2yz)(2x2
2
x
+
2yz
cyc
cyc

X
X
2 x2 +
xy
cyc

y)

3xy + 4yz)
!2

1.4. THE CYH TECHNIQUES

91

Nn ta ch cn chng minh
X
X
2 x2 +
xy
cyc

cyc

!2

X
3 (x2 + 2yz)(2x2

X
X
, 13 x3 y + 4 xy 3
cyc

8
<x
0 v y
:
z
X
Mz (x
cyc

Mz (3a

2b

y)

X
a)2 + 4 Mz (a

Mz (c

cyc

Mz (a

cyc

Mz (x

cyc

c)2

b)2 +

Mz (a

cyc

= 6

cyc

y)2

cyc

= 4

cyc

8
<2x y = 7a
26xy 13x2 v Mx ; My tng t. t 2y z = 7b th ta c
:
2z x = 7c
= 4a + 2b + c
= 4b + 2c + a ; thay vo bt ng thc trn, ta c
= 4c + 2a + b

vi Mz = 5y 2 +

a; b; c

X
X
2 x4 + 15 x2 y 2

cyc

3xy + 4yz)

cyc

X
b)2 + 3 Mz (c

a)2

cyc

(a

b) (3Mx

(a

b)2 (41z 2 + 78yz

b)(a

c)

cyc

Mz (b

c)2

cyc

2My + 6Mz )

cyc

X
cyc

= 343

(a

9y 2

b)2 (143b2 + 180c2

88x2

52zx + 156xy)

127a2 + 254ab + 288bc + 144ca)

cyc

X
X
X
= 1372 4 a4 + 199 a3 b + 28 ab3
cyc

cyc

cyc

S dng bt ng thc AM-GM, ta c


X
4 a4

cyc

X
4abc a

cyc

cyc

X
X
28 a3 b + 28 ab3
cyc

X
33 a2 b2

cyc

X
56 a2 b2
cyc

!
X
199abc a
cyc

92

CHNG 1. TM TI MT S K THUT GII TON


X
171 a3 b

X
171abc a

X
23abc a

cyc

23

a2 b2

cyc

cyc

cyc

Cng ln lt v vi v 4 bt ng thc ny, ta thu c


4

X
cyc

X
X
a4 + 199 a3 b + 28 ab3
cyc

33

cyc

X
199abc a

a2 b2

cyc

0:

cyc

Bt ng thc c chng minh xong. ng thc xy ra khi v ch khi a1 = a2 =


a3 = 1:
i khi, ta khng nht thit phi thm vo chuyn t s thnh s khng m v
n l mt s khng m nhng "kh ln", ta s s bt i 1 vi gi tr bin n
thnh s khng m nhng c gi tr "ln va phi"

V d 1.70 Cho cc s khng m a; b; c, khng c 2 s no ng thi bng 0: Chng


minh rng
1
1
1
+
+
a2 + bc b2 + ca c2 + ab

3(a + b + c)2
:
2(a2 + b2 + c2 )(ab + bc + ca)
(Phm Hu c)

Li gii. Bt ng thc tng ng vi

X a2 + b2 + c2
cyc

,3+

cyc

a2 + bc

X b2 + c2
cyc

a2 + bc

!2

ab

cyc

3
bc

cyc

cyc

!2

ab

1.4. THE CYH TECHNIQUES

93

S dng bt ng thc Cauchy Schwarz, ta c


#2
"
P 2
2
(b + c
bc)
X b2 + c2 bc
cyc
P 2
a2 + bc
(b + c2 bc)(a2 + bc)
cyc
cyc

2
=

Do , ta ch cn chng minh c
2
3+

ab

cyc

cyc

a2

cyc

cyc

a2 +

cyc

ab

cyc

ab

ab

cyc

!2

ab

!2

cyc

ab

cyc

cyc

4abc

!2

cyc

4abc

2
a

cyc

3
P

ab

cyc

cyc

Do tnh thun nht, ta c th chun ha cho a+b+c = 1. t q = ab+bc+ca; r = abc,


khi bt ng thc tr thnh
3+
Nu 1

5q)2
4r

3
2q

q2

4q, ta c
3+

Nu 4q

(2

(2
q

5q)2
q 2 4r

3
2q

3+

(2
q

5q)2
q2

3
(5 11q)(1 4q)
=
2q
2q(1 q)

1, s dng bt ng thc Schur bc 3, ta c r


3+

(2
q

5q)2
q 2 4r

3
2q

3+
=

3(1

(2
q

q2

4q 1
9 ,

5q)2
4(4q 1)
9

do

3
2q

3q)(4q 1)(11 4q)


2q(4 7q 9q 2 )

0:

Bt ng thc c chng minh. ng thc xy ra khi v ch khi a = b = c hoc


a = b; c = 0 hoc cc hon v tng ng.
V d 1.71 Cho cc s khng m a; b; c; khng c 2 s no ng thi bng 0: Chng
minh rng
5a2

1
+ 2
ab + 5b2
5b

1
+ 2
bc + 5c2
5c

1
ca + 5a2

a2

1
:
+ b2 + c2

94

CHNG 1. TM TI MT S K THUT GII TON


(Vasile Cirtoaje)

Hng dn.

5(a2 + b2 + c2 )
5a2 ab + 5b2

1=

5c2 + ab
ab + 5b2

5a2

0:

V d 1.72 Cho cc s khng m a; b; c tha mn a2 + b2 + c2 = 1: Chng minh rng


bc
ca
ab
+ 2
+ 2
+1 b +1 c +1

a2

3
:
4
(Phm Kim Hng)

Hng dn.
1

1 + a2 2bc
3a2 + (b c)2
2bc
=
=
a2 + 1
a2 + 1
a2 + 1

0:

V d 1.73 Cho cc s thc a; b; c tha mn a2 + b2 + c2 = 1: Chng minh rng


1
3 + a2

2bc

1
1
+
3 + b2 2ca 3 + c2 2ab

9
:
8

(Vasile Cirtoaje, Wolfgang Berndt)


Hng dn.
1

2
3 + a2

2bc

1 + a2
3 + a2

2bc
3a2 + (b c)2
=
2bc
3 + a2 2bc

0:

V d 1.74 Cho cc s khng m a; b; c; d; khng c 3 s no ng thi bng 0: Chng


minh rng
a b
b c
c d
d a
+
+
+
a + 2b + c b + 2c + d c + 2d + a d + 2a + b
Hng dn.

2(a b)
3a + c
+1=
a + 2b + c
a + 2b + c

0:

0:

V d 1.75 Cho cc s khng m a; b; c; khng c 2 s no ng thi bng 0: Chng


minh rng
a2 bc
b2 ca
c2 ab
+
+
2b2 3bc + 2c2
2c2 3ca + 2a2
2a2 3ab + 2b2

0:
(Vasile Cirtoaje)

1.4. THE CYH TECHNIQUES


Hng dn.

95

a2 + 2(b c)2
a2 bc
+
1
=
2b2 3bc + 2c2
2b2 3bc + 2c2

0:

i xng ha
Trong ton hc, bao gi cng c mt quan nim l i xng d gii hn hon v. K
thut ny cng da trn nn tng , t mt bt ng thc cha i xng, chng ta
s tm cch s dng bt ng thc Cauchy Schwarz hoc Holder a n tr v i
xng, ri gii.
V d 1.76 Cho cc s khng m a; b; c; khng c 2 s no ng thi bng 0: Chng
minh rng
r
r
r
a
b
c
3
p :
+
+
a+b
b+c
c+a
2
(Vasile Cirtoaje)
Li gii. S dng bt ng thc Cauchy Schwarz, ta c
Xr a
a+b
cyc

!2

"

X
cyc

#"

(a + c)

X
cyc

Suy ra

#
4
a
ab
cyc
cyc
a
=
(a + b)(a + c)
(a + b)(b + c)(c + a)

Mt khc, theo bt ng thc AM-GM th


!
!
X
X
(a + b)(b + c)(c + a) =
a
ab
cyc

abc

cyc

Xr a
a+b
cyc

!2

8
9

X
cyc

X
cyc

ab

9
:
2

ng thc xy ra khi v ch khi a = b = c:


V d 1.77 Cho cc s dng a; b; c: Chng minh rng
r
r
r
a
b
c
+
+
4a + 4b + c
4b + 4c + a
4c + 4a + b

1:
(Phm Kim Hng)

96

CHNG 1. TM TI MT S K THUT GII TON

Li gii. S dng bt ng thc Cauchy Schwarz, ta c


Xr
cyc

a
4a + 4b + c

!2

"

X
cyc

cyc

cyc

cyc

cyc

(4a + 4b + c)(4b + 4c + a)(4c + 4a + b)

Ta cn chng minh
!
!
X
X
X
2
9
a
a +8
ab
cyc

a
(4a + 4b + c)(4a + b + 4c)
!
P 2
P
a + 8 ab

(4a + b + 4c)

9
=

(4a + 4b + c)(4b + 4c + a)(4c + 4a + b)

cyc

,7

a3 + 3

cyc

ab(a + b)

39abc:

cyc

Bt ng thc ny hin nhin ng theo bt ng thc AM-GM. ng thc xy ra


khi v ch khi a = b = c:
V d 1.78 Cho cc s khng m a; b; c; d; khng c 3 s no ng thi bng 0: Chng
minh rng
r
r
r
r
a
b
c
d
4
p :
+
+
+
a+b+c
b+c+d
c+d+a
d+a+b
3
(Phm Vn Thun)
Li gii. S dng bt ng thc Cauchy Schwarz, ta c
Xr
cyc

"

a
a+b+c

!2

#"

(a + b + d)(a + c + d)

cyc

X
cyc

a
(a + b + c)(a + b + d)(a + c + d)

2[2(a + b + c + d) + (a + c)(b + d)][(a + c)(b + d) + ac + bd]


(a + b + c)(b + c + d)(c + d + a)(d + a + b)

Ta cn chng minh
P (a; b; c; d) = 8(a + b + c)(b + c + d)(c + d + a)(d + a + b)
[2(a + b + c + d)2 + (a + c)(b + d)][(a + c)(b + d) + ac + bd]

1.4. THE CYH TECHNIQUES

97

y l mt hm bc nht theo ac; bd, nn ta d dng kim tra c


P (a; b; c; d)

P (a + c; b; 0; d)
P

a+c
a+c
; b;
;d
2
2

min P (a + c; b; 0; d); P

min P (a + c; b + d; 0; 0); P

a+c
a+c
; b;
;d
2
2
a+c
a+c
; b + d;
;0 ;P
min P
2
2

a + c;

b+d
b+d
; 0;
2
2

a+c b+d a+c b+d


;
;
;
2
2
2
2

Do , ta ch cn xt cc trng hp sau l
Trng hp 1. c = d = 0, bt ng thc tr thnh
8ab(a + b)2

3ab(2a2 + 5ab + 2b2 )

, ab(2a2 + ab + 2b2 )

hin nhin ng.


Trng hp 2. a = c; d = 0, bt ng thc tr thnh
16a(2a + b)(a + b)2

6a(a + b)(a + 2b)(4a + b)

, 2a(a + b)(4a2

3ab + 2b2 )

hin nhin ng.


Trng hp 3. a = c; b = d, bt ng thc tr thnh
8(2a + b)2 (a + 2b)2

12(2a2 + 5ab + 2b2 )(a2 + 4ab + b2 )

, 4(a + 2b)(2a + b)(a

b)2

hin nhin ng.


ng thc xy ra khi v chi khi a = b = c = d:
V d 1.79 Cho cc s khng m a; b; c; khng c 2 s no ng thi bng 0: Chng
minh rng
r
r
r
a2
b2
c2
+
+
1:
4a2 + ab + 4b2
4b2 + bc + 4c2
4c2 + ca + 4a2
(Bin Zhao)

98

CHNG 1. TM TI MT S K THUT GII TON

Li gii. S dng bt ng thc Cauchy Schwarz, ta c


X
cyc

"

a2
4a2 + ab + 4b2

!2

#"

(4a2 + ac + 4c2 )

cyc

Ta cn chng minh
"
#"
X
X
2
2
(4a + ac + 4c )
cyc

cyc

X
cyc

a2
(4a2 + ab + 4b2 )(4a2 + ac + 4c2 )

a2
(4a2 + ab + 4b2 )(4a2 + ac + 4c2 )

Bng khai trin trc tip, ta thy bt ng thc ny tng ng vi


X
X
X
8
a3 b3 + 8
a4 bc + 3abc
ab(a + b) 66a2 b2 c2 :
cyc

cyc

cyc

hin nhin ng theo bt ng thc AM-GM. ng thc xy ra khi v ch khi a =


b = c hoc c = 0; ab ! 0 hoc cc hon v tng ng.
i bin c th s dng bt ng thc Cauchy Schwarz-Holder
Bt ng thc c rt nhiu nt l v c o. Mt bt ng thc dng ny, ta
khng th s dng bt ng thc Cauchy Schwarz-Holder gii m khi i bin th
li gii c bng chng! iu ny cng gp phn to nn v p li cun ca bt
ng thc.
V d 1.80 Cho cc s dng a; b; c: Chng minh rng
a3
b3
c3
+
+
a3 + abc + b3
b3 + abc + c3
c3 + abc + a3

1:
(Nguyn Vn Thch)

Li gii. t x = ab ; y = ac ; z =

c
b

) x; y; z > 0; xyz = 1 v bt ng thc tr thnh

x3 +

1
x3 + x2 z + 1

cyc

cyc

1
x
y

+1

1.4. THE CYH TECHNIQUES


,

99
X
cyc

x2

yz
+ yz + zx

S dng bt ng thc Cauchy Schwarz, ta c

X
cyc

yz
2
x + yz + zx

cyc

yz

!2

yz(x2 + yz + zx)

= 1:

cyc

ng tc xy ra khi v ch khi a = b = c hoc c ! 0; ab ! 0 hoc cc hon v tng


ng.
V d 1.81 Cho cc s dng a; b; c: Chng minh rng
r
r
r
a2
b2
c2
+
+
2
2
2
2
2
a + 7ab + b
b + 7bc + c
c + 7ca + a2

1:

(L Hu in Khu)
Li gii. t x = ab ; y = cb ; z =

a
c

) x; y; z > 0; xyz = 1 v bt ng thc tr thnh


p

cyc

1
x2 + 7x + 1

Li do x; y; z > 0; xyz = 1 nn tn ti m; n; p > 0 sao cho x =


m2 n2
p4 , bt ng thc c vit li thnh
X
cyc

m4
p
m8 + 7m4 n2 p2 + n4 p4

n2 p2
m4 ; y

p2 m2
n4 ; z

S dng bt ng thc Holder, ta c


!2
!
X
X
m4
p
m(m8 + 7m4 n2 p2 + n4 p4 )
8 + 7m4 n2 p2 + n4 p4
m
cyc
cyc

X
cyc

m3

!3

Ta cn chng minh

X
cyc

sym

m3

!3

(5m6 n3 + 2m3 n3 p3

m(m8 + 7m4 n2 p2 + n4 p4 )

cyc

7m5 n2 p2 ) +

(m6 n3

m4 n4 p)

0:

sym

Bt ng thc cui hin nhin ng theo bt ng thc AM-GM. ng thc xy ra


khi v ch khi a = b = c hoc c ! 0; ab ! 0 hoc cc hon v tng ng.

100

CHNG 1. TM TI MT S K THUT GII TON

V d 1.82 Cho cc s dng a; b; c: Chng minh rng


1
1
1
+
+
a(a + b) b(b + c) c(c + a)

p
3

2 a2 b2 c2

Li gii. Do tnh thun nht, ta c th chun ha cho abc = 1, khi tn ti


x; y; z > 0 sao cho a = xy ; b = xz ; c = yz . Bt ng thc tr thnh
X
cyc

x2

y2
+ yz

3
2

S dng bt ng thc Cauchy Schwarz, ta c

y2
2
x + yz

!2

y3 z =

cyc

Mt khc
X

y2

cyc

X
cyc

y2

!2

cyc

x2 y 2 +

cyc

!2

y3 z

cyc

x2 y 2 =

cyc

cyc

1X 2
(x
2 cyc

1X 2
(x
2 cyc
z2

y 2 )2

2xy + yz + zx)2

Do

X
cyc

y2
2
x + yz

cyc

x2 y 2 +

cyc

!2

cyc

y3 z

cyc

1
3

cyc

y2

!2

!2

1
3

cyc

y2

!2 =

3
:
2

ng thc xy ra khi v ch khi a = b = c:


Nhn xt 9 Chng ta thng t a = xy ; b = yz ; c = xz nhng y ta li t
a = xy ; b = xz ; c = yz . Tht ra c 2 php t trn l tng ng nhng chng ta nn
c gng t lm sao biu thc trn t cng c lp vi cc bin khc th cng d nhn
hn, s thun li hn cho chng ta khi gii ton. C th, nu t a = xy ; b = yz ; c = xz
P y2 z
3
th bt ng thc tr thnh
x(y 2 +zx)
2 : R rng l kh nhn hn php t kia.
cyc

1.4. THE CYH TECHNIQUES

101

V d 1.83 Cho cc s dng a; b; c: Chng minh rng


1
1
1
p
+ p
+ p
a a+b b b+c c c+a

3
:
2abc
(Phan Thnh Nam)

Li gii. Do tnh thun nht, ta c th chun ha cho abc = 1, khi tn ti


x; y; z > 0 sao cho a = xy ; b = xz ; c = yz . Bt ng thc tr thnh
X
cyc

p
y y
p
x(x2 + yz)

3
p
2

S dng bt ng thc Cauchy Schwarz, ta c

X
cyc

v
Xp
xy(x2 + yz)
cyc

Suy ra

p
y y

p
x(x2 + yz)

Pp

cyc

!2

xy(x2 + yz)

cyc

v
!
!
u
X
X
u X
t
xy
x2 +
xy
cyc

cyc

cyc

v
!
!
u
X
X
X
1 u
xy
x2 +
xy
= p t 2
2
cyc
cyc
cyc
!
!2
X
X
X
1
2
2
p
p
x +3
xy
x
2 2 cyc
3 2 cyc
cyc
X
cyc

p
y y

x(x2

+ yz)

3
p :
2

ng thc xy ra khi v ch khi a = b = c:


V d 1.84 Cho cc s dng a; b; c tha mn abc = 1: Chng minh rng
a2

1
+
a + 1 b2

1
+
b + 1 c2

1
c+1

3:
(V nh Qu)

102

CHNG 1. TM TI MT S K THUT GII TON

Li gii. Trc ht, ta s chng minh rng vi mi x; y; z > 0 tha mn xyz = 1 th


x2

1
1
1
+ 2
+ 2
+x+1 y +y+1 z +z+1

Tht vy, do x; y; z > 0; xyz = 1 nn tn ti cc s m; n; p > 0 sao cho x =


pm
mn
n2 ; z = p2 . Bt ng thc trn c vit li l
X
cyc

m4
m4 + m2 np + n2 p2

np
m2 ; y

T
sPdng bt ng thc Cauchy Schwarz v bt ng thc quen thuc
P y,
n 2 p2
m2 np, ta c
cyc

cyc

X
cyc

m4
4
2
m + m np + n2 p2

cyc

!2

(m4 + m2 np + n2 p2 )

cyc

cyc

m4 + 2

cyc

!2

n 2 p2

cyc

Tr li bi ton ban u, s dng bt ng thc trn ta c


X

1
cyc a4

,
,
,
,

1
a2

a4
+ a2 + 1

X 2(a2 + 1)
a4 + a2 + 1
cyc

cyc

a4

X (a2 + a + 1) + (a2
cyc

X
cyc

(a2

a2

+1

+a+

1)(a2

a + 1)
a + 1)

X
1
+
+ a + 1 cyc a2

Li s dng bt ng thc trn mt ln na, ta c

1
4
a+1
P
1
cyc

X
cyc

a2

1
a+1

ng thc xy ra khi v ch khi a = b = c = 1:

3:

a2 +a+1

1, v v th

=1

1.4. THE CYH TECHNIQUES

103

V d 1.85 Cho cc s dng x; y; z: Chng minh rng


y
z
x
+
+
y(x2 + 2y 2 ) z(y 2 + 2z 2 ) x(z 2 + 2x2 )

3
:
xy + yz + zx
(Dng c Lm)

Li gii. t a = x1 ; b = y1 ; c =

1
z

th bt ng thc tr thnh

ab3
2a2 + b2
cyc

3abc
a+b+c

b2
c(2a2 + b2 )
cyc

3
a+b+c

S dng bt ng thc Cauchy Schwarz, ta c


"
#"
#
X
X
b2
2
2
2
b c(2a + b )
c(2a2 + b2 )
cyc
cyc
Nn ta ch cn chng minh c
!2
X
2
a

cyc

a5 +

cyc

X
cyc

cyc

cyc

ab +

cyc

X
cyc

cyc

a b

cyc

3 2

a b +

2 3

(ab + c a )

cyc

2 3

a b

cyc

X
X
2 a4 b + 6 a2 b2 c

cyc

cyc

2 3

X
cyc

cyc

a (b + c )

cyc

a4 b

cyc

X
2 a2 b2 c
cyc

cyc

!2

cyc

X
X
ab4 + 2 a3 b2 + 2 a2 b3

cyc

b2 c(2a2 + b2 )

S dng bt ng thc AM-GM, ta c


X
X
X
a5 +
a3 b2 =
a3 (a2 + b2 )
X

X
2 a3 bc
cyc

X
2 a2 b2 c
cyc

Cng tng ng v vi v 3 bt ng thc trn, ta suy ra c pcm. ng thc xy


ra khi v ch khi x = y = z:
V d 1.86 Cho cc s dng x; y; z: Chng minh rng
1
1 1
+ +
x y z

x2

x
y
z
+ 2
+ 2
+ 2yz
y + 2zx z + 2xy

:
(Dng c Lm)

104

CHNG 1. TM TI MT S K THUT GII TON

Li gii. t a = x1 ; b = y1 ; c =
a+b+c
,

3a

3abc
2a2 + bc
,3

+ 3b

1
z

th bt ng thc tr thnh

3abc
3abc
3abc
+
+
2a2 + bc 2b2 + ca 2c2 + ab
3abc
2b2 + ca

+ 3c

3abc
2c2 + ab

a3
b3
c3
+ 2
+ 2
+ bc 2b + ca 2c + ab

2(a + b + c)

a+b+c

2a2

S dng bt ng thc Cauchy Schwarz, ta c


a3
b3
c3
+
+
2a2 + bc 2b2 + ca 2c2 + ab
=

(a2 + b2 + c2 )2
a(2a2 + bc) + b(2b2 + ca) + c(2c2 + ab)
(a2 + b2 + c2 )2
2(a3 + b3 + c3 ) + 3abc

Ta cn chng minh
3(a2 + b2 + c2 )2

(a + b + c)[2(a3 + b3 + c3 ) + 3abc]

Chun ha cho a + b + c = 1: t q = ab + bc + ca; r = abc th bt ng thc tr


thnh
3(1 2q)2 2(1 3q + 3r) + 3r
, (1

3q)2 + 3(q 2

3r)

0:

Bt ng thc cui hin nhin ng nn ta c pcm. ng thc xy ra khi v ch khi


x = y = z:
V d 1.87 Cho cc s khng m x; y; z; khng c 2 s no ng thi bng 0: Chng
minh rng
r
r
r
r
p
x
y
z
xyz
+
+
+2 2 2 3
2:
y+z
z+x
x+y
(x + y)(y + z)(z + x)
(Dng c Lm)
Li gii. t x = a2 ; y = b2 ; z = c2 (a; b; c

0), ta phi chng minh


p
X
2 2 2 3 abc
a
p
+p
2
2
2
2
b +c
(a + b2 )(b2 + c2 )(c2 + a2 )
cyc

S dng bt ng thc Holder, ta c


!2 "
#
X
X
a
2
2
p
a(b + c )
b2 + c2
cyc
cyc

X
cyc

!3

1.4. THE CYH TECHNIQUES

105

cyc

v
u
!3
u
P
u
a
u
u
u P cyc
t a(b2 + c2 )

a
+ c2

b2

cyc

Mt khc, theo bt ng thc Cauchy Schwarz,


X

cyc

v
X

b2

cyc

Suy ra
2

cyc

2 2

a b

cyc

a2 b2

cyc

2 2

a b

cyc

!2

ab2

!2

a b

cyc

X
cyc

a b

cyc

1
2

!2

a b+

cyc

ab

cyc

ab

cyc

!2

!2

T , bt ng thc AM-GM cho ta


2

(a + b )(b + c )(c + a )

8
9

X
cyc

2 2

a b

cyc

"
#2
2 X 2
a (b + c)
9 cyc

Ta cn chng minh
v
u
!3
u
P
u
a
u
p
u
2 2 2 3 abc
u P cyc
t a(b2 + c2 ) + v
#2
u "
u P
cyc
t2
a2 (b + c)
9

cyc

Do tnh thun nht, ta c th chun ha cho a + b + c = 1. t q =

ab; r = abc,

cyc

khi theo bt ng thc Schur bc 3 v bt ng thc Newton, ta c

q2
3

106

CHNG 1. TM TI MT S K THUT GII TON

max 0; 4q9

. Bt ng thc trn tr thnh


p

,q

3r

,
2q

p
3 4 3 2 r
+
q 3r
3r

1
q

p
q

3r
p

9 2

h
, f (r) = 2q

2q
2 r

9 2

9 2

i2

2
2 r

(v 2q
2 r

i2

+ 3r

Do f (r) l hm li nn
n
2
o
2
f (r) max f q3 ; f (0)
4
n
o
2
f (r) max f q3 ; f 4q9 1

Ta c

f
Nu 1

q2
3

p
2 2 q(3q

= 3

4q, ta c

1)

0)

nu 1

4q

nu 4q

1
i
2q)

1, ta c
4q

f
Suy ra, nu 1

1
9

17

p
12 2
(4q
3

1)(3q

1)

4q, th
f (r)

Nu 4q

2 r

h
p
1) 6q 2 + 3 + 2 2 (1

f (0) = q(4q
Nu 4q

9 2

max f

q2
3

q2
3

;f

; f (0)

1, th
f (r)

max f

4q

1
9

0:

Bt ng thc c chng minh. ng thc xy ra khi v ch khi x = y = z hoc


x = y; z = 0 hoc cc hon v tng ng.
Nhn xt 10 Mt iu rt l l vi bi ton ny, nu ta dng Holder trc tip
!2 "
#
!3
Xr x
X
X
2
x (y + z)
x
y+z
cyc
cyc
cyc

1.4. THE CYH TECHNIQUES

107

Ri i n chng minh kt qu
v
u
!3
u
P
u
x
u
u
p
u P cyc
t x2 (y + z) + 2 2 2

cyc

xyz
(x + y)(y + z)(z + x)

nhng bt ng thc ny li khng ng.


Th nhng sau khi ta dng php t n x = a2 ; y = b2 ; z = c2 th ta li c th p
dng Holder mt cch kh hiu qu.

V d 1.88 Cho cc s x; y; z
p

2x2

0; xyz = 1: Chng minh rng

1
1
1
+p
+p
2
2
+ 6x + 1
2z + 6z + 1
2y + 6y + 1

1:

(Nguyn Vn Thch)

Hng dn. t x =

bc
a2 ; y

X
cyc

ca
b2 ; z

ab
c2

(a; b; c

0); bt ng thc tr thnh

a2
a4 + 6a2 bc + 2b2 c2

S dng bt ng thc Holder

X
cyc

a2
p
a4 + 6a2 bc + 2b2 c2

!2

cyc

!3

a2 (a4 + 6a2 bc + 2b2 c2 )

cyc

V d 1.89 Cho cc s a; b; c; d > 0: Chng minh rng


X

abc
(a
+
d)(b
+ d)(c + d)
cyc

1
:
2
(Nguyn Vn Thch)

Hng dn. t a = x1 ; b = y1 ; c = z1 ; d = 1t ; bt ng thc tr thnh


X
cyc

x3
(x + y)(x + z)(x + t)

1
2

108

CHNG 1. TM TI MT S K THUT GII TON

S dng bt ng thc Cauchy Schwarz

X
cyc

x3
(x + y)(x + z)(x + t)

x2

cyc

!2

x(x + y)(x + z)(x + t)

cyc

V d 1.90 Cho cc s x; y; z; k > 0; xyz = 1: Chng minh rng


r
r
r
3
x
y
z
4
4
p
+
+ 4
:
4
y+k
z+k
x+k
k+1
(V Quc B Cn)
Hng dn. t x =

a5
b5 ; y

X
cyc

c5
a5 ; z

b5
c5

(a; b; c > 0); bt ng thc tr thnh

a5=2
p
b5=4 4 c5 + ka5

p
4

3
k+1

S dng bt ng thc Holder


X
cyc

a5=2
p
4 5
5=4
b
c + ka5

!4 "

cyc

+3

X
cyc

(c + ka )

cyc

Hy chng minh b
X a2

X a2
cyc

!5

P
15 a2
cyc
P
:
a
cyc

The CYH technique


K thut CYH3 l k thut quan trng nht m chng ti mun gii thiu n cc
bn trong bi vit ny. N ch yu c dng gii cc dng ton cn thc, mt
dng ton rt kh gii. tng ca n cng ging nh k thut tham s ha nhng
thay v chn nhng tham s c nh, ta s chn nhng tham s chy. tng ca
k thut ny, chng ti xut pht t vic gii bi ton sau ca Jack Garfunkel, mt
nh ton hc M, ng l tc gi ca nhiu bt ng thc kh m hin nay vn cha
c li gii
3 Cn

yu Hng

1.4. THE CYH TECHNIQUES

109

V d 1.91 Cho cc s khng m a; b; c; khng c 2 s no ng thi bng 0: Chng


minh rng
b
c
5p
a
p
+p
+p
a + b + c:
4
c
+
a
a+b
b+c
(Jack Garfunkel)
Li gii. S dng bt ng thc Cauchy Schwarz, ta c
!2 "
#"
#
X
X
X
a
a
p
a(5a + b + 9c)
(a + b)(5a + b + 9c)
a+b
cyc
cyc
cyc
!2 "
#
X
X
a
= 5
a
(a + b)(5a + b + 9c)
cyc
cyc
Ta cn chng minh
X
cyc

!"

X
cyc

a
(a + b)(5a + b + 9c)

5
16

Nhng bt ng thc ny hin nhin ng v


!"
#
X
X
5
a
A+B
a
=
16
(a
+
b)(5a
+
b
+
9c)
C
cyc
cyc
trong
A =

X
cyc

B = 243

ab(a + b)(a + 9b)(a


X
cyc

a3 b2 c + 835

X
cyc

3b)2

a2 b3 c + 232

a4 bc + 1230a2 b2 c2

cyc

C = 16(a + b)(b + c)(c + a)(5a + b + 9c)(5b + c + 9a)(5c + a + 9b) > 0:


ng thc xy ra khi v ch khi a3 = 1b = 0c hoc cc hon v tng ng.
Chng ta hy phn tch li gii trn, tng cng ging nh k thut tham s ha,
chng ta s thm vo cc b s c dng ma + nb + pc; mb + nc + pa; mc + na + pb vi
m; n; p 0 khi s dng bt ng thc Cauchy Schwarz
#
!2 "
#"
X
X
X
a
a
p
a(ma + nb + pc)
(a + b)(ma + nb + pc)
a+b
cyc
cyc
cyc
Ch l bt ng thc ban u c ng thc xy ra ti im a = 3; b = 1; c = 0
(chng ta bit c iu ny l do nh chng ti ni, i vi mt bt ng thc

110

CHNG 1. TM TI MT S K THUT GII TON

hon v vng quanh th gn nh im nhy cm lun c dng (x; y; 0)). Do bt


ng thc Cauchy Schwarz m ta p dng trn cng phi xy ra ng thc ti y,
ngoi ra nh bit l ng thc xy ra bt ng thc Cauchy Schwarz xy ra khi
v ch khi
p
p
p
a(ma + nb + pc)
b(mb + nc + pa)
c(mc + na + pb)
q
= q
= q
a
(a+b)(ma+nb+pc)

c
(c+a)(mc+na+pb)

b
(b+c)(mb+nc+pa)

Mc tiu ca chng l chn cc s m; n; p sao cho im (3; 1; 0) tha mn phng


trnh ny, tc l
p

3 (3 m + 1 n + 0 p)
q
=
3
(3+1)(3 m+1 n+0 p)

p
p

1 (1 m + 0 n + 3 p)
q
1
(1+0)(1 m+0 n+3 p)

0 (0 m + 3 n + 1 p)
q
0
(0+3)(0 m+3n+1
_
p)

, 2(3m + n) = m + 3p

Hn na, t dng

, 5m + 2n = 3p
!
P 2
P
P
m a + (n + p) ab
cyc

mt nhn xt l nu biu thc m

cyc

cyc

cyc

a + (n + p)

a
(a+b)(5a+b+9c)

cyc

, chng ta rt ra

ab c dng k

cyc

!2

th bt

ng thc sau khi s dng Cauchy Schwarz s d chng minh hn, t ta rt ra


tng l chn m; n; p sao cho 2m = n + p: T y, kt hp vi phng trnh trn,
ta rt ra c m = 95 p; n = 19 p; t ta chn c m = 5; n = 1; p = 9.
Nhn xt 11 Chng ta khng th dng k thut tham s ha y c v sau khi
s dng Cauchy Schwarz-Holder xong th bt ng thc khng cn i xng hay hon
v g c, bt ng thc s cng kh chng minh hn. tng tham s chy c xut
pht t y.
V d 1.92 Cho cc s khng m a; b; c; khng c 2 s no ng thi bng 0: Chng
minh rng
1
1
1
9
(ab + bc + ca)
+
+
:
2
2
2
(b + c)
(c + a)
(a + b)
4
(Iran MO 1996, Ji Chen)

1.4. THE CYH TECHNIQUES

111

Li gii. iu
P 1g gi m cho ta vic s dng Cauchy Schwarz bi ny? Chnh l
biu thc
(b+c)2 ; n c dng tng ca cc bnh phng nn ta hy th gii bi
cyc

ton ny bng Cauchy Schwarz xem sao


"
#"
#
X
X
1
2
(ma + nb + nc)
(b + c)2
cyc
cyc

X ma + nb + nc
b+c

cyc

!2

ng thc xy ra khi v ch khi


ma + nb + nc
1
b+c

mb + nc + na
1
c+a

mc + na + nb
1
a+b

bt ng thc ban u, ta thy ng thc xy ra khi a = b = c = 1 v a = b =


1; c = 0 nn phi chn $m,n$ sao cho phng trnh trn cng tha mn c iu
ny. Hin nhin l n tha khi a = b = c = 1 nn ta ch cn xt im th 2 l (1; 1; 0),
ta phi c
m+n
m+n
2n
=
= 1 , m = 3n ) m = 3; n = 1
1
1
2
T y, li gii ca ta cho bt ng thc trn nh sau
S dng bt ng thc Cauchy Schwarz, ta c
#
"
#"
#
!"
X
X
X
X
X
1
1
=
(3a + b + c)2
11
a2 + 14
ab
2
(b
+
c)
(b + c)2
cyc
cyc
cyc
cyc
cyc
!2
X 3a + b + c
b+c
cyc
!2
X a
= 9 1+
b+c
cyc
Ta cn chng minh
4 1+

X
cyc

a
b+c

!2

11

P
a2 + 14 ab
cyc
cyc
P
ab
cyc

Do tnh thun nht, ta c th chun ha cho a + b + c = 1, t q =

ab; r = abc,

cyc

khi theo bt ng thc Schur bc 3, ta c r max 0; 4q9


thnh
2
1+q
11 8q
4
2
q r
q

. Bt ng thc tr

112

CHNG 1. TM TI MT S K THUT GII TON

Nu 1
4

4q th
1+q
q r

Nu 4q

11

8q

1+q
q

11

8q
q

(4

3q)(1
q2

4q)

1 th

1+q
q r

11

8q

q
=

(1

1+q
q

4q 1
9

!2

11

8q
q

3q)(4q 1)(11
q(5q + 1)2

17q)

0:

ng thc xy ra khi v ch khi a = b = c hoc a = b; c = 0 hoc cc hon v tng


ng.
Nhn xt 12 Bt ng thc trn l mt bt ng thc rt ni ting v v p cng
nh kh ca n. Hin nay c rt nhiu li gii cho n nhng li gii bng cch s
dng Cauchy Schwarz nh th ny th cha c cp trong bt c ti liu no c.
Cc bn thy khng, ci n gin nht nhng li c th tr thnh ci mnh m nht!

V d 1.93 Cho cc s khng m a; b; c tha mn a + b + c = 1: Chng minh rng


1
q
a + (b

c)2
4

+q

b+

(c a)2
4

1
+q
2
c + (a 4b)

5:

(Walther Janous)
Li gii. Mc tiu ln nht ca ta khi gii bt ng thc cha cn lun l tm cch
loi b cn thc, v trong bi ny cng vy. V ta ny sinh tng bnh phng 2
v, ri s dng bt ng thc Holder nh sau
2
X
1
4
q
cyc
a + (b

c)2
4

32 "
5

(ma + nb + nc)

a+

cyc

"

X
cyc

#3

(ma + nb + nc)

c)2

(b
4

= (m + 2n)3

1.4. THE CYH TECHNIQUES


ng thc xy ra khi v ch khi
h
(ma + nb + nc)3 a +
q

113

(b c)2
4

a+

(b

c)2
4

h
(mb + nc + na)3 b +

(a b)2
4

b+

(c

a)2
4

h
(mc + na + nb)3 c +

(c a)2
4

c+

(a

b)2
4

Ngoi ra, ng thc bi ton ban u xy ra khi a = 1; b = c = 0 (i vi cc bi


ton i xng, thng thng chng ta c 2 im nhy cm l (x; x; y) v (x; y; 0), cc
bn hy xt th 2 trng hp ny th s tm c ng thc nh trn) nn ta phi
chn m; n; p sao cho im (1; 0; 0) tha mn phng trnh trn, tc l
n3 1
n3 1
m3
= 1 4 = 1 4
p1
p1
1
4

, 2m = n ) m = 1; n = 2
V li gii ca ta nh sau
2
X
1
4
q
cyc
a + (b

Ta cn chng minh
5

c)2
4

32 "
5

(a + 2b + 2c)

"

#3

(a + 2b + 2c)

cyc

(a + 2b + 2c)3 a +

cyc

t q = ab + bc + ca; r = abc, khi ta c q 2


3q 2
4

5q

a+

cyc

c)2

(b
4

= 125

c)2

(b
4

3r. Bt ng thc tr thnh

3r 11

q
4

Ta c
5q

3q 2
4

3r 11

q
4

5q

3q 2
4

q 2 11

q
1
= q(20
4
4

47q + q 2 )

ng thc xy ra khi v ch khi a = 1; b = c = 0 hoc cc hon v tng ng.

0:

114

CHNG 1. TM TI MT S K THUT GII TON

Nhn xt 13 Vi cc bi ton dng cn thc th ny, ta khng bit nn bt u t


u gii chng nhng t by gi vi k thut ny, chng ta hon ton c th c t
tin gii chng!

V d 1.94 Cho cc s x; y; z > 1; x1 +


p

1+

1
y

1+

+
p

1
z

= 2: Chng minh rng


p

x + y + z:

Li gii. Vi bi ton ny, thng thng chng ta s p dng Cauchy Schwarz theo
li t nhin l
p
p
p
p
x 1+ y 1+ z 1
3(x + y + z 3)
Ri i n vic chng minh
p
3(x + y + z

3)

,x+y+z

x+y+z

9
:
2

Nhng bt ng thc ny li ngc chiu v cng theo bt ng thc Cauchy Schwarz,


ta c
9
9
x+y+z
1
1
1 = 2:
+
+
x
y
z
Do li i ny khng c hiu qu, chng ta ny sinh tng thm cc tham s vo
s dng bt ng thc Cauchy Schwarz nh sau
r
r
r
p
p
p
x 1
y 1
z 1
x 1+ y 1+ z 1 =
a
+ b
+ c
a
b
c
s
x 1 y 1 z 1
(a + b + c)
+
+
a
b
c
T y, nu ta n iu kin bi ton mt t, ta c th chn c
y; c = z v khi
s
s
x 1 y 1 z 1
x 1 y
(a + b + c)
+
+
=
(x + y + z)
+
a
b
c
x
y
s
1
1
=
(x + y + z) 3
x y
p
=
x+y+z

a = x; b =

1
z

1
z

1.4. THE CYH TECHNIQUES

115

Bi ton c gii. ng thc xy ra khi v ch khi x = y = z = 23 :


Do cc biu thc dng tuyn tnh ma + nb + pc; mb + nc + pa; mc + na + pb d dng
chn c cc gi tr ca m; n; p hn cc biu thc khc nn ta thng dng chng
gii, nhng i khi trong mt vi trng hp vic s dng chng khng mang li
hiu qu m ta phi s dng cc biu thc ph khc (vic chn cc biu thc ny
khng c mu mc m phn ln da vo kinh nghim ca ngi lm ton)
V d 1.95 Cho cc s khng m a; b; c; khng c 2 s no ng thi bng 0: Chng
minh rng
p

1
1
1
+p
+p
a2 + bc
b2 + ca
c2 + ab

1
1
1
+
+
b+c c+a a+b

(V Quc B Cn)
Li gii. S dng bt ng thc Cauchy Schwarz, ta c
!2
"
#"
#
X
X (a + b)(a + c) X
1
1
p
a2 + bc
(a + b)(a + c)
a2 + bc
cyc
cyc
cyc
!
P
!
2
a
X a(b + c)
cyc
+3
=
(a + b)(b + c)(c + a) cyc a2 + bc
Ta cn chng minh
2

cyc

X a(b + c)

(a + b)(b + c)(c + a)

X a(b + c)
cyc

a2 + bc

X a(b + c)
cyc

X
cyc

(a

a2 + bc

b)(a

cyc

+3

a2 + bc

+3

a2 + 3

cyc

cyc

1
a+b

!2

ab

cyc

(a + b)(b + c)(c + a)

cyc

cyc

a4

a2 b2

cyc

(a + b)(b + c)(c + a)

cyc

c)

!2

1
1
+
a2 + bc (b + c)(a + b + c)

116

CHNG 1. TM TI MT S K THUT GII TON

Khng mt tnh tng qut, gi s a


(c

a)(c

b)

0)a

a
b (b

c)

1
1
+
c2 + ab (a + b)(a + b + c)

0: Khi

v
1
1
+
+ bc (b + c)(a + b + c)
1
1
+(b a)(b c) 2
+
b + ca (c + a)(a + b + c)
(a b)(b c)
a
a
b
+
2
2
b
a + bc (b + c)(a + b + c) b + ca
c(a b)2 (a + b)(b c)[(a b)2 + ab + bc + ca]
=
0:
(a2 + bc)(b2 + ca)(b + c)(c + a)(a + b + c)

(a

b)(a

c)

a2

b
(c + a)(a + b + c)

Vy ta c pcm. ng thc xy ra khi a = b = c:


V d 1.96 Cho cc s khng m a; b; c; khng c 2 s no ng thi bng 0: Chng
minh rng
p
p
p
3(a + b + c) 2
a2 + bc + b2 + ca + c2 + ab :
(Phm Kim Hng)

Li gii. S dng bt ng thc Cauchy Schwarz, ta c


!2
"
#
X
X
Xp
2
2
a + bc
(3a + 4bc + ab + ac)
cyc

cyc

cyc

Ta cn chng minh

,8

a4 (b2 + c2 )

cyc

!2

X
cyc

3a2

a2 + bc
+ 4bc + ab + ac

a2 + bc
3
2 + 4bc + ab + ac
3a
4
cyc
X
X
X
16
a3 b3 11abc
a3 + 43abc
a2 (b + c) + 18a2 b2 c2
cyc

Y
, 8 (a
cyc

cyc

a2 + bc
2
3a + 4bc + ab + ac
!

b) + abc

cyc

X
cyc

cyc

X
cyc

a + 22

X
cyc

ab

0:

Bt ng thc cui hin nhin ng. ng thc xy ra khi v ch khi a = b; c = 0


hoc cc hon v tng ng.

1.4. THE CYH TECHNIQUES

117

Nhn xt 14 tng ca li gii ny nh sau


Chng ta thy l ng thc xy ra khi v ch khi a = b; c = 0, khi c mt im c
bit l
a2 + bc = b2 + ca = c2 + ab
Do , khi ta dng Cauchy Schwarz kh cn
!2
Xp
a2 + bc

"

cyc

#"

g(a; b; c)

cyc

X a2 + bc
g(a; b; c)
cyc

Ta cn chn g(a; b; c); g(b; c; a); g(c; a; b) sao cho ng thc trong bt ng thc ny
cng t ti a = b; c = 0. Vi ch trn, ta thy l nu ta chn g(a; b; c) c dng
a2 + bc + k(mt i lng i xng vi a; b; c) th r rng ng thc ban u vn c
m bo (cc i lng i xng ny cng n gin cng tt, s thun li hn cho
chng ta trong vic chng minh bt ng thc sau, chng ta c th chn cc i lng
nh (a + b + c)2 ; a2 + b2 + c2 ; ab + bc + ca). Ngoi ra, ta thy bn v tri ca bt
2
ng thc ban u c s xut hin ca (a + b +
Pc) nn bt ng thc sau khi s dng2
g(a; b; c) cng c dng m(a + b + c)
Cauchy Schwarz s d chng minh hn nu
cyc

t , ta d dng thy c mt trng hp hin nhin tha l k = 31 v i lng


i xng thm vo l ab + bc + ca: T dn n li gii kh c sc nh trn.

V d 1.97 Cho cc s khng m a; b; c; khng c 2 s no ng thi bng 0: Chng


minh rng
(b + c)2
(c + a)2
(a + b)2
+
+
6:
a2 + bc
b2 + ca
c2 + ab
(Darij Grinberg)
Li gii. S dng bt ng thc Cauchy Schwarz, ta c
"

X (b + c)2
cyc

a2 + bc

#"

"

!2

(b + c) (a + bc)

cyc

(b + c)

cyc

#2

=4

a +

cyc

cyc

Ta cn chng minh
2

a +

cyc

,2

X
cyc

a4 + 2abc

X
cyc

X
cyc

ab

a+

X
cyc

(b + c)2 (a2 + bc)

cyc

ab(a2 + b2 )

X
cyc

a2 b2

!2

ab

118

CHNG 1. TM TI MT S K THUT GII TON

Bt ng thc ny c suy ra t bt ng thc Schur bc 4


X
X
X
2
a4 + 2abc
a 2
ab(a2 + b2 )
cyc

cyc

v
3

cyc

ab(a2 + b2 )

cyc

a2 b2 :

cyc

vi bt ng thc cui hin nhin ng theo bt ng thc AM-GM. ng thc xy


ra khi v ch khi a = b = c hoc a = b; c = 0 hoc cc hon v tng ng.
V d 1.98 Cho cc s khng m a; b; c tha a + b + c = 1: Chng minh rng
a+b
b+c
c+a
+
+
ab + 1 bc + 1 ca + 1

9
:
5
(Michael Rozenberg)

Li gii. Bt ng thc tng ng vi


X (1 + a)(1 + b)
cyc

1 + ab

, (1 + a)(1 + b)(1 + c)

X
cyc

24
5

1
(1 + c)(1 + ab)

S dng bt ng thc Cauchy Schwarz, ta c


#"
#
"
X
X
1
2
(3 c) (1 + c)(1 + ab)
(1 + c)(1 + ab)
cyc
cyc

"

24
5

(3

cyc

Ta cn chng minh
40(1 + a)(1 + b)(1 + c)

(3

c)2 (1 + c)(1 + ab)

cyc

t q =

ab; r = abc. Ta c

cyc

(1 + a)(1 + b)(1 + c) = 2 + q + r
X

(3

c)2 (1 + c)(1 + ab) = (8

2q)r + 26 + 16q

cyc

Bt ng thc tr thnh
(16 + 6q)r + 2

8q

#2

c)

= 64

1.4. THE CYH TECHNIQUES

119

Nu 1 4q, bt ng thc l hin nhin. Nu 4q


q)
bc 4, ta c r (4q 1)(1
, do
6
(16 + 6q)r + 2

8q

(16 + 6q)
=

1
(4q
3

(4q

1)(1

ng thc xy ra khi v ch khi a = b = c =


v tng ng.

1
3

1, s dng bt ng thc Schur


1)(1
6

q)

+2

3q)(q + 2)

8q

0:

hoc a = b = 12 ; c = 0 hoc cc hon

V d 1.99
Cho cc s khng m a; b; c tha a + b + c = 1: Chng minh rng vi
p
3
k=1
,
ta
c
2
p
p
p
p
a + k(b c)2 + b + k(c a)2 + c + k(a b)2
3:
(Phan Thnh Nam)

Li gii. S dng bt ng thc Cauchy Schwarz, ta c


v
2
32
u
p !
Xu
3
ta + 1
4
(b c)2 5
2
cyc
"
#
#"
p
X
Xa+ 1
32 (b c)2
1
a+ p
a + p13
3
cyc
cyc
"
#
p
X
p
a
2
3 X (b c)2
3+1
+
=
2
a + p1
a + p1
cyc
cyc
3

Ta cn chng minh
p

3+1

"

X
cyc

#
p
a
2
3 X (b c)2
+
2
a + p13
a + p13
cyc

t q = ab + bc + ca =; r = abc, khi ta c q 2 3r. Bt ng thc tr thnh


p
p
9 2 + 3 r q 6q + 3
0
Ta c
9 2+

3 r

q 6q +

3 2+

3 q2

ng thc xy ra khi v ch khi a = b = c =


v tng ng.

1
3

q 6q +

3 =

3q(3q

1)

0:

hoc a = 1; b = c = 0 hoc cc hon

120

CHNG 1. TM TI MT S K THUT GII TON

V d 1.100 Cho cc s dng a; b; c; d tha (a + b + c + d)


Chng minh rng
1
1
1
1
+ 2+ 2+ 2
a2
b
c
d

(a2 + b2 + c2 + d2 )

1
a

1
b

1
c

1
d

= 20:

36:
(Phm Kim Hng)

Li gii. S dng bt ng thc Cauchy Schwarz, ta c


X 1
a2
cyc

!"

(b + c + d

a)2

cyc

Ta cn chng minh
4

a2

cyc

Xb+c+d

cyc

!2

= 144

a)2

(b + c + d

cyc

,0

0:

hin nhin ng.


V d 1.101 Cho cc s khng m a; b; c; khng c 2 s no ng thi bng 0: Chng
minh rng
b+c
c+a
a+b
6
+
+
:
2a2 + bc 2b2 + ca 2c2 + ab
a+b+c
(Vasile Cirtoaje)
Li gii. S dng bt ng thc Cauchy Schwarz, ta c
X
cyc

b+c
2a2 + bc

!"

(b + c) (2a + bc)

cyc

"

(b + c)

cyc

#2

Ta cn chng minh
2

X
cyc

a +

!2

ab

cyc

cyc

(b + c)3 (2a2 + bc)

cyc

Chun ha cho a + b + c = 1. t q = ab + bc + ca; r = abc, khi theo bt ng thc


Schur bc 3, ta c r max 0; 4q9 1 . Ta c
X
cyc

a +

X
cyc

!2

ab

X
cyc

= (1

q)2

1.4. THE CYH TECHNIQUES


X

121

(b + c)3 (2a2 + bc) = q + 2q 2 + 12qr

11r

cyc

Bt ng thc tr thnh
q)2

2(1

, 3(11
Nu 1

3(q + 2q 2 + 12qr
12q)r + (q + 2)(1

4q, bt ng thc l hin nhin. Nu 4q


3(11

12q)r + (q + 2)(1

4q)

4q)

1, ta c

3(11

12q)

5
(4q
3

1)(1

11r)

4q

1
9

3q)

+ (q + 2)(1

4q)

0:

ng thc xy ra khi v ch khi a = b = c hoc a = b; c = 0 hoc cc hon v tng


ng.
V d 1.102 Cho cc s khng m a; b; c tha a + b + c = 1: Chng minh rng
r
s
X
(b c)2
1X
a+
3+
(b c)2 :
4
2
cyc
cyc
(Phan Thnh Vit)
Li gii. S dng bt ng thc Cauchy Schwarz, ta c
"

X
cyc

a+

c)2

(b
4

#2

"
=

#"

(a + 1)

cyc

X 4a + (b

Xa+

(b c)2
4

a+1

cyc

c)2

a+1

cyc

Ta cn chng minh
X 4a + (b

a+1

cyc

(b

c)2

cyc

X
cyc

c)2

(b

1
2
c)2

3+

1
a+1

1X
(b
2 cyc

+3

c)2

X 4a
a+1
cyc

X 1 3a
a 1
+
2(a + 1) cyc 1 + a

122

CHNG 1. TM TI MT S K THUT GII TON

Ta c
X1
cyc

3a X b + c 2a X
=
=
(b
1+a
1+a
cyc
cyc

1
1+c

c)

1
1+b

X
cyc

(b c)2
(b + 1)(c + 1)

Do , bt ng thc tng ng vi
X
cyc

X
(b c)2
+
(b
(b + 1)(c + 1) cyc

c)2

a 1
2(a + 1)

1
a 1
+
(b + 1)(c + 1) 2(a + 1)
X
,
x(b c)2 0

c)2

(b

cyc

cyc

trong x = 2(a + 1) + (a
Do tnh i xng, gi s a
x

2(a + 1) + (a
1 3
(a
=
4

1)(b + 1)(c + 1) v y; z tng t.


b c, khi a 31 c; 21 b, ta c
1) 1 +

7a2 + 23a

1)

b+c
2

= 2(a + 1) + (a

1) 1 +

y = 2(b + 1) + (b2

1)(a + c

b + 1) + (1

2(b + 1) + (b

1)(a + c

b + 1)

= 2(b + 1) + (b

1)(2

2b) = 2b(b + 1)(2

b)(a
b)

b)(b

c)

y + z = 2(b + c + 2) + (a + 1)[(b 1)(c + 1) + (c 1)(b + 1)]


= 2(3 a) + 2(a + 1)(bc 1) 2(3 a) 2(a + 1) = 4(1
X
)
x(b c)2 (y + z)(a b)2 0:

a)

cyc

ng thc xy ra khi v ch khi a = b = c =


v tng ng.

1
3

hoc a = 1; b = c = 0 hoc cc hon

Nhn xt 15 Chng ta c mt kt qu tng t nh sau


r
r
r
(b c)2
(c a)2
(a b)2
a+
+ b+
+ c+
4
4
4
Khng mt tnh tng qut, gi s a b c: Xt 2 trng hp sau
Trng hp 1. b 61 ) a = 1 b c 1 2b 23 ; khi ta c
r
r
p
(b c)2
2
a+
a
4
3

3
2

1.4. THE CYH TECHNIQUES


r

b+

a)2

(c
4

c+

123
r

b)2

(a
4

)
Trng hp 2. b

cyc

a+

b+

=
X

(2a

1)2

(3a

a+

c)2

(b

2
+
3

4
r

b
4
r

c)2
(9a

b+c+

4)(3a
12

2) + 7

3
7
>
12
2

1
6;

bnh phng 2 v, ta c bt ng thc tng ng l


s
X
1X
(b c)2
(a c)2
5
2
(a b) + 2
a+
b+
4 cyc
4
4
4
cyc

S dng bt ng thc Cauchy Schwarz, ta c


s
X p
X
(a c)2
1
(b c)2
b+
2
2
ab + j(a
a+
4
4
4
cyc
cyc

b)(a

Nn ta ch cn chng minh
1X
(a
4 cyc

b)2 + 2

Xp

ab +

cyc

, (a

c)2 + 2

1X
j(a
2 cyc

Xp

cyc

5
4

c)j

5
4

ab

cyc

S dng bt ng thc AM-GM, ta c


X
Xp
p
ab = 2
2
(a + b + c) ab
cyc

b)(a

p
(a + b) ab

cyc

ab

cyc

Nn ta ch cn chng minh c
(a
, (a
,

1
(a
4

c)2 + 4(ab + bc + ca)

b + c)(5b

5
4

c)2 + 4(ab + bc + ca)

c)

0,

1
(a
4

5
(a + b + c)2
4
b + c)(6b

Bt ng thc cui hin nhin ng. Vy ta c pcm.

1)

0:

c)j

(2a

b
4
r

c)2
7
12

124

CHNG 1. TM TI MT S K THUT GII TON

V d 1.103 Cho cc s khng m a; b; c; khng c 2 s no ng thi bng 0: Chng


minh rng
p

1
1
1
+p
+p
4a2 + bc
4b2 + ca
4c2 + ab

2
:
ab + bc + ca
(V Quc B Cn)

Li gii. S dng bt ng thc Holder, ta c


X
cyc

1
4a2 + bc

!2 "

(b + c)3 (4a2 + bc)

cyc

"

#3

(b + c)

cyc

=8

!3

cyc

Ta cn chng minh
X

2
X

cyc

!3

cyc

ab

cyc

ab(a3 + b3 )

(b + c)3 (4a2 + bc)

cyc

a2 b2 (a + b) + 14abc

cyc

ab(a

cyc

a2

cyc

b)2 (a + b) + 14abc

a2

0:

cyc

hin nhin ng. ng thc xy ra khi v ch khi a = b; c = 0 hoc cc hon v tng


ng.
V d 1.104 Cho cc s khng m a; b; c; tt c khng ng thi bng 0: Chng minh
rng
(a + b)2
(b + c)2
(c + a)2
3
+
+
:
a2 + 2b2 + 3c2
b2 + 2c2 + 3a2
c2 + 2a2 + 3b2
2
(Dng c Lm, V Quc B Cn)
Li gii. S dng bt ng thc Cauchy Schwarz, ta c
#"
#
"
X
X
(a + b)2
2 2
2
2
(2a + b) (a + 2b + 3c )
a2 + 2b2 + 3c2
cyc
cyc
"
#2
!2
X
X
X
2
(a + b)(2a + b) = 9
a +
ab
cyc

cyc

cyc

1.4. THE CYH TECHNIQUES

125

Ch rng
X
(2a + b)2 (a2 + 2b2 + 3c2 )
cyc

= 6

a4 + 4

cyc

a3 b + 8

cyc

a +8

cyc

ab3 + 24

cyc

ab(a + b ) + 24

cyc

a2 b2 + 12

cyc

2 2

a b + 12

cyc

a2 bc

cyc

a2 bc

cyc

Ta cn chng minh
3

X
cyc

a +

X
cyc

!2

ab

,2

a4 + 4

cyc

ab(a2 + b2 )

ab(a2 + b2 ) + 12

cyc

cyc

,2

X
3

b)2 +

cyc

a2 b2 + 6

cyc

a2 b2 + 6

cyc

ab(a

a2 bc

a2 bc

cyc

cyc

a2 b2 + 6

cyc

a2 bc

0:

cyc

hin nhin ng. ng thc xy ra khi v ch khi


ng.

a
1

b
0

c
0

hoc cc hon v tng

V d 1.105 Cho cc s khng m a; b; c; tt c khng ng thi bng 0: Chng minh


rng
b2
c2
2
a2
+
+
:
2a2 + (b + c)2
2b2 + (c + a)2
2c2 + (a + b)2
3
(Darij Grinberg)
Li gii. Bt ng thc tng ng vi
X

2a2
2a2 + (b + c)2

(b + c)2
2a2 + (b + c)2

cyc

cyc

5
3
5
3

S dng bt ng thc Cauchy Schwarz, ta c


"
#"
#
X
X
(b + c)2
2
2
2
(a + 3b + 3c) [2a + (b + c) ]
2a2 + (b + c)2
cyc
cyc
"
#2
X
(b + c)(a + 3b + 3c)
cyc

126

CHNG 1. TM TI MT S K THUT GII TON

Ta cn chngm inh
3

"

#2

(b + c)(a + 3b + 3c)

cyc

(a + 3b + 3c)2 [2a2 + (b + c)2 ]

cyc

Chun ha cho a + b + c = 1. t q = ab + bc + ca; r = abc, ta c


X
(b + c)(a + 3b + 3c) = 6 4q
cyc

(a + 3b + 3c)2 [2a2 + (b + c)2 ] =

cyc

2a)2 (3a2

(3

cyc

= 12

a4

44

cyc

a3 + 55

cyc

84r + 24q 2

2a + 1)
X

a2

cyc

26q + 20

Bt ng thc tr thnh
3(6

4q)2

5( 84r + 24q 2

, 210r + (1

4q)(4 + 9q)

Nu 1 4q, bt ng thc l hin nhin. Nu 4q


bc 3 ta c r 4q9 1 , do
210r + (1

4q)(4 + 9q)

4q

210

1
9

26q + 20)

+ (1

1, s dng bt ng thc Schur

4q)(4 + 9q) =

1
(4q
3

1)(58

27q)

0:

ng thc xy ra khi v ch khi a = b; c = 0 hoc cc hon v tng ng.


V d 1.106 Cho cc s khng m a; b; c; tt c khng ng thi bng 0: Chng minh
rng
a2
b2
c2
1
+ 2
+ 2
:
2
2
2
5a + (b + c)
5b + (c + a)
5c + (a + b)2
3
(V Quc B Cn)
Li gii. Bt ng thc tng ng
X
cyc

X
cyc

!"

X
cyc

4a2
5a2 + (b + c)2

5
3

#
X
1
bc
+2
2
2
2 + (b + c)2
5a + (b + c)
5a
cyc

5
3

1.4. THE CYH TECHNIQUES

127

S dng bt ng thc Cauchy Schwarz, ta c


"

1
2
5a + (b + c)2

cyc

"

#2

(a + 3b + 3c)

cyc

(a + 3b + 3c)2 [5a2 + (b + c)2 ]

cyc

49
= P

!2

cyc

(a + 3b + 3c)2 [5a2 + (b + c)2 ]

cyc

X
cyc

bc
2
5a + (b + c)2

cyc

!2

bc

bc[5a2 + (b + c)2 ]

cyc

bc

cyc

!2

cyc

Ta cn chng minh
49
P

cyc

a2

!2

cyc

(a + 3b + 3c)2 [5a2 + (b + c)2 ]

bc

cyc

cyc

!2

5
3

cyc

Chun ha cho a + b + c = 1. t q = ab + bc + ca; r = abc, ta c


X

(a + 3b + 3c)2 [5a2 + (b + c)2 ] = 23

20q + 48q 2

144r

cyc

Bt ng thc tr thnh

23
Nu 1

49(1 2q)
+ 2q
20q + 48q 2 144r

5
3

4q, ta c

23

49(1 2q)
+ 2q
20q + 48q 2 144r

5
3

49(1 2q)
5
+ 2q
23 20q + 48q 2
3
8(4 3q)(1 4q)(1 + 3q)
=
3(23 20q + 48q 2 )

128

CHNG 1. TM TI MT S K THUT GII TON

Nu 4q

23

4q 1
9 .

1, s dng bt ng thc Schur bc 3 ta c r


5
3

49(1 2q)
+ 2q
20q + 48q 2 144r

Suy ra

49(1 2q)
23 20q + 48q 2 16(4q
8(1 3q)(4q 1)(2 q)
=
3(13 28q + 16q 2 )

1)

+ 2q

5
3

0:

ng thc xy ra khi v ch khi a = b = c hoc a = b; c = 0 hoc cc hon v tng


ng.
V d 1.107 Cho cc s khng m a; b; c; khng c 2 s no ng thi bng 0: Chng
minh rng
r
r
r
p
3 6
a2 + ab + b2
b2 + bc + c2
c2 + ca + a2
+
+
:
c2 + ab
a2 + bc
b2 + ca
2
(Nguyn Vn Thch)
Li gii. S dng bt ng thc Holder, ta c
X
cyc

!"

a2 + ab + b2
c2 + ab

Ta cn chng minh
X

16
27

16
,
27
Do a2 + ab + b2

cyc

3
4 (a

16
27
16
,
27

X
cyc

!3

cyc

cyc

a2 + ab + b2

X (a + b)(a2 + 2ab + b2 )(c2 + ab)

!3

a2 + ab + b2

X (a + b)3 (c2 + ab)

!3

cyc

cyc

!3

cyc

16
,
27

X (a + b)3 (c2 + ab)

a2 + ab + b2

cyc

ab(a + b) +

cyc

X ab(a + b)(c2 + ab)


cyc

a2 + ab + b2

+ b)2 , nn ta ch cn chng minh c


X
cyc

!3

!3

ab(a + b) +

cyc

cyc

ab(a + b) +

4 X ab(c2 + ab)
3 cyc
a+b

X c
4 X a2 b2
4
+ abc
3 cyc a + b 3
a+b
cyc

1.4. THE CYH TECHNIQUES


Li do

4a2 b2
a+b

16
27

129

ab(a + b), nn ta ch cn chng minh


!3
X
X
X c
1X
4
a
2
ab(a + b) +
ab(a + b) + abc
3 cyc
3
a+b
cyc
cyc
cyc
16
,
9

X
cyc

!3

ab(a + b) + 4abc

cyc

X
cyc

c
a+b

Chun ha cho a + b + c = 1. t q = ab + bc + ca; r = abc, khi theo bt ng thc


2
q)
r (4q 1)(1
. Bt ng thc tr thnh
Schur v bt ng thc Newton, ta c q3
6
16
9

7(q

1+q
q r

3r) + 4r

, f (r) = 297r2 + (52

16q + 63q 2

324q)r

V f (r) l hm li nn

f (r)

max f

q2
3

;f

(4q

1)(1
6

q)

Mt khc, ta c
f
f

(4q

1)(1
6

q2
3

=
q)

1
q(3q
3
=

1)(33q 2

1
(3q
12

1)(528q 3

v
528q 3

280q 2 + 29q + 5 = q 3
= q3
q

97q + 48)

"

5
29
+ 2
q3
q
1
3
q
1
q

= q 3 29

3
1
q

280q 2 + 29q + 5)

280
+ 528
q
5
44
+
148 + 84
q2
q
!
#
5
44
148 + 84
2 + 1
1
3

3 + 84

0:

Bt ng thc c chng minh. ng thc xy ra khi v ch khi a = b = c.


V d 1.108 Cho cc s khng m a; b; c: Chng minh rng
a2
b2
c2
+
+
a2 + b2 + ab + ca b2 + c2 + bc + ab c2 + a2 + ca + bc

3
:
4

(Michael Rozenberg)

130

CHNG 1. TM TI MT S K THUT GII TON

Li gii. S dng bt ng thc Cauchy Schwarz, ta c


!"
#
X
X
a2
2
2
2
(2a + c) (a + b + ab + ca)
a2 + b2 + ab + ca
cyc
cyc
"
#2
!2
X
X
X
2
a(2a + c) = 2
a +
ab
cyc

cyc

cyc

Nn ta ch cn chng minh
4 2

a +

cyc

,4

!2

a2 b2 +

ab

cyc

a4 + 6

cyc

(2a + c) (a2 + b2 + ab + ca)

cyc

cyc

a3 b

cyc

ab3

3abc

cyc

cyc

S dng bt ng thc Vasile, ta c


8

ab

cyc

8
3

8
3

cyc

!2

Nn ta ch cn chng minh
4

a +6

cyc

2 2

a b +

cyc

a b

cyc

cyc

4X 4 2X 2 2 X 3
a +
a b +
a b
3 cyc
3 cyc
cyc

!2
3abc

3abc

cyc

cyc

S dng bt ng thc AM-GM, ta c


4X 4
a
3 cyc
2

a2 b2

4X 2 2
a b
3 cyc
2abc

cyc

X
cyc

cyc

a3 b

abc

a:

cyc

Cng tng ng v vi v 3 bt ng thc trn, ta thu c bt ng thc trn.


Vy ta c pcm. ng thc xy ra khi v ch khi a = b = c:

1.4. THE CYH TECHNIQUES

131

V d 1.109 Cho cc s dng a; b; c: Chng minh rng


2b3

b
c
a
+ 3
+ 3
2
2
+ c a 2c + a b 2a + b2 c

a2

3
:
+ b2 + c2
(Dng c Lm)

Li gii. S dng bt ng thc Cauchy Schwarz, ta c


!"
#
"
#2
X
X
X
a
a(a + c)2 (2b3 + c2 a)
a(a + c)
2b3 + c2 a
cyc
cyc
cyc
X

a +

cyc

Nn ta ch cn chng minh c
!2
X
X
2
a +
ab
cyc

cyc

cyc

Bng khai trin trc tip, ta d thy


X
a(a + c)2 (2b3 + c2 a)
cyc

a4 (b2 + c2 ) + 4

cyc

X
cyc

a4 (b2 + c2 ) + 4

cyc

X
cyc

a4 (b2 + c2 ) + 4

cyc

!2

ab

cyc

X
3 a(a + c)2 (2b3 + c2 a)
cyc

X
X
a3 b3 + 2 a3 b2 c + 4 a2 b3 c
cyc

cyc

X
X
a3 b3 + 2abc a2 (b + c) + 2abc a3
cyc

a3 b3 + 2abc

cyc

X
cyc

a2

cyc

cyc

Khng mt tnh tng qut gi s a + b + c = 1 v t q = ab + bc + ca; r = abc th ta


c
!
!
X
X
X
X
4 2
2
3 3
2
a (b + c ) + 4 a b + 2abc
a
a
cyc

X
cyc

2 2

a b

!
2

cyc

X
cyc

cyc

cyc

!
!
X
X
X
3 3
2
+ 4 a b + 2abc
a
a

= (1 2q)(q
2r) + 4(q
2
3
= q + 2q
12qr + 9r2

cyc

cyc

3qr + 3r ) + 2r(1

cyc

2q)

3r2

3a2 b2 c2

132

CHNG 1. TM TI MT S K THUT GII TON

Ta cn chng minh
(1

q)2

2q)(1

3(q 2 + 2q 3

4q + 2q 2

,1

8q 3 + 36qr

, (1

12qr + 9r2 )

4q)(1 + q ) + 2(9q + 2)r + [q

27r2

4q + 2(9q

27r2 ]

2)r

Ta c
q2

4q 3 + 2(9q

2)r

27r2 = (a

(4q 1)(1 q)
6

v theo bt ng thc Schur bc 4 th r


(1

4q)(1 + q 2 ) + 2(9q + 2)r

c)2 (c

a)2

1
(4q
3

1)2 (1

nn

4q)(1 + q 2 ) + 2(9q + 2)

(1
=

b)2 (b

3q)

(4q

1)(1
6

q)

0:

Bt ng thc c chng minh xong. ng thc xy ra khi v ch khi a = b = c


hoc a = b; c = 0 v cc hon v.
V d 1.110 Cho a; b; c l di 3 cnh ca mt tam gic. Chng minh rng
a
b
c
+
+
2b2 + ca 2c2 + ab 2a2 + bc

3
:
a+b+c
(V Quc B Cn)

Li gii. S dng bt ng thc Cauchy Schwarz, ta c


X

a
2
2b + ca
cyc

!"

a(a + c) (2b + ca)

cyc

X
cyc

a +

!2

ab

cyc

Bng khai trin trc tip, ta c


X
cyc

a(a + c) (2b + ca)

"
#
X
X
1 X 4
3 2
2
a (b + c) + 5 a (b + c ) + 12abc ab
2 cyc
cyc
cyc
X
1
+ (a b)(b c)(c a) a2
2
cyc

Do a; b; c l di 3 cnh tam gic nn ta d dng chng minh c


X
X
(a b)(b c)(c a)
a3 + 3abc
a2 (b + c)
cyc

cyc

1.4. THE CYH TECHNIQUES


)

133

a(a + c)2 (2b2 + ca)

cyc

"
#
X
X
1 X 4
3 2
2
a (b + c) + 5 a (b + c ) + 12abc ab
2 cyc
cyc
cyc
"
#
!
X
X
1 X 3
a + 3abc
a2 (b + c)
a2
+
2 cyc
cyc
cyc

1
=
2
2

X
cyc

!
X
X
X
3 2
2
2
a + 5 a (b + c ) + 3abc a + 10abc ab
5

cyc

14 X 3
a
=
2
cyc

cyc

X
cyc

a2

+4

a2 b2

cyc

cyc

cyc

+ 3abc

cyc

Chun ha cho a + b + c = 1; v t ab + bc + ca = q; r = abc )


X
cyc

cyc

+4

2 2

a b

cyc

X
cyc

+ 3abc

1
3

!2 3
a 5

1
4,

ta c

!2

cyc

= (1 2q)(1 3q + 3r) + 4(q 2


= 1 5q + 10q 2 2(1 + 3q)r

2r) + 3r

Ta cn chng minh
2(1

q)2

3[1

5q + 10q 2

, 6(1 + 3q)r + (4q


Theo bt ng thc Schur bc 4, ta c 6r
6(1 + 3q)r + (4q

1)(1

7q)

1)(1
(4q

(1 + 3q)(4q
= (1 3q)(4q

2(1 + 3q)r]
7q)
1)(1

0
q) nn

1)(1 q) + (4q
1)(q + 2) 0:

1)(1

7q)

Bt ng thc c chng minh xong. ng thc xy ra khi v ch khi a = b = c


hoc a = b; c = 0 v cc hon v.
V d 1.111 Cho cc s dng a; b; c. Chng minh rng vi mi k
s X
Xp
X
2
2
a + kab + b
4
a2 + (3k + 2)
ab:
cyc

cyc

2, ta c

cyc

(Michael Rozenberg)

134

CHNG 1. TM TI MT S K THUT GII TON

Li gii. S dng bt ng thc Cauchy Schwarz, ta c


!2
"
#
!
X
X a2 + kab + b2
Xp
a2 + kab + b2
(a + b)
a+b
cyc
cyc
cyc
!
!
X
X a2 + kab + b2
= 2
a
a+b
cyc
cyc
Ta cn chng minh

a+b

cyc

,2

X a2 + kab + b2

(a + b) + 2(k

,2

P
a2 + (3k + 2) ab
cyc
cyc
P
a
cyc

X ab
2)
a+b
cyc

cyc

,2

X
cyc

cyc

ab
a

cyc

a+b

, 2abc

a+

P
2) ab
cyc
P
a

cyc

X ab(a + b + c)
cyc

3(k

cyc

X ab
a+b
cyc

ab

cyc

1
a+b

ab

cyc

S dng bt ng thc Cauchy Schwarz, ta c


2abc

X
cyc

1
a+b

2abc

X1
cyc

1 1
+
a b

ab:

cyc

Bt ng thc c chng minh. ng thc xy ra khi v ch khi a = b = c:


V d 1.112 Cho cc s khng m a; b; c tha mn a + b + c = 1: Chng minh rng
r
p
p
p
2
2
2
2
a b + 3c + b c + 3a + c a + 3b
:
3
Li gii. Ta c b sau
2

X
cyc

ab + 5

X
cyc

a2 b

11
9

1.4. THE CYH TECHNIQUES

135

Tht vy, bt ng thc tng ng vi


X
X
11
a3 + 15
ab2 + 12abc
cyc

30

cyc

xy + y 2 )c + 11x3

cyc

Gi s c = minfa; b; cg, t a = c + x; b = c + y (x; y


18(x2

a2 b

0), bt ng thc tr thnh

30x2 y + 15xy 2 + 11y 3

Ta cn chng minh
f (x) = 11x3

30x2 y + 15xy 2 + 11y 3

Ta c
f 0 (x) = 33x2

60xy + 15y 2

p
p
10 + 3 5 y
10 3 5 y
f (x) = 0 , x =
_x=
11
11
0

T y, ta d dng kim tra c


f (x)

!
p
p
10 + 3 5 y
9 109 30 5 y 3
=
11
121

Tr li bi ton ca ta, s dng bt ng thc Cauchy Schwarz, ta c


X p
a b + 3c2
cyc

!2

"
=

#"

a(1 + c)

cyc

1+

ab

cyc

1+

X
cyc

cyc

cyc

1+c

X
X ab
+3
ab
1+c
cyc
cyc

!6
6X ab
X
6
ab 6
+3
ab
6 cyc 1 + c
cyc
4

X ab
X
=
ab
1+c
cyc
cyc
X

X a(b + 3c2 )

a2 b

abc

X
cyc

11
45

1
c+1
2X
5

cyc

X
cyc

ab

ab

!
X ca
3
1+c
cyc
!2 3
P
7
3
ab
7
cyc
P
P 2 7
7
ab +
a b7
5
cyc
cyc
9
abc
4

136

CHNG 1. TM TI MT S K THUT GII TON

Ta cn chng minh

1+

X
cyc

!6
6 X
6
ab
ab 64
6 cyc
4

3
9
abc
4

7
7
7
P 7
3
+5
ab 7
5
cyc
ab

cyc

11
45

!2 3

2
3

t q = ab + bc + ca; r = abc, th bt ng thc ny tr thnh


4q

9
r
4

2
3(q + 1)

3q 2
3
11
5 q + 45

2
3(q + 1)

9
135q 2
4q + r +
4
27q + 11
(4q 1)(1 q)
.
6

Theo bt ng thc Schur bc 4, ta c r

Suy ra

9
135q 2
2
4q + r +
3(q + 1)
4
27q + 11
2
9 (4q 1)(1 q)
135q 2
4q +
+
3(q + 1)
4
6
27q + 11
2
3
2
7 60q 87q
36q
135q
=
+
24(q + 1)
27q + 11
(1 3q)(324q 3 57q 2 240q + 77)
=
24(q + 1)(27q + 11)
Do , ta ch cn chng minh c
324q 3

57q 2

240q + 77

y l mt hm gim theo q nn
324q 3

57q 2

240q + 77

324

1
33

57

1
32

240

1
8
+ 77 = > 0:
3
3

Bt ng thc c chng minh. ng thc xy ra khi v ch khi a = b = c = 31 :


V d 1.113 Cho cc s dng a; b; c: Chng minh rng
a
b
c
+
+
2
2
b+c
c+a
a + b2

9
:
3+a+b+c
(Trn Quc Lut)

1.4. THE CYH TECHNIQUES

137

Li gii. S dng bt ng thc Cauchy Schwarz, ta c


X
cyc

a
b + c2

!"

a(b + c )(2a + 2b + c)

cyc

a +3

cyc

!2

ab

cyc

Nn ta ch cn chng minh
2

a +3

cyc

!2

ab

cyc

3+

cyc

a(b + c2 )(2a + 2b + c)2

cyc

Bt ng thc ny c suy ra t cc bt ng thc sau


2

a +3

cyc

ab

cyc

a +3

cyc

!2

!2

ab

cyc

ab(2a + 2b + c)2

cyc

cyc

c2 a(2a + 2b + c)2

cyc

a) Trc ht, ta s chng minh

a +3

cyc

!2

ab

cyc

ab(2a + 2b + c)2

cyc

Do tnh thun nht, ta c th gi s a + b + c = 1: t q = ab + bc + ca; r = abc th


ta c
!2
X
X
2
2
a +3
ab
= (2 q)2
cyc

ab(2a + 2b + c)2 =

cyc

cyc

ab(2

(2

q)2

c)2 =

cyc

ab(4

4c + c2 ) = 4q

11r

cyc

Bt ng thc tr thnh
3(4q

, 33r + 4

11r)

16q + q 2

Theo bt ng thc Schur, ta c nn


33r + 4

16q + q 2

33

4q

1
9

+4

16q + q 2 =

1
(1
3

q)(1

3q)

138

CHNG 1. TM TI MT S K THUT GII TON

b) Tip theo, ta s chng minh


2

a +3

cyc

!2

ab

cyc

cyc

c2 a(2a + 2b + c)2

cyc

y cng l mt bt ng thc thun nht nn ta cng c th chun ha cho a+b+c =


1 v t q = ab + bc + ca; r = abc tng t nh trn, khi ta c
!2
!
X
X
X
2
2
a +3
ab
a = (2 q)2
cyc

cyc

cyc

V
X

c2 a(2a + 2b + c)2

cyc

c2 a(2

c)2 =

cyc

a b

cyc

M
X

a3 b =

cyc

a2 b

cyc

a b

cyc

X
3

a b

(1

q)

cyc

cyc

cyc

a2 b

c2 a(4

cyc

a b+

abc

cyc

a2 b2

cyc

a b

+ (1

cyc

q 2 + (1 + q)r

a4 b

cyc

a=

cyc

4c + c2 )

a2 b

q2 + r

cyc

ab

cyc

+ abc

ab

cyc

cyc

Nn
X

c2 a(2a + 2b + c)2

cyc

= 4

a b

cyc

= (1

q)

a b

q +r

cyc

a b + 3q + (q

q)

a2 b

q 2 + (1 + q)r

cyc

3)r

cyc

S dng kt qu quen thuc

4
27

a2 b

cyc

(2

q)2

9 (1

q)

r; ta ch cn chng minh c
4
27

r + 3q 2 + (q

3)r

1.4. THE CYH TECHNIQUES

139

, 9(2
Nu 1

q)r +

8
q
3

26q 2

4q th ta c
8
3

Nu 4q
9(2

8
3

8
q
3

26q 2

1 th theo bt ng thc Schur, ta c 9r

q)r +

8
3

8
q
3

26q 2

(2

q)(4q

1) +

8
3

8
q
3

4q

1 nn

26q 2 =

2
(17q
3

2)(1

3q)

0:

Bt ng thc c chng minh xong. ng thc xy ra khi v ch khi a = b = c:


V d 1.114 Cho cc s khng m a; b; c tha mn a + b + c = 1: Chng minh rng
a2
b2
c2
+
+
b + 27c3
c + 27a3
a + 27b3

1
:
4
(V Quc B Cn)

Li gii. S dng bt ng thc Cauchy Schwarz, ta c


X

a2
b + 27c3

hX

i
(2a + b)2 (b + 27c3 )

a2 +

ab

t q = ab + bc + ca; r = abc th ta c
2

27

b(2a + b)2 =

c3 (2a + b)2

a3 + 4

a2 +

ab

a2 (b + c) = 1

= (2

3q)2

3q + 3r + 4(q

3r) = 1 + q

9r

i
X
X
a2 b2 (a + b) + 4abc
a2 + 3
a3 b2
h X
i
X
X
X
X
= 27
a
a2 b2 + 4abc
a2 abc
ab + 3
a3 b2
h
i
X
= 27 q 2 2r + 4(1 2q)r qr + 3
a3 b2
h
i
X
= 27 q 2 + (2 9q)r + 3
a3 b2
27

hX

140

CHNG 1. TM TI MT S K THUT GII TON

v
X

a3 b2

ab

a2 b

abc

a2

X
a2 b (1 q)r = q
a
X
X
X
= q
a3 b +
a2 b2 + abc
a

= q

X
abc
ab
X
a2 b
(1
(1

q)r

X
1 X 2 2 X 2 2
+
a b + abc
a
(1
a
3
1
1
= q (1 2q)2 + q 2 r
(1 q)r = q(1
3
3

q)r

) 27

c3 (2a + b)2

27 q 2 + (2
=

9q)r + q(1

q)r

4q + 7q 2 )

4q + 7q 2 )

3r

3q + 7q 2 )]

27[ (9q + 1)r + q(1

Do ta ch cn chng minh c
3q)2

4(2

1+q

9r + 27[ (9q + 1)r + q(1

, 9(4 + 37q)r + 15

76q + 117q 2

Theo bt ng thc Schur bc 3, ta c 9r

4q

3q + 7q 2 )]

189q 3

1 nn

9(4 + 37q)r + 15 76q + 117q 2 189q 3


(4 + 27q)(4q 1) + 15 76q + 117q 2
= (1 3q)2 (11 21q) 0:

189q 3

Bt ng thc c chng minh xong. ng thc xy ra khi v ch khi a = b = c = 31 :


V d 1.115 Cho cc s a; b; c

0; a + b + c = 3: Chng minh rng

1
1
1
+
+
ab2 + 8 bc2 + 8 ca2 + 8

1
:
3
(Vasile Cirtoaje)

Hng dn. S dng bt ng thc Cauchy Schwarz,


X
cyc

1
ab2 + 8

X
cyc

(ab + 8)(a + 2c + 5)

"

X
cyc

#2

(a + 2c + 5)

= 576:

1.4. THE CYH TECHNIQUES


V d 1.116 Cho cc s a; b; c

141
0: Chng minh rng

1
1
1
+
+
2a2 + bc 2b2 + ca 2c2 + ab

6
:
a2 + b2 + c2 + ab + bc + ca
(Vasile Cirtoaje)

Hng dn. S dng bt ng thc Cauchy Schwarz

X
cyc

1
2a2 + bc

1
4a2

+ bc

#2

(b + c)

cyc

(b + c)2 (2a2 + bc)

cyc

V d 1.117 Cho cc s a; b; c
p

"

+p

0: Chng minh rng


1
1
+p
2
+ ca
4c + ab

4
:
a+b+c

4b2

(Phm Kim Hng)


Hng dn. S dng bt ng thc Holder

X
cyc

1
4a2 + bc

V d 1.118 Cho cc s a; b; c
p

2a2

!2

"

#3

(b + c)

cyc

(b + c)3 (4a2 + bc)

cyc

0: Chng minh rng

1
1
1
+p
+p
2
2
+ ab + bc
2b + ca + ab
2c + ca + ab

9
:
2(a + b + c)

Hng dn. S dng bt ng thc Holder


X
cyc

1
p
2
2a + ab + bc

!2

cyc

27(a + b + c)3
:
(b + 2c)3 (2a2 + ab + bc)

142

CHNG 1. TM TI MT S K THUT GII TON

V d 1.119 Cho a; b; c l di 3 cnh ca mt tam gic. Chng minh rng


r
b
c
a
3
p
+p
+p
(a + b + c)
2
c+a
a+b
b+c
(V Quc B Cn)
Hng dn. S dng bt ng thc Cauchy Schwarz,
X
cyc

a
p
a+b

!2

"

b2

a(a + b + 2c)

cyc

V d 1.120 Cho cc s a; b; c
p

#"

X
cyc

#
a
:
(a + b)(a + b + 2c)

0: Chng minh rng

a
b
c
+p
+p
2
2
+ 15ca
c + 15ab
a + 15bc

3
:
4
(Park Doo Sung)

Hng dn. S dng bt ng thc Holder

X
cyc

a
p
b2 + 15ca

V d 1.121 Cho cc s a; b; c
p

b2

!2

"

#3

a(a + 2b)

cyc

a(a + 2b)3 (b2 + 15ca)

cyc

0: Chng minh rng

a
b
c
+p
+p
2
2
2
2
+ 3c
c + 3a
a + 3b2

3
:
2
(Vasile Cirtoaje)

Hng dn. S dng bt ng thc Holder

X
cyc

a
p
b2 + 3c2

!2

cyc

"

cyc

#3

a(2a + b)

a(2a + b)3 (b2 + 3c2 )

1.5. THE HYBERBOLIC FUNCTIONAL TECHNIQUE


V d 1.122 Cho cc s a; b; c
p

143

0: Chng minh rng

b
c
a
+p
+p
2
2
+ ca
3c + ab
3a + bc

3b2

3
:
2
(Vasile Cirtoaje)

Hng dn. S dng bt ng thc Holder

X
cyc

1.5
1.5.1

a
p
3b2 + ca

!2

"

cyc

#3

a(a + c)

a(a + c)3 (3b2 + ca)

cyc

The Hyberbolic functional technique


Li ni u

K thut ny c v l kh mi m nu cc bn ch xem tn ca k thut thi nhng


thc ra n tng c gii thiu rt nhiu ln trn cc din n, cc tp ch vi
ci tn phng php tip tuyn chng minh bt ng thc (chng hn nh [5]).
Nhng, trong cc bi vit , cc tc gi u cha khai thc tht trit cc tnh
cht ca tip tuyn k thut tr nn mnh hn v c s dng nhiu hn trong
chng minh bt ng thc. y, trong bi vit ny, chng ti xin c gii thiu
cng cc bn mt s tm ti ca mnh trong vic lm mnh k thut trn.

1.5.2

Mt s v d m u

chng minh mt bt ng thc f (x1 ) + f (x2 ) +


+ f (xn ) 0; m vic nh
gi f (x) gp nhiu kh khn th chng ta s tm mt hm g(x) d nh gi hn sao
cho f (x) g(x) v ta ch cn vic phi chng minh bt ng thc cn li cht hn
nhng d hn l
g(x1 ) + g(x2 ) +
+ g(xn ) 0:
V d 1.123 Cho cc s khng m a; b; c; khng c 2 s no ng thi bng 0: Chng
minh rng
b
c
3
a
+
+
:
b+c c+a a+b
2
(Nesbitt)

144

CHNG 1. TM TI MT S K THUT GII TON

Li gii. Chun ha cho a + b + c = 3; bt ng thc tr thnh


a
3
Vi mi x

b
3

c
3

3
2

3; ta c
4x
3

x
3(x
,
3
Do

4a
3

4b
3

4c
3

(3a

3x

1)2
x

0:

1) + (3b

1) + (3c

1) = 6

Bt ng thc c chng minh.


V d 1.124 Cho cc s khng m a; b; c; tt c khng i bng 0: Chng minh rng
(2a + b + c)2
(2b + c + a)2
(2c + a + b)2
+
+
2a2 + (b + c)2
2b2 + (c + a)2
2c2 + (a + b)2

8:
(USAMO 2003)

Li gii. Chun ha cho a + b + c = 3; bt ng thc tr thnh


(3 + a)2
(3 + b)2
(3 + c)2
+ 2
+ 2
2
2
+ (3 a)
2b + (3 b)
2c + (3 c)2

2a2

,
Vi mi x

X a2 + 6a + 9
cyc

a2

2a + 3

24

3; ta c
x2 + 6x + 9
4x + 4
x2 2x + 3
(4x + 3)(x 1)2
,
0
x2 2x + 3

Do

X a2 + 6a + 9
cyc

a2

2a + 3

(4a + 4) = 24

cyc

Bt ng thc c chng minh.


V d 1.125 Cho cc s dng a; b; c tha mn abc = 1: Chng minh rng
p
p
p
p
a2 + 1 + b2 + 1 + c2 + 1
2(a + b + c):

(Gabriel Dospinescu)

1.5. THE HYBERBOLIC FUNCTIONAL TECHNIQUE


Li gii. Xt hm s f (x) =

f 0 (x) =

x2 + 1

x+

p1
2

145

ln x vi x > 0; ta c

i
h
p
x) 1 x + 2x2 + 2x2 2(1 + x2 )
p
p
p
x 2(x2 + 1) 2x2 + x2 + 1

(1

f 0 (x) = 0 , x = 1

T y d thy
f (x) f (1) = 0 8x > 0
p
p
1
) x2 + 1
2x p ln x
2

Do
p
p
p
a2 + 1 + b2 + 1 + c2 + 1

2(a + b + c)

p
1
p (ln a + ln b + ln c) = 2(a + b + c)
2

Bt ng thc c chng minh.


Cu hi t ra l lm sao chng ta c th chn c cc hm g(x) thch hp?
Tht ra, y hm g(x) c la chn da vo iu kin rng buc cc bin ca bi
ton, chng hn nh nu iu kin l x1 + x2 +
+ xn = n th g(x) = k(x 1); nu
xn = 1 th g(x) = k ln x
x21 + x22 +
+ x2n = n th g(x) = k(x2 1), v nu x1 x2
vi k l hng s m ta s chn sau. ( y ta gi s bt ng thc c ng thc xy
ra ti x1 = x2 =
= xn = 1). y nu f c o hm v lin tc li x = 1 th
k = f 0 (1): Nhng trong mt vi trng hp, ta khng cn phi tnh o hm lm g
m vn c th d dng chn bng php bin i tng ng, chng hn nh bt
ng thc Nesbitt, chng ta cn chn sao cho
x
3

x
, (x

k(x
1)

1) +

1
2

3
2(3

x)

8x 2 (0; 3)
k

bt ng thc ny khng i du khi x chy qua gi tr 1 th ta phi chn k sao


cho
3
k=0
2(3 x)
c nghim x = 1 (nu khng th bt ng thc s khng ng), t suy ra k = 43 :
V d 1.126 Cho cc s thc a; b; c tha mn a + b + c = 6: Chng minh rng
a4 + b4 + c4

2(a3 + b3 + c3 ):

146

CHNG 1. TM TI MT S K THUT GII TON

V d 1.127 Cho cc s khng m a; b; c tha mn a + b + c = 3: Chng minh rng


p
p
p
a + b + c ab + bc + ca:
(Russia MO 2002)
Hng dn.
2(ab + bc + ca) = 9

a2

b2

c2 :

V d 1.128 Cho cc s dng a; b; c tha mn a + b + c = 1: Chng minh rng


a
b
c
+
+
1 + bc 1 + ca 1 + ab
Hng dn.
bc

9
:
10

(b + c)2
(1 a)2
=
:
4
4

V d 1.129 Cho cc s dng a; b; c tha mn abc = 1: Chng minh rng


a2 + b2 + c2 + 9(ab + bc + ca)

10(a + b + c):
(Vasile Cirtoaje)

V d 1.130 Cho cc s dng a; b; c. Chng minh rng


(b + c a)2
(c + a b)2
(a + b c)2
+ 2
+ 2
2
2
2
a + (b + c)
b + (c + a)
c + (a + b)2

3
:
5
(Japan MO 1997)

1.5.3

t vn

i vi nhng bi ton khng cht, cch lm trn t ra rt hiu qu nhng i vi


nhng bi ton tng i cht th chng ta kh lng dng n gii quyt, bi v
bt ng thc f (x) g(x) lc ny khng phi lun ng na m n ch ng trong
mt s trng hp. Chng hn nh vi bi ton sau
V d 1.131 Cho cc s thc a; b; c tha mn a + b + c = 1: Chng minh rng
a
b
c
+
+
a2 + 1 b2 + 1 c2 + 1

9
:
10

1.5. THE HYBERBOLIC FUNCTIONAL TECHNIQUE

147

Bng cch tng t, chng ta thit lp c bt ng thc


36x + 3
50

x
x2 + 1

3
4

Nhng tic thay bt ng thc ny ch ng trong trng hp x


36x + 3
50

x
(4x + 3)(3x 1)2
=
x2 + 1
50(x2 + 1)

trong khi gi thit li yu cu chng ta chng minh bi ton vi cc s thc ty .


V th, cch lm ny khng pht huy c tc dng.
Nhng chng ta cng c th gii quyt bi ton bng cch chia thnh tng trng
hp nh gii.
Vi bi ton trn, chng ta c th gii quyt n nh sau
3
Li gii. Trng hp 1. Nu min fa; b; cg
4 ; khi s dng bt ng thc
36x + 3
50

x
x2 + 1

3
4

8x

Ta d dng suy ra kt qu bi ton.


Trng hp 2. Gi s tn ti mt s trong ba s a; b; c nh hn
Khi , s dng bt ng thc AM-GM, ta c
b
+1

1
2

a
a2 + 1

2
5

b2

3
4;

chng hn c <

3
4:

Nn bt ng thc ca ta s ng nu

,a
Do , ta ch cn xt trng hp 2
trng hp 2 a; b 12 ; khi ta c

1
_a 2
2
1
a
2 : Hon ton tng t, ta ch cn xt

3
>c=1
4
)
Do
a2

c2

c
+1

a
b
c
+ 2
+ 2
+1 b +1 c +1

3
10
1 1
+
2 2

3
7
9
=
<
:
10
10
10

Bt ng thc c chng minh xong. ng thc xy ra khi v ch khi a = b = c = 31 :

148

CHNG 1. TM TI MT S K THUT GII TON

V d 1.132 Cho cc s dng a; b; c tha mn abc = 1: Chng minh rng


3a2

1
+ (a

1)2

3b2

1
+ (b

1)2

3c2

1
+ (c

1:

1)2

(L Hu in Khu)
Li gii. Xt hm s f (x) =

1
3x2 +(x 1)2

f 0 (x) =

1
3

2
3

ln x; ta c

2(x 1)(16x3 1)
3x(4x2 2x + 1)2

T y d thy f 0 (x) = 0 khng phi ch c mt nghim x = 1 m cn c thm mt


1
nghim na l x = 2 p
nn chc chn rng bt ng thc f (x) 0 m ta ang mong
3
2
i s khng phi lun ng. Vy ta hy xem xt xem n ng trong trng hp no?
Bng kim tra trc tip, ta thy c f (x) 0 8x 21 : T y, dn n li gii nh
sau
Nu min fa; b; cg 12 khi ta c
X
cyc

By gi, ta gi s a
X
cyc

3a2

3a2

1
2;

1
+ (a

1
+ (a

1)2

X
cyc

1
3

2
ln a
3

= 1:

khi ta c
1)2

3a2

1
+ (a

1)2

1
2a(2a

1:

1) + 1

nn bt ng thc cho lun ng. Ta c pcm.


V d 1.133 Cho cc s khng m a; b; c; khng c 2 s no ng thi bng 0: Chng
minh rng
V d 1.134
a2
b2
c2
+
+
5a2 + (b + c)2
5b2 + (c + a)2
5c2 + (a + b)2

1
:
3
(V Quc B Cn)

Li gii. Gi s

a+b+c=1
a b c)a

1
3

6a2

cyc

, bt ng thc tr thnh

a2
2a + 1

1
3

1.5. THE HYBERBOLIC FUNCTIONAL TECHNIQUE


1
8;

Nu c

khi ta c

27

1
3

X
cyc

6a2

a2
2a + 1

X (1
cyc

1
8;

1
6

12q

27a2
6a2 2a + 1

cyc

=
Nu c

149

3a)2 (8a 1)
6a2 2a + 1

ta c
X
cyc

6a2

a2
2a + 1

1 2b
6c2
2a
+ 2
2
2a + 1 6b
2b + 1 6c
2c + 1
b+c a
a+c b
6c2
=
+
6a2 2a + 1 6b2 2b + 1 6c2 2c + 1
1
2(a b)2 (2 3c)
+c
+
=
(6a2 2a + 1)(6b2 2b + 1)
6a2 2a + 1 6b2
1
1
6c
c
+
6a2 2a + 1 6b2 2b + 1 6c2 2c + 1
1
1
c
+
1
6a2 2a + 1 6b2 2b + 1

6a2

1
2b + 1

6c2

6c
2c + 1

Do , ta ch cn chng minh
6a2

1
+
2a + 1 6b2

1
2b + 1

Nu b 13 th bt ng thc ny hin nhin ng, nu b


Cauchy Schwarz
6a2

1
+
2a + 1 6b2

1
2b + 1

th theo bt ng thc

4
+
2(a + b) + 2
2
3(a2 + b2 ) (a + b) + 1
2(a + b + c)2
3(a2 + b2 ) + c(a + b + c)
6(a2

1
3

b2 )

Ta cn chng minh
2(a + b + c)2

3(a2 + b2 ) + c(a + b + c)

150

Do b

CHNG 1. TM TI MT S K THUT GII TON

1
3

3(a2 + b2 )

, (a + b + c)(2a + 2b + c)
nn 3b

a; suy ra
2(a + b)2 = 3(a2 + b2 ) + b(a

(a + b + c)(2a + 2b + c)

b) + a(3b

a)

3(a2 + b2 ):

Bt ng thc c chng minh xong. ng thc xy ra khi a = b = c hoc a =


b; c = 0 v cc hon v.
V d 1.135 Cho cc s khng m x; y; z tha mn x2 + y 2 + z 2 = 3: Chng minh
rng
x5 x2
y5 y2
z5 z2
+
+
0:
x5 + y 2 + z 2
y 5 + z 2 + x2
z 5 + x2 + y 2
(Vasile Cirtoaje)
Li gii. Bt ng thc tng ng
X
cyc

1
x5 + y 2 + z 2
,

X
cyc

3
x2 + y 2 + z 2
1
x2 + 3

x5

1:

S dng bt ng thc AM-GM, ta c


x5 =

2x6
x2 + 1

x6
x

t a = x2 ; b = y 2 ; c = z 2 ) a + b + c = 3 th ta ch cn chng minh
X

2a3
cyc a+1

,
,

2a3

cyc

X (a
cyc

a+3

a+1
a2 + 2a + 3

1)2 (3 + 3a 2a2 )
a2 + 2a + 3

2a3

Gi s a b c ) a 1 c: Xt 2 trng hp
Trng hp 1. b + c 1 ) a 2; khi ta c
3 + 3a

2a2

Nn bt ng thc ng.

0;

3 + 3b

2b2

0;

3 + 3c

2c2

1.5. THE HYBERBOLIC FUNCTIONAL TECHNIQUE


Trng hp 2. b + c
2a3

a2 + 2a + 3

1)a

151

2; ta c
= 2a3

5(a + 1)

a2

a3 2
)

2a3

2 = a3 2

3a
1
2

2
23

c+1
c2 + 2c + 3

4
5

3
22

1
a

3
a2

2
a3

1 3
a >0
2

1
5

a+1
a2 + 2a + 3

Do ta ch cn chng minh
b+1
+
b2 + 2b + 3 2c3

2b3

Nhng bt ng thc ny hin nhin ng v vi mi ta c

Nu x
4x3

1
2

2
5

x+1
x2 + 2x + 3

2x3
, 4x3

(x + 1)(2x

1)
1
2;

th bt ng thc hin nhin ng, nu x


(x + 1)(2x

1)

4x3

2(2x

1)

2x2

ta c

2(2x

1) = 2(x

1)2

0:

Bt ng thc c chng minh.


V d 1.136 Cho cc s khng m a; b; c tha mn a + b + c = 3: Chng minh rng
p

a2

1
1
1
+p
+p
2
2
3a + 3
b
3b + 3
c
3c + 3

3:
(Nguyn Vn Thch)

Li gii. Chng ta thit lp c bt ng thc sau


p
Gi s a

c)a

x2

2
3x + 3

c: Nu c

cyc

Xt trng hp ngc li, c

a2
p

x + 1 8x
p

5 1
2

2
3a + 3

5 1
2 ;

5 1
2

th ta c
X

(a + 1) = 6

cyc

ta xt 2 trng hp nh

152

CHNG 1. TM TI MT S K THUT GII TON

Nu b

1; ta c
a2

3a + 3 =

b2
c2

1
3a + 3

Nu b

1)2

1; xt hm s f (x) =

cyc

a2

f "(x) =

8x2
4(x2

3
4

3
4

b)(1 b) + 1 1
!2
p
p
5 1
5 1
16
+2
= p
2
2
5+1
p
5+1
2
p +1+
<3
2
3

c)2 + 2

3c + 3 = (1

3b + 3 = (2

3
2

1
x2 3x+3

vi x 2 [1; 2]; ta c

24x + 15
< 0 8x 2 [1; 2]
3x + 3)3=2

Nn t y, theo bt ng thc Jensen, ta c


f (a) + f (b)

2f

a+b
2

= 2f (t) = p

t2

2
3t + 3

Ta cn chng minh
p

t2

2
+p
3t + 3
(3

,p
,

36(t

1)2 (36t6

t2

1
2t)2

3(3

2t) + 3

2
1
+p
2
3t + 3
4t
6t + 3

252t5 + 749t4 1202t3 + 1099t2


(t2 3t + 3)2 (4t2 6t + 3)2

3
546t + 117)

0:

Bt ng thc cui hin nhin ng. Vy ta c pcm.


Cch chia nhng trng hp v i n nhng nh gi khi qut nh cc v d rt
hay v c o nhng rt kh thc hin. V vy, chng ta cn tm mt con ng
khc gii quyt bi ton tt hn khi m chng ta khng th tm c mt nh
gi no c.

1.5.4

Gii quyt vn

Cch gii quyt vn ca chng ta y hon ton tng t nh trn. Trc ht,
chng ta s tm iu kin bt ng thc f (x)
g(x) c tha mn vi mi
x1 ; x2 ; :::; xn . Trong trng hp ngc li, tn ti mt bin xn sao cho f (xn ) g(xn );
chng ta s tm mt hm g 0 (x) sao cho f (x) g 0 (x) vi mi x1 ; x2 ; :::; xn 1 (cch tm

1.5. THE HYBERBOLIC FUNCTIONAL TECHNIQUE

153

hm g 0 (x) cng phi da trn gi thit ca bi ton a ra). Khi , chng ta s a


bi ton v chng minh
g 0 (x1 ) + g 0 (x2 ) +

+ g 0 (xn

1)

+ f (xn )

T y, chng ta s da trn mi quan h rng buc gia x1 ; x2 ; :::; xn gii quyt


bi ton.
Mt iu cn lu l s tn ti rt nhiu hm g 0 (x) tha mn f (x) g 0 (x): Vy ta
lm th no chn c hm g 0 (x) thch hp? Chng ti s khng ni r ra iu
ny, cc bn hy xem xt tht k cc v d di y s hiu ra ngay cch lm ca
chng ta.
V d 1.137 Cho cc s khng m a; b; c tha mn a + b + c = 3. Chng minh rng
1
1
1
+
+
2a2 + 3 2b2 + 3 2c2 + 3

3
:
5
(V Quc B Cn)

Li gii. Khng mt tnh tng qut, ta c th gi s a


Trng hp 1. Nu c 14 , ta c
5
+3

2x2

4
1 + (x
5
)
)

Trng hp 2. Nu c
15
2
2x + 3

cyc

1
4,

4
(x
5

5
+3

2a2

cyc

0 8x

1
4

1
4

1) 8x
4
(a
5

c. Xt 2 trng hp

1) = 3

ta c

4
2 + (2x
5
)

5
+3

2(4x 1)(x 1)2


5(2x2 + 3)

1) =

2x2

3) =

(4x + 1)(2x 3)2


5(2x2 + 3)

4
2 + (3
5

15
+3

2x2

15
15
+
2a2 + 3 2b2 + 3

4
4 + (6
5

2x) 8x
2a

Ta cn chng minh
15
8
+ c
+3 5

2c2

0 8x
0

8
2b) = 4 + c
5

154

CHNG 1. TM TI MT S K THUT GII TON

hin nhin ng v
15
8
+ c
2
2c + 3 5

5=

2c[8c2 + 23c + 12(1


5(2c2 + 3)

4c)]

0:

ng thc xy ra khi v ch khi a = b = c = 1 hoc a = b = 32 ; c = 0 hoc cc hon


v tng ng.
V d 1.138 Cho cc s khng m a; b; c; d tha mn a + b + c + d = 2: Chng minh
rng
1
1
1
16
1
+
+
+
:
3a2 + 1 3b2 + 1 3c2 + 1 3d2 + 1
7
(Vasile Cirtoaje)
Li gii. Khng mt tnh tng qut, gi s a
1
Trng hp 1. Nu d 12
, ta c
7
3x2 + 1

4+

24
(2x
7

1) =

7
3x2 + 1
X
7

)
)
Trng hp 2. Nu d
7
2
3x + 1

1
12 ,

)
)

3a2 + 1

cyc

3+

d, ta xt 2 trng hp

3(12x 1)(2x 1)2


7(3x2 + 1)

1
12

0 8x

24
1
(2x 1) 8x
7
12
X
24
(2a 1) = 16
4
7
cyc

ta c

12
(3x
7

2) =

7
+1

3x2

3+

7
7
7
+
+
3a2 + 1 3b2 + 1 3c2 + 1

(12x + 1)(3x 2)2


7(3x2 + 1)
12
(2
7
9+

3x) 8x
12
(6
7

3a

0 8x

0
3b

3c) = 9 +

36
c
7

Ta cn chng minh
7
36
+ d
3d2 + 1
7

hin nhin ng v
7
36
+ d
+1
7

3d2

7=

3d[36d2 + 95d + 12(1


7(3d2 + 1)

12d)]

0:

Bt ng thc c chng minh. ng thc xy ra khi v ch khi a = b = c = d =


hoc a = b = c = 23 ; d = 0 hoc cc hon v tng ng.

1
2

1.5. THE HYBERBOLIC FUNCTIONAL TECHNIQUE

155

V d 1.139 Cho cc s khng m a; b; c; tt c khng ng thi bng 0: Chng minh


rng
b2
c2
2
a2
+ 2
+ 2
:
2
2
2
2a + (b + c)
2b + (c + a)
2c + (a + b)2
3
(Darij Grinberg)
Li gii. Chun ha cho a + b + c = 1 v gi s a b
thnh
X
2
a2
2
3a
2a
+
1
3
cyc
Ta xt 2 trng hp
Trng hp 1. Nu b

1
6,

3x2
3x2 2x + 1
)
)

c, khi bt ng thc tr

ta c
4
(2x
3

1) =

3x2
3x2 2x + 1

3a2
3b2
+
3a2 2a + 1 3b2 2b + 1

(6x 1)(2x 1)2


3(3x2 2x + 1)
4
(1
3
2

2x) 8x
4
(2
3

2a

1
6
2b) = 2

Mt khc
3c2
2c + 1

3c2
)

X
cyc

1
6

Trng hp 2. Nu

3x2

2+

c)a=1

x2
2x + 1
)

3b2

c[(1

3c2
2c + 1

3c2

8
c=
3

3c)(17 24c) + 7]
9(3c2 2c + 1)
3c2

3c2
2c + 1

2
3,

8
c
3

3x2

x2
2x + 1

0 8x

2
x 8x
9

b2
c2
+ 2
2b + 1 3c
2c + 1

3a2

2
a
9

2
2
(b + c) =
9
9
4
9

1
6

1
6

Ta cn chng minh
a2
2a + 1

ta c

2
x(6x 1)(2 x)
x=
9
9(3x2 2x + 1)

1
6

0 8x

2
a
9

8
c
3

156

CHNG 1. TM TI MT S K THUT GII TON

hin nhin ng v

3a2

a2
2a + 1

2
a
9

4
=
9

2)(6a2 + 3a 4) + 4
27(3a2 2a + 1)

(3a

0:

Bt ng thc c chng minh. ng thc xy ra khi v ch khi a = b; c = 0 hoc


cc hon v.
V d 1.140 Cho cc s khng m a; b; c; tt c khng ng thi bng 0: Chng minh
rng
b2
c2
1
a2
+
+
:
5a2 + (b + c)2
5b2 + (c + a)2
5c2 + (a + b)2
3
(V Quc B Cn)
Li gii. Chun ha cho a + b + c = 1 v gi s a b
thnh
X
a2
1
2
6a
2a
+
1
3
cyc
Ta xt 2 trng hp
Trng hp 1. Nu c

6x2

1
8;

9x2
2x + 1

1
8,

6x2
2x + 1

(8x 1)(3x 1)2


3(6x2 2x + 1)

1) =

9x2
2x + 1

9a2
6a2 2a + 1

cyc

)
)

4
(3x
3

6x2

Trng hp 2. Nu c

6x2

th

c, khi bt ng thc tr

4
(1
3
X
1
cyc

1
8

3x) 8x
4
(1
3

1
8

0 8x

3a) = 3

th

6x2

2
(2x
3

1) =

6x2
2x + 1

6a2
6b2
+
6a2 2a + 1 6b2 2b + 1

(6x + 1)(2x 1)2


3(6x2 2x + 1)
2
(1
3

2x) 8x
2
(2
3

Ta cn chng minh
6c2

6c2
2c + 1

4
c
3

2a

0 8x

0
2b) = 2

4
c
3

1.5. THE HYBERBOLIC FUNCTIONAL TECHNIQUE

157

hin nhin ng v
6c2

6c2
2c + 1

2c[12c2 + 3c + 2(1 8c)]


3(6c2 2c + 1)

4
c=
3

0:

Bt ng thc c chng minh. ng thc xy ra khi v ch khi a = b = c hoc


a = b; c = 0 hoc cc hon v tng ng.
V d 1.141 Cho cc s khng m a; b; c; d tha mn a + b + c + d = 4. Chng minh
rng
(a2 + 1)(b2 + 1)(c2 + 1)(d2 + 1) (a + 1)(b + 1)(c + 1)(d + 1):
(Phan Thnh Vit)
Li gii. Khng mt tnh tng qut, gi s a
b
ng vi
X
[ln(a2 + 1) ln(a + 1)]

d. Bt ng thc tng

cyc

Ta xt 2 trng hp
Trng hp 1. Nu a
x 2, ta c

2, xt hm s f (x) = ln(x2 + 1)
f 0 (x) =

T y, ta d dng kim tra c f (x)


[ln(a2 + 1)

x=

ln(a + 1)]

X1
cyc

g 0 (x) =

(a

2. Suy ra

1) = 0

b c d, xt hm s g(x) = ln(x2 + 1)
2, ta c
(3x

1) vi

minff (1); f (2)g = 0 8x

cyc

Trng hp 2. Nu a 2 ) 2
7
1) 65
(3x 2) ln 13
15 with x

1
2 (x

(x 1)(3 x2 )
2(x2 + 1)(x + 1)

f 0 (x) = 0 , x = 1
X

ln(x + 1)

ln(x +

2)(43 + 10x 7x2 )


65(x2 + 1)(x + 1)

g 0 (x) = 0 , x =
T y, ta d dng kim tra c g(x)

2
3

2
3

= 0 8x

2. Suy ra

[ln(b2 + 1) ln(b + 1)] + [ln(c2 + 1) ln(c + 1)] + [ln(d2 + 1)


7
13
21
13
(3b + 3c + 3d 6) + 3 ln
=
(2 a) + 3 ln
65
15
65
15

ln(d + 1)]

158

CHNG 1. TM TI MT S K THUT GII TON

Ta cn chng minh
h(a) = ln(a2 + 1)
Ta c

ln(a + 1) +

21
(2
65

a) + 3 ln

13
15

2)(43 + 10a 7a2 )


65(a2 + 1)(a + 1)
p
5 + 326
0
h (a) = 0 , a =
7

h0 (a) =

(3a

T y, ta d thy h(a) minfh(2); h(4)g > 0 8a 2.


Bt ng thc c chng minh. ng thc xy ra khi v ch khi a = b = c = d = 1.
V d 1.142 Cho cc s khng m a; b; c; khng c 2 s no ng thi bng 0: Chng
minh rng
r
r
r
48a
48b
48c
1+
+ 1+
+ 1+
15:
b+c
c+a
a+b
(Vasile Cirtoaje)
Li gii. Chun ha cho a + b + c = 1 v gi s a b
thnh
r
X 1 + 47a
15
1 a
cyc
Ta xt 2 trng hp
Trng hp 1. Nu c
1 + 47x
1 x

Trng hp 2. Nu c
1 + 47x
1 x

2
27 ,

c, khi bt ng thc tr

th
2

54
7
12(27x 2)(3x 1)2
x+
=
0 81
5
5
25(1 x)
r
1 + 47x
54
7
2
)
x+
81 x
1 x
5
5
27
r
X 1 + 47a X 54
7
)
x+
= 15
1
a
5
5
cyc
cyc
2
27 ,

2
27

th
2

96
1
48(48x + 1)(2x 1)2
x+
=
7
7
49(1 x)
r
1 + 47x
96
1
)
x+
81 x
1 x
7
7

0 81
0

1.5. THE HYBERBOLIC FUNCTIONAL TECHNIQUE

1 + 47a
+
1 a

Ta cn chng minh

1+47c
1 c

= t2 (t

1 + 47b
1 b

r
11
5

0) )

96
2
(a + b) + = 14
7
7

1 + 47c
1 c

1+

96
c
7

96
c
7

1, bt ng thc tr thnh
t

96(t2 1)
7(t2 + 47)

1+
1)(7t2

, (t

159

96t + 233)

0:

hin nhin ng do 11
t 1:
5
Bt ng thc c chng minh. ng thc xy ra khi v ch khi a = b = c hoc
a = b; c = 0 hoc cc hon v tng ng.
V d 1.143 Cho cc s khng m a; b; c tha mn a + b + c = 1. Chng minh rng
1 4a2
1 4b2
1 4c2
+
+
2
2
1 + 3a 3a
1 + 3b 3b
1 + 3c 3c2

1:
(Michael Rozenberg)

Li gii. Khng mt tnh tng qut, gi s a


Trng hp 1. Nu c 91 , th
1 4x2
1 + 3x 3x2

1
3

3
(1
5
)

X
cyc

Trng hp 2. Nu c
1 4x2
1 + 3x 3x2

1 4a2
1 + 3a 3a2

1
9,

(1 9x)(1
15(1 + 3x

1 4x2
1 + 3x 3x2

c. Ta xt 2 trng hp
3x)2
3x2 )

1 3
+ (1
3 5

0 8x 2

1
;1
9

3x)

X 1 3
+ (1
3 5
cyc

3a) = 1

th

8
(1
7
)

3x) =

2x) =

(12x + 1)(2x 1)2


7(1 + 3x 3x2 )

1 4x2
1 + 3x 3x2

1 4a2
1 4b2
+
1 + 3a 3a2
1 + 3b 3b2

8
(1
7
8
(2
7

0 8x 2 [0; 1]

2x)
2a

2b) =

16
c
7

160

CHNG 1. TM TI MT S K THUT GII TON

Ta cn chng minh
1 4c2
16
1
c+
7
1 + 3c 3c2
c(48c2 41c + 5)
,
0:
7(1 + 3c 3c2 )
hin nhin ng do c 91 :
Bt ng thc c chng minh. ng thc xy ra khi v ch khi a = b = c =
a = b = 12 ; c = 0 hoc cc hon v tng ng.

1
3

hoc

V d 1.144 Cho cc s khng m a; b; c; d tha mn a + b + c + d = 1. Chng minh


rng
(1 + 2a)(1 + 2b)(1 + 2c)(1 + 2d)
125
:
(1 a)(1 b)(1 c)(1 d)
8
(Vasile Cirtoaje)
Li gii. Khng mt tnh tng qut, gi s a b
tng ng vi
X
[ln(1 + 2a) ln(1 a)]

d)d

3 ln

5
2

cyc

1
4.

Bt ng thc

Ta xt 2 trng hp
Trng hp 1. Nu c x0 vi x0 l nghim thuc (0; 0:09) ca phng trnh ln(1 +
9
2x) ln(1 x) 10
(3x 1) ln 52 = 0, xt hm s f (x) = ln(1 + 2x) ln(1 x)
9
5
1) ln 2 with x x0 , ta c
10 (3x
f 0 (x) =

3(6x 1)(3x
10(1 + 2x)(1
1
_
6
min f

1)
x)

f 0 (x) = 0 , x =
T y, ta d dng kim tra c f (x)

1
3
; f (x0 ) = 0 8x

x=
1
3

[ln(1 + 2a) ln(1 a)] + [ln(1 + 2b) ln(1 b)] + [ln(1 + 2c)
9
5
27
5
(3a + 3b + 3c 3) + 3 ln =
c + 3 ln
10
2
10
2
Ta cn chng minh
g(d) = ln(1 + 2d)
Ta c
g 0 (d) =

ln(1

d)

3(6d 1)(3d
10(1 + 2d)(1

27
d
10
1)
d)

x0 . Suy ra
ln(1

c)]

1.5. THE HYBERBOLIC FUNCTIONAL TECHNIQUE

161

1
6
T y, ta d dng kim tra c g(d) min g(0); g 41 = 0.
Trng hp 2. Nu x0
c d, xt hm s h(x) = ln(1 + 2x) ln(1
1) 2 ln 2 vi x 0, ta c
g 0 (d) = 0 , d =

h0 (x) =

x)

3
2 (2x

3x(2x 1)
(1 + 2x)(1 x)

h0 (x) = 0 , x = 0
T y, ta d dng kim tra c h(x)

_
1
2

x=

1
2

= 0 8x

0. Suy ra

[ln(1 + 2a) ln(1 a)] + [ln(1 + 2b) ln(1 b)]


3
(2a + 2b 2) + 4 ln 2 = 3c 3d + 4 ln 2
2
Ta cn chng minh
k(c) + k(d)

0
3
2

trong k(x) = ln(1 + 2x) ln(1 x) 3x + 2 ln 2


chng minh c k(x) 0 8x x0 . Ta c
k 0 (x) =
Suy ra k(x) nghch bin vi mi x
k(x)

3x(2x 1)
(1 + 2x)(1 x)

=
>

x0 , do o

k(x0 ) = ln(1 + 2x)


=

ln 52 . V v th, ta ch cn

ln(1

x)

3x + 2 ln 2

3 5
ln
2 2

9
5
3 5
(3x0 1) + ln
ln
3x0 + 2 ln 2
10
2
2 2
3
9
1 5
x0 + 2 ln 2
ln
10
10 2 2
3
9
1 5
0:09 + 2 ln 2
ln
0:0011 > 0:
10
10 2 2

Bt ng thc c chng minh. ng thc xy ra khi v ch khi a = b = c = 31 ; d = 0


hoc cc hon v tng ng.
V d 1.145 Let a; b; c; d be nonnegative real numbers such that a + b + c + d = 2.
Prove that
a2
b2
c2
d2
16
+
+
+
:
2
2
2
2
2
2
2
2
(a + 1)
(b + 1)
(c + 1)
(d + 1)
25
(Ji Chen)

162

CHNG 1. TM TI MT S K THUT GII TON

Li gii. Khng mt tnh tng qut, gi s a


Trng hp 1. Nu 12d3 + 11d2 + 32d 4, th
X
cyc

a2
(a2 + 1)2

24
(2a
125
=

Trng hp 2. Nu 4
x2
(x2 + 1)2

X (12a3 + 11a2 + 32a


125(a2

cyc

24 X
(2a
125 cyc

a2
(a2 + 1)2

12d3 + 11d2 + 32d

1) +

x2
(x2 + 1)2

540
2197

1
8,

32d ) d

b2
c2
d2
+
+
(b2 + 1)2
(c2 + 1)2
(d2 + 1)2

2
3

1)2

ta c

2)2

36
169

540
(b + c + d
2197

4)(2a
1)2

16
16
=
25
25

540
2
36
x
2197
3
169
(60x3 + 92x2 + 216x + 27)(3x
=
2197(x2 + 1)2
)

d. Ta xt 2 trng hp

4
25

1)

cyc

8x
2) +

0 8x

0
108
=
169

540
108
a+
2197
169

Ta cn chng minh
(a2

a2
+ 1)2

540
108
a+
2197
169

169a2
(a2 + 1)2

540
a
13

16
25
4
25

Ta c
169a2
(a2 + 1)2

540
a
13

4
25

540
4
169a2
4
169a2
a
=
36a
(a2 + 1)2
15
25
(a2 + 1)2
25
4 + 900a 4217a2 + 1800a3 + 4a4 + 900a5
=
25(a2 + 1)2
2
4 + 8 900a
4217a2 + 1800a3 + 4a4 + 900a5
25(a2 + 1)2
2
4 + 2983a + 1800a3 + 4a4 + 900a5
=
< 0:
25(a2 + 1)2

Bt ng thc c chng minh. ng thc xy ra khi v ch khi a = b = c = d = 12 :

1.5. THE HYBERBOLIC FUNCTIONAL TECHNIQUE


V d 1.146 Cho cc s khng m a1 ; a2 ; :::; an (n
n. Chng minh rng
1)(a31 + a32 +

(n

+ a3n ) + n2

163

2) tha mn a1 +a2 +
1)(a21 + a22 +

(2n

+an =

+ a2n ):
(Vasile Cirtoaje)

Li gii. Nu n = 2, bt ng thc tr thnh ng thc. Nu n = 3, t q =


a1 a2 + a2 a3 + a3 a1 ; r = a1 a2 a3 , bt ng thc tr thnh
2(27

9q + 3r) + 9
, 3r + 9

5(9

4q

2q)

y chnh l bt ng thc Schur bc 3.


Suy ra, ta ch cn xt trng hp n 4. Khng mt tnh tng qut, gi s a1
an . Ta xt 2 trng hp
1
Trng hp 1. Nu an
n 1 , ta c
(n

1)x3

1)x2 + (n + 1)(x

(2n

1)x3 + n

) (n
)

n
X

1)x3

n
X1

n
X

1)a3i + n]

[(n

i=1

(n

[(2n

1)a2i

1)x

1]

0 8x

1) 8x

(n + 1)(ai

1)]

(2n

n2
n

1)a2i +

i=1

n
X1
i=1

i=1

n(n 2)
n2
[(n
1)x
n]
+
(n 1)2
n 1
2
[(n 1)x n] [(n 1)x + 1]
=
(n 1)2

1)x2 +

1)

n
X
1) a2i

ta c

1)x2 +

1)a3i

(2n

i=1

n 1,

(2n

, (n

(n + 1)(x

n
X
1) a3i + n2

(2n

1)x3

1)2 [(n

i=1

Trng hp 2. Nu an

) (n

1)x2

(2n

, (n

(n

1) + n = (x

a2

a3i

(2n

n(n 2)
[n
(n 1)2

1
n2
n

1
n
X1

1)

i=1

(n

1)x] 8x

n 1
n(n 2) X
[n
(n 1)2 i=1

a2i + n2

0 8x

(n

n(n 2)
an
n 1

0
0

1)ai ]

164

CHNG 1. TM TI MT S K THUT GII TON

Ta cn chng minh
(n
,

an [(n

1)a3n

1)a2n +

(2n

1)(2

an )[1

1)an ] + n2

(n
n

n(n 2)
an
n 1

4n + 2]

0:

hin nhin ng do n 4:
Bt ng thc c chng minh.

1.5.5

Mt s m rng

M rng th nht
V d 1.147 Cho cc s khng m a; b; c tha mn a + b + c = 3: Chng minh rng
1
9

ab

1
9

bc

1
9

3
:
8

ca

Li gii. Bi ton ny c c im gn ging dng ca cc v d trn nhng chng ta


khng th dng cch ca cc bi trn gii n v cc bin trong bt ng thc dng
f (x1 ) + f (x2 ) + f (x3 ) 0 khng phi l a; b; c lin quan n gi thit ca bi ton
m l ab; bc; ca. Mt l do na l khi ta c gng thit lp bt ng thc f (x) g(x)
trong g(x) c dng k(x 1) th bt ng thc ny b ngc chiu (cc bn hy
kim tra). Tuy nhin, chng ta c th dng dng sau gii bi ton ny, l thit
lp g(x) c dng m(x2 1) + n(x 1), chng ta ch vic thit tm cc tham s m; n
sao cho f (x) g(x) c tha mn. C th, bi ny, chng ta s tm m; n sao cho
8
9

, (x

m(x2

1) + n(x

1) m(x + 1) + n

1) + 1
1
9

Ging nh cc bi trc, ta s chn m; n sao cho phng trnh m(x + 1) + n 9 1 x = 0


c mt nghim l 1, t ta suy ra c n = 18 2m, thay vo bt ng thc trn
v phn tch ra, ta c bt ng thc tng ng l
(x

1)2 (72m

8mx)

By gi hy ch rng 3 > max fab; bc; cag v a; b; c 0 nn ta ch cn tm m sao


cho bt ng thc trn ng vi mi x 2 [0; 3] l (nu cn ta c th dng nh gi
mnh hn l 94 max fab; bc; cag, tc l tm bt ng thc ng vi mi x 2 0; 49
nhng v bi ny khng cht nn ta khng cn phi nh gi qu cht nh th). Cho
x = 0 ) m > 0, do
72m

8mx

72m

24m = 48m

1.5. THE HYBERBOLIC FUNCTIONAL TECHNIQUE

165

1
1
T y, r rng nu ta chn m = 48
) n = 12
th bt ng thc ng. Vy
1
1
m = 48 ; n = 12 v ta thit lp c bt ng thc

8
9

1 2
(x
48

1) +

V li gii ca ta nh sau
D dng chng minh c 9 8 x
ny vi ch l max fab; bc; cag
X
cyc

1
48

ab

1 2
(x + 4x + 3)
48

1) + 1 =

1
2
48 (x + 4x + 3)
9
4 < 3; ta c

8
9

1
(x
12

8x 2 [0; 3]. S dng bt ng thc

a2 b2 + 4

cyc

ab

cyc

43
16

Do ta ch cn chng minh
X

a2 b2 + 4

cyc

t x = ab + bc + ca ) x
Nu 4x 9; ta c
X
cyc

Nu 4x

a2 b2 + 4

max 0; 4x3

ab = x2 + 4x

15
9

x2 + 4x

6abc

cyc

9; ta c
X
X
a2 b2 + 4
ab = x2 + 4x
cyc

ab

cyc

3; abc

x2 + 4x

6abc

225
< 15
16

2(4x

9)

cyc

(x

1)(x

3) + 15

15

Bt ng thc c chng minh.


V d 1.148 Cho cc s khng m a; b; c tha a4 + b4 + c4 = 3: Chng minh rng
1
4

ab

1
4

bc

1
4

ca

1:
(Moldova TST 2005)

Li gii. D dng chng minh c vi mi x


3
4

3
2

th

1
(2x2 + x + 12)
15

166

CHNG 1. TM TI MT S K THUT GII TON


q

Ch l max fab; bc; cag


X
cyc

3
2

<

nn

1
15

3
4

3
2

ab

2 2

a b +

cyc

ab + 36

cyc

Mt khc, ta c
X

a2 b2

cyc

a4 = 3;

cyc

s X
3
a2 b2

ab

cyc

cyc

T y d dng suy ra pcm.


V d 1.149 Cho cc s khng m a; b; c; d tha mn a2 + b2 + c2 + d2 = 4: Chng
minh rng
1
1
1
1
+
+
+
2:
3 abc 3 bcd 3 cda 3 dab
(Phm Kim Hng)
8
p
3 3

Li gii. D thy max fabc; bcd; cdag

thc

2
3

D thy n =

1
2

m(x2

1)2 (6m

2mx

Ta cn c
6m

)m
p

3>

5
3

1) + 1

2mx)

6m

16
p m
3 3

0; suy ra
6m

Do

1) + n(x

2m; khi bt ng thc tng ng


(x

D thy m

nn chng ta cn thit lp bt ng

16
p m 0
3 3
1
p
6 316
3

nn ta ch cn chn sao cho


m

1
6

16
5

)m=

=
5
14

5
14

1.5. THE HYBERBOLIC FUNCTIONAL TECHNIQUE


)n=

167

3
14

Nh vy, ta thit lp c bt ng thc


5x2

2
3

3x + 12
14

8x

8
p
3 3

S dng cc bt ng thc ny ln lt, ta suy ra ta ch cn chng minh


X
X
5
a2 b2 c2 3
abc 8
cyc

cyc

Chng minh bt ng thc ny bng cch dng k thut hm li.


M rng th hai
i vi nhng bt ng thc i xng, chng ta c th lm nh cc cch trn, phn
ln u gip chng ta i n kt qu. Nhng i vi cc bt ng thc hon v, li
cha cn thc th mi chuyn li khng n gin nh th. Chng ta khng th thit
lp nhng hm s trung gian bc nht hay bc hai ri da vo chng chng minh
bi ton ban u c. Tuy nhin, trong mt s trng hp, ta c th tm cc hm
phn thc trung gian (trong mt s trng hp, ta cng c th thit lp hm bc 2)
nh gi cc biu thc trong cn nhm gip ta loi b cn thc, iu ny gip
ta d dng hn trong vic gii bi ton. thit lp c cc hm phn thc ny,
chng ta c rt nhiu cch, nhng tt hn ht l ta hy i t bt ng thc suy
ra bt ng thc, chng hn t bt ng thc Cauchy Schwarz, ta c
p
2x2 + 2y 2 x + y 8x; y 0
Nh vy, ta c

p
2x2 + 2y 2

(x y)2
y=p
2x2 + 2y 2 + x + y

(x y)2
2(x + y)

V nh vy, ta thit lp c bt ng thc


p
2x2 + 2y 2

x+y+

(x y)2
3x2 + 2xy + 3y 2
=
2(x + y)
2(x + y)

V n gip ta gii c bi ton rt kh sau


V d 1.150 Cho cc s dng x; y; z: Chng minh rng
p
p
p
x+ y+ z
x
y
z
p
p
+p
+p
:
x+y
y+z
z+x
2
(Walther Janous)

168

CHNG 1. TM TI MT S K THUT GII TON

Li gii. Ch rng vi mi x; y

0, ta c
p
2(x2 + y 2 )

3x + 2xy + 3y 2
2(x + y)
Tht vy, ta c

(3x2 + 2xy + 3y 2 )2 8(x + y)2 (x2 + y 2 ) = (x y)4 0


p
p
p
Tr li bi ton, t a = x; b = y; c = z. Bt ng thc tr thnh
X
cyc

1X
a
2 cyc

a2

2(a2

b2 )

S dng bt ng thc trn, ta ch cn chng minh c


X
cyc

X
cyc

,
t x =

3c b
3b2 +2bc+3c2

4a2 (a + b)
3a2 + 2ab + 3b2

8a2 (a + b)
3a2 + 2ab + 3b2
X (a
cyc

3a2

cyc

3a + b

b)2 (3b a)
+ 2ab + 3b2

v y; z tng t. Ta phi chng minh


X
x(b c)2 0
cyc

Trng hp 1. Nu a

c, khi ta c y
2

0. Ta s chng minh

a y + 2b x

v
x + 2z

Tht vy, bt ng thc th nht tng ng vi


a2 (3a c)
2b2 (3c b)
+ 2
2
+ 2ac + 3c
3b + 2bc + 3c2

3a2

Nu 3c b, bt ng thc l hin nhin. Nu b


mt hm nghch bin theo b; do
a2 (3a c)
2b2 (3c b)
+ 2
2
+ 2ac + 3c
3b + 2bc + 3c2

3a2

3c, ta c th kim tra c y l

a2 (3a c)
2a2 (3c a)
+ 2
2
+ 2ac + 3c
3a + 2ac + 3c2
2
a (a + 5c)
=
0
2
3a + 2ac + 3c2
=

3a2

1.5. THE HYBERBOLIC FUNCTIONAL TECHNIQUE

169

Bt ng thc th 2 tng ng vi
2(3b a)
3a c
+ 2
3a2 + 2ac + 3c2
3a + 2ab + 3b2

y l mt hm nghch bin theo c nn


3a c
2(3b a)
+ 2
3a2 + 2ac + 3c2
3a + 2ab + 3b2

3a b
2(3b a)
+ 2
3a2 + 2ab + 3b2
3a + 2ab + 3b2
a + 5b
=
0
2
3a + 2ab + 3b2

Ch rng
(a

a2
(b
b2

c)2

c)2 ;

c)2

(a

(a

b)2

Nn
2

x(b

c)2 = [2x(b

c)2 + y(a

c)2 ] + [y(a

c)2 + 2z(a

b)2 ]

cyc

2x(b
=

a2
(b
b2

c)2 + y

a2 y + 2b2 x
(b
b2

c)2 + [y(a

c)2 + (a

b)2 + 2z(a

b)2 (y + 2z)

b)2 ]

Trng hp 2. Nu c b a, th ta c x; z
0. Nu y
0, bt ng thc l hin
nhin. Nu y 0, tc l c 3a, xt nhng trng hp nh sau
i) Nu 2b c + a, ta s chng minh
z(a
,

b)2 + y(a

c)2

(a b)2 (3b a) (a c)2 (3a c)


+ 2
3a2 + 2ab + 3b2
3a + 2ac + 3c2

y l mt hm ng bins theo b nn
(a b)2 (3b a)
3a2 + 2ab + 3b2

a+c 2
2

a+c
2

3a2 + a(a + c) + 3

a
a+c 2
2

(a c)2 (a + 3c)
2(19a2 + 10ac + 3c2 )

Mt khc
(a c)2 (3a c)
(a c)2 (a + 3c)
+ 2
2
2
2(19a + 10ac + 3c )
3a + 2ac + 3c2
(a c)2 (3a + c)(39a2 2ac + 3c2 )
=
2(3a2 + 2ac + 3c2 )(19a2 + 10ac + 3c2 )

170

CHNG 1. TM TI MT S K THUT GII TON

)
ii) Nu a + c

b)2 + y(a

) z(a
2

x(b

2b v c

c) = x(b

cyc

2b

c)2

c) + [z(a

0
2

c)2 ]

b) + y(a

p
3 + 2 3 a, ta s chng minh
z + 3y

3
x+ y
2
Bt ng thc th nht tng ng vi
3a2

3(3a c)
3b a
+ 2
2
+ 2ab + 3b
3a + 2ac + 3c2

y l mt hm ng bin theo c nn
3a c
3a2 + 2ac + 3c2

3a (2b a)
3a2 + 2a(2b a) + 3(2b

a)2

2(a2

2a b
2ab + 3b2 )

Mt khc, ta c
3(2a b)
3b a
+
2(a2 2ab + 3b2 ) 3a2 + 2ab + 3b2
16a3 + 13a2 b 6ab2 + 9b3
=
2(a2 2ab + 3b2 )(3a2 + 2ab + 3b2 )

Bt ng thc th hai tng ng vi


3(3a c)
2(3c b)
+ 2
3b2 + 2bc + 3c2
3a + 2ac + 3c2

y l mt hm ng bin theo a nn
Nu c 2b, ta c
2(3c b)
3(3a c)
+ 2
3b2 + 2bc + 3c2
3a + 2ac + 3c2

Nu 2b

2(3c b)
3(3 0 c)
+
3b2 + 2bc + 3c2
3 02 + 2 0 c + 3c2
3c2 4bc 3b2
0
=
c(3b2 + 2bc + 3c2 )

c, ta c
3b2

2(3c b)
3(3a c)
+ 2
2
+ 2bc + 3c
3a + 2ac + 3c2
2(3c b)
3[3(2b c) c]
+
3b2 + 2bc + 3c2
3(2b c)2 + 2(2b c)c + 3c2
15b3 + 44b2 c 13bc2 6c3
=
0
2(3b2 2bc + c2 )(3b2 + 2bc + 3c2 )

1.5. THE HYBERBOLIC FUNCTIONAL TECHNIQUE

171

Tip theo, s dng bt ng thc AM-GM, ta c


c)2

(a

(a

b)2 + (b

(a

b)2 + (b

(a

b)2 + (b

=
Ta c

x(b

3
b)2 + (b
2

3(a

c)2

(b

cyc

b)2 + z(b

c)2

3
c)2 x + y + (a
2

p
iii) Nu a + c 2b v 3 + 2 3 a
Schwarz, ta c
x(c

c)2 + 2(a b)(b c)


p
1
2(a b) p (b c)
c)2 + 2
2
1
c)2 + 2(a b)2 + (b c)2
2

a)2

2b

[(c

b
4,

a)a
b) + (b
1
1
x + z

b)2 (z + 3y)

s dng bt ng thc Cauchy

a)]2

xz(c a)2
x+z

Ta cn chng minh

xz
+y 0
x+z
, xy + yz + zx 0
X
X
X
3
, f (c) = 9
ab + 22
a2 bc 12
a2 b2
cyc

cyc

cyc

a3 b

cyc

D dng kim tra c f (c) l hm ng bin, suy ra


f (c)

f (b) = 2b(3a3

a2 b + 25ab2

3b3 )

0:

Bt ng thc c chng minh. ng thc xy ra khi v ch khi x = y = z:


Nhn xt
ta p
cng c mt li gii khc rt hay ca Peter Scholze nh sau
p 16 Chng
p
t a = x; b = y; c = z v bnh phng 2 v, ta c bt ng thc tng ng
X
cyc

X
a4
a2 b2
p
+
2
a2 + b2
(a2 + b2 )(b2 + c2 )
cyc

S dng bt ng thc sp xp li cho 2 dy


p

a2 b2
b2 c2
c2 a2
;p
;p
a2 + b2
b2 + c2
c2 + a2

1
2

X
cyc

!2

172

CHNG 1. TM TI MT S K THUT GII TON

v
p
ta c

X
cyc

a2 b2
a2 + b2

1
1
1
;p
;p
a2 + b2
b2 + c2
c2 + a2
p

1
2
b + c2

X
cyc

a2 b2
a2 + b2

1
+ b2

a2

X a2 b2
=
a2 + b2
cyc

Ta cn chng minh
X
cyc

X 2a2 b2
a4
+
a2 + b2
a2 + b2
cyc

X 4a2 b2
+
a2 + b2
a2 + b2
cyc

X a4 + b4

,2
,

!2

cyc

cyc

X 4a2 b2
2a4
+
a2 + b2
a2 + b2
cyc

X
cyc

!2

1
2

X
cyc

X
cyc

X 2a2 b2
a +
a2 + b2
cyc
2

a2 + b2
2

cyc

cyc

X
cyc

2a2 b2
a2 + b2

!2

!2

!2

0:

hin nhin ng.

Mt cch khc thit lp hm phn thc trung gian l s dng bt ng thc


AM-GM, chng hn ta c
p
p
2(Ax + By) 8x2 + y 2
(Ax + By)2 + 8x2 + y 2
2 8x2 + y 2 =
8A; B; x; y 0
Ax + By
Ax + By
Chng ta s thit lp mt bt ng thc c ng thc xy ra khi x = y; khi ta
phi c A + B = 3. Ngoi ra, bt ng thc ny c cht th chng ta nn chn
A; B sao cho bt ng thc ny c ng thc ti 2 im. V ta cn dng bt ng
thc ny gii cc bi ton hon v cha cn thc (ch l cc bt ng thc ny
thng c nhng im "nhy cm" l (x; y; 0)) nn tt hn ht l chng ta s chn

1.5. THE HYBERBOLIC FUNCTIONAL TECHNIQUE

173

A; B sao cho bt ng thc trn c ng thc ti x = 1; y = 0 hoc x = 0; y = 1: Nu


p
ta chn A; Bpsao cho bt ng thc c ng thc ti x = 1; y = 0 th ta c A = 2 2
v B = 3 2 2; nhng gi tr ny l v s gy tr ngi cho cc tnh ton ca ta. Nu
ta chn A; B sao cho bt ng thc ti x = 0; y = 1 th ta c A = 2; B = 1 v ta
thit lp c bt ng thc
p
8x2 + y 2

(2x + y)2 + 8x2 + y 2


6x2 + 2xy + y 2
=
2(2x + y)
2x + y

V ta gii c bi ton sau (cng rt kh)


V d 1.151 Cho cc s khng m a; b; c; khng c 2 s no ng thi bng 0: Chng
minh rng
p
p
p
(a + b + c)2 a 8b2 + c2 + b 8c2 + a2 + c 8a2 + b2 :
(V Quc B Cn)

Li gii. Ch rng
(6b2 + 2bc + c2 )2
(2b + c)2
)

(8b2 + c2 ) =

4b2 (b c)2
(2b + c)2

6b2 + 2bc + c2
= 3b + c
2b + c

8b2 + c2

3bc
2b + c

Do , ta ch cn chng minh c
X

!2

cyc

, 3abc

X
cyc

3bc
2b + c

a 3b + c

cyc

X
1
+
a2
2b + c cyc

X
cyc

S dng bt ng thc Cauchy Schwarz, ta c


X
cyc

1
2b + c

3
P

bc

cyc

Do , ta ch cn chng minh
9abc X 2
P +
a
a
cyc
cyc

cyc

bc

174

CHNG 1. TM TI MT S K THUT GII TON


,

a3 + 3abc

cyc

bc(b + c):

cyc

y chnh l bt ng thc Schur bc 3.


Vy ta c pcm. ng thc xy ra khi v ch khi a = b = c.
i khi chng ta cng c th bt u t vic s dng lin phn s, chng hn xut
pht t bt ng thc hin nhin sau
p
4x2 + y 2 2x + y 8x; y 0
Ta c

p
4x2 + y 2

2x

=
=

4xy
p
2
4x + y 2 + 2x + y
4xy
2(2x + y) p 2 4xy
2

4x +y +2x+y

=
Chng hn, ta s s dng ng thc
p
4x2 + y 2 2x y =
kt hp vi

2(2x + y)

4xy
p

4xy
4x2 +y 2 +2x+y

p
4x2 + y 2 2x + y; ta thit lp c
p
4xy
4x2 + y 2 2x y =
2(2x + y) p

4xy
4x2 +y 2 +2x+y

4xy
2(2x + y)

p
4x2 + y 2

4xy
2(2x+y)

2xy(2x + y)
4x2 + 3xy + y 2
2xy(2x + y)
4x2 + 3xy + y 2
(2x + y)(4x2 + xy + y 2 )
4x2 + 3xy + y 2

2x + y
=

Ta gii c bi ton sau


V d 1.152 Cho cc s khng m a; b; c; khng c 2 s no ng thi bng 0: Chng
minh rng
p
p
p
3
(a + b + c)2 a 4b2 + c2 + b 4c2 + a2 + c 4a2 + b2 :
4
(V Quc B Cn)

1.5. THE HYBERBOLIC FUNCTIONAL TECHNIQUE

175

Li gii. Ch rng
(2b + c)2 (4b2 + bc + c2 )2
(4b2 + 3bc + c2 )2
p
4b2 + c2

4b2

(2b + c)(4b2 + bc + c2 )
= 2b + c
4b2 + 3bc + c2

Do , ta ch cn chng minh c
!2
X
3 X
a 2b + c
a
4 cyc
cyc
, 8abc

4b3 c3
(4b2 + 3bc + c2 )2

c2 =

2bc(2b + c)
4b2 + 3bc + c2

2bc(2b + c)
4b2 + 3bc + c2

X
2b + c
+
3
a2
4b2 + 3bc + c2
cyc

cyc

bc

cyc

chng minh bt ng thc ny, ta ch cn chng minh c


8

X
cyc

4b2

27
P
a

2b + c
+ 3bc + c2

cyc

khi , bt ng thc trn l mt h qu ca bt ng thc Schur v


X
27abc
P +3
a2
a
cyc

bc

cyc

cyc

a3 + 3abc

cyc

bc(b + c)

cyc

Do , tt c chng ta phi lm by gi l chng minh


8

X
cyc

,64

a5 b + 32

cyc

4b2

ab5 + 68

ab(16a2

a2 b4

128

cyc

a + 147

cyc

,4

27
P
a

cyc

cyc

+ abc 132

2b + c
+ 3bc + c2

X
cyc

ab

243

cyc

ab + 8b2 )(a

a4 b2 + 60

cyc

a b

cyc

b)2 + 4

cyc

a2 b2 (a2

+ abc 132

cyc

a + 147

X
cyc

ab

243

X
cyc

a3 b3
!

396abc

11ab + 34b2 )

cyc

a b

396abc

176

CHNG 1. TM TI MT S K THUT GII TON


,4

ab(16a2

ab + 8b2 )(a

b)2 +

cyc

a2 b2 (2a

11b)2 + 15

cyc

+ abc 132

a + 147

cyc

a2 b4

cyc

ab

243

cyc

a b

396abc

cyc

S dng bt ng thc AM-GM, ta c


X

a2 b4

abc

cyc

a2 b

cyc

v
X

a2 b2 (2a

11b)2 =

cyc

1X 2 2
[a b (2a 11b)2 + b2 c2 (2b
2 cyc
X
ab2 c(2a 11b)(2b 11c)

11c)2 ]

cyc

= abc 121

a b+4

cyc

ab

22

cyc

66abc

cyc

Ta cn chng minh
121

a2 b + 4

cyc

ab2

22

cyc

+ 132

, 107

X
cyc

a + 147

ab

a3 + 151

ab2

cyc

a b

a2 b

cyc

a2 b

396abc

cyc

107

cyc

243

cyc

cyc

66abc + 15

cyc

cyc

, 110

a3

a2 b

462abc

cyc

X
cyc

a + 151

X
cyc

ab

462abc

0:

hin nhin ng theo bt ng thc AM-GM.


Vy ta c pcm. ng thc xy ra khi v ch khi a = b; c = 0 hoc cc hon v tng
ng.
V d 1.153 Cho cc s khng m a; b; c; khng c 2 s no ng thi bng 0: Chng
minh rng
p
p
p
a2 + b2 + c2 + ab + bc + ca a 3b2 + c2 + b 3c2 + a2 + c 3a2 + b2 :

1.5. THE HYBERBOLIC FUNCTIONAL TECHNIQUE

177

Li gii. Ch rng
(2b2 + bc + c2 )2
(b + c)2
)
Ta cn chng minh

a2 +

cyc

ab

cyc

, 2abc

c2 =

b2 (b c)2
(b + c)2

2b2 + bc + c2
= 2b + c
b+c

3b2 + c2

3b2

cyc

cyc

2bc
b+c

2bc
b+c

a 2b + c

X
1
+
a2
b + c cyc

ab

cyc

S dng bt ng thc Cauchy Schwarz, ta c


X
cyc

Ta cn chng minh

1
b+c

ab

cyc

a3 + 3abc

cyc

cyc

cyc

9abc X 2
P +
a
a
cyc
,

9
P

bc(b + c):

cyc

hin nhin ng v y chnh l bt ng thc Schur bc 3.


Vy ta c pcm. ng thc xy ra khi v ch khi a = b = c:
Nhn xt 17 Mt cch tng qut, ta c kt qu sau vi mi k > 0
2

a + b + c + ab + bc + ca

k+1
2

p
p
p
a kb2 + c2 + b kc2 + a2 + c ka2 + b2

(V Quc B Cn, Vasile Cirtoaje)


Tht vy, s dng bt ng thc Cauchy Schwarz, ta c
X p
a kb2 + c2
cyc

!2

X
cyc

!"

X
cyc

a(kb + c )

178

CHNG 1. TM TI MT S K THUT GII TON

T y, ta thy bt ng thc trn c suy ra t 2 bt ng thc sau


!2
!
!
X
X
X
X
2
2
a +
ab
4
a
a b
cyc

cyc

a +

cyc

cyc

!2

ab

cyc

X
cyc

cyc

ab

cyc

Ta s chng minh bt ng thc th nht, bt ng thc thc 2 c chng minh


tng t. Ta c bt ng thc tng ng
X
X
X
X
a4
a2 b2 + 2
ab3 2
a3 b 0
cyc

cyc

cyc

cyc

Gi s c = min fa; b; cg ; t a = c + x; b = c + y vi x; y
tr thnh
4(x2

xy + y 2 )c2 + 4[x(x

y)2 + y 3 ]c + (x2

0 th bt ng thc ny
xy

y 2 )2

0:

Bt ng thc ny hin nhin ng nn ta c pcm.


Chng ta c kt qu tng qut ca bt ng thc
!2
!
!
X
X
X
X
a2 +
ab
4
a
a2 b
cyc

cyc

cyc

cyc

l
[(q + r)a + (r + p)b + (p + q)c]2

4(p + q + r)(pbc + qca + rab)

vi p; q; r l cc s khng m v a; b; c l cc s thc ty . Chng minh bt ng thc


ny nh sau
Gi s a = maxfa; b; cg, khi ta c
[(q + r)a + (r + p)b + (p + q)c]2
= [(q r)a + (r + p)b
Cho a; b; c

4(p + q + r)(pbc + qca + rab)


(p + q)c]2 + 4qr(a b)(a c)

0 v p = bk ; q = ck ; r = ak ; ta c
"
#2
!
!
X
X
X
k
k
k+1
a (a + b)
4
a
a
b
cyc

cyc

Vi k = 1; ta thu c bt ng thc trn.


Vi k = 1; ta c
s
c a
b
+ + + 3 2 (a + b + c)
a b
c

cyc

1 1 1
+ +
:
a b
c

1.6. CC DNG TNG BNH PHNG

179

V d 1.154 Cho cc s khng m a; b; c; khng c 2 s no ng thi bng 0: Chng


minh rng
r
r
r
p
p
p
a3
b3
c3
a+ b+ c
p
+
+
:
a2 + ab + b2
b2 + bc + c2
c2 + ca + a2
3
(L Trung Kin)
Hng dn. S dng bt ng thc AM-GM, ta c
p
p
2 3(x4 + x2 y 2 + y 4 ) = 2 (x2 + xy + y 2 ) 3(x2 xy + y 2 )
(x2 + xy + y 2 ) + 3(x2 xy + y 2 )
= 2(2x2 xy + 2y 2 )
v ta thit lp c bt ng thc
2x2

1.6

xy + 2y 2

p
3(x4 + x2 y 2 + y 4 ):

Cc dng tng bnh phng

K thut ny da trn mt kt qu hin nhin ca bt ng thc l x2 0 8x 2 R;


n c th gip ta gii c nhng bi ton m nu dng k thut thng thng th
rt kh (thng thng y l cc bt ng thc bc 4). Chng ta c nh l sau
nh l 1.6 Xt bt ng thc sau vi cc bin thc a; b; c
X
X
X
X
X
m
a4 + n
a2 b2 + p
a3 b + g
ab3 (m + n + p + g)
a2 bc
cyc

cyc

cyc

cyc

khi bt ng thc ny ng nu

m>0
3m(m + n)

Chng minh. Vit li bt ng thc nh sau


!
X
X
X
m
a4
a2 b2 + (m + n)
a2 b2
cyc

cyc

cyc

+g

ab3

cyc

Ch rng

X
cyc

a4

X
cyc

a2 bc

cyc

a2 b2 =

cyc

p2 + pg + g 2

X
cyc

a2 bc

1X 2
(a
2 cyc

b2 )2

+p

X
cyc

a3 b

X
cyc

a2 bc

180

CHNG 1. TM TI MT S K THUT GII TON


X

a3 b

cyc

a2 bc =

cyc

b3 c

cyc

a2 bc =

cyc

ab3

cyc

a2 bc =

cyc

bc(a

cyc

cyc

b2 )(ab + ac

ca3

ab2 c =

cyc

ca(a2

ca(a2

b2 )

cyc

b2 )

1X 2
(a
3 cyc

b2 )

2bc)

X
1
(ab + bc + ca)
(a2
3
cyc

cyc

b2 )

1X 2
(a
3 cyc
X

bc(a2

X
1
(a2
b ) + (ab + bc + ca)
3
cyc

cyc

b2 )(ab + bc

b2 )

2ca)

Do , bt ng thc trn tng ng vi


mX 2
(a
2 cyc

b2 )2 +

1X 2
(a
3 cyc

b2 )[(p

+(m + n)

g)ab

(2p + g)bc + (p + 2g)ca]

2 2

a b

cyc

cyc

a bc

Mt khc
X

a2 b2

cyc

a2 bc =

cyc

6(p2

X
1
[(p
2
+ pg + g ) cyc

g)ab

(2p + g)bc + (p + 2g)ca]2

Bt ng thc c vit li thnh


mX 2
(a
2 cyc

b2 )2 +
+

6(p2

1 X
[3m(a2
18m cyc
+

1X 2
(a
3 cyc

b2 )[(p

X
m+n
[(p
2
+ pg + g ) cyc
b2 ) + (p

g)ab

g)ab
g)ab

(2p + g)bc + (p + 2g)ca]


(2p + g)bc + (p + 2g)ca]2

(2p + g)bc + (p + 2g)ca]2

3m(m + n) p2 pg g 2 X
[(p
18m(p2 + pg + g 2 )
cyc

g)ab

(2p + g)bc + (p + 2g)ca]2

1.6. CC DNG TNG BNH PHNG

181

m>0
3m(m + n)
ng thc trn hin nhin ng. nh l c chng minh.

T y, ta c th d dng kim tra c vi

th bt

p2 + pg + g 2

V d 1.155 Cho cc s thc a; b; c: Chng minh rng


(a2 + b2 + c2 )2

3(a3 b + b3 c + c3 a):
(Vasile Cirtoaje)

Li gii. Bt ng thc tng ng


X
X
a4 + 2
a2 b2
cyc

cyc

T y, ta c m = 1 > 0; n = 2; p =
3m(m + n)

p2

a3 b

3; g = 0, ta c

g 2 = 3 1 (1 + 2)

pg

cyc

( 3)2

( 3) 0

02 = 0:

Do , theo nh l ca ta, bt ng thc c chng minh.


V d 1.156 Cho cc s thc a; b; c: Chng minh rng
p 3
p
3 1 abc(a + b + c)
3(a b + b3 c + c3 a):
a4 + b4 + c4 +
(V Quc B Cn)
p

Li gii. Ta c m = 1 > 0; n = 0; p =
3m(m + n)

p2

3; g = 0 v
p

g 2 = 3 1 (1 + 0)

pg

02 = 0:

Do bt ng thc cn chng minh ng.


V d 1.157 Cho cc s thc a; b; c: Chng minh rng
7(a4 + b4 + c4 ) + 10(a3 b + b3 c + c3 a)

0:
(Phm Vn Thun)

Li gii. Ta s chng minh kt qu mnh hn l


7

X
cyc

, 86

X
cyc

a4

51

X
cyc

a + 10

a b

cyc

a2 b2 + 101

X
cyc

a3 b

17
27
34

X
cyc

X
cyc

!4

ab3

102

X
cyc

a2 bc

182

CHNG 1. TM TI MT S K THUT GII TON


8
m = 86 > 0
>
>
<
n = 51
)
p = 101
>
>
:
g = 34

Mt khc, ta c
3m(m + n)

p2

g 2 = 3 86 (86

pg

1012

51)

( 34)2 = 1107 > 0:

101 ( 34)

Bt ng thc c chng minh.


V d 1.158 Cho cc s thc a; b; c tha mn abc = 1: Chng minh rng
a2

1
+
a + 1 b2

1
+
b + 1 c2

1
c+1

3:
(V nh Qu)

Li gii. Do abc = 1 nn tn ti cc s x; y; z sao cho a = xy ; b = yz ; c =


thc tr thnh
X
x2
3
2
x
xy + y 2
cyc
,
,

X
cyc

4
X
cyc

(x
x2

2y)2
xy + y 2

bt ng

3x2
xy + y 2

x2

cyc

3x2
xy + y 2

x2

x
z

S dng bt ng thc Cauchy Schwarz, ta c


"

X
cyc

(x
x2

2y)2
xy + y 2

#"

(x

2y)2

#2

2y) (x

"

xy + y )

cyc

(x

2y)

cyc

#2

Ta cn chng minh
"
, 10

X
cyc

(x

cyc

x4 + 39

X
cyc

x2 y 2

(x

2y)2 (x2

xy + y 2 )

cyc

25

X
cyc

x3 y

16

X
cyc

xy 3

X
cyc

x2 yz

1.6. CC DNG TNG BNH PHNG


T y, ta c m = 10 > 0; n = 39; p =
3m(m+n) p

183

25; g =

16 v

( 25) ( 16) ( 16)2 = 189 > 0:

pg g = 3 10 (10+39) ( 25)

Bt ng thc c chng minh.


V d 1.159 Cho cc s thc a; b; c: Chng minh rng
(c + a b)2
(a + b c)2
(b + c a)2
+ 2
+ 2
2
2
2
a + (b + c)
b + (c + a)
c + (a + b)2

3
:
5
(Japan MO 1997)

Li gii. t x = b + c

a; y = c + a
X

b; z = a + b

c; bt ng thc tr thnh

(y +

cyc

,2

z)2

X
cyc

x2

4x
+ (2x + y + z)2

3
5

x2
+ (x + y + z)2

3
5

S dng bt ng thc Cauchy Schwarz, ta c

X
cyc

x2
x2 + (x + y + z)2

,4

x4 + 14

cyc

cyc

!2

x2 y 2

cyc

T y, ta c m = 4 > 0; n = 14; p =
3m(m + n)

p2

pg

cyc

!2

x2 [x2 + (x + y + z)2 ]

cyc

Ta cn chng minh
10

x2 [x2 + (x + y + z)2 ]

cyc

x3 y

cyc

6; g =

g 2 = 3 4 (4 + 14)

xy 3

6xyz

cyc

cyc

6 v
( 6)2

( 6) ( 6)

( 6)2 = 108 > 0:

Bt ng thc c chng minh. ng thc xy ra khi v ch khi a = b = c:


V d 1.160 Cho cc s thc a; b; c: Chng minh rng
(2a + b + c)2
(2b + c + a)2
(2c + a + b)2
+ 2
+ 2
2
2
2
2a + (b + c)
2b + (c + a)
2c + (a + b)2

8:
(USA MO 2003)

184

CHNG 1. TM TI MT S K THUT GII TON

Li gii. Bt ng thc tng ng


X

(2a + b + c)2
2a2 + (b + c)2

cyc

,
t x = b + c

a; y = c + a
X
cyc

X (b + c a)2
2a2 + (b + c)2
cyc

b; z = a + b
2(y +

z)2

1
1
2

c; khi bt ng thc tr thnh

x2
+ (2x + y + z)2

1
8

S dng bt ng thc Cauchy Schwarz, ta c

X
cyc

x2
2
2(y + z) + (2x + y + z)2

Ta cn chng minh
X

cyc

,2

x4 + 5

cyc

!2

p2

x2 y 2

!2

x2 [2(y + z)2 + (2x + y + z)2 ]

x2 [2(y + z)2 + (2x + y + z)2 ]

cyc

pg

cyc

cyc

x3 y

cyc

T y, ta c m = 2 > 0; n = 5; p =
3m(m + n)

2; g =

g 2 = 3 2 (2 + 5)

xy 3

cyc

3xyz

cyc

2 v
( 2)2

( 2) ( 2)

( 2)2 = 30 > 0:

Bt ng thc c chng minh. ng thc xy ra khi v ch khi a = b = c:


V d 1.161 Cho cc s a; b; c
p

0; a + b + c = 1: Chng minh rng

a
b
c
+p
+p
2
2
4a + 5b
4b + 5c
4c + 5a2

3
p :
17
(V Quc B Cn)

Li gii. S dng bt ng thc Cauchy Schwarz,


!2
!
!
X
X
X
X
a
a
a
p
a
=
2
2
4a
+
5b
4a
+
5b2
4a
+
5b
cyc
cyc
cyc
cyc

1.6. CC DNG TNG BNH PHNG


nn ta ch cn chng minh

X
cyc

185

9
17

a
4a + 5b2

X
cyc

b2
4a + 5b2

3
17

Li theo bt ng thc Cauchy Schwarz, ta c


!2
!2
P 2
P 2
b
a
X
cyc
cyc
b2
!
!
P 2
=
b (4a + 5b2 )
4a + 5b2
P
P 2
P
cyc
cyc
4
a
ab + 5 a4
cyc

cyc

cyc

Ta cn chng minh
17

X
cyc

!2

a4 + 11

p2

pg

cyc

12

X
cyc

X
cyc

a2 b2

cyc

ab3

cyc

ab

+ 15

a4

cyc

a2 bc

cyc

T y,
m=1>0
3m(m + n)

g 2 = 3 1 (1 + 11)

02

( 6)2 = 0

0 ( 6)

nn bt ng thc trn ng. ng thc xy ra khi a = b = c = 31 :


V d 1.162 Cho cc s thc a; b; c: Chng minh rng
a4 + b4 + c4 + a3 b + b3 c + c3 a

2(a3 b + b3 c + c3 a):
(Vasile Cirtoaje)

V d 1.163 Cho cc s thc a; b; c: Chng minh rng


a(a + b)3 + b(b + c)3 + c(c + a)3

8
(a + b + c)4 :
27

(Phm Vn Thun, V Quc B Cn)


V d 1.164 Cho cc s thc a; b; c: Chng minh rng
1
a4 + b4 + c4 + (ab + bc + ca)2
3

2(a3 b + b3 c + c3 a):
(Phm Kim Hng)

186

CHNG 1. TM TI MT S K THUT GII TON

1.7

Hm li, hm bc nht

Hm li c nhng tnh cht rt c bit m c th gip chng ta gii ton mt cch


hiu qu. Di y l mt s tnh cht m chng ti cho l cn thit v ph hp vi
chng trnh ton THPT
nh l 1.7 Nu f (x) li trn [a; b] th
f (x)

max ff (a); f (b)g

f (x)

min ff (a); f (b)g :

Nu f (x) lm trn [a; b] th

Tnh cht ny c suy ra t nh ngha ca hm li. T tnh cht ny, ta suy ra


chng minh mt bt ng thc
f (x1 ; x2 ; :::; xn )

Vi f (x1 ; x2 ; :::; xn ) li trn [a; b] cho tng bin x1 ; x2 ; :::; xn v x1 ; x2 ; :::; xn 2 [a; b];
ta ch cn xt bt ng thc ti
x1 =
= xk = a
xk+1 =
= xn = b

(k = 0; 1; :::n)

Tng t nu f (x) l hm lm.


nh l 1.8 Nu f (x) l hm li v kh vi cp 2 trn khong I th
f (x)

f (y) + f 0 (y)(x

y)

8x; y 2 I

Nu f (x) l hm lm v kh vi cp 2 trn khong I th


f (x)

f (y) + f 0 (y)(x

y)

8x; y 2 I:

Tnh cht ny ta c th chng minh d dng bng nh l Lagrange. Tnh cht 2


c dng chng minh cc bt ng thc dng
f (x1 ) + f (x2 ) +

+ f (xn )

f (y1 ) + f (y2 ) +

+ f (yn )

kt hp vi k thut nhm Abel, hoc cc bt ng thc dng


m1 f (x1 ) + m2 f (x2 ) +

+ mn f (xn )

m1 f (y1 ) + m2 f (y2 ) +

trong ta c
m1 f 0 (y1 ) = m2 f 0 (y2 ) =
= mn f 0 (yn )
x1 + x2 +
+ xn = y1 + y2 +
+ yn
T tnh cht th hai, chng ta suy ra c h qu sau

+ mn f (yn )

1.7. HM LI, HM BC NHT

187

H qu 1.6 Nu f (x) li v kh vi cp 2 trn khong I th vi mi x; y; z 2 I tha


x y z; ta c
f (x) + f (z) f (y) + f (x + z y)
Nu f (x) lm v kh vi cp 2 trn khong I th vi mi x; y; z 2 I tha x
c
f (x) + f (z) f (y) + f (x + z y):

z; ta

Chng minh. Ta s chng minh kt qu ny trong trng hp f (x) l hm li


(trng hp hm lm chng minh tng t).
Nu y x + z y; theo tnh cht 2, ta c
f (x)

f (z)

) f (x) + f (z)
Nu x + z

f (x + z
= f (x + z

f (y) + f 0 (y)(x
y) + f 0 (x + z
y) f 0 (x + z

f (y) + f (x + z
f (y) + f (x + z

y)

y)[z
y)(x

y) + [f 0 (y)
y)

(x + z
y)

y)]

f 0 (x + z

y)](x

y)

y; theo tnh cht 2, ta c


f (x)

f (x + z
= f (x + z
f (z)

) f (x) + f (z)

y) + f 0 (x + z
y) + f 0 (x + z

y)[x
y)(y

f (y) + f 0 (y)(z

f (y) + f (x + z
f (y) + f (x + z

(x + z
z)

y)]

y)

y) + [f 0 (x + z
y)

y)

f 0 (y)](y

z)

H qu ca ta c chng minh xong.


C th thy nhng tnh cht ny c pht biu rt n gin v nh nhng, nhng
ng dng ca chng th li rt ln. Chng ta xt 1 vi v d
V d 1.165 Cho cc s a; b; c 2 [1; 2]: Chng minh rng
a3 + b3 + c3

5abc:
(Ton hc tui tr)

188

CHNG 1. TM TI MT S K THUT GII TON

Li gii. Xt P (a; b; c) = a3 + b3 + c3 5abc, r rng P (a; b; c) l hm li ln lt


theo cc bin a; b; c; do ta ch cn xt cc trng hp sau l
Trng hp 1. a = b = c = 2; ta c P (a; b; c) = 16 < 0:
Trng hp 2. a = 1; b = c = 2; ta c P (a; b; c) = 3 < 0:
Trng hp 3. a = b = 1; c = 2; ta c P (a; b; c) = 0:
Trng hp 4. a = b = c = 1; ta c P (a; b; c) = 2 < 0:
Do bt ng thc cn chng minh ng. ng thc xy ra khi a = b = 1; c = 2
v cc hon v.
k
P

V d 1.166 Cho dy dng x1 ; x2 ; :::; xn tha

xi

i=1

minh rng
x21 + x22 +

1
4

+ x2n

1+

1
+
2

k 8k = 1; 2; :::; n: Chng
1
n

:
(Titu Andreescu)

Li gii. R rng hm s f (x) = x2 l hm li, nn theo tnh cht 2, ta c


h
i
p
p
p
p
p
p
i
i 1 + f0
i
i 1 xi
i
i 1
8i = 1; 2; :::; n
f (xi ) f

Do
n
X

n h
X

x2i

i=1

1 + f0

i=1

n
X

n
X

1 +

i=1

f0

xi

i=1

i
p

xi

ii
1

S dng k thut nhm Abel, ta c


n
X

f0

i=1

n
X1 h

f0

+f 0

n
X1 h

n
p

f0

n
p

f0

i=1

"

xi

+f 0

2
i
p i X
xj
i 4
j=1

n
X

xj

i=1

n
X

n
X
i=1

i=1

f0

i+1

i+1

i=1

xj

0
i
p i X
i @
xj
j=1

i
X
p
j=1

iA

p
j

1 5

1.7. HM LI, HM BC NHT


Do

1>

n
X1 h

f0

i+1
p

i=1

n
X

x2i

i=1

i=1

>

i 8i = 1; 2; :::; n v f (x) li nn ta c
0
1
i
i X
p
p
p
1
f0
i+1
i @
iA
xj

j=1

T y, ta c
n
X

189

n
X

p
p

i+

p
p

1 =
1

n
X
p

i=1

i=1

4i

i=1

n
X
4i

4i

1X1
:
4 i=1 i

Bt ng thc c chng minh xong.


V d 1.167 Cho cc s khng m a; b; c; khng c 2 s no ng thi bng 0: Chng
minh rng
p
p
p
3(a + b + c) 2
a2 + bc + b2 + ca + c2 + ab :
(Phm Kim Hng)

Li gii. Nu abc = 0; gi s c = 0 th d thy bt ng thc hin nhin ng. Nu


abc > 0; ta chun ha cho abc = 1 v gi s a b c; khi tn ti cc s thc
x y z sao cho a = ex ; b = ey ; c = ez tha
p x + y + z = 0; bt ng thc tr thnh
f (x) + f (y) + f (z) 0 vi f (t) = 3et 2 et + e t : Ta c
f "(t) =

6e3t=2 (e3t + 1)3=2 4e6t 14e3t


2e2t (e2t + e t )3=2

f "(t) = 0 , 6e3t=2 (e3t + 1)3=2 = 4e6t + 14e3t + 1


, 36e3t (e3t + 1)3 = (4e6t + 14e3t + 1)2

, 36(e

3t

+ 1)3 = (4 + 14e

2 2

, 36(u + 1) = (4 + 14u + u )
, g(u) = u4

3t

+e

6t 2

(u = e

3t

9u3 + 96u2 + 4u

> 0)

20 = 0

R rng g(u) l hm ng bin, li c g(0) = 20 < 0; g(1) = 73 > 0; nn tn ti duy


nht u0 2 (0; 1) tha mn g(u0 ) = 0; suy ra f "(t) c ng mt nghim t0 , t y d
thy f (t) li trn [t0 ; +1) v lm trn ( 1; t0 ]:
Trng hp 1. Nu y t0 ; khi s dng bt ng thc Jensen, ta c
f (x) + f (y)

2f

x+y
2

190

CHNG 1. TM TI MT S K THUT GII TON

Ta cn chng minh
x+y
+ f (z) 0
2
q
p
p
p
, 6 ab + 3c 4 ab + c ab + 2 c2 + ab
q
2
p
2
p
p
, 6 ab + 3c
4 ab + c ab + 2 c2 + ab
r
p
p
2
, 15ab + 20c ab + 5c
ab + c ab (c2 + ab)
16
2f

2
p
p
c2 + ab + 3c 4 ab c
0
p
Bt ng thc cui hin nhin ng do ab c:
Trng hp 2. Nu t0 y; khi ta c t0 y y + z t0 nn theo h qu ca ta

,8

p
ab + c ab

f (y) + f (z)

f (t0 ) + f (y + z

t0 )

Mt khc, theo bt ng thc Jensen th


f (x) + f (t0 )

2f

x + t0
2

N n ta ch cn chng minh
2f

x + t0
2

+ f (y + z

t0 )

0:

y chnh l trng hp 1 m ta xt trn. Bi ton c gii quyt xong. ng


thc xy ra khi v ch khi 2 trong 3 s bng nhau, s cn li bng 0.
V d 1.168 Cho cc s khng m a; b; c; d tha mn a + b + c + d = 4: Chng minh
rng
1
1
1
1
4
+
+
+
:
48 11abc 48 11bcd 48 11cda 48 11dab
37
(V Quc B Cn)
( 2
( 2
z
z
x
0
42
4 =x,a=b .
Li gii. t x = ab; y = cd; z = a+b; t = c+d )
;
vi
t
t2
y 0
4
4 =y ,c=d
Bt ng thc tng ng
f (x; y)

=
=
=

96 11xt
96 11yz
+
2304 528xt + 121x2 y 2304 528yz + 121xy 2
1
1
96 11yz
+
+
48 11xc 48 11xd 2304 528yz + 121xy 2
1
1
96 11xt
+
+
48 11ya 48 11yb 2304 528xt + 121x2 y

4
37

1.7. HM LI, HM BC NHT

191

C th kim tra c f (x; y) l hm li cho tng bin x; y; chng hn


242d2
2(96 11yz)
242c2
+
+
0
(48 11xc)3
(48 11xd)3
(2304 528yz + 121xy 2 )
2 2
t2
z
42 = x; 4 = y
6 z
6
= x; y = 0
Do , ta ch cn xt cc trng hp sau l 6 4
. C th thy vic
2
4 x = 0; t = y
4
x=y=0
xt cc trng hp ny tng ng vi vic xt cc trng hp di y
4
4
Trng hp 1. a = c = 0; bt ng thc tr thnh 48
37 :
Trng hp 2. a = b; c = 0; bt ng thc tr thnh
f "x (x; y) =

1
+
16 48

1
11abd

4
37

S dng bt ng thc AM-GM, ta c


1
+
16 48

1
11abd

Trng hp 3. a = b; c = d ) c = 2

1
+
16 48

1
11

4
:
37

a; bt ng thc tr thnh

2
2
+
2
1 a c 1 ac2
Thay c = 2

a+b+d 3
3

4
37

a vo v thu gn, ta c bt ng thc tng ng


44(a 1)2 (48 22a 33a2 + 44a3 11a4 )
37(48 22a2 + 11a3 )(48 44a + 44a2 11a3 )

0:

Bt ng thc ny hin nhin ng do a


2: Ta c pcm. ng thc xy ra khi
a = b = c = d = 1 hoc ba trong bn s a; b; c; d bng 34 ; s cn li bng 0.
V d 1.169 Cho cc s dng a; b; c tha mn a + b + c = 3: Tm gi tr nh nht
ca biu thc
P = a3 + 2b3 + 3c3 :
Li gii. R rng hm f (x) = x3 l hm li, do
f (a)

f (A) + f 0 (A)(a

A)

f (b)

f (B) + f 0 (B)(b

B)

f (c)

f (C) + f 0 (C)(c

C)

192

CHNG 1. TM TI MT S K THUT GII TON


f (A) + 2f (B) + 3f (C) + f 0 (A)(a

)P

A) + 2f 0 (B)(b

B) + 3f 0 (C)(c

C)

f (A) = 2f (B) = 3f (C)


:
A+B+C =3

tng ca ta l chn cc s dng A; B; C sao cho


Khi , ta s c
P

f (A) + 2f (B) + 3f (C)

Nh vy, vic ca ta cn li ch l gii h phng trnh

f 0 (A) = 2f 0 (B) = 3f 0 (C)


:
A+B+C =3

H ny rt d gii nn xin c dnh cho cc bn.


V d 1.170 Cho cc s dng a1 ; a2 ; :::; an : Chng minh rng
(a21

(a1 + a2 +
+ an )2
2
+ 1)(a2 + 1)
(a2n + 1)

(n

1)n
nn 2

:
(Vasile Cirtoaje)

Li gii. B . Cho f l mt hm lm trn [a; b], khi vi mi x1 ; x2 ; :::; xn 2 [a; b]


tha mn x1 + x2 +
+ xn (n 1)a b ta c
f (x1 ) + f (x2 ) +

+ f (xn )

(n

1)f (a) + f (x1 + x2 +

+ xn

(n

1)a)

Chng minh. Ta s chng minh bng quy np theo n: Nu n = 1; bt ng thc l


hin nhin. Gi s khng nh ng vi n; ta s chng minh n cng ng vi n + 1:
Tht vy, gi s xn+1 = max fx1 ; xn ; :::; xn+1 g ; khi theo gi thit quy np, ta c
f (x1 ) + f (x2 ) +

+ f (xn )

(n

1)f (a) + f (x1 + x2 +

+ xn

(n

1)a)

Ta cn chng minh
f (xn+1 ) + f (x1 + x2 +

+ xn

(n

1)a)

f (a) + f (x1 + x2 +

+ xn+1

na)

Ta c
b

x1 + x2 +

+ xn+1

na

x1 + x2 +

+ xn

(n

1)a

Nn theo h qu ca ta
f (xn+1 ) + f (x1 + x2 +

+ xn

(n

1)a)

f (a) + f (x1 + x2 +

B c chng minh.
Tr li bi ton ca ta, bt ng thc cho tng ng vi
!
n
n
X
X
f (ai ) 2 ln
ai + (n 2) ln n (n 1) ln(n
i=1

i=1

+ xn+1

1)

na) :

1.7. HM LI, HM BC NHT

193

vi f (x) = ln(1 + x2 ): Ta c
f 00 (x) =

2(1 x2 )
(1 + x2 )2

nn f (x) li trn (0; 1] v lm trn [1; +1): Khng mt tnh tng qut, gi s an
an 1
a1 : Nu tn ti m sao cho am > 1; khi gi k l ch s nh nht sao
cho ak > 1; ta c an an 1
ak > 1 ak 1
a1 : Theo b trn, ta
c
f (ak ) + f (ak+1 ) +

+ f (an )

(n

k)f (1) + f (ak + ak+1 +

+ an

Nn ta ch cn chng minh bt ng thc cho trong trng hp 1


khi theo bt ng thc Jensen, ta c
f (a1 ) + f (a2 ) +

+ f (an

1)

a1 + a2 +

nf

an

+ an

(n
1

1)n
nn

(x2 + 1)n

a1 ;

V t y, ta suy ra c ta ch cn chng minh bt ng thc khi a1 = a2 =


an 1 = x; tc l
(n

k))

(a2n + 1)

1)x + an ]2

[(n

S dng bt ng thc AM-GM, ta c


(n

1)n

(x2 + 1)n

1)2 x2 + 1]

[(n
n

n 2

[(n

1
n

+n

n
n

2
1

n 1

2 2

1) x + 1]

Li c
[(n

1)2 x2 + 1](a2n + 1)

[(n

1)x + an ]2 :

Nn bt ng thc cn chng minh ng. ng thc xy ra khi v ch khi a1 = a2 =


= an = 1:
V d 1.171 Cho x; y; z 2 [0; 1]: Tm gi tr ln nht ca biu thc
P = 2(x3 + y 3 + z 3 )

x2 y

y2 z

z 2 x:

V d 1.172 Cho cc s khng m a; b; c; d tha mn a + b + c + d = 4: Chng minh


rng
abc + bcd + cda + dab + a2 b2 c2 + b2 c2 d2 + c2 d2 a2 + d2 a2 b2 8:
(Phan Thnh Nam)

194

CHNG 1. TM TI MT S K THUT GII TON

V d 1.173 Cho tam gic nhn ABC: Chng minh rng


cos

B
C
A
+ cos + cos
2
2
2

4
p
3

1 + sin

A
B
C
sin sin
2
2
2

:
(Jack Garfunkel)

V d 1.174 Cho cc s dng a; b; c tha mn a + 3b + 4c = 1: Tm gi tr ln nht


ca biu thc
p
p
p
3
P = 3 a + 4 b + 3 c:
V d 1.175 Cho tam gic nhn ABC: Tm gi tr nh nht ca biu thc
P = tan A + 2 tan B + 5 tan C:
(VMEO 2005)
V d 1.176 Cho cc s dng a; b; c tha mn 21ab + 2bc + 8ca 12: Tm gi tr
nh nht ca biu thc
1 2 3
P = + + :
a b
c
(Vietnam TST 2001)
V d 1.177 Cho dy dng a1 ; a2 ; :::; an tha mn

k
P

ai

i=1

Chng minh rng


1
1
+
+
a1
a2

1
an

n
:
n+1

k
P

i=1

i(i + 1) 8k = 1; 2; :::; n:

(Ton hc tui tr)


V d 1.178 Cho cc s dng a1 ; a2 ; :::; an : Chng minh rng
a1 + a2 +
2
(a1 + 1)(a22 + 1)

+ an
(a2n + 1)

(2n 1)n
2n nn 1

1
2

:
(Vasile Cirtoaje)

Tip theo, ta s i n mt tnh cht c bn ca hm tuyn tnh bc nht, l


mi hm bc nht u n iu. ngha ca iu ny l g? Chng ta hy xt mt
trng hp c th, xt hm f (x) = ax + b vi x 2 [c; d]; nu a 0; th r rng f (x)
l hm ng bin, do f (c) f (x) f (d): Nu a 0 th f (x) l hm nghch bin,
cho nn f (d) f (x) f (c): Nh vy, ta c
nh l 1.9 Cho hm f (x) = ax + b (vi a; b l cc s thc ty ); khi vi mi
x 2 [c; d]; ta c
min ff (c); f (d)g f (x) max ff (c); f (d)g :

1.7. HM LI, HM BC NHT

195

Tnh cht ny rt n gin nhng li kh hiu qu trong vic gii ton. N gip ta
thu gn khong xt t[c; d] n vic xt 2 im cc bin l x = c v x = d:
V d 1.179 Cho cc s khng m x; y; z tha mn x + y + z = 1: Chng minh rng
x3 + y 3 + z 3 +

1
:
4

15
xyz
4

Li gii. Bin i bt ng thc v dng tng ng


f (yz) =

27
x
4

1
1 yz + (1
4

2x)2

D thy y l mt hm bc nht theo yz, hn na d thy


(y + z)2
4

yz

Da trn nh l trn, ta ch cn xt bt ng thc ti yz =


yz = 0 l . Ta c
f (0) =

1
(1
4

2x)2

0;

x)2

(1

3
x(1
16

(y+z)2
4

3x)2

(1 x)2
4

0:

Nn bt ng thc cn chng minh hin nhin ng.


V d 1.180 Cho cc s khng m x; y; z tha mn x + y + z = 1: Chng minh rng
xy + yz + zx

2xyz

7
:
27

V d 1.181 Cho cc s khng m a; b; c; d: Chng minh rng


r

a
+
a+b+c

b
+
b+c+d

c
+
c+d+a

d
d+a+b

4
p :
3

(Phm Vn Thun)
V d 1.182 Cho cc s khng m a; b; c tha mn a + b + c = 1: Chng minh rng
a
b
c
+
+
1 + bc 1 + ca 1 + ab

9
:
10

196

1.8

CHNG 1. TM TI MT S K THUT GII TON

Quy np

Quy np l mt trong nhng k thut rt hay ca bt ng thc. tng rt n


gin nh sau, chng minh mt bt ng thc cho n bin, chng ta s chng minh
bt ng thc cho mt trng hp c th, ri gi nh rng n ng trong trng
hp n = k; khi ta s chng minh n ng cho n = k + 1; da trn c s ny chng
ta c kt lun n ng vi mi n:
V d 1.183 Cho cc s dng x1 ; x2 ; :::; xn > 0 (n
n
X
i=1

x2i
x2i + xi+1 xi+2

3): Chng minh rng

trong xn+1 = x1 ; xn+2 = x2 :


Li gii. Trc ht, ta s chng minh bt ng thc ng khi n = 3; tht vy, ta
cn chng minh
X x2
2
x2 + yz
cyc
X yz
1
,
x2 + yz
cyc
S dng bt ng thc Cauchy Schwarz, ta c

X
cyc

yz
2
x + yz

X
cyc

yz
2
2x + yz

yz

cyc

!2

yz(2x2 + yz)

=1

cyc

Tip theo, ta gi s bt ng thc ng khi n = k (k


ng khi n = k + 1; tc l
k+1
X
x2i
k
2
x + xi+1 xi+2
i=1 i

3), ta s chng minh n cng

Gi s xk+1 = max fx1 ; x2 ; :::; xk+1 g : S dng gi thit quy np, ta c


k
X
i=1

x2i
x2i + xi+1 xi+2

Nn chng minh bt ng thc trn, ta ch cn chng minh


x2k+1
x2k
+ 2
+ 2
2
xk+1 + x1 x2
xk + xk+1 x1
xk

x2k 1
1 + xk xk+1

x2k
2
xk + x1 x2

x2k

x2k 1
1 + xk x1

1.8. QUY NP
,

197
x2k+1
x2k+1 + x1 x2

+x2k
,

x2k

+ x2k

x2k

1
+
xk x1
1

1
+ x1 x2

x2k

1
+ xk+1 x1

x2k

1
+
xk xk+1
1

x1 x2
x2k x1 (xk+1 x2 )
+
+
x2k+1 + x1 x2
(x2k + x1 x2 )(x2k + xk+1 x1 ) (x2k

x2k 1 xk (xk+1 x1 )
2
1 + xk x1 )(xk 1 + xk xk+1 )

0:

Bt ng thc cui hin nhin ng. T y ta c pcm.


V d 1.184 Cho cc s dng a1 ; a2 ; :::; an tha mn a1 a2
rng
1
1
1
+
+
+
n 1 + a1
n 1 + a2
n 1 + an

an = 1: Chng minh
1:
(Vasile Cirtoaje)

Li gii. Ta chng minh kt qu tng qut hn


1
1
+
+
mn + a1
mn + a2

1
mn + an

1 8mn

Vi n = 1 th bt ng thc hin nhin ng. Gi s bt ng thc ng vi n = k;


khi ta s chng minh bt ng thc cng ng cho n = k + 1: Tht vy, gi s
p
ak+1 = max fa1 ; a2 ; :::; ak+1 g ) b = k a1 a2
ak 1: t bi = abi 8i = 1; 2; :::; k )
mk+1
b1 b2
bk = 1: Ch l b
mk+1 n > n 1 nn theo gi thit quy np, ta c
1
mk+1 + a1
1
=
b

mk+1 + a2
mk+1 + ak
1
1
+ mk+1
+
+
mk+1
+ b1
+ b2
b
b
k
k
=
mk+1
mk+1 + b
b
+1
b

1
mk+1
b

+ bk

Do ta ch cn chng minh
k
1
+
mk+1 + b mk+1 + ak+1
,
, (b

k
mk+1 + b

1)2

k
X1

bk
bk mk+1 + 1

[mk+1 (k + 1)

n
mk+1 + 1
n
mk+1 + 1

i(mk+1 + 1)]bk

i=1

Bt ng thc cui hin nhin ng do mk+1


minh ng. Ta c pcm.

n do bt ng thc cn chng

198

CHNG 1. TM TI MT S K THUT GII TON

V d 1.185 Cho cc s dng a1 ; a2 ; :::; an (n


minh rng
a21 + a22 +

+ a2n

p
n
(n

2) tha mn a1 a2

2n
1)n

(a1 + a2 +

an = 1: Chng

+ an

n):

Li gii. Ta s chng minh kt qu tng qut hn


a21 + a22 +

+ a2n

mn (a1 + a2 +

+ an

n) 8mn

Vi n = 2; bt ng thc tr thnh
a21 + a22

m2 (a1 + a2

1
a21

m2 a1 +

, a21 +
,

1)2 [(a1 + 1)2


a21

(a1

1
a1

22
(2 1)2

2n
1)n

2)
2

m2 a1 ]

Bt ng thc ny hin nhin ng v m2

p
n
(n

0
= 4:

Gi s bt ng thc ng khi n = k (k 2); ta s chng minh bt ng thc ng


p
khi n = k + 1: Tht vy, gi s ak+1 = min fa1 ; a2 ; :::; ak+1 g ) b = k a1 a2
ak 1:
ai
t bi = b 8i = 1; 2; :::; k ) b1 b2 bk = 1: Ta c mk mk+1 ; tht vy vi k = 2; 3;
bt ng thc hin nhin ng, xt vi k > 3; ta c
2k

p
k

1)k

(k

,1+
,

1+

k2

kk
1
=
k+1
(k 1)
k 1
T y, ta c

mk+1
b

1+

mk+1

b21 + b22 +

p
k
+ b2k

k(k+1)

1
k2

2k
(k 1)k

>2

1
1+

k
k 1
kk
(k 1)k+1

1
k

p
k

k(k+1)

Do k > 3 nn
1+

k+1

1
k2

mk+1

k 1

1
k

<

e
k

1+

nn theo gi thit quy np

mk+1
(b1 + b2 +
b

+ bk

k)

1
k

<2

1.8. QUY NP

199

, a21 + a22 +
,

a21

a22

+ a2k
+

a2k

kb2

mk+1 (a1 + a2 +

mk+1 (a1 + a2 +

+ ak )

+ ak

kb)

kmk+1 b

kb

Do ta ch cn phi chng minh


kb2

kmk+1 b + a2k+1

, kb2 + a2k+1

, kb2 + a2k+1
, f (b) = kb2 +

1
b2k

mk+1 (ak+1

mk+1 (ak+1 + kb

2(k + 1)
p
k+1
k

1)

2(k + 1)
p (ak+1 + kb
k+1
k

1
k

1
+ kb
bk

1)
k

1)
k

1)

0:

D dng chng minh c bt ng thc ny, t ta c pcm.


V d 1.186 Cho cc s dng a1 ; a2 ; :::; an tha mn a1 + a2 + + an = n: Chng
minh rng
1
1
1
8(n 1)
(1 a1 a2
an ):
+
+
+
n
a1
a2
an
n2
(Phm Kim Hng)
Li gii. Tng t cc bi trc, ta cng chng minh kt qu tng qut hn
1
1
+
+
a1
a2

1
an

mn (1

a1 a2

an ) 8mn

8(n

1)
n

Vi n = 1;bt ng thc hin nhin ng. Gi s bt ng thc ng vi ta s chng


minh n cng ng khi n = k + 1: Tht vy gi s ak+1 = max fa1 ; a2 ; :::; ak+1 g )
+ak
b = a1 +a2 +
1: t bi = abi 8i = 1; 2; :::; k ) b1 + b2 +
+ bk = n: Ch l
k
mk+1 bk+1 ak+1

mk+1 bk ak+1
= mk+1 =

ak+1 + kb
k+1
8(k 1)
k2

k+1

mk+1

8k
(k + 1)2

Do , s dng gi thit quy np, ta c


1
1
+
+
b1
b2
1
1
+
+
a1
a2
1
1
,
+
+
a1
a2
,

1
bk

(mk+1 bk+1 ak+1 )(1

b1 b2

bk )

1
k
a1 a2
ak
mk+1 bk ak+1 1
ak
b
bk
1
k
+
+ mk+1 a1 a2
ak+1
+ mk+1 bk ak+1
ak
b
+

200

CHNG 1. TM TI MT S K THUT GII TON

Cui cng ta phi chng minh


k
1
+ mk+1 bk ak+1 +
b
ak+1
,
,

1
k
+
b
ak+1

mk+1 (1

mk+1 [1

1
k
+
b
k+1

kb

, mk+1

b(k + 1

Bt ng thc ny hin nhin ng v


8
>
< mk+1
b 1
>
: b(k + 1

mk+1

ak+1 bk )
kb)bk ]

(k + 1

n(n + 1)
kb)(1 + 2b +

+ kbk

1)

8k
(k+1)2

:
kb)

(k+1)2
4k

Vy ta c pcm.

V d 1.187 Cho cc s dng a1 ; a2 ; :::; an tha mn a1 + a2 + + an = n: Chng


minh rng
1
1
3 2
1
(a + a22 +
+ a2n n):
+
+
+
n
a1
a2
an
n 1
(Phm Kim Hng)
V d 1.188 Cho cc s dng a1 ; a2 ; :::; an tha mn a1 + a2 + + an = n: Chng
minh rng
(n 1)(a21 + a22 +
+ a2n ) + na1 a2
an n2 :
V d 1.189 Cho cc s dng a1 ; a2 ; :::; an tha mn a1 a2
2n 1
rng vi mi 0 < kn
(n 1)2 ; ta c
p

1
1
+p
+
1 + kn a1
1 + kn a2

+p

1
1 + kn an

an = 1: Chng minh

n
:
1 + kn
(Vasile Cirtoaje)

Chng 2

Sng to bt ng thc
Bi ton 2.1 Cho cc s dng a; b; c tha mn a + b + c = 3: Chng minh rng
1
1
1
+
+
a3 + b2 + c b3 + c2 + a c3 + a2 + b

1:
(Dng c Lm)

Li gii. S dng bt ng thc Cauchy Schwarz, ta c


X

1
3 + b2 + c
a
cyc

a + b2 + c3
(a3 + b2 + c)(a + b2 + c3 )
cyc
P
P
3 + a2 + a3
2
3
X a+b +c
cyc
cyc
=
!2
2
2
2
2
(a + b + c )
P 2
cyc
a

cyc

Nn ta ch cn chng minh c
X
cyc

!2

3+

a2 +

cyc

a3

cyc

t q = ab + bc + ca; r = abc th bt ng thc ny tng ng vi


(9

2q)2

, 3(r

3 + (9
1) + (q

2q) + (27
3)

4(q

9q + 3r)
3)2

0:

Bt ng thc cui hin nhin ng nn ta c pcm. ng thc xy ra khi v ch khi


a = b = c = 1:
201

202

CHNG 2. SNG TO BT NG THC

Bi ton 2.2 Cho cc s dng a; b; c tha mn a + b + c = 3. Chng minh rng


a
b
c
+
+
3a2 + abc + 27 3b2 + abc + 27 3c2 + abc + 27

3
:
31

Li gii. S dng bt ng thc Schur, ta c


3abc

4(ab + bc + ca)

Nn ta ch cn chng minh c
X
cyc

,
,
,
vi s =

a
3a2 +

X
cyc

9a2

cyc

4(ab+bc+ca) 9
3

+ 27

3
31

3a
+ 4(ab + bc + ca) + 72

9a2

3
31

31a(a + b + c)
+ 4(ab + bc + ca) + 72

X 9a2 + 4(ab + bc + ca) + 8(a + b + c)2

31a(a + b + c)

a2 + s

cyc

4(ab+bc+ca)+72
:
9

X (7a + 8c + 10b)(c
cyc

,
,

(a

b)

cyc

X
cyc

(a

b)2

a) (7a + 8b + 10c)(a
a2 + s

8a + 7b + 10c
b2 + s

7a + 8b + 10c
a2 + s

b)

8a2 + 8b2 + 15ab + 10c(a + b) + s


(a2 + s)(b2 + s)

0:

Bt ng thc c chng minh xong. ng thc xy ra khi v ch khi a = b = c = 1:

Bi ton 2.3 Cho cc s dng a; b; c: Chng minh rng


a + 2b
b + 2c
c + 2a
+
+
c + 2b
a + 2c b + 2a

a
b
c
+
+
b+c c+a a+b

(Dng c Lm)

203
Li gii. Bt ng thc tng ng vi
X a + 2b
c + 2b

cyc

2
,

Do 2

a3

cyc

a2 b

cyc

cyc

ab(a + b)
X

"

X
3 a2 b
0 v

cyc

X
2 a3

cyc

a + 3abc

"

ab(a + b)

X
, 2 a3

cyc

ab2 +

cyc

cyc

(2a + b)(2b + c)(2c + a)


(a + b)(b + c)(c + a)

2 nn ta ch cn chng minh c

X
2 2 a3

cyc

ab(a + b)

cyc

cyc

X
3 a2 b

cyc

a3

(a + b)(b + c)(c + a)

(2a+b)(2b+c)(2c+a)
(a+b)(b+c)(c+a)

cyc

(2a + b)(2b + c)(2c + a)

cyc

a3 + 3abc

cyc

X
, 2 a3 + 3abc
P

X a
2
b+c
cyc

a2 b

ab(a + b)

cyc

3abc

3abc

0:

cyc

Theo bt ng thc AM-GM, ta c


X
2 a3
cyc

X
2 ab2
cyc

0;

a2 b

cyc

Bt ng thc c chng minh. ng thc xy ra khi v ch khi a = b = c:


Bi ton 2.4 Cho cc s dng a; b; c tha mn (a + b + c)
minh rng
p
p
p
7+8 2 5 5
a b
c
7+5 5
+ +
2
b
c a
2

1
a

1
b

p
8 2

1
c

= 10: Chng

(Phm Kim Hng, V Quc B Cn)


Li gii. Do tnh thun nht, khng mt tnh tng qut gi s a + b + c = 1; t
q = ab + bc + ca; r = abc th ta c q = 10r: Ta c
a b
c
ab2 + bc2 + ca2
+ + =
b
c a
abc

204

CHNG 2. SNG TO BT NG THC

Do , chng minh bt ng thc bn tri, ta ch cn xt n trong trng hp


c b a l , t
p
q 3r + q 2 4q 3 + 2(9q 2)r 27r2
c
ab2 + bc2 + ca2
a b
+ +
=
=
b
c a
abc
2r
v
u
q 2
q
q 2 4q 3 + 2(9q 2) 10 27 10
7 1u
=
+ t
q 2
2 2
10
s
q
p
1
7 1
7 1
253 40 10q +
253 80 10
=
+
+
2 2
q
2 2
p
p
7+8 2 5 5
=
2
Tip theo, ta s chng minh bt ng thc bn phi, r rng ta ch cn xt n trong
trng hp a b c l , khi
p
q 3r
q 2 4q 3 + 2(9q 2)r 27r2
c
ab2 + bc2 + ca2
a b
+ +
=
=
b
c a
abc
2r
v
u
q 2
q
2
4q 3 + 2(9q 2) 10 27 10
7 1u
tq
=
q 2
2 2
10
s
q
p
1
7 1
7 1
253 40 10q +
253 80 10
=
2 2
q
2 2
p
p
7+5 5 8 2
=
:
2
Bt ng thc c chng
minh
xong.
ng
thc btp ngp thc
bn tri xy
p
p
p
p
p
2 2 5
10
10
10 5 2+2 5
;
b
=
;
a
=
v cc hon
ra khi v ch khi c = 10 10+5
20
10
20
v tng
ng.
ng
thc

bt
ng
thc
bn
phi
xy
ra
khi
v
ch
khi a =
p
p
p
p
p
p
p
10+5 2 2 5
10
10 5 2+2 5
10
10
; b = 10 ; c =
v cc hon v tng ng.
20
20
Bi ton 2.5 Cho cc s dng a; b; c tha mn a + b + c = 3: Chng minh rng
p
p
p
a4 + b4 + c4 + 4
ab + bc + ca
15:
(Dng c Lm)
Li gii. Trc ht, ta s chng minh rng
P
P
a
a
X 1
cyc
cyc
P + P 2
a+b
ab 2 a
cyc
cyc

cyc

205

X c(a + b) + ab
a+b

cyc

cyc

a+

cyc

X ab
,
a+b
cyc

ab

cyc

a2

cyc

cyc

ab

cyc

a2

cyc

S dng bt ng thc Cauchy Schwarz, ta c

X ab
a+b
cyc

!2

ab

cyc

ab(a + b)

cyc

Nn ta ch cn chng minh
2

cyc

ab

cyc

cyc

b)2

ab(a

!"

ab(a + b)

cyc

cyc

Tr li bi ton ca ta, s dng bt ng thc GM-HM, ta c


Xp

ab

cyc

X ab
a+b
cyc

Nn ta ch cn chng minh c
X

X ab
a4 + 8
a+b
cyc
cyc

,
,
Theo trn, ta c

X
cyc

X
cyc

a4 +

a4 +

15

8 X ab(a + b + c)
3 cyc
a+b

X 1
8X
8
ab + abc
3 cyc
3
a+b
cyc

1X 1
3 cyc a + b

1
1
P + P 2
ab 2 a

cyc

cyc

15
15

206

CHNG 2. SNG TO BT NG THC

Nn ta ch cn chng minh c
X

a4 +

cyc

8X
3 cyc

1 C
B 1
ab + 8abc @ P + P 2 A
ab 2 a

t q = ab + bc + ca; r = abc th ta c
X
a4 = 81

15

cyc

cyc

36q + 2q 2 + 12r

cyc

Nn bt ng thc tng ng vi
33
Nu 9

50
12r(3 + 4q q 2 )
q + q2 +
3
q(9 2q)

4q th ta c
50
q + q2
3

33

nn bt ng thc ng.
Nu 4q 9 th theo bt ng thc Schur bc 4, ta c r
chng minh
2(4q 9)(9 q)(3 + 4q
50
q + q2 +
33
3
3q(9 2q)
, (99

50q + 3q 2 )q(9
, (q

2q) + 2(4q

9)(9

(4q 9)(9 q)
18

q2 )

q)(3 + 4q

3)(2q + 11q

117q + 162)

117q + 162

, f (q) = 2q + 11q

nn ta ch cn

0
q2 )

D thy f (q) l hm li nn
f (q)

max f (3); f

9
4

= max

36;

729
32

< 0:

Bt ng thc c chng minh xong. ng thc xy ra khi v ch khi a = b = c = 1:


Bi ton 2.6 Cho cc s dng a; b; c tha mn a b + c: Chng minh rng
s
a
b
c
abc
+
+
+
2:
b+c c+a a+b
(a + b)(b + c)(c + a)
(Dng c Lm)

207
Li gii. Do a

b + c nn

(a + b)(a + c)(b + c)
abc

b+c
bc
a+b+c+
bc
a
b+c
bc
(a b c)(ab + ac bc)
2b + 2c +
+
bc
b+c
a(b + c)
b+c
bc
2(b + c)2
2b + 2c +
=
+1 9
bc
b+c
bc

=
=

) (a + b)(a + c)(b + c)

9abc

Do
VT

X a
+
=
b+c
cyc

p
abc(a + b)(b + c)(c + a)
(a + b)(b + c)(c + a)

X a
3abc
+
b + c (a + b)(b + c)(c + a)
cyc

(a

b c)(a + b c)(a b + c)
(a + b)(b + c)(c + a)

2
0:

Bt ng thc c chng minh xong. ng thc xy ra khi v ch khi a = b; c = 0


hoc a = c; b = 0 hoc a = 2b = 2c:
Bi ton 2.7 Cho cc s khng m a; b; c, khng c 2 s no ng thi bng 0: Chng
minh rng
r
r
r
2
2
2
1
3 a + bc
3 b + ca
3 c + ab
+
+
2+ p
:
3
2
2
2
2
2
2
b +c
c +a
a +b
2
(V Quc B Cn)
Li gii. Khng mt tnh tng qut, gi s a
r
3

a2 + bc
+
b2 + c2

r
3

b2 + ca
c2 + a2

b c; ta s chng minh
( r
)
2
2
3 4(a + b )
max 2;
c2 + ab

Ch rng
(a2 + bc)(b2 + ca)
(a2 + c2 )(b2 + c2 )
nn

r
3

a2 + bc
+
b2 + c2

r
3

b2 + ca
c2 + a2

(a2 + c2 )(b2 + c2 )
=1
(a2 + c2 )(b2 + c2 )
s

26

(a2 + bc)(b2 + ca)


(a2 + c2 )(b2 + c2 )

208

CHNG 2. SNG TO BT NG THC

Ta cn chng minh
r
r
2
2
2
a2 + bc
3 4(a + b )
3 b + ca
+
2
2
2
2
2
b +c
c +a
c + ab
s
!
r
r
2
2
2
2
a2 + bc b2 + ca
3 b + ca
3 a + bc
3 (a + bc)(b + ca)
+ 2
+3
+
, 2
b + c2
c + a2
(a2 + c2 )(b2 + c2 )
b2 + c2
c2 + a2
r
3

4(a2 + b2 )
c2 + ab
Li c
b2 + ca
b2 + c2

a2 + bc
c(a
=
a2 + c2

b)(a2 + b2 + c2 + ab ac
(a2 + c2 )(b2 + c2 )

b2 + ca
a2 + bc
1
b2 + c2
a2 + c2
Do , theo bt ng thc AM-GM,
s
!
r
r
2
2
2
2
3 a + bc
3 b + ca
3 (a + bc)(b + ca)
3
+
(a2 + c2 )(b2 + c2 )
b2 + c2
c2 + a2

bc)

(a2 + bc)(b2 + ca)


(a2 + c2 )(b2 + c2 )

6(a2 + bc)
a2 + c2
T , ta ch cn chng minh c
a2 + bc b2 + ca 6(a2 + bc)
+ 2
+ 2
b2 + c2
c + a2
a + c2
, f (c) + g(c)

trong

f (c) = (a + 7b)c5 + 3(a2


3

g(c) = (a

b)(3b + 2ab + 4a b

4(a2 + b2 )
c2 + ab

b2 )c4 + 2(a + b)(a + 3b)bc3


3

2 3

2 3

3a )c + (b a + 6b a + a b )c + ab(a

b)4

Ta s chng minh rng g(c)


0. Nu 3b3 + 2ab2 + 4a2 b 3a3
0, iu ny l
3
2
2
hin nhin. Nu 3b + 2ab + 4a b 3a3 0, khi do g(c) l hm lm theo c nn
g(c) min fg(0); g(b)g, m
g(0) = ab(a

b)4

0;

g(b) = b

1
(a
4

b)[(2a2

6ab

b2 )2 + 43b4 ] + 8b5

Khng nh c chng minh. Tr li bi ton ca ta, c 2 trng hp xy ra


2
2
+b2
1
Nu ac2 +ab
2 , ac 2+ab
+b2
2 , khi t khng nh trn, ta d dng i n kt lun.

Nu

a2 +b2
c2 +ab

2, khi t khng nh trn, ta ch cn chng minh


r
r
2
2
2
1
3 4(a + b )
3 c + ab
+
2+ p
3
c2 + ab
a2 + b2
2

209
p
p
3
2.
Bt ng thc ny hin nhin ng do hm 3 4x + x1 l hm tng vi mi x
Bt ng thc c chng minh xong. ng thc xy ra khi v ch khi a = b; c = 0
hoc cc hon v.
Bi ton 2.8 Cho a; b; c l cc s khng m tha mn a + b + c = 3. Chng minh
rng
3
a2 b + b2 c + abc 4:
2
(Vasile Cirtoaje, V Quc B Cn)
Li gii. Nu a

2b th
(a + c)2 b

a2 b

bc(a
3
abc =
2

b2 c

3
) a2 b + b2 c + abc
2

(a + c)2 b = 4
a+c
2

4
Nu 2b

a+c
2

2b + 2c)
2

a+c a+c
b
2
2
3
+b
=4

a, bt ng thc tng ng vi
f (c) = 4(a + b + c)3

Ta c

27a2 b

27b2 c

81
abc
2

3
[8(a + b + c)2 9b(3a + 2b)]
2
3 p
f 0 (c) = 0 , c = p
b(3a + 2b) a b
2 2
f 0 (c) =

Do 2b
f (c)

3
p

a nn
f

2 2

b(3a + 2b)

3 p
p
b(3a + 2b)
2 2

a + b, v ta d dng kim tra c

2b)2 (2a + b)
q
3
4 a2 + 5ab + 2b2 + b(3a+2b)
2
27ab(a

0:

Bt ng thc c chng minh. ng thc xy ra khi v ch khi a = 2; b = 1; c = 0


hoc a = 0; b = 2; c = 1.
Bi ton 2.9 Cho a; b; c l cc s khng m, khng c 2 s no ng thi bng 0.
Chng minh rng
a2

1
1
1
+ 2
+ 2
2
2
+ ab + b
b + bc + c
c + ca + a2

9
:
(a + b + c)2
(Vasile Cirtoaje)

210

CHNG 2. SNG TO BT NG THC

Li gii. Bt ng thc c vit li nh sau


X (a + b + c)2
a2 + ab + b2
cyc
,
,

X
cyc

X
cyc

X c(a + b + c) + ab + bc + ca
cyc

c
2
a + ab + b

a2 + ab + b2
!
!
X
X
+
ab
2
cyc

cyc

6
1
2
a + ab + b2

S dng bt ng thc Cauchy Schwarz, ta c

X
cyc

X
cyc

c
2
a + ab + b2

!2

cyc

1
a2 + ab + b2

cyc

c(a2 + ab + b2 )

cyc

cyc

ab

= P

!2

c2 (a2 + ab + b2 )

cyc

=
P

!2

ab

cyc

cyc

!2

cyc

ab

cyc

cyc

Nn ta ch cn chng minh c
!2
P
a

cyc

cyc

ab

3abc

cyc

ab

cyc

!2

!2

3abc

cyc

Do tnh thun nht, ta c th gi s a + b + c = 1. t q =

ab; r = abc, bt ng

cyc

thc tr thnh

q
1
+ 2
q
2q
3r

S dng bt ng thc Schur bc 4, ta c r


1
q
+ 2
q
2q
3r

1
+
q
2q 2

6
(4q 1)(1 q)
,
6

q
(4q 1)(1 q)
2

6=

nn

(1 3q)(4q 1)2
q(8q 2 5q + 1)

0:

Bt ng thc c chng minh. ng thc xy ra khi v ch khi a = b = c:

211
Bi ton 2.10 Cho cc s khng m a; b; c; khng c 2 s no ng thi bng 0:
Chng minh rng
b4
c4
a4
+
+
a2 + ab + b2
b2 + bc + c2
c2 + ca + a2

a3 + b3 + c3
:
a+b+c
(Phan Thnh Vit)

Li gii. Ta c
a3 + b3 + c3
= a2 + b2 + c2
a+b+c

ab

bc

ca +

3abc
a+b+c

Nn bt ng thc cho tng ng vi


X

a2

cyc

a4
+ ab
+ ab + b2
X
cyc

a2

3abc
a+b+c

a2

ab3
+ ab + b2

3abc
a+b+c

S dng bt ng thc Cauchy Schwarz, ta c


!
!
X
X a2 + ab + b2
ab3
cyc

a2 + ab + b2

ab

cyc

cyc

!2

Nn ta ch cn chng minh
X
cyc

,
,

!2

3abc X a2 + ab + b2
a + b + c cyc
ab

X
cyc

X
cyc

!3

c(a2 + ab + b2 )

cyc

X
cyc

a2

X
cyc

ab

0:

iu ny hin nhin ng. ng thc xy ra khi v ch khi a = b = c hoc c = 0; ab ! 0


hoc cc hon v tng ng.
Bi ton 2.11 Cho a; b; c; d l cc s khng m tha mn a + b + c + d = 1: Chng
minh rng
a
b
c
d
3
p
+p
+p
+p
:
2
a+b
b+c
c+d
d+a
(Mircea Lascu)

212

CHNG 2. SNG TO BT NG THC

Li gii. Khng mt tnh tng qut, ta c th gi s a + c


S dng bt ng thc Jack Garfunkel, ta c
p

5p
5p
a+b+c=
1
4
4

b
c
a
+p
+p
c+a
a+b
b+c

a
b
+p
a+b
b+c
c
d
a
p
+p
+p
a+c
c+d
d+a
d
c
+p
)p
c+d
d+a
)p

b+d ) x = a+c

5p
1
4
5p

c
c+a
5p
a+c+d=
1
4
4
5p
a
1 b p
4
a+c
d

1
2:

Suy ra
X
cyc

5 p
1
4

a
a+b
=

b+

p
5p
2(2 b d)
c+a
4
p
p
p
( x 1) (17 x 7)
3
3
p
x= p
+
:
p
p
2
2
2 2 5 x + 1 + 2 (2 x + 3)

5p
2(x + 1)
4

a+c

Bt ng thc c chng minh xong. ng thc khng xy ra.


Bi ton 2.12 Cho a; b; c; d l cc s khng m, khng c 3 s no ng thi bng
0: Chng minh rng
p

a
b
c
d
+p
+p
+p
a+b+c
b+c+d
c+d+a
d+a+b

5p
a + b + c + d:
4

Li gii. Gi s d = minfa; b; c; dg v t x = a + c; khi ta d thy


p

a
d
+p
a+b+c
d+a+b
p

a
d
x
+p
=p
a+b+d
d+a+b
x+b

b
b+c+d

b
b+c

Nn ta ch cn chng minh
p

x
b
c
+p
+p
x+b
b+c
c+d+a
,p

x
b
c
+p
+p
c+x
x+b
b+c

5p
a+b+c+d
4
5p
x+b+d
4

y chnh l bt ng thc Jack Garfunkel nn bt ng thc cho c chng


minh. ng thc xy ra khi v ch khi a3 = 1b = 0c = d0 hoc cc hon v tng ng.

213
Bi ton 2.13 Cho a; b; c l cc s dng. Chng minh rng
2(a + b + c)(bc + ca + ab)
a2 + b2 + c2 + ab + bc + ca

((2a + b)(2b + c)(2c + a)) 3 :


(Sung Yoon Kim)

Li gii 1. S dng bt ng thc Cauchy Schwarz, ta c a2 b + b2 c + c2 a


(ab+bc+ca)2
. Suy ra
a+b+c
) (2a + b)(2b + c)(2c + a)

= 2(a + b + c)(ab + bc + ca) + 3abc + 2(a2 b + b2 c + c2 a)


2(a + b + c)(ab + bc + ca) + 3abc +

2(ab + bc + ca)2
a+b+c

Ta cn chng minh
2(a + b + c)(bc + ca + ab)
a2 + b2 + c2 + ab + bc + ca
2(ab + bc + ca)2
2(a + b + c)(ab + bc + ca) + 3abc +
a+b+c

1
3

Do tnh thun nht, ta c th chun ha cho a+b+c = 1: t q = ab+bc+ca; r = abc;


khi theo bt ng thc Schur bc 3, ta c r
max 0; 4q9 1 . Bt ng thc
tr thnh
1
2q
(3r + 2q 2 + 2q) 3
1 q
Nu 1

4q; th

(2q 2 + 2q) 3

(3r + 2q 2 + 2q) 3
v
2q 2 + 2q

Nu 4q

8q 3
(1 q)3

4q 2 + 2q 3
(1 q)3
2q(1 3q)
0
(1 q)3
2q(1

q4 )

2q

2q(1 2q 4q 2 )
(1 q)3

1; th
2

(3r + 2q + 2q)
v

6q 2 + 10q
3

1
3

2q + 2q +

8q 3
(1
=
(1 q)3

4q

1
3

3q)(2q 4

1
3

6q 2 + 10q
3

2q 3 + 3q 2 + 10q
(1 q)3

1)

1
3

0:

214

CHNG 2. SNG TO BT NG THC

ng thc xy ra khi v ch khi a = b = c:


Li gii 2. S dng bt ng thc Holder, ta c
[(a + b + c)(bc + ca + ab)]3
= [(a + b)(b + c)(c + a) + abc]3
[(a + b)3 + a3 ][(b + c)3 + b3 ][(c + a)3 + c3 ]
= (2a + b)(2b + c)(2c + a)(a2 + ab + b2 )(b2 + bc + c2 )(c2 + ca + a2 )
Ta cn chng minh
8(a2 + ab + b2 )(b2 + bc + c2 )(c2 + ca + a2 )

(a2 + b2 + c2 + bc + ca + ab)3

Bt ng thc ny c th c chng minh bng php khai trin.


Li gii 3. Ta s chng minh kt qu mnh hn l
a + 2b
b + 2c
c + 2a
+
+
a + 2c b + 2a
c + 2b

3(a2 + b2 + c2 )
ab + bc + ca

Cng 3 vo hai v, ta vit c bt ng thc trn dng


2(a + b + c)

1
1
1
+
+
a + 2c b + 2a c + 2b

3(a2 + b2 + c2 )
+3
ab + bc + ca

T y, s dng bt ng thc AM-GM, ta c th suy ra bt ng thc ban u ca


bi ton.
Do (a + b + c)2 = (a + 2b)(a + 2c) + (b c)2 nn
a+b+c
a + 2b
(b c)2
=
+
a + 2c
a + b + c (a + 2c)(a + b + c)
Xa+b+c

cyc

a + 2c

=3+

X
cyc

(b c)2
(a + 2c)(a + b + c)

Do , bt ng thc trn tng ng vi


6+2

X
cyc

(b c)2
(a + 2c)(a + b + c)

, x(b
vi x =

3(a+b+c)
4(ab+bc+ca)

1
a+2c

c)2 + y(c

3(a2 + b2 + c2 )
+3
ab + bc + ca

a)2 + z(a

b)2

v y; z tng t.

Khng mt tnh tng qut, gi s b l s hng nm gia a v c; tc l (b a)(b c)

0:

215
T , ta c (a b)(a c)
3(ab + bc + ca) + (a b)(a
3(ab + bc + ca)
= a + 2b
a + 2c
nn x
Nu a

0 v (c a)(c
c), ta c
(a

b)(a c)
a + 2c

b)

0: Ch rng (a + 2b)(a + 2c) =

a + 2b

2(a + b + c)

0: Tng t, ta cng c z 0:
b c; khi d thy y 0 do
1
b + 2a

1
a+b+c

a+b+c
3(ab + bc + ca)

3(a + b + c)
4(ab + bc + ca)

nn bt ng thc ng.
Nu c b a v nu y 0 nn bt ng thc ng. Gi s y
x + 2y =
do

9(a + b + c)
4(ab + bc + ca)

2
4a + b + 4c
1
+
=
a + 2c b + 2a
(a + 2c)(b + 2a)

v
z + 2y =
do

9(a + b + c)
4

9(a + b + c)
4(ab + bc + ca)

2
2a + 5b + 2c
1
+
=
c + 2b b + 2a
(c + 2b)(b + 2a)
c)2 + y(c

2
b + 2a

2(a + b + c)
ab + bc + ca
1
c + 2b

2(a

b)2 + 2(b

a)2 + z(a

b)2

(x + 2y)(b

0
9(a + b + c)
4(ab + bc + ca)

2
b + 2a

2(a + b + c)
ab + bc + ca

c)2

T y, vi ch rng (a
x(b

1
a + 2c

0; khi ta c

0
9(a + b + c)
4(ab + bc + ca)

c)2 ; ta c
c)2 + (z + 2y)(a

b)2

0:

Bt ng thc c chng minh.


Bi ton 2.14 Cho cc s dng a; b; c: Chng minh rng
3(a2 + b2 + c2 )
a+b+c

a(a + b) b(b + c) c(c + a)


+
+
:
a+c
b+a
c+b
(Phm Hu c)

Li gii 1. Ch rng
X a(a + b)
cyc

a+c

X ac
a+c
a+c
cyc
cyc
!
!
X
X a
X ab
=
a
a+c
a+b
cyc
cyc
cyc
!
!
X
X c
X ab
=
a
3
a+c
a+b
cyc
cyc
cyc
=

X a(a + b + c)

216

CHNG 2. SNG TO BT NG THC

Ta c th vit li bt ng thc nh sau


P
!
!
3 a2
X
X c
X ab
cyc
P
+
a
+
a+b
a+c
a
cyc
cyc
cyc

cyc

cyc

S dng bt ng thc Cauchy Schwarz, ta c

X ab
a+b
cyc

!2

ab

cyc

ab(a + b)

cyc

cyc

c
a+c

!2

c(a + c)

a2

cyc

cyc

3abc

!3

!2

cyc

ab

cyc

cyc

3abc

!2

ab

cyc

ab

!2

ab

cyc

cyc

cyc

!2

cyc

cyc

Ta cn chng minh
P

cyc

ab

cyc

!2

cyc

cyc

v
X

3
ab

cyc

cyc

Do tnh thun nht, ta c th chun ha cho a+b+c = 1: t q = ab+bc+ca; r = abc;


khi theo bt ng thc Schur bc 3, ta c r 4q9 1 . Bt ng thc tr thnh
3(1

,
Ta c

q2
1
+
q 3r 1 q

6q

2q) +

1
q2
+
q 3r 1 q

q2
1
+
q 3r 1 q
q2
q

4q 1
3

6q

1
1

6q =

(1 3q)2
1 q

0:

Bt ng thc c chng minh xong. ng thc xy ra khi v ch khi a = b = c:

217
Li gii 2. Bt ng thc cho tng ng vi
X a(a + b)(a + b + c)

3(a2 + b2 + c2 )

a+c

cyc

X ab(a + b)
a+c

cyc

X
cyc

Gi x; y; z l mt hon v ca a; b; c sao cho x


xy(x + y)

a2

ab

cyc

xz(x + z)

z; khi vi ch rng
yz(y + z)

S dng bt ng thc sp xp li, ta c


X ab(a + b)
cyc

xy(x + y) yz(y + z) zx(z + x)


+
+
y+z
y+x
z+x

a+c

xy(x + y) yz(y + z)
+
+ xz
y+z
y+x

By gi, ta thy
xy(x + y) yz(y + z)
+
y+z
y+x

xy(x z) yz(z x)
+
y+z
x+y
y(x z)2 (x + y + z)
=
(x
(x + y)(y + z)

(xy + yz) =

z)2

T y, ta c
X ab(a + b)
cyc

a+c

(x

z)2 + (xy + yz + xz)

2(x2 + y 2 + z 2 )

(xy + yz + zx):

Bt ng thc c chng minh.


Bi ton 2.15 Cho cc s khng m a; b; c; khng c 2 s no cng bng 0: Chng
minh rng
r
r
r
a
b
c
3
+
+
:
b + 3c
c + 3a
a + 3b
2
(Vasile Cirtoaje)
2

b
a
c
= z4 ; c+3a
= y4 ; b+3c
=
Li gii. t a+3b
, ta d dng kim tra c ng thc sau

x2
4 ;

vi x; y; z l cc s khng m. Khi

16 = 7x2 y 2 z 2 + 3(x2 y 2 + x2 z 2 + y 2 z 2 )

218

CHNG 2. SNG TO BT NG THC

v ta cn chng minh
x+y+z

Gi s x + y + z < 3; khi tn ti k > 1 sao cho kx + ky + kz = 3. t kx = u; ky =


v; kz = w th u + v + w = 3 v
16 =

3(u2 v 2 + u2 w2 + v 2 w2 )
7u2 v 2 w2
+
< 7u2 v 2 w2 + 3(u2 v 2 + u2 w2 + v 2 w2 )
6
k
k4

Nhng ta d dng chng minh c


16

7u2 v 2 w2 + 3(u2 v 2 + u2 w2 + v 2 w2 )

vi mi u; v; w
0 tha mn u + v + w = 3; iu ny dn n mu thun nn bt
ng thc cn chng minh ng.
ng thc xy ra khi v ch khi a = b = c:
Bi ton 2.16 Cho cc s dng a; b; c: Chng minh rng
p
p
p
3
3
3
(a + b)2 (b + c)2 (c + a)2
(a b)2 (b c)2 (c a)2 + 4 a2 b2 c2 :

(Sung Yoon Kim)

Li gii. S dng bt ng thc AM-GM, ta c


s
s
b)2 (b c)2 (c a)2
a2 b2 c2
3 (a
+43
2
2
2
2
(a + b) (b + c) (a + c)
(a + b) (b + c)2 (a + c)2
s
s
2 (b
2 (c
2
(a
b)
c)
a)
ab
bc
= 3
+43
(a + b)2 (b + c)2 (c + a)2
(a + b)2 (b + c)2
1 X (a b)2
4X
ab
+
2
3 cyc (a + b)
3 cyc (a + b)2
=

ca
(c + a)2

4ab
1 X (a b)2
+
= 1:
3 cyc (a + b)2
(a + b)2

Bt ng thc c chng minh. ng thc xy ra khi v ch khi a = b = c:


Bi ton 2.17 Cho a; b; c l cc s dng tha mn a2 b2 + b2 c2 + c2 a2 = 3: Chng
minh rng
r
r
r
a + bc2
b + ca2
c + ab2
3
+
+
:
2
2
2
abc
(Sung Yoon Kim)

219
Li gii. S dng bt ng th AM-GM, ta c
X
cyc

a + bc2
2

X
cyc

1
4

ab

1X
2 cyc
!

1 ab + b2 c2
b
2

1 ab + b2 c2
+
b
2

X1 X
+
ab + 3
a cyc
cyc

v
1X 2 2
(a b + 1) = 3
2 cyc

cyc

P
ab
X1
cyc
)
=
a
abc
cyc
Do
X
cyc

a + bc2
2

1
4

6
+6
abc

3
abc

3
3
+
2abc 2

3
:
abc

Bt ng thc c chng minh. ng thc xy ra khi v ch khi a = b = c = 1:


Bi ton 2.18 Cho k > 0 l mt hng s cho trc. Tm hng s ln nht sao
cho vi mi a b c d 0 tha a + b + c + d = 1; bt ng thc sau ng
a b
b c
c d
d a
+
+
+
k+a+b k+b+c k+c+d k+d+a

(a

b)(b

c)(c

d):
(Shalex)

Li gii. Cho a =
tr thnh

1
4

+3 ;b =

2
Cho

! 0+ , ta c

1
4

+ ;c =

(2k + 1)3

1
4

;d =

64
64(2k + 1)

64
(2k + 1)3

1
4

3 vi

! 0+ , bt ng thc

220

CHNG 2. SNG TO BT NG THC

Mt khc, t x = a

b; y = b

c; z = c

d ) x; y; z

0, ta c

a b
b c
c d
d a
+
+
+
k+a+b k+b+c k+c+d k+d+a
a b
c d
b c
d a
=
+
+
+
k+a+b k+c+d
k+b+c k+d+a
= [k(a

b+c

d) + (a2

b2 + c2

d2 )]

1
1
(k + a + b)(k + c + d) (k + b + c)(k + c + d)
[(k + a + b)x + (k + c + d)z](x + y)(y + z)
=
(k + a + b)(k + b + c)(k + c + d)(k + d + a)
p
8xyz (k + a + b)(k + c + d)
(k + a + b)(k + b + c)(k + c + d)(k + d + a)
8xyz
= p
(k + a + b)(k + c + d)(k + b + c)2 (k + d + a)2
64
xyz:
(2k + 1)3
vi bt ng thc cui ng theo bt ng thc AM-GM v gi thit a + b + c + d = 1:
T , ta i n kt lun
64
:
max =
(2k + 1)3
n
P

Bi ton 2.19 Cho x1 ; x2 ; :::; xn l cc s dng tha mn

xi = 1: Chng minh

i=1

rng

n q
X
x2i + x2i+1

2
2

i=1

1
n
P

i=1

x2i
xi+1

Li gii. Bt ng thc cho tng ng


n
X

xi + xi+1

n
X

x2i

i=1

q
x2i + x2i+1
x2i

i=1 x
i+1

+ xi +

xi
xi+1

x2i

x2i+1

2
2

1
n
P

i=1

x2i
xi+1

1
n
P
x2i
2 + 2 xi+1
i=1

221
S dng bt ng thc Cauchy Schwarz, ta c
1"
0
#
n
n
q
2
2
X
X
x
x
x
i
i
i
2
2
A
@
q
+ xi +
xi + xi+1
x2i
xi
xi+1
xi+1
2 + x2
+
x
+
x
i=1
i=1 x
i
i+1
i
xi+1
i+1
!2
n
X
xi
=1
i=1

Ta cn chng minh
n
X
i=1

xi q 2
xi + x2i+1
xi+1
,

n
X
i=1

Do

xi xi+1

x2i
xi+1

x2i + x2i+1 + xi

q
x2i + x2i+1

xi + xi+1
p
2

Nn ta ch cn chng minh c
n
X
i=1

n
X
i=1

xx
p i i+1
1 + 2 xi + xi+1

xx
p i i+1
1 + 2 xi + xi+1
,

n
X
i=1

p
2

n
X
i=1

2
2

!
p
p
2 1
2 2
xi +
xi+1
2
2

1 (xi xi+1 )2
p
1 + 2 xi + xi+1

0:

Bt ng thc c chng minh xong. ng thc xy ra khi v ch khi x1 = x2 =


= xn = n1 :
Bi ton 2.20 Cho cc s thc a; b; c; d: Chng minh rng
!
!
X
X
5X 4 X 2 2
3
a +
a b
a
a + (a b)(b c)(c
2 cyc
sym
cyc
cyc

d)(d

a):

(Phm Minh Khoa)

222

CHNG 2. SNG TO BT NG THC

Li gii. Do
5X 4 X 2 2
a +
a b
2 cyc
sym
X

cyc

cyc

cyc

b2 )2

!2

1X 2
(a
2 sym

b)2

cyc

!2

Nn ta c th vit li bt ng thc nh sau


X
1X 2
b2 )2
(a
ab(a b)2 + (a
2 sym
sym
,

1X
(a
2 sym

1
2 (a

Ch rng a2 + b2
1X
(a
2 sym

b)2 (a2 + b2 )

(a

ab(a

sym

b)(b

b)(b

c)(c

c)(c

d)(d

d)(d

a)

a)

b)2 , nn
1X
(a
4 sym

b)2 (a2 + b2 )

b)4

1
[(a b)4 + (b c)4 + (c d)4 + (d
4
j(a b)(b c)(c d)(d a)j
(a b)(b c)(c d)(d a):

a)4 ]

Bt ng thc c chng minh xong.


Bi ton 2.21 Cho cc s dng a; b; c tha a2 + b2 + c2 = 3: Chng minh rng
r
r
r
p
a2
b2
c2
+
+
3:
2
2
2
a +b+c
b +c+a
c +a+b
Li gii. Theo bt ng thc Cauchy Schwarz, ta c a + b + c 3. T
!2
!
!
r
X
X
X
a2
a
a
a2 + b + c
a2 + b + c
cyc
cyc
cyc
!
!
X
X
a
a
a2 + 31 (b + c)(a + b + c)
cyc
cyc
Ta cn chng minh
X
cyc

X
cyc

a
a2 + 31 (b + c)(a + b + c)

223
Bt ng thc ny thun nht nn ta c th b qua gi thit a2 + b2 + c2 = 3 v
chun ha cho a + b + c = 1; khi bt ng thc tr thnh
X
cyc

Ta c

X
cyc

3a
3a2 a + 1

3a2

3=

3a
a+1

X
cyc

3a2

3a
a+1

2a

X (2a + 1)(3a
cyc

3(3a2

1)2
a + 1)

1
3
0:

Bt ng thc c chng minh. ng thc xy ra khi v ch khi a = b = c = 1:


Bi ton 2.22 Cho a; b; c l cc s dng. Chng minh rng
a
b
c
+
+
b + c2
c + a2
a + b2

1
4

a
b
c
+ 2+ 2
c2
a
b

a b
c
+ +
b
c a

Li gii. S dng bt ng thc AM-GM, ta c


b
c
a
+
+
b + c2
c + a2
a + b2

a
c
b
p + p + p
2
ca
2
ab
2 bc
p p
p p
p
1
a
a
b
b
c
p
=
+p
+p
2
c a
a
b c
s
1
a
b
c
a b
+ +
+ 2+ 2
2
2
c
a
b
b
c

p !
c
b
c
:
a

Bt ng thc c chng minh. ng thc xy ra khi v ch khi a = b = c = 1:


Bi ton 2.23 Cho cc s dng a; b; c: Chng minh rng
r
r
r
a b
c
a2 + c2
b2 + a2
c2 + b2
+ +
+
+
:
2
2
2
2
b
c a
b +c
c +a
a2 + b2
(V Quc B Cn)
Li gii 1. Bnh phng 2 v, ta c th vit li bt ng thc nh sau
!
r
X a2
2b a2 + c2
b2 + c2
+
2
0
b2
a
b2 + c2
a2 + c2
cyc

224

CHNG 2. SNG TO BT NG THC

Suy ra, ta ch cn chng minh


2b
a2
+
b2
a
,

a2 + c2
b2 + c2

b2 + c2
a2 + c2
r

a3 c2 + 2b5 + 2b3 c2 ab2 c2


ab2 (b2 + c2 )

0
b2 + c2
a2 + c2

S dng bt ng thc AM-GM, ta c


a3 c2 + 2b3 c2
) a3 c2 + 2b5 + 2b3 c2

3ab2 c2
ab2 c2

Ta cn phi chng minh

b3 + ac2
a(b2 + c2 )

2b2 (b3 + ac2 )

b2 + c2
a2 + c2

Nhng bt ng thc ny hin nhin v


(b3 + ac2 )2 (a2 + c2 )

a2 (b2 + c2 )3 = c2 (a

b)2 (a2 c2 + 2b3 a + 2c2 ab + b4 )

ng thc xy ra khi v ch khi a = b = c:


Li gii 2. Trc ht, ta s chng minh bt ng thc sau
r
r
b
a
a2 + c2
b2 + c2
+
+
2
2
b
a
b +c
a2 + c2
r
r
a
b
a2 + c2
b2 + c2
, +
2
+
2
2
2
b
a
b +c
a2 + c2
p
p
2
a2 + c2
b2 + c2
(a b)2
p
,
ab
(a2 + c2 )(b2 + c2 )

(a2 b2 )2
p
p
p
2
(a2 + c2 )(b2 + c2 ) a2 + c2 + b2 + c2
p
p
p
2
, (a2 + c2 )(b2 + c2 )
a2 + c2 + b2 + c2
ab(a + b)2
,

(a

b)2
ab

Bt ng thc cui hin nhin ng v


p
p
a2 + c2 + b2 + c2 a + b;
T , ta c

Xa
cyc

Xb
+
b
a
cyc

X
cyc

(a2 + c2 )(b2 + c2 )

a2 + c2 X
+
b2 + c2
cyc

b2 + c2
a2 + c2

a2 b2

0:

225
Gi s

cyc

a
b

<

P q a2 +c2
b2 +c2

cyc

, khi ta c

cyc

Xa
cyc

,
Do

cyc

b
a

>

<

X
cyc

b
a

>

P q b2 +c2

cyc

a2 +c2 .

cyc

a2 +c2

Xb
X
X a2 + c2
+
2
+
2
<
b2
a
b2 + c2
cyc
cyc
cyc

b2 + c2
a2 + c2

nn
X a2
cyc

iu ny mu thun v

Nhng

a2 + c2
b2 + c2

X a2

P q b2 +c2

cyc

b2

X a2
cyc

b2

<

X a2 + c2
cyc

b2 + c2

X a2 + c2
cyc

b2 + c2

Tht vy, gi s c = minfa; b; cg, th bt ng thc ny tng ng vi


1
a2 b2

1
(a2 b2 )2
(a2 + c2 )(b2 + c2 )
1
1
(a2
+ 2 2
a c
(a2 + b2 )(a2 + c2 )

c2 )(b2

c2 )

hin nhin ng. Vy nn ta phi c


Xa
cyc

X
cyc

a2 + c2
:
b2 + c2

Bi ton 2.24 Cho cc s khng m a; b; c tha mn a + b + c = 1: Chng minh rng


p
p
p
p
a + (b c)2 + b + (c a)2 + c + (a b)2
3:

(Phan Thnh Nam)

Li gii. Bnh phng 2 v, bt ng thc cho c th c vit li nh sau


X
Xp
(b c)2 + 2
[a + (b c)2 ][b + (a c)2 ] 2
cyc

cyc

226

CHNG 2. SNG TO BT NG THC

S dng bt ng thc Cauchy Schwarz, ta c


Xp
X
Xp
[a + (b c)2 ][b + (a c)2 ]
ab +
j(a
cyc

cyc

Nn ta ch cn chng minh c
X
Xp
X
(b c)2 + 2
ab + 2
j(a
cyc

cyc

c)(b

c)j

cyc

c)(b

c)j

cyc

Khng mt tnh tng qut, gi s a b c; khi


X
Xp
X
(b c)2 + 2
ab + 2
j(a c)(b c)j 2
cyc

cyc

cyc

= 4(a

c)

2 1

Xp
cyc

ab

p
p 2
+
b
c
h
i
p
p 2
p 2
p 2
p
p
4(a c)2 2 a
c =2 a
c
2 a+ c
1
p 2
p
p 2
p
c [2(a + c) 1] = 2 a
c (a b + c) 0:
2 a

= 4(a

c)2

Bt ng thc c chng minh xong. ng thc xy ra khi v ch khi a = b = c =


hoc a = b = 21 ; c = 0 hoc cc hon v.

1
3

Bi ton 2.25 Cho cc s dng a; b; c tha a + b + c = 3: Chng minh rng


a
b
c
+
+
2b + 1 2c + 1 2a + 1

1
:
abc
(V Quc B Cn)

Li gii. Bt ng thc cho tng ng vi


3

a
2b + 1

+ 3

b
2c + 1

+ 3

c
2a + 1

1
abc

7b + c 7c + a 7a + b
1
+
+
+
9
2b + 1 2c + 1 2a + 1 abc
c 1
a 1
b 1
2
1
1
1
+
+
+
+3 5
+
+
2b + 1 2c + 1 2a + 1
abc
2a + 1 2b + 1 2c + 1
,

,2

Nn chng minh bt ng thc cho, ta ch cn chng minh 2 bt ng thc sau


a 1
b 1
c 1
+
+
2b + 1 2c + 1 2a + 1

227
2
+3
abc

1
1
1
+
+
2a + 1 2b + 1 2c + 1

a) Trc ht, ta s chng minh


c 1
a 1
b 1
+
+
2b + 1 2c + 1 2a + 1
, 2(a2 + b2 + c2

ab

bc

ca) + 4(a2 b + b2 c + c2 a)

, a3 + b3 + c3 + 12(a2 b + b2 c + c2 a)
,

6(ab2 + bc2 + ca2 ) + 21abc

a3 + b3 + c3 + 3(a2 b + b2 c + c2 a + ab2 + bc2 + ca2 )


6(ab2 + bc2 + ca2 a2 b b2 c c2 a)

, (a + b + 7c)(a b)2 + (b + c + 7a)(b


18(a b)(b c)(c a)

21abc

c)2 + (c + a + 7b)(c

T y, gi s a = min fa; b; cg v t b = a + x; c = a + y (x; y


dng bin i bt ng thc v dng tng ng l
9(x2

12

xy + y 2 )a + x3 + 3x2 y + y(3x

y)2

a)2

0); ta c th d

Nn bt ng thc hin nhin ng.


b) Tip theo, ta s chng minh
2
+3
abc

1
1
1
+
+
2a + 1 2b + 1 2c + 1

t q = ab + bc + ca; r = abc th ta c
1
1
1
4q + 15
+
+
=
2a + 1 2b + 1 2c + 1
8r + 4q + 7
Nn bt ng thc tng ng vi
3r + 2
r
,

2(1

5(4q + 15)
8r + 4q + 7

r)(4q + 7 12r)
r(8r + 4q + 7)

Bt ng thc cui hin nhin ng v 3


ra khi v ch khi a = b = c = 1:

0:

3r: Vy ta c pcm. ng thc xy

228

CHNG 2. SNG TO BT NG THC

Bi ton 2.26 Cho cc s dng a; b; c tha a

c v abc = 1: Chng minh rng

1
1
1
+ 3:
+
a b2
c

a + b2 + c3

(Rachid)
Li gii. T gi thit, ta suy ra c
1
c3

, (c3 a
Do c3 a

c2 ba

1=c

1: Bt ng thc cho tng ng


1
a

1
b2

b2

1
1
+ 3
a c

b4

+ c3
1)

0 nn nu b

(c3 a
Nu 1

1; bc

1)

b2

1 th bt ng thc hin nhin ng v

1
1
+
a c3

b4

b2

b; ta c

c3 a + b4

1
a

a(c3 + b3 )

1
b2

abc(b + c)

) c3 a

2=b+c
b4

bc

,b
a th bt ng thc hin nhin ng. Xt trng hp ngc
Nn nu
li a b2 ; khi ta c
c3 a

c(1

b4 )

c3 b2 + cb4 c 1
= c3 b2 + cb4 c (abc)4=3 (abc)5=3
c3 b2 + cb4 c (b2 c)4=3 (b2 c)5=3
= b2 c(c2 + b2 b4=3 c2=3 b2=3 c4=3 ) 0

) c3 a

c(1

b4 )

Li c
c
1
b2 c a
b a
=
0:
2
2
a b
ab
ab2
Bt ng thc c chng minh xong. ng thc xy ra khi v ch khi a = b = c = 1:
Bi ton 2.27 Cho cc s dng x; y; z tha x2 + y 2 + z 2 = 1: Chng minh rng
1
1
1
+ 2+ 2
x2
y
z

2
xyz

p
6 3:
(Ji Chen)

229
Li gii. Bt ng thc tng ng vi
yz
zx xy
+
+
x
y
z
,

yz
zx xy
+
+
x
y
z

zx
t a = yz
x ;b = y ;c =
ng thc tr thnh

xy
z

p
6 3 xyz

2+9

2
p
3

1 xyz

2
p
3

2+9

1 abc

2
p
3

1 abc
=

2+

(2

p)

(2

p)

2
p
3
2
p
3
2
p
3

3; bt

Nu p
2 th bt ng thc hin nhin ng. Nu 2
thc Schur bc 3, ta c 9abc p(4 p2 ); suy ra
2+9

v p = a + b + c th ta c ab + bc + ca = 1 v p
p

3 th theo bt ng

p2 )

1 p(4

1 p(p + 2)

3+2

1 = 0:

Bt ng thc c chng minh xong. ng thc xy ra khi v ch khi x = y = z =


p1 :
3
Bi ton 2.28 Cho cc s dng x; y; z: Tm hng s k > 0 ln nht sao cho
3yz +

z)2

(y
k

3zx +

x)2

(z
k

3xy +

y)2

(x
k

xyz(x + y + z)3 :
(Ji Chen)

Li gii. t a = x1 ; b = y1 ; c =
3ab +

b)2

(a
k

1
z

3bc +

th ta c a; b; c > 0 v bt ng thc trn tr thnh


c)2

(b
k

3ca +

a)2

(c
k

(ab + bc + ca)3

Cho c ! 0; a = b = 1 th bt ng thc tr thnh


3
k2

1,k

Tip theo, ta s chng minh rng


27 ab +

(a b)2
p
3 3

bc +

(b c)2
p
3 3

ca +

(c a)2
p
3 3

(ab + bc + ca)3

230

CHNG 2. SNG TO BT NG THC


1
p
3 3

t t =

th ta c
ab +

cyc

(a b)2
p
3 3
b)2

ab + t(a

cyc

= [ab + t(a b)2 ][t2 (b c)2 + tbc(c a)2 + tca(b c)2 + abc2 ]
Y
X
X
= t3 (a b)2 + t2
bc(a b)2 (a c)2 + tabc
c(a b)2 + a2 b2 c2
cyc

t2

cyc

b)2 (a

bc(a

c)2 + tabc

cyc

cyc

b)2 + a2 b2 c2

c(a

cyc

"
1 X
bc(a
=
27 cyc

X
p
c) + 3 3abc
c(a

b) (a

2 2 2

b) + 27a b c

cyc

v
X

!3

27a2 b2 c2

ab

cyc

a3 b3

cyc

3a2 b2 c2

!"

ab

cyc

bc(a

+ 3abc

"

ab(a + b)

c) + 3abc

cyc

6abc

cyc

b)(a

b)2

c(a

cyc

Nn ta ch cn chng minh
X

bc(a

b)2 (a

cyc

bc(a

X
p
c(a
c)2 + 3 3abc

cyc

!"

c)(a2

2ab

X
b)(a

ab

b)2

cyc

bc(a

b)(a

c) + 3abc

cyc

2ac) + 3

c(a

b)2

cyc

1 abc

c(a

b)2

b)(a

c)

cyc

cyc

Ta c
3

1 abc

X
cyc

c(a

b)2

2abc

X
cyc

c(a

b)2 = 2

X
cyc

(b + c)(a

231
V
X

bc(a

c)(a2

b)(a

cyc

abc

2ab

2ac) + 2abc

(b + c)(a

b)(a

c)

cyc

a(a

b)(a

c)

0:

cyc

Bt ng thc c chng minh xong. Ta i n kt lun


p
kmax = 3:

Bi ton 2.29 Cho cc s x; y; z > 0: Chng minh rng


y
z
x
+ +
x y
z

x2
y2
z2
+
+
x2 + 2yz
y 2 + 2zx z 2 + 2xy

:
(Dng c Lm)

Li gii. Ta s s dng kt qu sau


B . Vi mi a; b; c > 0; ta c
9(a2 + b2 + c2 )
(a + b + c)2

c
a b
+ +
b
c a

(V Quc B Cn)
Chng minh. Bt ng thc tng ng
X
cyc

X a2
cyc

X a2

X
cyc

X
cyc

cyc

(a

Xa
cyc

X ab
cyc

b)2

X ab
cyc

1
c
+
b 2ab

a2

cyc

cyc

9
a

cyc

a2

cyc

cyc

cyc

3
a+b+c

a2

cyc

X
cyc

cyc

232

CHNG 2. SNG TO BT NG THC


,

3c
t z = 2a
b + b +
, khi ta c

x=

c
a

cyc

c2
ab

(a

b)2

c
c2
2a 3c
+
+ +
b
b
a ab

4 v x; y tng t. R rng ta ch cn xt a

2b 3a a a2
+
+ +
c
c
b
bc

2+3+1+1

4=3>0

y+z

2y + z

b2
c2
2a 3b b 3c 3c
+
+ +
+
+
+
8
b
a
c
b
a
ca ab
2a 3b b 3c 3c
b2
b2
c2
=
+
+ +
+
+
+
+
b
a
c
b
a
2ca 2ca ab
3 b
2a 3b b 3c 3c
+
+ +
+
+ p
8
3
b
a
c
b
a
4 a
2a
9b
b 3c 3c
+
+ +
+
8
b
2a c
b
a
a
b
3c
a
9b
2b 3c
=
+
+
+
+
+
+
b
3c
a
b
2a
3c
b
p
p
p
3+3 2+2 2 8=5 2 5>0
=

2a 6b 2b 3c 5c 2b2
c2
+
+
+
+
+
+
12
b
a
c
b
a
ca
ab
2a 6b 2b 3c 5c
b2
b2
c2
=
+
+
+
+
+
+
+
b
a
c
b
a
ca ca ab
2a 6b 2b 3c 5c 3b
+
+
+
+
+
12
b
a
c
b
a
a
2a 9b 2b 3c 5c
=
+
+
+
+
12
b
a
c
b
a
a
b
5c
a 9b
9b 3c
=
+
+
+
+
+
+
b
5c
a
b
a
5c
b
r
r
3
3
3+6+6
12 = 6
3>0
5
5

12

12

c l

233
x + 2y

Do
Nu y

0 th ta c
X
z(a

2b2
a 6b 4b 3a 4c a2
+
+
+
+
+
+
12
b
a
c
c
a
bc
ca
a 6b 4b 3a 4c
a2
2b2
+
+
+
+
+
+
12
b
a
c
c
a
b a b a
2a 8b 4b 3a 4c
=
+
+
+
+
12
b
a
c
c
a
c
a 4b
4b
a
c
+
= 2
+
+
+3
+
12
b
a
c
c
a
a
8 + 4 + 6 + 0 12 = 6 > 0
=

b)2

y(a

c)2 + z(a

b)2

x(b

c)2 + y[2(b

(y + z)(a

b)2

cyc

Nu y

0 th ta c
X
z(a b)2

c)2 + 2(a

b)2 ] + z(a

b)2

cyc

c)2 + (z + 2y)(a

(x + 2y)(b

B c chng minh xong.


Tr li bi ton ca ta, t a = x1 ; b = y1 ; c =
Xa
cyc

Xa
cyc

X
cyc

+6

1
z

bc
2a2 + bc
9

S dng bt ng thc Cauchy Schwarz, ta c

X
cyc

a2
2a2 + bc

a2 +

cyc

Mt khc, theo b trn, ta c


Xa
cyc

cyc

cyc

cyc

!2

cyc

0:

th bt ng thc tr thnh

a2
2a2 + bc

cyc

b)2

a2
!2

bc

234

CHNG 2. SNG TO BT NG THC

Nn ta ch cn chng minh c

cyc

t t =

cyc

bc
!2

1
3

!2 +

!2

cyc

a2 +

cyc

cyc

cyc

a2

bc

cyc

th bt ng thc tr thnh

3(1

2t) +

2
2t) + t

2(1

3t)2
3t

2(1
2

0:

Bt ng thc cui hin nhin ng nn ta c pcm. ng thc xy ra khi v ch khi


x = y = z:
Bi ton 2.30 Cho cc s khng m a; b; c; khng c 2 s no ng thi bng 0:
Chng minh rng

a3

a4
b4
c4
+ 3
+ 3
3
3
+b
b +c
c + a3

a+b+c
:
2
(Vasile Cirtoaje)

Li gii. S dng bt ng thc Cauchy Schwarz, ta c


X

a4
a3 + b3

cyc

!"

a (a + b )

cyc

cyc

!2

Ta cn chng minh

cyc

!2

X
cyc

!"

X
cyc

a (a + b )

235

S dng bt ng thc Vasile

ab

cyc

X
cyc

1
3

cyc

!2

, ta c

!
!
ab
X
X
X
3
2
2
2
a (a + b ) =
a + P
ab +
a b
abc
a
a cyc
cyc
cyc
cyc
cyc
P 2
3
!2
!
ab
X
X
X
X
1
cyc
5
2
2
2
2
a
+
a b 5 abc
a + P 4
a
a 3 cyc
cyc
cyc
cyc
2

cyc

cyc

Do tnh thun nht, ta c th chun ha cho a+b+c = 1. t q = ab+bc+ca; r = abc,


khi ta c
P 2
3
!2
!
ab
X
X
X
X
1
cyc
a2
+
a2 b2 5 abc
a2
a5 + P 4
a 3 cyc
cyc
cyc
cyc
cyc

1
[3(4
3

v
X

a3

cyc

!2

14q + 11q 2 + 7q 3 ]

5q)r + 3

3q + 3r)2

= (1

Ta phi chng minh


2(1

3q + 3r)2

, 54r2 + 3(8

1
[3(4
3

5q)r + 3

22q + 43q 2

28q)r + 3

4q 1
9

S dng bt ng thc Schur bc 3, ta c r


28q)r tng vi mi r 4q9 1 , ta c
f (r) + 3

22q + 43q 2

7q 3

9qr

b)2 (b
r

c)2 (c
1 h
9q
27

a)2

7q 3

9qr

1
+3
9
22
49
q + q2
3
3

22q + 43q 2
7q 3

0, ta suy ra c

2 + 2(1

v ch rng f (r) = 54r2 + 3(8

4q

f
= 1

T bt ng thc (a

14q + 11q 2 + 7q 3 ]

p
3q) 1

3q

9qr

7q 3

9qr

236

CHNG 2. SNG TO BT NG THC

Do , ta ch cn chng minh
1

22
49
q + q2
3
3

7q 3

1 h
q 9q
3

2 + 2(1

p
3q) 1

3q

p
1
0
(1 3q) 7q 2 11q + 3 2q 1 3q
3
S dng bt ng thc AM-GM, ta c
p
7q 2 11q + 3 2q 1 3q 7q 2 11q + 3 (q 2 + 1 3q) = 2(1

q)(1

3q)

0:

Bt ng thc c chng minh. ng thc xy ra khi v ch khi a = b = c:

Bi ton 2.31 Cho cc s dng a; b; c: Chng minh rng


p
p
p
3
3
3
(a + b)2 (b + c)2 (c + a)2
(a b)2 (b c)2 (c a)2 + 4 a2 b2 c2 :

(Sung Yoon Kim)

Li gii. S dng bt ng thc Cauchy Schwarz, ta c


s
s
b)2 (b c)2 (c a)2
a2 b2 c2
3 (a
+43
2
2
2
2
(a + b) (b + c) (a + c)
(a + b) (b + c)2 (a + c)2
s
s
2 (b
2 (c
2
(a
b)
ab
c)
a)
bc
= 3
+43
(a + b)2 (b + c)2 (c + a)2
(a + b)2 (b + c)2
ab
4X
1 X (a b)2
+
2
3 cyc (a + b)
3 cyc (a + b)2
=

ca
(c + a)2

1 X (a b)2
4ab
+
= 1:
3 cyc (a + b)2
(a + b)2

ng thc xy ra khi v ch khi a = b = c:


Bi ton 2.32 Cho cc s dng a; b; c tha mn abc = 1: Tm hng s k ln nht
sao cho bt ng thc sau ng
1
1
1
+
+
a(1 + bc)2
b(1 + ca)2
c(1 + ab)2

k
3
+
(1 + ab)(1 + bc)(1 + ca) 4

k
8

(V Quc B Cn)
Li gii 1. Cho a = 2; b = 1; c = 21 ; ta c k 4: Ta s chng minh y l gi tr
m ta cn tm, tc l
X
1
16
4
1+
2
a(1
+
bc)
(1
+
ab)(1
+
bc)(1 + ca)
cyc

237
,4
t x =

1 a
1+a ; y

X
cyc

1 b
1+b ; z

(1

a
(a + 1)2

1 c
1+c ;

x)(1

1+

16
(1 + a)(1 + b)(1 + c)

th ta c x; y; z 2 [ 1; 1] v
y)(1

z) = (1 + x)(1 + y)(1 + z)

) x + y + z + xyz = 0
Bt ng thc tr thnh
X

(1

x2 )

1 + 2(1 + x)(1 + y)(1 + z)

cyc

, x2 + y 2 + z 2 + 2(xy + yz + zx) + 2(x + y + z + xyz)


, (x + y + z)2

hin nhin ng.


Bt ng thc c chng minh. Vy ta i n kt lun
kmax = 4:

Li gii 2. Tng t nh trn, ta cn phi chng minh


4

X
cyc

1
a(1 + bc)2

1+

16
(1 + ab)(1 + bc)(1 + ca)

V a; b; c > 0; abc = 1 nn tn ti cc s x; y; z > 0 sao cho a =


ng thc tr thnh
X
cyc

xy
(x + y)2

1
4xyz
+
4 (x + y)(y + z)(z + x)

x
y;b

= yz ; c =

z
x.

Bt

238

CHNG 2. SNG TO BT NG THC

Ta c
4xyz
(x + y)(y + z)(z + x)
2[(x + y)(y + z)(z + x) xy(x + y) yz(y + z) zx(z + x)]
=
(x + y)(y + z)(z + x)
P 2
(x + yz)(y + z)
cyc
= 2
(x + y)(y + z)(z + x)
!
X x
X y
x
z
= 2
+
x+y x+z
x+y x+z
cyc
cyc
!
!
#
"
X y
X x
X x
y
= 2
x+y
x+z
x+y x+y
cyc
cyc
cyc
!
!
X x
X y
X
xy
= 2
+
x+y
x+z
(x + y)2
cyc
cyc
cyc
Bt ng thc tng ng vi
X
cyc

x
x+y

X
cyc

y
x+z

9
4

S dng bt ng thc AM-GM, ta c


X
cyc

x
x+y

X
cyc

y
x+z

1
4

X
cyc

X y
x
+
x+y
x+y
cyc

!2

9
:
4

Bi ton 2.33 Cho cc s khng m a; b; c tha mn a2 + b2 + c2 = 1: Tm gi tr


ln nht ca biu thc
P = (a

b)(b

c)(c

a)(a + b + c):
(V Quc B Cn)

Li gii. Nu a
P = (c
b(c

b
b)(b

c
a)(c

0; th P

0: Nu c

0; th

b)(b a)(c2 + bc a2
1
1
bc)(b2 + bc)
(b2 + c2 )2 =
4
4

a)(a + b + c) = (c

b)(c2 + bc) = (c2

ab)

239
Mt khc, cho a = 0; b = sin 8 ; c = cos 8 ; ta c P = 14 : Vy nn
max P =

1
:
4

Bi ton 2.34 Cho cc s dng a; b; c; d: Chng minh rng


b(a + c) c(b + d) d(c + a) a(d + b)
+
+
+
c(a + b) d(b + c) a(c + d) b(d + a)

4:
(V Quc B Cn)

Li gii. Vit li bt ng thc nh sau


b
d
+
c(a + b) a(c + d)

(a + c)

, (abc + abd + acd + bcd)


1 1 1 1
+ + +
a b
c d

c
a
+
d(b + c) b(d + a)

+ (b + d)

a+c
b+d
+
ac(a + b)(c + d) bd(b + c)(d + a)
!
1
1
1
1
a + c
b + d
1
1 + 1
1
1
1
1
1
a + b
c + d
b + c
d + a

4
4

S dng bt ng thc AM-GM, ta c

1
a

1
a
1
b

1
c
1
c

1
d

1
b

1
b
1
c

1
d
1
d

1
a

4
+

1
a

1
a

1
a

1
b

1
c

1
b

+ 1c + d1
4
:
+ 1c + d1

1
b

4
1
a

1
b

1
c

1
d

1 2
d

Bt ng thc c chng minh. ng thc xy ra khi v ch khi a = c v b = d:


Bi ton 2.35 Cho cc s dng a; b; c: Chng minh rng
a2 + bc
b2 + ca
c2 + ab
+
+
a2 + (b + c)2
b2 + (c + a)2
c2 + (a + b)2

18 a2 + b2 + c2
:
5 (a + b + c)2
(Phm Hu c)

Li gii. Bt ng thc cho tng ng vi


X (b + c)2 bc
18 a2 + b2 + c2
+
2
2
a + (b + c)
5 (a + b + c)2
cyc

240

CHNG 2. SNG TO BT NG THC

Do (b + c)2

4bc, nn ta ch cn chng minh c


X
cyc

(b + c)2
6 a2 + b2 + c2
+
4[a2 + (b + c)2 ] 5 (a + b + c)2

S dng bt ng thc Cauchy Schwarz, ta c


X
cyc

(b + c)2
+ (b + c)2 ]

4[a2

(a + b + c)2
(a + b + c)2
P 2
=
2
2
2
[a + (b + c) ]
2(a + b + c2 ) + (a + b + c)2

cyc

Chun ha cho a + b + c = 1. t x = a2 + b2 + c2 ) 3x
1
6x
+
2x + 1
5

, x(3x

0:

1)

1, ta phi chng minh

ng thc xy ra khi v ch khi a = b = c:


Bi ton 2.36 Cho cc s dng a; b; c; d: Chng minh rng
(a + b)(a + c)(a + d)(b + c)(b + d)(c + d)

4(abc + bcd + cda + dab)2 :


(V Quc B Cn)

Li gii. S dng bt ng thc Cauchy Schwarz, ta c


(abc + bcd + cda + dab)2

(ac + bc + ad + bd)(ab2 c + bcd2 + ac2 d + a2 bd)


= (a + b)(c + d)(ab2 c + bcd2 + ac2 d + a2 bd)

(abc + bcd + cda + dab)2

(bc + bd + ac + ad)(a2 bc + bc2 d + acd2 + ab2 d)


= (a + b)(c + d)(a2 bc + bc2 d + acd2 + ab2 d)

Cng tng ng v vi v 2 bt ng thc trn, ta c


2(abc + bcd + cda + dab)2
(a + b)(c + d)(ab2 c + bcd2 + ac2 d + a2 bd + a2 bc + bc2 d + acd2 + ab2 d)
= (a + b)2 (c + d)2 (ab + cd)
Tng t, ta cng c
2(abc + bcd + cda + dab)2

(a + c)2 (b + d)2 (ac + bd)

2(abc + bcd + cda + dab)2

(a + d)2 (b + c)2 (ad + bc)

241
Nhn tng ng v vi v, ta c
8(abc + bcd + cda + dab)6

(ab + cd)(ac + bd)(ad + bc)

(a + b)2

sym

Mt khc, theo bt ng thc AM-GM th


4(ab + cd)(ac + bd)

(ab + cd + ac + bd)2 = (a + d)2 (b + c)2

Tng t,
4(ab + cd)(ad + bc)

(a + c)2 (b + d)2

4(ac + bd)(ad + bc)

(a + b)2 (c + d)2

Do
64(ab + cd)2 (ac + bd)2 (ad + bc)2

(a + b)2

sym

, (ab + cd)(ac + bd)(ad + bc)


Kt hp vi bt ng thc 8(abc+bcd+cda+dab)6

1 Y
(a + b)
8 sym
(ab+cd)(ac+bd)(ad+bc)

(a+

sym

b)2 , ta suy ra c
8(abc + bcd + cda + dab)6

"
#"
#
Y
1 Y
1 Y
(a + b)
(a + b)2 =
(a + b)3 :
8 sym
8
sym
sym

T y, ta suy ra pcm. ng thc xy ra khi v ch khi a = b = c = d:


Bi ton 2.37 Cho cc s dng a; b; c: Chng minh rng
s
a b
c
1 1 1
3+ + +
2 (a + b + c)
+ +
:
b
c a
a b
c
(Phm Hu c)
Li gii. t x3 = a; y 3 = b; z 3 = c. S dng bt ng thc Schur bc 3, ta c
3+

c
x y z
x3
y3
z3
a b
+ +
= 3
+ 3 + 3 + 3
b
c a
y z x y
z
x
=

X x2
cyc

yz

X xz
cyc

y2

X x2
cyc

y2

y
z
+
z
x

242

CHNG 2. SNG TO BT NG THC

Mt khc, theo bt ng thc AM-GM th


X x2
cyc

yz

X xz
cyc

y2

x3 + y 3 + z 3
(xz)3 + (yx)3 + (zy)3
+
xyz
(xyz)2
s
(x3 + y 3 + z 3 )(x3 z 3 + y 3 x3 + z 3 y 3 )
2
x3 y 3 z 3
s
1
1
1
+ 3+ 3
= 2 (x3 + y 3 + z 3 )
x3
y
z
s
1 1 1
= 2 (a + b + c)
+ +
:
a b
c

ng thc xy ra khi v ch khi a = b = c:


Bi ton 2.38 Cho cc s dng a; b; c. Chng minh rng
abc + 2(a2 + b2 + c2 ) + 8

5(a + b + c):
(Trn Nam Dng)

Li gii. S dng bt ng thc AM-GM v bt ng thc Schur bc 3, ta c


6abc + 12(a2 + b2 + c2 ) + 48

30(a + b + c)
2

= 3(2abc + 1) + 12(a + b2 + c2 ) + 45 5 2 (a + b + c) 3
p
3
9 a2 b2 c2 + 12(a2 + b2 + c2 ) + 45 5[(a + b + c)2 + 9]
9abc
= p
+ 7(a2 + b2 + c2 ) 10(ab + bc + ca)
3
abc
27abc
+ 7(a2 + b2 + c2 ) 10(ab + bc + ca)
a+b+c
3[4(ab + bc + ca) (a + b + c)2 ] + 7(a2 + b2 + c2 ) 10(ab + bc + ca)
= 4(a2 + b2 + c2

ab

bc

ca)

0:

ng thc xy ra khi v ch khi a = b = c = 1:


Bi ton 2.39 Cho cc s khng m a; b; c tha mn ab + bc + ca + 6abc = 9: Tm
hng s k nh nht sao cho bt ng thc sau ng
a + b + c + kabc

k + 3:
(V Quc B Cn)

243
Li gii. Cho a = b = 3; c = 0, ta c k 3. Ta s chng minh y l gi tr m ta
cn tm, tc l
a + b + c + 3abc 6
t p = a + b + c; q = ab + bc + ca; r = abc th ta c q + 6r = 9. S dng bt ng
thc AM-GM, ta c p2 3q 9. Bt ng thc tr thnh
p + 3r
, 2p
Nu p
Nu 6

6
q

6, bt ng thc l hin nhin.


p 3 v nu p2 4q, th
2p

Nu6 p
Do

3 v nu p2

(p
p2
=
4

2p

2)(6
4

p)

+3

4q, theo bt ng thc Schur bc 3, ta c r


27 = 3q + 18r
) 2p

2p

0.

p2 )

3q + 2p(4q

p(4q p2 )
9

2p3 + 27
8p + 3

Ta cn chng minh
2p

2p3 + 27
8p + 3

, (p + 1)(p

3)(p

3
6)

0:

hin nhin ng. Vy ta i n kt lun


kmin = 3:

Bi ton 2.40 Cho cc s dng a; b; c tha mn a + b + c = 3: Chng minh rng


3(a4 + b4 + c4 ) + a2 + b2 + c2 + 6

6(a3 + b3 + c3 ):
(Vasile Cirtoaje)

Li gii. Bt ng thc tng ng vi


X
3a4 6a3 + a2 + 4a

cyc

X
cyc

(a

1)2 (3a2

2)

244

CHNG 2. SNG TO BT NG THC


X

,
,

c)2 (3a2

(2a

2)

b)(a

c)(4a2 + b2 + c2

cyc

(a

4)

cyc

Khng mt tnh tng qut, gi s a


4a2 + b2 + c2

0; khi ta d dng kim tra c

4b2 + c2 + a2

4c2 + a2 + b2

Mt khc
4c2 + a2 + b2

4c2 +

(a + b)2
2

4=

1)2

(3c

Do
4a2 + b2 + c2

4b2 + c2 + a2

4c2 + a2 + b2

T y, vit li bt ng thc nh sau


(a

c)(4a2 + b2 + c2

b)[(a

4)
+ (c

(b

c)(4b2 + a2 + c2
2

a)(c

4)]

b)(4c + a + b

4)

0:

Ta thu c kt qu cn chng. ng thc xy ra khi v ch khi a = b = c = 1 hoc


a = b = 34 ; c = 13 hoc cc hon v tng ng.
Bi ton 2.41 Cho a; b; c l di 3 cnh ca mt tam gic. Chng minh rng
c+a
a+b
b+c
+
+
a2 + bc b2 + ca c2 + ab

3(a + b + c)
:
ab + bc + ca
(V Quc B Cn)

Li gii. Vit li bt ng thc nh sau


X
cyc

,
,

b+c
a2 + bc

1
a

X (a

(a

cyc

b)(a c)
a(a2 + bc)

b)(a

c)

cyc

X (a
cyc

X1
a
cyc

3(a + b + c)
ab + bc + ca

X bc(a b)(a c)
abc(ab + bc + ca)
cyc

1
a(a2 + bc)

1
a(ab + bc + ca)

b)(a c)(b + c
a2 + bc

a)

245
Gi s a

c; do a; b; c l di 3 cnh ca mt tam gic nn


a

Do a

b(a

b)
c

c nn
X (a

b)(a c)(b + c
a2 + bc

cyc

(b
(b

= (a
(a
=

(a

a)

c)(b

a)(c + a b) (c a)(c b)(a + b c)


+
b2 + ca
c2 + ab
c)(b a)(c + a b) b(a b)(b c)(a + b c)
+
b2 + ca
c(c2 + ab)
b(a + b c) c + a b
b)(b c)
c(c2 + ab)
b2 + ca
b(c + a b) c + a b
b)(b c)
c(c2 + ab)
b2 + ca
3
3
b)(b c)(c + a b)(b
c )
0:
c(c2 + ab)(b2 + ca)

ng thc xy ra khi v ch khi a = b = c hoc


ng.

a
2

b
1

c
1

hoc cc hon v tng

Bi ton 2.42 Chng minh rng vi mi a; b; c 2 R v a2 + b2 + c2 = 9; ta c


3 min fa; b; cg

abc + 1:
(Virgil Nicula)

Li gii. Gi s c

a; bt ng thc tr thnh
abc + 1

Nu a

3a

0; s dng bt ng thc AM-GM, ta c

abc + 1

3a

a(b2 + c2 )
+1
2

Nu a
0; v c
b
p
nhin. Nu 3 a

3a =

a2 )

a(9
2

+1

3a =

(a + 1)2 (2
2

a)

p
1
a; ta c 3
a
0: Nu a
3 ; bt ng thc l hin
1
;
ta
c
th
d
dng
kim
tra
c
3
p
p
bc a b2 + c2 a2 = a 9 2a2

Do , ta phi chng minh

a2

p
9

2a2 + 1

3a

246

Nu 1

CHNG 2. SNG TO BT NG THC

1
3;

, f (a) = 2a6

6a + 1

ta c

2f (a) = a2 (a 1) 4a(a
p
Nu 3 a 1; th
f (a)

9a4 + 9a2

2a6

1)2 + 12a2

9a4 + 9a2

18a + 7 + (1

5 = (a2

1)(2a4

3a)(7a2

7a2 + 2)

6a + 3)

3<0

Bt ng thc c chng minh. ng thc xy ra khi v ch khi a =


hoc cc hon v tng ng.

1; b = c = 2

Bi ton 2.43 Cho cc s khng m x; y; z tha mn x2 + y 2 + z 2 = 1. Chng minh


rng
r
r
r
p
x
y
z
+
+
2 2:
3
3
3
x + yz
y + zx
z + xy
(V Quc B Cn)
Li gii. t x2 = bc; y 2 = ca; z 2 = ab (a; b; c 0) th ab + bc + ca = 1 v bt ng
thc tr thnh
p
1
1
1
p
+p
+p
2 2
a + bc
b + ca
c + ab
Do tnh i xng, ta c th gi s a
Nu a

p1 :
3

0; khi ta c a

2; ta s chng minh rng


p

v
p

1
1
+p
b + ca
c + ab

p
trong t = (a + b)(a + c)
Tht vy, ta c
(a + t2 )

1
a + bc

1
a + t2
2

t(a + 1)

0 ) t2 + 2at = 1:

(a + bc) = a

a+b

a+c

S dng bt ng thc AM-GM, ta c


p

1
1
+p
b + ca
c + ab

Suy ra ta ch cn chng minh


t2 (a + 1)2

2
p
4
(b + ac)(c + ab)

(b + ac)(c + ab)

247
Ta c
t2 (a + 1)2

(b + ac)(c + ab) = a(a + 1)2


a(b

c)2

=
Mt khc
p

a+

1
a+
a

!2

a+b+

a+c
a(b c)2
p
p
2
(a + 1)2
a+b+ a+c
p
p
2
a+b+ a+c
a+b

a+c

bc
a

v
p

(a + 1)

a+

1
a+
a

!2

Tip theo, ta phi chng minh

Do a

p
1
2 a2 + 1
a
p
p
a2 + 1
a2 + 1 2
=

= a2 + 1

p
p
22 + 1
22 + 1

p
1
2
p
+p
2 2
a + t2
t(a + 1)
r
2
t
,p
+
2
3
2
2t
t2 + 1
1 + 2t t
!2
r
t
2
4
, p
+
2t3 t2 + 1
1 + 2t t2
p
p
, 4 (1 + 2t t2 )(2t3 t2 + 1)
t(7 6t 7t2 + 20t3

2)1

t2 )(2t3

t2 + 1) = 1 + t4 (5

7 6t 7t2 + 20t3 8t4 = 7


p
p
) 4 (1 + 2t t2 )(2t3 t2 + 1)
t(7
a

p1 ;
3

p
4 5
>0
2

8t4 )

4t, do

(1 + 2t

Nu 2

1
a
1
9
=
2

2t) + 2t(1

6t

t2 (7

6t

7t2 + 20t3

20t)

t)

8t4

8t4 )

s dng bt ng thc AM-GM, ta c

1
1
1
+p
+p
a + bc
b + ca
c + ab

3
p
6
(a + bc)(b + ca)(c + ab)

p
7 t

248

CHNG 2. SNG TO BT NG THC

Do , ta ch cn chng minh c
729
512

(a + bc)(b + ca)(c + ab)


t p = a + b + c; r = abc
p 2
p
p
Mt khc, ly u =
3
2u + v = p;

0; khi t a 2; ab + bc + ca = 1; ta suy ra 52
p
3
p+2 p2 3
v v =
; ta d dng kim tra c
3
p
5 + 2 13
6

u + 2uv = 1;

1
p
3

13

u2 v

3:

Ta c
(a + bc)(b + ca)(c + ab) = r2 + (p2

2p

1)r + 1 = f (r)

D thy f (r) l hm li nn
f (r)

max f (0); f (u2 v) = max 1; f (u2 v)

Ta cn phi chng minh


f (u2 v)

729
512

, v 2 (u + 1)2 (v + u2 )
, g(u) =

u2 )2 (2u3
u

(1 + 2u

729
512
u2 + 1)

729
64

Ta c
g 0 (u) =

(1

) g(u)

3u2 )(1 + 2u

p
6

13

u2 )(4u3
u2
=

14141

7u2 + 2u

1)

p
559 13
729
<
:
1458
512

Bt ng thc c chng minh. ng thc xy ra khi v ch khi x = y =

p1 ; z
2

=0

hoc cc hon v tng ng.


Bi ton 2.44 Cho cc s khng m x; y; z tha mn xy + yz + zx = 1: Chng minh
rng
p
1
1
1
2 6
p
+p
+p
:
3
2x2 + 3yz
2y 2 + 3zx
2z 2 + 3xy

249
Li gii. t a = yz; b = zx; c = xy (a; b; c 0), khi t xy + yz + zx = 1; ta c
a + b + c = 1. Bt ng thc tr thnh
p
Xr
2 2
a
r P
3a2 + 2bc
3 a
cyc
cyc

Ta s chng minh

X
cyc

X
cyc

a
3a2 + 2bc

4
P

cyc

ab
(3a2 + 2bc)(3b2 + 2ca)

4
P

cyc

Tht vy, ta c
X
cyc

X
cyc

a
3a2 + 2bc

4
P

18
a

(b

cyc

cyc

(3a2

4
9

cyc

18
=

(b

c)2 + 15

cyc

cyc

cyc

cyc

!2

!3

!2

Q
Q
c)2 + 21 a(b + c) + 7 a2
cyc
cyc
!
P
Q
3
a
(3a2 + 2bc)
cyc

ab
+ 2bc)(3b2 + 2ca)

!2

a + 36

cyc

a(b + c) + 22

cyc

(3a2

a2

cyc

+ 2bc)

cyc

Mt khc, vi mi m; n; p 0; ta c
p
m+n+p =
m2 + n2 + p2 + 2(mn + np + pm)
q
p
=
m2 + n2 + p2 + 2 m2 n2 + n2 p2 + p2 m2 + 2mnp(m + n + p)
q
p
m2 + n2 + p2 + 2 m2 n2 + n2 p2 + p2 m2
q
q
q
a
b
c
S dng bt ng thc ny vi m = 3a2 +2bc
; n = 3b2 +2ca
; p = 3c2 +2ab
; ta c
Xr
cyc

a
2
3a + 2bc

v
uX
u
t
cyc

s
X
a
ab
+
2
2
2
3a + 2bc
(3a + 2bc)(3b2 + 2ca)
cyc

250

CHNG 2. SNG TO BT NG THC

T 2 bt ng thc ch ra trn, ta c
v
s
uX
Xr
X
u
a
a
ab
t
+
2
2
2
2
3a + 2bc
3a + 2bc
(3a + 2bc)(3b2 + 2ca)
cyc
cyc
cyc
v
v
p
u
u
u 4
2 2
4
u P + 2u
!2 = r P :
u
u3 a
u
u
3 a
P
t
t cyc
9
a
cyc
cyc

Bt ng thc c chng minh xong.


Bi ton 2.45 Cho cc s khng m a; b; c tha mn a + b + c = 3: Chng minh rng
q
q
q
q
p
p
p
p
2
2
2
2
2
2
a+ b +c + b+ c +a + c+ a +b
3
2 + 1:

(Phan Hng Sn)

Li gii. Sau khi bnh phng 2 v v thu gn, ta c th vit li bt ng thc nh


sau
r
p
p
Xp
X
p
a + b2 + c2 b + a2 + c2
a2 + b2 + 2
9 2+6
cyc

cyc

S dng bt ng thc Cauchy Schwarz, ta c


X
cyc

p
a + b2 + c2

1 X
= p
2 cyc
1 X
p
2 cyc

p
b + a2 + c2

X
cyc

1 a+3
1

ab + 3 =

b+c
a+ p
2

a+c
b+ p
2

1 b+3

2 1 Xp
9
p
ab + p
2 cyc
2

Do , ta cn chng minh
Xp
a2 + b2 + 2
cyc

Xp

ab

cyc

chng minh bt ng thc ny, ta ch cn chng minh bt ng thc sau vi mi


x; y 0
p
p
x4 + y 4 + 2
2 xy x2 + y 2

251
Tht vy, ta c
p

2(x4 + y 4 ) + 2

= (x

y)2

(x

1 xy

y)
p

2(x2 + y 2 )
(x + y)2

2(x4 + y 4 ) + x2 + y 2

(x + y)2

2(x2 + y 2 )2 + x2 + y 2
y)2

1 xy(x
x2 + y 2

2+1

2+1

0:

ng thc xy ra khi v ch khi a = b = c = 1:


Bi ton 2.46 Cho cc s dng a; b; c: Chng minh rng
bc2
ca2
ab2
+
+
+a+b+c
c2
a2
b2

6(a2 + b2 + c2 )
:
a+b+c

Li gii. S dng bt ng thc Cauchy Schwarz, ta c


"
#
"
#"
#
#"
X ab2 X
X a(b2 + c2 ) X
X
1
1
=
+
a
c2
a(b2 + c2 )
c2
a(b2 + c2 )
cyc
cyc
cyc
cyc
cyc
!2
X1
a
cyc
Suy ra, ta ch cn chng minh c
X

cyc

Nu

1
c

cyc

,
2(a+b)
ab ;

X (a
cyc

X1
a
cyc
X1
a
cyc
2

b)
ab

!2

!2

X
cyc

X1
9
a
cyc

a2

!"

X
cyc

X
cyc

1
a(b2 + c2 )

2a
b2 + c2

1 1 1
3(a + b)(c2 ab)
+ + + 2
a b
c (a + c2 )(b2 + c2 )

X1
a
cyc

th

1 1 1
3(a + b)(c2 ab)
+ + + 2
a b
c (a + c2 )(b2 + c2 )

1 1 1
+ +
a b
c

3(a + b)
ab

252

CHNG 2. SNG TO BT NG THC

Nu

2(a+b)
ab

1
c

,c

ab
2(a+b) ;

abc(a2 + c2 )(b2 + c2 )

th

1 1 1
3(a + b)(c2 ab)
+ + + 2
a b
c (a + c2 )(b2 + c2 )

= c4 (ab + bc + ca) + abc2 (a2 + b2 ) + (a + b)(a2 + b2 + 3ab)c3


+ a3 b3

2a2 b2 c(a + b)

abc2 (a2 + b2 ) + (a + b)3 c3 + a3 b3

2a2 b2 c(a + b)

a3 b3 (a2 + b2 )
+ (a + b)3 c3 + a3 b3 2a2 b2 c(a + b)
4(a + b)2
9
9
(a + b)3 c3 + a3 b3 + a3 b3 2a2 b2 c(a + b)
16
16
r
81 2 2
3
3
a b c(a + b) 2a2 b2 c(a + b) 0:
256
ng thc xy ra khi v ch khi a = b = c:
Bi ton 2.47 Cho cc s thc a; b; c khc 0 tha mn a2 + b2 + c2 = (a
c)2 + (c a)2 : Chng minh rng
P (a; b; c) =
1
12

Q(a; b; c) =

c
a b
+ +
b
c a

b)2 + (b

a2 b + b2 c + c2 a
(a + b + c)3

5
:
36
(V Quc B Cn)

Li gii. Nu tn tai 2 trong 3 s a; b; c, chng hn a; b < 0, ta c th thay a; b; c bi


a; b; c; khi ta c th thy iu kin bi ton v 2 biu thc P Q vn khng
thay i, do ta ch cn xt bi ton trong trng hp trong 3 s a; b; c, tn ti t
nht 2 s dng, chng hn a; b > 0. Nu c < 0 th
b)2 + (b

(a

c)2 + (c

a)2

(a2 + b2 + c2 ) = (a

b)2 + c2

2c(a + b) > 0

Do , ta phi c c > 0. T y, vi gi thit a = maxfa; b; cg, ta c


0 = (a
=

b)2 + (b c)2 + (c a)2 (a2 + b2 + c2 )


p
p
p
p
p
p
p
p
a+ b+ c
a+ b
c
b+ c
)

a=

b+

c+

253
Do , bt ng thc tr thnh
p

b+

c
+

b
p

1
12

t t =

b
c

b+

c
p

b
+ p
c

c
b+

b + b2 c + c2

b+

c
p

b+

5
36

+b+c

> 0; ta phi chng minh


(t + 1)2
1
+ t2 +
2
t
(t + 1)2
2
3

(t + 1)4 t2 + t4 + (t + 1)2
(t2 + t + 1)3

10
9

Nhng chng hin nhin ng v


(t + 1)2
1
+ t2 +
t2
(t + 1)2
2
3

5 , (t3 + t2

(t + 1)4 t2 + t4 + (t + 1)2
, (t3 + 3t2
(t2 + t + 1)3

(t + 1)4 t2 + t4 + (t + 1)2
(t2 + t + 1)3

10
, (t3
9

3t2

1)2

2t

1)2
6t

0
1)2

Bt ng thc c chng minh xong.


Bi ton 2.48 Cho cc s dng a; b; c; n: Chng minh rng
a2 + bc
b+c

b2 + ca
c+a

c2 + ab
a+b

an + bn + cn :
(V Quc B Cn)

Li gii. Do tnh i xng, ta c th gi s a


a2 + bc
a(b + c)

c > 0, khi ta c

v
(a2 + bc)(b2 + ca)

ab(a + c)(b + c) = c(a

b)2 (a + b)

254

CHNG 2. SNG TO BT NG THC


n

a2 + bc
a(b + c)

a2 + bc
a(b + c)

b2 + ca
b(c + a)

b2 + ca
b(c + a)

(a2 + bc)(b2 + ca)


ab(a + c)(b + c)

c2 + ab
c(a + b)

n=2

Do
x
vi x =

a +bc
a(b+c)

0;

1; y =

x+y

0;

x+y+z

b +ca
b(c+a)

c +ab
c(a+b)

1; z =

1.

Bt ng thc cho tng ng vi


an x + bn y + cn z

Bt ng thc ny hin nhin ng v


an x + bn y + cn z = (an

bn )x + (bn

cn )(x + y) + cn (x + y + z)

0:

ng thc xy ra khi v ch khi a = b = c hoc n ! 0:


Bi ton 2.49 Tm gi tr ln nht ca biu thc
P =

a
b
c
d
+ + +
c
d a
b

abcd
(ab + cd)2
a
b

trong a; b; c; d l cc s thc phn bit tha mn


(Phm Vn Thun)
Li gii. t x =
do

a
b

= dc ; y =

b
c

t)2 =

(4

a
d

b
c

v t = y+ y1 , suy ra jtj

x+
)t

)t

1
x

= x2 +
2_t

c
d

d
a

= 4 v ac = bd:

2; th ta c x+ x1 +t = 4;

1
+2
x2

2_t=2_t

Ta c
P =

a
b
c
d
+ + +
c
d a
b

= t(4

t)

a2
d2

abcd
=
x+
(ab + cd)2
1
= t(4 t)
2
+ 1 cb2 + 1

Nu y > 0; th t 2 v do t
phi c t 6; suy ra
P =

(t

1
x

y+

1
y

b2 d 2
(a2 + d2 )(b2 + c2 )

1
t2

6 hoc t = 2; nhng do a; b; c; d phn bit nn ta

6)(36t3 + 72t2
36t2

6)

433
36

433
36

255
Nu y < 0; th t
P =

2; suy ra
49
49
433
<
4
4
36
p
p
2 2; b = 3 + 2 2; c = 1; d = 1; ta c

(t + 2)[ (t + 2)(9 2t)(7


4t2

433
Do P
36 : Mt khc, cho a =
433
P = 36 : Vy ta i n kt lun

2t) + 124]

433
:
36

max P =

Bi ton 2.50 Cho cc s khng m a; b; c; khng c 2 s no ng th bng 0:


Chng minh rng
p
p
p
a2 + bc + b2 + ca + c2 + ab :
3(a + b + c) 2
(V Quc B Cn)

Li gii 1. Bnh phng 2 v, bt ng thc tng ng vi


X
Xp
X
a2 + 14
ab 8
5
(a2 + bc)(b2 + ca)
cyc

cyc

cyc

S dng bt ng thc Cauchy Schwarz, ta c


s X
Xp
(a2 + bc)(b2 + ca)
3
(a2 + bc)(b2 + ca)
cyc

cyc

Do , ta ch cn chng minh c
5

a + 14

cyc

!2

ab

cyc

24

(a2 + bc)(b2 + ca)

cyc

Do tnh thun nht, ta c th chun ha cho a + b + c = 1. t q =

ab; r = abc,

cyc

khi theo bt ng thc Schur bc 3, ta c r


thnh
384r + (25 52q)(1
Nu 1

4q, bt ng thc l hin nhin. Nu 4q

384r +(25 52q)(1 4q)

384

4q

1
9

max 0; 4q9
4q)

. Bt ng thc tr

1 th

+(25 52q)(1 4q) =

1
(4q 1)(156q +53)
3

0:

256

CHNG 2. SNG TO BT NG THC

ng thc xy ra khi v ch khi a = b; c = 0 hoc cc hon v tng ng.


Li gii 2. Khng mt tnh tng qut, ta c th gi s a b c, khi theo bt
ng thc AM-GM, ta c
p
2 a2 + bc

p
2 b2 + ca

a+c+

a2 + bc
c(b a)
= 2a + c +
a+c
a+c

b+c+

b2 + ca
c(a b)
= 2b + c +
b+c
a+c

p
p
) 2 a2 + bc + 2 b2 + ca
p
2 ab + c2

2a + 2b + 2c +

b+c+

c(a b)2
(a + c)(b + c)

ab + c2
b+c

Nn ta ch cn chng minh c
3(a + b + c)

2a + 3b + 3c +
ab + c2
b+c

,a
,

c(a b)2
(a + c)(b + c)
c(a b)2
(a + c)(b + c)

c(a c)
b+c

, (a

ab + c2
c(a b)2
+
b+c
(a + c)(b + c)

(a

b)2

0:

a+

c
2

c)(a + c)

Bt ng thc ny hin nhin ng v


a+c
Li gii 3. Gi s a

c. V
p
a2 + bc

p
p
b2 + ca + c2 + ab

p
2(b2 + c2 + ab + ac)

Nn ta ch cn chng minh c
p
2 2(b2 + c2 + ab + ac)
, a2

2(b + 2c)a + b2

a + 3b + 2c
4c2 + 12bc

257
, (a
hin nhin ng.
Li gii 4. Gi s a

2c)2 + 8c(b

Nn ta ch cn chng minh c

V (a

b+c
2

2s)2

p
2(b2 + c2 + ab + ac)

p
p
a2 + bc + 2(b2 + c2 + ab + ac)

(s

0:

c. Ta c

p
p
b2 + ca + c2 + ab

t s =

c)

3
(a + b + c)
2

a) v p = bc, bt ng thc tr thnh

p
p
2 a2 + p + 4 2s2 p + as 3(a + 2s)
p
p
, 4 2s2 p + as 3(a + 2s) 2 a2 + p
h
i2
p
, 16(2s2 p + as)
3(a + 2s) 2 a2 + p
p
, 12(a + 2s) a2 + p 13a2 + 20as + 4s2 + 20p
p
, 12(a + 2s)
a2 + p a
(a 2s)2 + 20p

0 v

p
a2 + p

INn ta ch cn chng minh

p
a= p
2
a +p+a

p
2a

6(a + 2s)p
20p
a
, p(6s 7a) 0:
hin nhin ng.
Bi ton 2.51 Cho x; y; z 2 [ 1; 1] tha mn x + y + z = 0. Chng minh rng
p
p
p
1 + x + y 2 + 1 + y + z 2 + 1 + z + x2 3:

(Phan Thnh Nam)

Li gii. Bnh phng 2 v, ta c th vit li bt ng thc nh sau


X
Xp
x2 + 2
(1 + x + y 2 )(1 + y + z 2 ) 6
cyc

cyc

258

CHNG 2. SNG TO BT NG THC

S dng bt ng thc Cauchy Schwarz, ta c


Xp
X
Xp
(1 + x + y 2 )(1 + y + z 2 )
(1 + x)(1 + y) +
jyzj
cyc

cyc

Xp

cyc

(1 + x)(1 + y)

cyc

yz

cyc

Ta cn chng minh
X

x2

cyc

xy + 2

cyc

Xp
(1 + x)(1 + y)

cyc

t a2 = 1 + x; b2 = 1 + y; c2 = 1 + z (a; b; c 0) ) a2 + b2 + c2 = 3. Bt ng thc
tr thnh
X
X
X
X
a4 2
a2 b2 + 2
a2 + 2
ab 9 0
cyc

cyc

,
,3

cyc

2 2

2 2

a b +
a2 bc +

a2 (a

b)(a

cyc

cyc

ab

ab

a2

cyc

cyc

a4 +

X
cyc

cyc

cyc

a b +2

cyc

cyc

cyc

cyc

ab(a2 + b2 )

cyc

cyc

a2 b2

b)2

0:

0
0

cyc

c) + 2

cyc

ab(a

cyc

Bt ng thc cui hin nhin ng theo bt ng thc Schur bc 4. ng thc xy


ra khi v ch khi x = y = z = 0:
Bi ton 2.52 Cho cc s dng a; b; c tha mn a + b + c = 3: Chng minh rng
a2

bc + (a

b2
+p
1)2
ca + (b

1)2

c2

+p

ab + (c

1)2

(Phan Hng Sn)

Li gii. S dng bt ng thc Cauchy Schwarz, ta c

X
cyc

a2
bc + (a

1)2

cyc

cyc

a2

3:

!2

bc + (a

1)2

259
v

X
cyc

a2

1)2 =

bc + (a

Xp

p
abc + a(a

a3

cyc

v
!"
u
X
u X
3
t
a
[abc + a(a
cyc

cyc

cyc

q2

ab; r = abc th ta c 9

1)2 ]

cyc

Do , ta ch cn chng minh c
!"
!4
X
X
X
2
3
9
a
[abc + a(a
a
t q =

1)2

1) ]

cyc

9r. Bt ng thc tr thnh

cyc

(9

Ta c
(9

2q)4

2q)4
9(27

9(27

9q + 3r)(12

9q + 3r)(12
4

= (9

2q)

(9

2q)

(81
(81

5q + 6r)
27q + 9r)(36
2

27q + q )(36

q)2 [14q 2 + 135(3

= (3

5q + 6r)

q)]

15q + 18r)
15q + 2q 2 )

0:

ng thc xy ra khi v ch khi a = b = c = 1:


Bi ton 2.53 Cho cc s khng m a; b; c tha mn a2 + b2 + c2 = 1: Chng minh
rng
p
a
b
c
1
+
+
2:
1 + bc 1 + ca 1 + ab
(Faruk Zejnulahi)
Li gii. Vi mi x 2 0; 21 , ta c
1

2
x
3

1
x(1 2x)
=
1+x
3(1 + x)
2
x
3

1
1
1+x
T y, ch rng maxfab; bc; cag 21 , ta c
)

X
cyc

a
1 + bc

X
cyc

a 1

2
bc
3

X
cyc

2abc = a(1

p
[a2 + (b + c)2 ][(1 2bc)2 + 1]
p
=
2(2b2 c2 2bc + 1)(1 + 2bc)
p
p
=
2[1 2b2 c2 (1 2bc)]
2

2bc) + b + c

260

CHNG 2. SNG TO BT NG THC

Mt khc, ta c
X
cyc

a
1 + bc

!2

cyc

3abc

cyc

cyc

cyc

=
v

!2

cyc

a + 3abc

cyc

v
X

1
6

!2

!2

X
cyc

cyc

cyc

X
cyc

1
3

1
6

cyc

cyc

ab

cyc

!2
!2
X
a 4
a
cyc

cyc

15

0:

1: Bt ng thc c chng minh xong.

cyc

Bi ton 2.54 Cho cc s dng a; b; c; d tha mn (a + c)(b + d)


minh rng
1
1
1
1
+
+
+
ab(1 + c) bc(1 + d) cd(1 + a) da(1 + b)

4abcd. Chng

32
:
(1 + a)(1 + b)(1 + c)(1 + d)
(Phm Kim Hng)

Li gii. t x = a1 ; y = 1b ; z = 1c ; t = d1 , th ta c x; y; z; t > 0 v (x + z)(y + t)


Bt ng thc tr thnh
xz

y
t
+
1+z
1+x

+ yt

x
z
+
1+y 1+t

32xyzt
(1 + x)(1 + y)(1 + z)(1 + t)

S dng bt ng thc AM-GM, ta c


xz

y
t
+
1+z
1+x

x
z
+
1+y 1+t
s
y
t
x
z
2 xyzt
+
+
1+z
1+x
1+y 1+t
s
xyzt(x + z + xt + yz)(y + t + xy + zt)
= 2
(1 + x)(1 + y)(1 + z)(1 + t)

+ yt

4.

261
Nn ta ch cn chng minh c
(x + z + xt + yz)(y + t + xy + zt)
Nu u4 = xyzt

256xyzt
(1 + x)(1 + y)(1 + z)(1 + t)

1 th ta c

(x + z + xt + yz)(y + t + xy + zt)
= (x + z)(y + t) + (x + z)(xy + zt) + (y + t)(xt + yz) + (xy + zt)(xt + yz)
p
p
4 xyzt + 2 (x + z)(y + t)(xy + zt)(xt + yz) + 4xyzt
p
4 xyzt + 8(xyzt)3=4 + 4xyzt = 4u2 + 8u3 + 4u4 = 4u2 (u + 1)2
(1 + x)(1 + y)(1 + z)(1 + t)

1 + (xyzt)1=4

= (1 + u)4

v
4u2 (1 + u)6

Nu u4 = xyzt

256u4

= 4u2 [(u + 1)6 64u2 ] = 4u2 [(u + 1)3 8u][(u + 1)3 + 8u]
= 4u2 (u 1)(u2 + 4u 1)[(u + 1)3 + 8u] 0

1 th ta c

(x + z + xt + yz)(y + t + xy + zt)
= (x + z)(y + t) + (x + z)(xy + zt) + (y + t)(xt + yz) + (xy + zt)(xt + yz)
p
4 + 2 (x + z)(y + t)(xy + zt)(xt + yz) + 4xyzt
p
4 + 8 xyzt + 4xyzt = 4(u2 + 1)2
(1 + x)(1 + y)(1 + z)(1 + t)
= 1 + (x + z) + (y + t) + (x + z)(y + t) + xz
+ yt + xz(y + t) + yt(x + z) + xyzt
p
p
p
5 + 2 (x + z)(y + t) + 2 xyzt + 2 xyzt(x + z)(y + t) + xyzt
p
p
2
9 + 6 xyzt + xyzt = 3 + xyzt = (3 + u2 )2
v
4(u2 + 1)2 (u2 + 3)2

256u4

= 4[(u2 + 1)2 (u2 + 3)2 64u4 ]


= 4[(u2 + 1)(u2 + 3) 8u2 ][(u2 + 1)(u2 + 3) + 8u2 ]
= 4(3 u2 )(1 u2 )[(u2 + 1)(u2 + 3) + 8u2 ] 0:

ng thc xy ra khi v ch khi a = b = c = d = 1:

262

CHNG 2. SNG TO BT NG THC

Bi ton 2.55 Cho cc s khng m a; b; c; khng c 2 s no ng thi bng 0:


Chng minh rng
r
r
r
r
a2 + bc
b2 + ca
c2 + ab
2(a2 + b2 + c2 )
1
+
+
+p :
2
2
2
2
2
2
b +c
c +a
a +b
ab + bc + ca
2
(V Quc B Cn)
Li gii. S dng bt ng thc AM-GM, ta c
vP
0v
12
P
u 2 P a2
u a2
2 a2
u
u
1 C
1
cyc
cyc
u cyc
Bu P
P
p
t
tP
+
+
2
+
=
@
A
ab
ab
ab 2
2
cyc

cyc

1
2+ p
2

cyc

a2

cyc

ab

cyc

Do , ta ch cn chng minh c
X
cyc

a2 + bc
b2 + c2

!2

X a2 + bc
cyc

b2

c2

1
2+ p
2

+2

X
cyc

(a2 + bc)(b2 + ca)


(a2 + c2 )(b2 + c2 )

2+

cyc

ab

1
2

cyc

Gi s a

c, ta s chng minh rng


r
r
a2 + c2
b2 + c2
+
2
2
b +c
c2 + a2
,
,

a2 + c2
b2 + c2
+ 2
+2
2
2
b +c
a + c2

a2 + c2
b2 + c2
+
b2 + c2
a2 + c2

(a2 b2 )2
(a2 + c2 )(b2 + c2 )

4c2
+ (a
c2 + ab

b)2

2(a2 + b2 )
c2 + ab
2(a2 + b2 )
c2 + ab

2(a2 + b2 )
c2 + ab
2(a

b)2 4c2
c2 + ab

(a + b)2
(a2 + c2 )(b2 + c2 )

c2

2
+ ab

4c2
(a b)2 (ab c2 )(a2 + b2 + 2c2 )
+
0
c2 + ab
(a2 + c2 )(b2 + c2 )(ab + c2 )
r
r
r
r
r
a2 + bc
b2 + ca
a2 + c2
b2 + c2
2(a2 + b2 )
)
+
+
b2 + c2
c2 + a2
b2 + c2
c2 + a2
c2 + ab
,

2+

1
2

263
Mt khc
s

(a2 + bc)(b2 + ca)


(a2 + c2 )(b2 + c2 )

Do
X
cyc

(a2 + bc)(b2 + ca)


=
(a2 + c2 )(b2 + c2 )
+

c2 + ab
a2 + b2

a2 + bc
+
b2 + c2

(a2 + bc)(b2 + ca)


(a2 + c2 )(b2 + c2 )

b2 + ca
c2 + a2

2+1

Ta cn phi chng minh


X a2 + bc
b2 + c2

cyc

X a + bc
b2 + c2
cyc
2

,
Nu

a2

cyc

b2

cyc

1
2+ p
2

a2

cyc

ab

cyc

5
2

1
2+ p
2

P
a2 2 ab
cyc
cyc
P
ab
cyc

ab, bt ng thc l hin nhin v

a2 + c2
b2 + c2
ab
+ 2
+ 2
2
2
b +c
a + c2
a + b2

5
2

c2

Nu

3
2+
2

cyc

X a2 + bc
cyc

a2

(2a2

ab + 2b2 )(a
2ab(a2 + b2 )

ab th ta c a

b+

b)2

5
2

a
b
ab
+ + 2
b
a a + b2

b + 3c v do 2 +

cyc

cn chng minh c

X a2 + bc
cyc

b2 + c2

5
2

5
2

P
a2 6 ab
cyc
cyc
P
ab
cyc

p1
2

< 3, nn ta ch

264
Ta c

CHNG 2. SNG TO BT NG THC


X a2 + bc
b2

cyc

b2 + c2
ab
a2 + c2
+ 2
+ 2
2
2
b +c
a + c2
a + b2

5
2

c2

5
2

(a2 b2 )2
(a b)2
(a2 + c2 )(b2 + c2 ) 2(a2 + b2 )
(a + b)2
1
= (a b)2
(a2 + c2 )(b2 + c2 ) 2(a2 + b2 )
3(a b)2 (a + b)2
4(a2 + c2 )(b2 + c2 )

P
a2 2 ab
(a
cyc
cyc
P
=
ab

b)2 + c(c 2a
P
ab

cyc

2b)

(a

cyc

Nn ta ch cn chng minh c

(a b)2 (a + b)2
4(a2 + c2 )(b2 + c2 )

(a

b)2 2ac
P
ab

cyc

b)2 2ac
P
ab

cyc

b)2

, (a
Nu a

(a + b)2
4(a2 + c2 )(b2 + c2 )

1
2ac
+
ab + bc + ca
ab + bc + ca

b + 5c; ta s chng minh


(a + b)2
4(a2 + c2 )(b2 + c2 )

1
ab + bc + ca

, f (c) = (a + b)2 (ab + bc + ca)

b)2

f (0) = ab(a
f

b
5

1
(a
625

b + 5c th
2ac

b)(21a3 + 1112a2 b
2
a(a
5

b)

4(a2 + c2 )(b2 + c2 )

Bt ng thc ny ng v f (c) l hm lm trn 0;

Nu a

2
(a
5

a b
5

0
362ab2 + 229b3 )

b)2

Suy ra, ta ch cn chng minh


4(a2

(a + b)2
+ c2 )(b2 + c2 )

3
5(ab + bc + ca)

265
, g(c) = 5(a + b)2 (ab + bc + ca)
Bt ng thc v g(c) l hm lm trn 0;
g(0) = ab(5a2
a

1
(5a4 + 313a3 b
27

12(a2 + c2 )(b2 + c2 )

a b
3

2ab + 5b2 )

150a2 b2 + 133ab3

85b4 )

0:

ng thc xy ra khi v ch khi a = b; c = 0 hoc cc hon v tng ng.


Bi ton 2.56 Cho cc s khng m a; b; c; khng c 2 s no ng thi bng 0:
Chng minh rng
a
b
c
+ 3
+ 3
b3 + c3
c + a3
a + b3

18
5(a2 + b2 + c2 ) ab

bc

ca

(Michael Rozenberg)
Li gii. Bt ng thc tng ng vi
X a(a + b + c)
cyc

X
cyc

b3 + c3

a
+
b3 + c3

X
cyc

P
18 a
cyc
P
P
5 a2
ab
cyc

a
bc + c2

b2

X
cyc

a2
b3 + c3

cyc

a2

!2

X
cyc

b2

a2 (b3 + c3 )

cyc

a
bc + c2

a2 b2

cyc

cyc

cyc

a(b2

P
18 a
cyc
P
P
5 a2
ab
cyc

S dng bt ng thc Cauchy Schwarz, ta c


P

cyc

!2

cyc

a2

cyc

cyc

bc + c2 )

cyc

!2

abc

ab

cyc

!2

cyc

cyc

ab

6abc

266

CHNG 2. SNG TO BT NG THC

Suy ra, ta ch cn chng minh c


!2
P 2
a
P

a2 b2

cyc

cyc

cyc

abc

ab

cyc

cyc

cyc

!2

ab

cyc

6abc

P
18 a
cyc
P
P
5 a2
ab
cyc

cyc

P
Do tnh thun nht, ta c th chun ha cho a + b + c = 1. t q =
ab; r = abc,
cyc
o
n
q)
. Bt ng thc tr
theo bt ng thc Schur bc 4, ta c r max 0; (4q 1)(1
6

thnh

(1

q2
Nu 1

2q)2
1
+
(q + 2)r q 6r

18
11q

4q th
(1
q2

Nu 4q
(1
q2

2q)2
1
+
(q + 2)r q 6r

18
5 11q

2q)2

(1

1
18
+
q2
q
5 11q
(1 4q)(11q 2 6q + 5)
=
q 2 (5 11q)

1 th
1
2q)2
+
(q + 2)r q 6r
q2
=

18
5 11q
(1 2q)2
(q + 2)

(4q 1)(1 q)
6
3

(6q

1
1)(1

q)

(1 3q)(4q 1)(112q
117q 2 + 39q 4)
2
3
(1 2q) (5 11q)(4q + 9q 2 9q + 2)

18
11q

0:

ng thc xy ra khi v ch khi a = b = c hoc a = b; c = 0 cc hon v tng ng.


Bi ton 2.57 Cho cc s dng a; b; c: Chng minh rng
r
r
r
r
a
b
c
3
ab + bc + ca
p
+
+
:
a+b
b+c
c+a
a2 + b2 + c2
2
(V Quc B Cn, Nguyn Vn Thch)
Li gii. Trc ht, ta s chng minh
P
3 a2
cyc
P
a
cyc

X 2a2
a+c
cyc

267
Tht vy, ta c
3

cyc

X 2a2
a+c
cyc

cyc

= 3

cyc

cyc

cyc

a+b

X c(a2 + b2 )

a2

X a2 + b2

cyc

cyc

a+b

cyc

ab(a b)2
(a + c)(b + c)

By gi, s dng bt ng thc bt ng thc ny v bt ng thc Holder, ta c


P
!2 "
#
3 a2
Xr a
X
cyc
P
a(a + b)(a + c)
a
a+b
cyc
cyc
cyc

X a2
2
a+c
cyc

Xr a
a+b
cyc

!2 "

a(a + b)(a + c)

cyc

cyc

!4

Ta cn chng minh
4

X
cyc

,4

X
cyc

!4

!4

27

!2 "

ab

cyc

27

!2 "

ab

cyc

a(a + b)(a + c)

cyc

X
cyc

cyc

Do tnh thun nht, ta chun ha cho a + b + c = 1. t q =

+ 3abc

ab; r = abc th ta c

cyc

q2

3r. Bt ng thc tr thnh


4

27q(1

2q + 3r)

Ta c
4

27q(1

2q + 3r)

27q(1

2q + q 2 ) = (4

3q)(1

3q)2

0:

ng thc xy ra khi v ch a = b = c:
Bi ton 2.58 Cho cc s dng a; b; c: Chng minh rng
X a2 + b2 + c2
a2 + 2b2 + 2c2
cyc

X b2 + bc + c2
:
a2 + (b + c)2
cyc
(Phm Hu c)

268

CHNG 2. SNG TO BT NG THC

Li gii. Bt ng thc tng ng vi


!"
X
X
X
1
a2 bc
2
a
2
2
2
a + (b + c)
a + (b + c)2
cyc
cyc
cyc
,

X
cyc

a2 bc
a2 + (b + c)2

cyc

!"

X
cyc

1
2
a + 2b2 + 2c2

(b c)2
[a2 + (b + c)2 ](a2 + 2b2 + 2c2 )

#
#

Do tnh thun nht, ta c th chun ha cho a2 + b2 + c2 = 1. Bt ng thc tr


thnh
X a2 bc X
(b c)2
1 + 2bc
(1 + 2bc)(1 + b2 + c2 )
cyc
cyc
Ch rng
X 2(a2

X 2a2 b2 c2 X (b c)2
bc)
=
+
1 + 2bc
1 + 2bc
1 + 2bc
cyc
cyc

cyc

(b2

c2 )

cyc

(b

c)2

cyc

1
1 + 2ab

1
1 + 2ca

X (b
cyc

c)2
1 + 2bc

1
2a(b + c)
+
(1 + 2ab)(1 + 2ac) 1 + 2bc

Do , bt ng thc tng ng vi
X

c)2

(b

cyc

(b

2a(b + c)
1
+
(1 + 2ab)(1 + 2ac) 1 + 2bc
c)2

cyc

2a(b + c)
(1 + 2ab)(1 + 2ac)

2
(1 + 2bc)(1 + b2 + c2 )

a2
(1 + 2bc)(1 + b2 + c2 )

Mt khc, ta c
2(1 + 2bc)(1 + b2 + c2 )
2

= [1 + (b + c)
2(b + c)(1

(1 + 2ab)(1 + 2ac)

2a(b + c)] + [(b + c)2


a) + 4bc(1

4a2 bc] + 4bc(b2 + c2 )

a2 ) + 4bc(b2 + c2 )

Nn ta ch cn chng minh c
X
cyc

(b

c)2

2a(b + c)
(1 + 2ab)(1 + 2ac)

2a2
(1 + 2ab)(1 + 2ac)

269
,

c)2

x(b

cyc

trong x = a(b + c a)(1 + 2bc) v y; z tng t.


Khng mt tnh tng qut, gi s a b c, khi ta c y; z
2

b(a

c)

c)2 (b + c

a(b

a(b

c) (b + c

1 + 2ac

c) (1 + 2bc)

0 v

1 + 2bc

c)2 (a + c

b)(1 + 2ac)

a)(1 + 2bc) + a(b

= 2ac(b

c) ;

a)(1 + 2bc) + b(a

a(b

c) (a + c

b)(1 + 2bc)

0:

ng thc xy ra khi v ch khi a = b = c:


Bi ton 2.59 Cho cc s dng x; y tha mn x + y 2
3x2 + 2y 3

x2 + y 3 : Chng minh rng

5:
(Ji Chen)

Li gii 1. T x + y 2

x2 + y 3 , ta c
5(x + y 2 )

(3x2 + 2y 3 ) + (2x2 + 3y 3 )

Ta s chng minh rng


5 + 2x2 + 3y 3
, (2x

3)(x

1) + [3y 2

Ch rng nu x; y 1 th ta c x + y
bi ton.
Nu y 1 x th ta c
(3

5(x + y 2 )

2x)(1

x)

2(y + 1)](y
2

1)

x + y , iu ny mu thun vi gi thit

x(1

y 2 (y

x)

1)

Do
(2x 3)(x 1) + [3y 2
Nu x

2(y + 1)](y

[4y 2

1)

2(y + 1)](y

1) = 2(y

1)2 (2y + 1)

y th
[2(y + 2)

3y 2 ](1

y 2 (1

y)

y)

x(x

1)

Do
(2x

3)(x

1) + [3y 2

2(y + 1)](y

1)

(3x

3)(x

1) = 3(x

1)2

0:

ng thc xy ra khi v ch khi x = y = 1:


Li gii 2. Ta c
5

3x2

2y 3 = 6(x

x2 + y 2

y 3 ) + 3(1

x)2 + 2(1

y)2 (1 + 2y)

0:

270

CHNG 2. SNG TO BT NG THC

Bi ton 2.60 Cho cc s khng m a; b; c tha mn a + b + c = 3. Chng minh rng


b
c
a
+
+
1 + a + b2
1 + b + c2
1 + c + a2

3
:
4
(V Quc B Cn)

Li gii. Xt 2 trng
sau
P 2hpP
Trng hp 1. Nu
a + ab2
cyc

X
cyc

9, s dng bt ng thc Cauchy Schwarz, ta c

cyc

a
1 + a + b2

!2

cyc

a(1 + a +

b2 )

3+

cyc

Trng hp 2. Nu

a2 +

9
a2

cyc

ab2

3
4

ab2

cyc

9, tc l

cyc

cyc

ab2

cyc

a2 b + 6abc

cyc

Bt ng thc tng ng vi
X
4
a(1 + b + c2 )(1 + c + a2 )

3(1 + a + b2 )(1 + b + c2 )(1 + c + a2 )

cyc

a2 b3 + 5

cyc

a3 b +

cyc

a3 + 9abc+

cyc

a2 b + 5

cyc

ab

cyc

a2 b2 + 3

cyc

Do

cyc

ab2

a2 + 3a2 b2 c2

cyc

a2 b + 6abc nn ta c

cyc

ab

cyc

X
cyc

cyc

a2 b3

ab

cyc

a2 b3

X
cyc

a b + 3abc

cyc

a3 b2 + 7

X
cyc

a2 b2 c +

ab

cyc

a3 bc

cyc

2X 2 2
7X 2 2
1X 3
a b (a + b) +
a b c+
a bc
3 cyc
3 cyc
3 cyc

271
t q = ab + bc + ca; r = abc th
2X 2 2
7X 2 2
1X 3
a b (a + b) +
a b c+
a bc = 2q 2 + (q
3 cyc
3 cyc
3 cyc
)

a2 b3

2q 2 + (q

9)r

9)r

cyc

Mt khc, ta c

X
cyc

a b

a bc = 3r;

cyc

!2

ab

cyc

a b

cyc

1 2
q
3

cyc

T nhng bt ng thc ny, ta ch cn chng minh c


[2q 2 + (q

9)r] + 15r + (27


,

1
9q + 3r) + 9r + q 2 + 5q
3
2
q(3
3

3r2

q) + (36 + q)r

3(q 2

6r) + 3(9

2q) + 3r2

0:

hin nhin ng do 3 q; 1 r.
Bt ng thc c chng minh. ng thc xy ra khi v ch khi a = 3; b = c = 0
hoc cc hon v tng ng.
Bi ton 2.61 Cho cc s dng a; b; c: Chng minh rng
a3 (b2 + 1) + b3 (c2 + 1) + c3 (a2 + 1)

8(a + b + c)

18:

(Nguyn Cng Minh)


Li gii. S dng bt ng thc Cauchy Schwarz, ta c
!2
!2
P 2
P
a b
ab
X
X
cyc
cyc
5 2
2
P
P
a b
;
a b
a
b
cyc
cyc
cyc

X
cyc

cyc

5 2

a b

!4

ab

cyc

cyc

!3

272

CHNG 2. SNG TO BT NG THC

Mt khc, bt ng thc cho tng ng vi


!
X
X
X
5 2
a b +
a +
a3 + 18
cyc

cyc

cyc

Do , theo trn v theo bt ng thc AM-GM, ta c


!4
P
2
ab
X
X
X
cyc
5 2
a
a b +
a
!3 +
P
cyc
cyc
cyc
a
cyc

Mt khc,
X

!3

cyc

) 18

+ 27 + 27

27

cyc

!2

ab

cyc

cyc

cyc

1
3

cyc

cyc

!3

Nn ta ch cn chng minh c
2

!2

ab

cyc

a3

cyc

cyc

2q 2 X 3
+
a
p
cyc

1
3

X
cyc

!3

1 3
p
3

trong p = a + b + c; q = ab + bc + ca. S dng bt ng thc Schur bc 3, ta c


X
cyc

a3

1
p(2p2
3

5q)

Suy ra
2q 2 X 3
+
a
p
cyc

1 3
p
3

2q 2
1
+ p(2p2
p
3

5q)

v p2 3q:
ng thc xy ra khi v ch khi a = b = c = 1:

1 3
(p2
p =
3

3q)(p2
3p

2q)

0:

273
Bi ton 2.62 Cho cc s khng m a; b; c; d tha mn a + b + c + d = 3: Chng
minh rng
p
ab(a + 2b + 3c) + bc(b + 2c + 3d) + cd(c + 2d + 3a) + da(d + 2a + 3b) 6 3:
(Phm Kim Hng)
Li gii. t
f (a; b; c; d) =

ab(a + 2b + 3c)

cyc

Ta c
f (a; b; c; d)

f (a + c; b; 0; d) = c(b

d)(a + c

d)

v
f (a; b; c; d)

f (0; b; a + c; d) =

a(b

d)(a + c

d)

Suy ra, nu (b d)(a + c b d)


0 th f (a; b; c; d)
f (a + c; b; 0; d) v nu
f (b d)(a + c b d) 0 th f (a; b; c; d) f (0; b; a + c; d), tc l
f (a; b; c; d)

maxff (a + c; b; 0; d); f (0; b; a + c; d)g

Tng t, ta c
f (a + c; b; 0; d)
f (0; b; a + c; d)

maxff (a + c; b + d; 0; 0); f (a + c; 0; 0; d)g


maxff (0; b + d; a + c; 0); f (0; 0; a + c; b + d)g

Mt khc, ta d dng kim tra c


p
6 3
p
maxff (0; b + d; a + c; 0); f (0; 0; a + c; b + d)g 6 3:
maxff (a + c; b + d; 0; 0); f (a + c; 0; 0; d)g

Bt p
ng thc
p c chng minh. ng thc xy ra khi v ch khi a = d = 0; b =
3
3; c = 3 hoc cc hon v tng ng.
Bi ton 2.63 Cho cc s dng a; b; c: Chng minh rng
r
r
r
r
a
b
c
4 3
+
+
(a + b + c):
a2 + b + 4
b2 + c + 4
c2 + a + 4
4
(Nguyn Cng Minh)
Li gii. S dng bt ng thc Cauchy Schwarz, ta c
!2
Xr
X
X
a
a
a
3
3
2
2
a
+
b
+
4
a
+
b
+
4
2a
+
b+3
cyc
cyc
cyc

274

CHNG 2. SNG TO BT NG THC

Ta cn chng minh
X

2a
2a + b + 3

b+3
+
2a + b + 3

cyc

cyc

vP
u
u a
t cyc
3

vP
u
u a
t cyc
3

S dng bt ng thc Cauchy Schwarz,

X
cyc

b+3
2a + b + 3

"

#2

(b + 3)

cyc

(b + 3)(2a + b + 3)

cyc

a+9

!2

a+9

cyc

cyc

!2

+ 12

a + 27

cyc

cyc

= P

a+3

cyc

a = 3t2 (t

0), ta phi chng minh

cyc

3t2 + 9
+t
3t2 + 3
,

t(t 1)2
t2 + 1

0:

hin nhin ng.


ng thc xy ra khi v ch khi a = b = c = 1 hoc a = b = c ! 0:
Bi ton 2.64 Cho cc s khng m a; b; c; khng c 2 s no ng thi bng 0:
Chng minh rng
s
s
s
p
a(b + c)
b(c + a)
c(a + b)
+
+
2:
(2b + c)(b + 2c)
(2c + a)(c + 2a)
(2a + b)(a + 2b)
Li gii. S dng bt ng thc Holder, ta c
s
"
#"
#
X
X a2 (2b + c)(b + 2c)
a(b + c)
(2b + c)(b + 2c)
b+c
cyc
cyc

X
cyc

!3

275
Ta cn chng minh
X

!3

cyc

,
,
Ch rng

X
X
cyc

cyc

!3

X a2 (2b + c)(b + 2c)


b+c

cyc

cyc

a2 (b + c) + 2abc

cyc

cyc

a (b + c) + 3abc

a2 (b + c) + 3abc =

cyc

X 2a
b+c
cyc

a
b+c

X 2a
abc
b+c
cyc

cyc

a3

a(a

b)(a

c)

cyc

3=

X (2a + b + c)(a
cyc

b)(a c)
(a + b)(b + c)(c + a)

Bt ng thc tng ng
X

x(a

b)(a

c)

cyc

trong x = a

abc(2a+b+c)
(a+b)(b+c)(c+a)

a2 (ab+ac+b2 +c2 )
(a+b)(b+c)(c+a)

Khng mt tnh tng qut, ta c th gi s a


a

v
x

y = (a
) x(a

b) 1
c)

y(b

0 v y; z tng t.

b
c

c, khi
0

abc
(a + b)(b + c)(c + a)
c)

(b

c)(x

y)

Mt khc, ta c th vit li bt ng thc nh sau


(a

b)[x(a

c)

y(b

c)] + z(a

c)(b

c)

0:

Bt ng thc c chng minh. ng thc xy ra khi v ch khi a = b = c hoc


a = b; c = 0 hoc cc hon v tng ng.
Nhn xt 18 Ch rng 2(b2 + c2 ) (2b + c)(b + 2c), nn
s
s
X
X
p
a(b + c)
a(b + c)
2
2
2
2(b + c )
(2b + c)(b + 2c)
cyc
cyc

276

CHNG 2. SNG TO BT NG THC

v ta thu c kt qu p sau
r
r
r
a(b + c)
b(c + a)
c(a + b)
+
+
b2 + c2
c2 + a2
a2 + b2

2:
(V Quc B Cn)

ng thc t c ti a = b; c = 0 v cc hon v ca n.

Bi ton 2.65 Cho cc s khng m a; b; c; khng c 2 s no ng thi bng 0:


Chng minh rng
2

b3
c3
a3
+
+
b+c c+a a+b

p
3abc(a + b + c):

2(a2 + b2 + c2 )

(Nguyn Cng Minh)

Li gii. S dng bt ng thc Holder, ta c

X a3
b+c
cyc

cyc

cyc

!"

!3

a2 (b + c)

cyc

Ta cn chng minh
2

cyc

cyc

!"

!3

#+

a2 (b + c)

cyc

3abc

cyc

a2

cyc

Do tnh thun nht, ta c th chun ha cho a+b+c =n1. t q = ab+bc+ca;


r = abc,
o
(4q 1)(1 q)
khi theo bt ng thc Schur bc 4, ta c r max 0;
. Bt ng thc
6
tr thnh

2(1
q

Nu 1

2q)3 p
+ 3r
3r

2(1

2q)

4q th
2(1
q

2q)3 p
+ 3r
3r

2(1

2q)3

2(1

2q)

2(1

q
=

2(1

2q)(1

q)(1
q

2q)
4q)

277
Nu 4q

1 th
2(1
q

2q)3 p
+ 3r
3r

2(1

2(1

2q)

2q)3

(4q 1)(1 q)
2

4(1 2q)3
=
+
4q 2 3q + 1

+
r

(4q

(4q

1)(1
2
1)(1
2

q)
q)

Ta cn chng minh
r
4(1 2q)3
(4q 1)(1 q)
+
2(1 2q)
2
4q
3q + 1
2
r
(4q 1)(1 q)
4(1 2q)3
,
2(1 2q)
2
4q 2 3q + 1
r
2(1 2q)(4q 1)(1 q)
(4q 1)(1 q)
,
2
4q 2 3q + 1
p
, 4q 2 3q + 1 2(1 2q) 2(4q 1)(1 q)

Theo bt ng thc AM-GM, ta c


p
4q 2 3q + 1 2(1 2q) 2(4q
4q 2

3q + 1

(1

1)(1

q)

2q)[2(4q

1) + 1

3q)2

2q] = 2(1

0:

Bt ng thc c chng minh. ng thc xy ra khi v ch khi a = b = c hoc


a = b; c = 0 hoc cc hon v tng ng.
Bi ton 2.66 Cho a; b; c l di 3 cnh ca mt tam gic. Chng minh rng
(a2 b + b2 c + c2 a)2

abc(a + b + c)(a2 + b2 + c2 ):
(V Quc B Cn)

Li gii 1. Bt ng thc tng ng vi


X
X
a4 b2
a4 bc abc(a
cyc

,
,

X
cyc

a4 (b

c)2 +

cyc

a4 (b

b)(a

c)(b

c)

cyc

a4 b2

cyc

c)2 + (a

b)(a

a2 b4

2abc(a

b)(a

c)(b

c)

cyc

c)(b

c)[(a + b)(b + c)(c + a)

2abc]

278

CHNG 2. SNG TO BT NG THC


,

a4 (b

c)2 + (a

b)(a

c)(b

cyc

c)

ab(a + b)

cyc

Do a; b; c l di 3 cnh ca mt tam gic nn tn ti cc s khng m x; y; z


sao cho a = y + z; b = z + x; c = x + y. Bt ng thc tr thnh
"
#
X
X
X
4
2
3
2
(y + z) (y z) + (x y)(y z)(z x) 2
x +5
x (y + z) + 12xyz
cyc

cyc

cyc

T y, gi s x = minfx; y; zg v t y = x + p; z = x + q, ta c th d dng vit li


bt ng thc nh sau
Ax4 + Bx3 + Cx2 + Dx + E

trong
A = 32(p2
B = 16(4p
C = 48(p

pq + q 2 )
3

5p q + pq + 4q 3 )
3

pq

2 2

3p q
3 2

q )

q)2 (p + q) + q 4 ]

p q + pq + 2q )

q) + pq(5p2
2

0
4

p q + pq + q ) = 48[p(p
4

= 8(p + q)[2(p
E = 2(p

2 2

p q

D = 8(p + q)(2p
3

13pq + 9q 2 )]

0:

Bt ng thc c chng minh. ng thc xy ra khi v ch khi


a=b=c
hoc
a = b + c; b =

p
p
3
100 + 12 69
2
+ p
p
3
6
3 100 + 12 69

1
3

c:

hoc cc hon v tng ng.


Li gii 2. Khng mt tnh tng qut, ta c th gi s (a b)(a c) 0. Xt hm
s sau
f (x) = (a2 + b2 + c2 )x2 2(a2 b + b2 c + c2 a)x + abc(a + b + c)
Ta c
f (a) = a(a

b)(a

c)(a

b + c)

v
lim f (x) = +1

x!1

Do , tn ti x0 sao cho f (x0 ) = 0, v v th bit thc ca f (x) phi khng m, tc


l
(a2 b + b2 c + c2 a)2 abc(a + b + c)(a2 + b2 + c2 ):

279
Nhn xt 19 S dng bt ng thc Cauchy Schwarz v bt ng thc trn, ta c
!2
P 2
a b
X a2 b
X
cyc
P 2
a2
c
a
bc
cyc
cyc
cyc

v ta c
a2 b b2 c
+
+
c
a
b
1 + b2
c

, a2

c2 a
a2 + b2 + c2
b
c
a
1 + c2
1
a
b

0:

y chnh l bt ng thc trong k thi chn i tuyn i thi ton quc t ca Moldova
2006.
Bi ton 2.67 Cho cc s khng m a; b; c tha mn a + b + c = 3. Chng minh rng
p
p
p
ab a + bc b + ca c 3:
(Vasile Cirtoaje)
Li gii. S dng bt ng thc Cauchy Schwarz, ta c
!2
!
!
X
X
X p
ab
a2 b
ab a
cyc

cyc

Ta cn chng minh
X

ab

cyc

,
,

X
cyc

ab

cyc

a b

cyc

cyc

!2
1
ab 4
3

X
cyc

a2

a b

1
3

!2

a b

cyc

!2

ab

cyc

cyc

cyc

cyc

cyc

!2
X
ab 4
a3 b +

S dng bt ng thc Vasile


X

cyc

X
cyc

!2

27
3

3abc5

27

. Ta cn phi chng minh

!2

ab

3abc5

27

280

CHNG 2. SNG TO BT NG THC

t q = ab + bc + ca, ta phi chng minh


q[(9

2q)2 + 3q 2

9abc]

Theo bt ng thc Schur bc 3, ta c 3r


4q
c
q[(9 2q)2 + 3q 2 3(4q
, (3

q)(7q 2

81

9. Do , ta ch cn chng minh
9)]

27q + 27)

81
0:

hin nhin ng do q 3.
ng thc xy ra khi v ch khi a = b = c = 1:
Nhn xt 20 T bi ny, ta suy ra kt qu sau phn trc
p
3(a2 + b2 + c2 )
b3
c3
a3
+
+
2
2
2
2
2
2
a +b
b +c
c +a
2
(V Quc B Cn)
y, chng ta c mt li gii khc bng cch s dng bt ng thc Holder nh sau
X

a3
2
a + b2

hX

(a2 + b2 )2 (a2 + c2 )3

hX

i3
i3
1 hX 2
a2 (a2 + c2 ) =
(a + b2 )2
8

Nn ta ch cn chng minh c
i3
i
1 hX 2
3 X 2 hX 2
(a + b2 )2
a
(a + b2 )2 (a2 + c2 )3
8
4
p
p
2
2 p
2
2
t x = a + b ; y = b + c ; z = c2 + a2 th bt ng thc tr thnh
p
p
p
p
p
p
(x + y + z)3 3 x + y + z xy x + yz y + zx z
Theo bt ng thc Cauchy Schwarz, ta c
p
p
p
p
x+ y+ z
3(x + y + z)
Nn ta ch cn chng minh c

(x + y + z)5=2

p
p
p
p
3 3 xy x + yz y + zx z

Do tnh thun nht nn ta c th chun ha cho x + y + z = 3: Khi bt ng thc


tr thnh
p
p
p
xy x + yz y + zx z 3:
y chnh l bt ng thc c chng minh trn. Vy nn bt ng thc cn chng
minh ng.

281

Bi ton 2.68 Cho a; b; c l di 3 cnh ca mt tam gic. Chng minh rng


a b
c
+ + +3
b
c a

a+b
b+c
c+a
+
+
b+c
c+a a+b

:
(V Quc B Cn)

Li gii. Bt ng thc tng ng vi


Xa
X c
+2
b
b+c
cyc
cyc
,

X
cyc

ab

Xa
cyc

+2

cyc

c
b+c

X
cyc

a
+3
b+c

X
cyc

ab

cyc

X bc2
+2
b
b+c
cyc

X ca2
cyc

X
cyc

!
a
+3
b+c

X abc
a2 + 2
b+c
cyc

S dng bt ng thc Moldova, ta c


X ca2

cyc

a2

cyc

Do , ta ch cn chng minh c
X bc2
b+c
cyc
,
,

X bc(c
cyc

a)
b+c

bc(c2

cyc

X
cyc

X abc
b+c
cyc

a2 b3

a2 )(a + b)
X

a2 b2 c:

cyc

hin nhin ng theo bt ng thc AM-GM.


ng thc xy ra khi v ch khi a = b = c:
Bi ton 2.69 Cho cc s khng m a; b; c; khng c 2 s no ng thi bng 0:
Chng minh rng
a2 + b2 + c2
3(a3 b + b3 c + c3 a)
+ 2 2
4:
ab + bc + ca a b + b2 c2 + c2 a2
(Bch Ngc Thnh Cng)

282

CHNG 2. SNG TO BT NG THC

Li gii. Bt ng thc tng ng vi


!
!
!
!
X
X
X
X
2
2 2
3
a
a b +3
a b
ab
cyc

,4

cyc

a4 b2 +

cyc

,4

cyc

a2 b4 + 3

cyc

4 2

a b +

cyc

a b

cyc

2 2

a4 bc + 3a2 b2 c2

a b (2a

3 3

a b

abc 4

cyc

b)

a3 b3 +

abc 4

ab +

ab +

cyc

2 2

a b

cyc

a3 b2 c + 4

a b

a b

a2 b3 c

cyc

cyc

cyc

S dng bt ng thc AM-GM, ta c


X
X
2
a2 b2 (2a b)2 =
[a2 b2 (2a

ab

cyc

cyc

cyc

cyc

cyc

cyc

cyc

cyc

2 4

3abc

cyc

3abc

cyc

b)2 + a2 c2 (2c

a)2 ]

cyc

a2 bc(2a

b)(2c

a)

cyc

= 2abc 4

ab +

cyc

2abc 4

X
cyc

a b

cyc

ab +

6abc

cyc

a b

cyc

3abc :

cyc

Bt ng thc c chng minh. ng thc xy ra khi v ch khi a = b = c hoc


b
c
a
1 = 2 = 0 hoc cc hon v tng ng.
Bi ton 2.70 Cho cc s khng m a; b; c; khng c 2 s no ng th bng 0:
Chng minh rng
b2 (c + a)
c2 (a + b)
a2 (b + c)
+
+
(b2 + c2 )(2a + b + c) (c2 + a2 )(2b + c + a) (a2 + b2 )(2c + a + b)

2
:
3

(Darij Grinberg)
Li gii. S dng bt ng thc Cauchy Schwarz, ta c
"

X
cyc

a2 (b + c)
2
(b + c2 )(2a + b + c)

#"

X a2 (b2 + c2 )(2a + b + c)
cyc

b+c

X
cyc

!2

283
Ta cn chng minh
X

cyc

,3

a4 + 6

cyc

,3

a2 b2

X a2 (b2 + c2 )(2a + b + c)
b+c

cyc

cyc

,3
,3

!2

cyc

cyc

a2 b2

cyc

a2 b2

b+c

X a3 [(b + c)2

cyc

2bc]

b+c

cyc

b+c

X a3 (b2 + c2 )
cyc

a2 b2

X a3 (b2 + c2 )
cyc

cyc

a4 + 2

a4 + 2

a2 b2 + 4

cyc

a4 + 2

cyc

a3 (b + c) + 8abc

cyc

X a2
b+c
cyc

S dng bt ng thc AM-GM, ta c


X a2
b+c
cyc
Do , ta ch cn chng minh c
X
X
3
a4 + 2
a2 b2
cyc

cyc

1X
a
2 cyc

a3 (b + c) + 4abc

cyc

cyc

Do tnh thun nht, ta c th chun ha cho a+b+c = 1. t q = ab+bc+ca; r = abc,


khi bt ng thc tr thnh
3(4r + 2q 2

4q + 1) + 2(q 2

2r)

, 16r + (3

4q)(1

S dng bt ng thc Schur bc 4, ta c r


16r + (3

4q)(1

4q)

16(4q

1)(1
6

q)

2q 2

4(q
4q)

(4q 1)(1 q)
,
6

+ (3

r) + 4r

4q)(1

suy ra
4q) =

1
(1
3

4q)2

0:

ng thc xy ra khi v ch khi a = b; c = 0 hoc cc hon v tng ng.


Nhn xt 21 Chng ta c cng c 1 cch khc (i hi chng ta phi c s kho
lo) chng minh bt ng thc
X
X
X
X
3
a4 + 2
a2 b2 4
a3 (b + c) + 4abc
a 0
cyc

cyc

cyc

cyc

284

CHNG 2. SNG TO BT NG THC

bng cch vit li n nh sau (sau khi gi s c = minfa; b; cg)


c4 + 2c2 (a + b

c)2 + (a

b)2 (3a2 + 3b2 + 2ab

4ac

4bc)

Bt ng thc ny hin nhin ng.


Bi ton 2.71 Cho cc s khng m a; b; c. Chng minh rng
b
c
a
+
+
b+c c+a a+b

3
7 maxf(a
+
2 16

b)2 ; (b c)2 ; (c
ab + bc + ca

a)2 g

(Phm Vn Thun, V Quc B Cn)


Li gii. Khng mt tnh tng qut, ta c th gi s a b
thnh
X a
3
7
(a c)2
+
b+c
2 16 ab + bc + ca
cyc
,

X a[a(b + c) + bc]
b+c

cyc

, a2 + b2 + c2 + abc

X
cyc

cyc

3
7
(ab + bc + ca) + (a
2
16

1
b+c

S dng bt ng thc AM-GM, ta c


X 1

b+c

c. Bt ng thc tr

c)2

3
7
(ab + bc + ca) + (a
2
16

c)2

9
2(a + b + c)

Nn ta ch cn chng minh c
a2 + b2 + c2 +
t a = c + x; b = c + y (x
(11x2

9abc
2(a + b + c)
y

3
7
(ab + bc + ca) + (a
2
16

c)2

0), khi bt ng thc ny tng ng vi

32xy + 32y 2 )c + (x + y)(3x

Hin nhin ng.


ng thc xy ra khi v ch khi a = b = c hoc
ng.

a
4

4y)2
b
3

c
0

0:
hoc cc hon v tng

Bi ton 2.72 Cho cc s dng a; b; c tha mn abc = 1. Tm gi tr ln nht ca


biu thc
1
1
1
P = 2
+ 2
+ 2
:
(a
a + 1)2
(b
b + 1)2
(c
c + 1)2
(V Quc B Cn)

285
Li gii. Khng mt tnh tng qut, gi s a
trng hp sau
Trng hp 1. Nu b 1, ta s chng minh

, 1

1
a + 1)2

(a2

, ab(a

1)(a2

1)(b

(a
2

, ab(a
Do a; b

1
b + 1)2

1
+ 2
a + 1)2
(b

(a2

(a2 b2

c ) a

(a2

b + 1)2

b + 2)(b2

b + 2)(a2 b2

ab + 1)2

(b2

1)(a + b)[2a2 b2
2 2

c. Xt nhng

c4
c + 1)2

1
=1+ 2
ab + 1)2
(c

1
b + 1)2
1
a + 1)2 (b2

1)(b

a + 2)(b

1+

(a2 b2

1
ab + 1)2

ab(a + b) + a2 + b2

b + 2)(a b

ab + 1)

2 2

ab(a + b) + a2 + b2

(a + b)[2a b

b + 2]

1, ta c
(a2
) ab(a2

a + 2)(b2

b + 2)(a2 b2

a + 2)(b2

b + 2)(a2 b2

ab + 1)
ab + 1)2

4;

2ab

a+b

2(a + b)(a2 b2

ab + 1)

Ta cn chng minh
2(a2 b2

2a2 b2

ab + 1)

, (a

ab(a + b) + a2 + b2

1)(b

1)(a + b)

b+2

hin nhin ng. T y, ta c


P

1+

c4
+ 2
c + 1)2
(c

(c2

Trng hp 2. Nu a

1
(b2

Do , nu a

b + 1)2

(c
(c2

9
;
16

(c2

c + 1)2

1
4

)b

1)2
c + 1)2

9
16

4, ta c
a + 1)2
)P

a ) bc

1
=3
c + 1)2

c. Ch rng

(a2

Nu 4

b + 2]

1
2,

(42

4 + 1)2 = 169

1
32
5417
+
=
169
9
1521

ta c 2 trng hp nh

286

CHNG 2. SNG TO BT NG THC

i) Nu c

1
2,

ta c
(c2

b2

b + 1 = b(1

c + 1)2

9
16
1) + (4b2 c2

2c)(b + 2bc

2bc + 1)

1
1
+2
1 + (4b2 c2
b(1 2c)
2
4
a2 2a + 4
= 4b2 c2 2bc + 1 =
a2
)P

) max P
ii) Nu 4

a4
16
+
= f (a)
2a + 4)2
9

1
+ 2
a + 1)2
(a

(a2

1
2,

2bc + 1)

max f (a)

4 a 1

xt hm s sau
g(x) =

1
ex + 1)2

(e2x

vi x 2 [ ln 2; 0]. Ta c
g 00 (x) =

2ex (ex 1)(8e2x + ex


(e2x ex + 1)4

1)

0 8x 2 [ ln 2; 0]

Do g(x) lm trn [ ln 2; 0]. By gi, ta hy ch rng ln b; ln c 2 [ ln 2; 0], nn


theo bt ng thc Jensen,
g(ln b) + g(ln c)

(b2

1
+ 2
b + 1)2
(c
)P

(a2

1
c + 1)2

2g

2
p
bc + 1

bc

1
+
a + 1)2
(a
) max P

ln b + ln c
2

(a

2a2
p
2
a + 1)

2a2
p
2 = h(a)
a + 1)

max h(a)

4 a 1

T nhng trng hp xt trn, ta i n kt lun (vi ch l ng thc lun


xy ra)
5417
max P = max
; max f (a); max h(a) :
4 a 1
1521 4 a 1

287
Bi ton 2.73 Cho cc s khng m a; b; c; d. Chng minh rng
X
a(a b)(a c)(a d) + abcd 0:
cyc

(Phm Kim Hng)


Li gii. Khng mt tnh tng qut, ta c th gi s a
tng ng vi
(a

b)[a(a

c)(a

d)

c)(a d) b(b c)(b d)] + c(c d)(a


+ d[abc + (d a)(d b)(d a)] 0

d. Bt ng thc
c)(b

c)

Ta c
a(a

b(b

c)(b

d) = (a

b)[a2 + ab + b2

(a + b)(c + d) + cd]

v
a2 + ab + b2 (a + b)(c + d) + cd
= (a b)(a + 2b c d) + (b
) a(a

c)(a

d)

b(b

c)(3b + c

2d) + c(c

c)(b

d)

d)

Mt khc
c(c

d)(a

c)(b

c)

v
abc + (d

a)(d

b)(d

a) = d[d2 + a(b

d) + b(c

d) + c(a

d)]

0:

Bt ng thc c chng minh. ng thc xy ra khi v ch khi a = b = c; d = 0


hoc a = b; c = d = 0 hoc cc hon v tng ng.
Bi ton 2.74 Cho cc s khng m a; b; c tha mn a + b + c = 2. Chng minh rng
bc
ca
ab
+
+
a2 + 1 b2 + 1 c2 + 1

1:
(Phm Kim Hng)

Li gii. Bt ng thc tng ng vi


X a
abc
+1
a2 + 1
cyc
Vi mi x

bc

cyc

0, ta c
1
2
x +1

1
x(x 1)2
1+ x=
2
x2 + 1

288

CHNG 2. SNG TO BT NG THC


1
x2 + 1

1
x
2

S dng bt ng thc ny, ta ch cn chng minh c


abc

1
a +1
2

a 1

cyc

, abc

X
cyc

bc

cyc

bc + 1

X
bc

cyc

t q = ab + bc + ca; r = abc. Bt ng thc tr thnh


qr + 1

q(q

1)(4
3

q)

(q 1)(4 q)
.
3

Theo bt ng thc Schur bc 4, ta c r


qr + 1

+1

q=

Do

1
(3
3

1)2

q)(q

0:

do q 34 < 3.
Bt ng thc c chng minh. ng thc xy ra khi v ch khi a = b = 1; c = 0
hoc cc hon v.
Bi ton 2.75 Cho a; b; c l cc s thc phn bit. Chng minh rng
1 + a2 b2
1 + b2 c2
1 + c2 a2
+
+
(a b)2
(b c)2
(c a)2

3
:
2
(Nguyn Vn Thch)

Li gii. Ta d dng kim tra c cc ng thc sau


X 1 + ab 1 + bc
= 1;
a b b c
cyc

X1

cyc

ab 1
b b

bc
=
c

Mt khc, vi mi x; y; z 2 R, ta c
x2 + y 2 + z 2

x2 + y 2 + z 2

xy + yz + zx;

2(xy + yz + zx)

Suy ra
X
cyc

X
cyc

1 + ab
a b
1 ab
a b

X 1 + ab 1 + bc
=1
a b b c
cyc
2

X1
cyc

ab 1
b b

bc
=2
c

289

)2

X 1 + a2 b2
b)2

(a

cyc

X
cyc

1 + ab
a b

X
cyc

1 ab
a b

3:

Bt ng thc c chng minh.


Bi ton 2.76 Cho cc s dng a; b; c: Chng minh rng
ab+c + bc+a + ca+b

1:

Li gii. Nu tn ti t nht mt trong 3 s a; b; c, chng hn a 1, th bt ng


thc hin nhin ng. Do , ta s xt bi ton trong trng hp 1 > a; b; c > 0.
Trng hp 1. Nu a + b + c
1 ) a + b; b + c; c + a
1, s dng bt ng thc
Bernoulli, ta c
1
ab+c

1+

b+c

1+

(1

a)(b + c)
b+c
a+b+c
1+
=
a
a
a
a
a+b+c
X
a
=1
a+b+c
cyc

) ab+c
X
)
ab+c
cyc

Trng hp 2. Nu a + b + c

1, li s dng bt ng thc Bernoulli, ta c

a + b(1
a

1
ab

) ab+c
Ta cn chng minh

X
cyc

a)

[a + b(1

1
ac

a + c(1
a

a2
a)][a + c(1

a2
a)][a + c(1

[a + b(1

a)]

a)

a)]

S dng bt ng thc Cauchy Schwarz, ta c

X
cyc

[a + b(1

v
X
cyc

a2
a)][a + c(1

a)]

cyc

[a + b(1

!2

a)][a + c(1

a)]

cyc

!2

[a + b(1

cyc

X
cyc

a)][a + c(1
!

ab

X
cyc

a)]
!

+ abc 3

X
cyc

0:

290

CHNG 2. SNG TO BT NG THC

T y, ta suy ra c kt qu bi ton ban u.


Bi ton 2.77 Cho cc s dng a; b; c: Chng minh rng
a(b + c) b(c + a) c(a + b)
+ 2
+ 2
a2 + bc
b + ca
c + ab

a2 + bc
b2 + ca
c2 + ab
+
+
:
a(b + c) b(c + a) c(a + b)

a+b+c
p
3
abc

(Phm Hu c)
Li gii. Trc ht, ta s chng minh
P

X a(b + c)

p
3

a2 + bc

cyc

cyc

abc

t a = x3 ; b = y 3 ; c = z 3 , v vi ch rng x3 + y 3
X a(b + c)
a2 + bc

cyc

X x3 (y 3 + z 3 )
x6 + y 3 z 3

cyc

Ta cn chng minh
X x3 (y 3 + z 3 )
x2 yz(x2 + yz)
cyc
,

xy(x + y) 8x; y

0, ta c

X x3 (y 3 + z 3 )
x2 yz(x2 + yz)
cyc
P

x3

cyc

xyz

X x2 (y 3 + z 3 )

x3

cyc

x2 + yz

cyc

Ta c
X

X x2 (y 3 + z 3 )

x3

cyc

x2 + yz

cyc

X x3 (x

X x2 (x3 + xyz
cyc

y)(x
2
x + yz

cyc

y3
x2 + yz

z)

X xy(x
cyc

z3)
y)2 (x + y)(xz + yz
(x2 + yz)(y 2 + zx)

xy)

Do tnh i xng, ta c th gi s z = minfx; y; zg, khi


X x3 (x

x2

cyc

y)(x
+ yz

z)

X xy(x
cyc

y)2 (x + y)(xz + yz
(x2 + yz)(y 2 + zx)

xy)

x (x y)(x z) y 3 (y z)(y x) xy(x y)2 (x + y)(xz + yz


+
+
x2 + yz
x2 + yz
(x2 + yz)(y 2 + zx)
z(x y)2 (x + y)(x3 + y 3 x2 z y 2 z)
=
0
(x2 + yz)(y 2 + zx)

xy)

291
Tip theo, ta s chng minh

Nu

X a2 + bc
a(b + c)
cyc
ab

p
3

cyc

cyc

p
3

abc

abc, khi theo bt ng thc Cauchy Schwarz v bt ng thc

cyc

AM-GM, ta c
X a2 + bc
X a2
X
b2 c2
=
+
a(b + c)
a(b + c) cyc abc(b + c)
cyc
cyc
!2
!2
P
P
a
ab
cyc

cyc

ab

2abc

cyc

Nu

p
3

v
!
u
u P
u
a
u
t cyc

ab

cyc

abc

cyc

ab

cyc

abc

cyc

p
3

abc

, ta c

cyc

X
cyc

!"

ab

X a2 + bc
a(b + c)
cyc

cyc

p
3

ab

cyc

abc

X
cyc

!"

ab

X a2 + bc
a(b + c)
cyc

X (ab + bc)(bc + ca)


cyc

ab + ca

X
cyc

!2

(ab + ca)

0:

cyc

Bt ng thc c chng minh.


Bi ton 2.78 Cho a; b; c l cc s khng m, khng c 2 s no ng thi bng 0
tha mn a + b + c = 1. Chng minh rng
p
p
4
27 3 1
a
b
c
p
p
+p
+p
:
c + 2a
a + 2b
b + 2c
2
(V Quc B Cn, Phm Kim Hng)

292

CHNG 2. SNG TO BT NG THC

Li gii. Nu c

X
cyc

a, khi theo bt ng thc Cauchy Schwarz, ta c

a
p
a + 2b

!2

cyc

cyc

a
a + 2b

X
cyc

a
a + 2b

Nn ta ch cn chng minh c
X

p
4

a
a + 2b

cyc

,
trong x = ab ; y = cb ; z =

a
c

X
cyc

27

!2

p
6 3

1
1 + 2x

) x; y

p
=6 3

z; xyz = 1. Ta c

1
1
1
1
2(x 1)(y 1)(4xy 1)
+
=
+
2x + 1 2y + 1
3 2xy + 1 3(2x + 1)(2y + 1)(2xy + 1)
1
1
1
z
+
= +
3 2xy + 1
3 z+2
)
Nu a

X
cyc

1
2x + 1

1
z
1
4
+
+
=
3 z + 2 2z + 1
3

c, t f (a; b; c) =

cyc

Trng hp 1. Nu a

4b, th

(a + c)2
a + c + 2b

a
a + 2b

) f (a; b; c)

a + 2c
a + 2b +

a+c
a + c + 2b

p
b
c
+p
b
b + 2c
c + 8b
p
a+c
p
+ b = f (a + c; b; 0)
a + c + 2b

b
c
+p
c + 2a
b + 2c

Trng hp 2. Nu 4b

xt 2 trng hp

a2
c(a2 + 4ab + ac + 2bc)
=
a + 2b
(a + 2b)(a + 2b + c)
)p

p a
;
a+2b

p
4
<6 3
3

3z
(z + 2)(2z + 1)

a, th
2
3c
2

a2
c(2a(4b a) + c(a + 2b))
=
a + 2b
2(a + 2b)(2a + 4b + 3c)

293
)p
p

a
a + 2b

c
b
+p
c + 2a
b + 2c
a + 2c
q
a + 2b +

) f (a; b; c)

p
3c
2

a + 2c
q
a + 2b +

3c
2

r
b
c
c
+p
b+
2
b + 2c
c + 2b
r
c
c
c
+ b + = f a + ;b + ;0
2
2
2

Do , trong mi trng hp, ta lun c th a bi ton v chng minh trong trng


hp c mt bin bng 0. Nh vy, chng minh bt ng thc cho, ta ch cn
xt n trong trng hp abc = 0, chng hn c = 0. Khi , ta phi chng minh
p
p
4
p
27 3 1
1 b
p
:
f (a; b; 0) = f (1 b; b; 0) = p
+ b
1+b
2
Ta d pdng chng minh c bt ng thc ny. ng thc xy ra khi v ch khi
p
2( 3 1 )
a = p3 ; b = 2 p3 3 ; c = 0 hoc cc hon v.
Bi ton 2.79 Cho cc s khng m x; y; z tha mn xy + yz + zx + xyz = 4. Chng
minh
1
x + y + z 3 + max (x y)2 ; (y z)2 ; (z x)2 :
4
(V Quc B Cn)
2a
2b
Li gii. t x = b+c
; y = c+a
;z =
ng thc tr thnh
X 2a
cyc

,
, (a

c)2
, (a

b+c

(a; b; c

3+

a
b+c

(a + b + c)
(a + b)2 (b + c)2

0) v gi s a
c
a+b

(a c)2 (a + b + c)2
(a + b)2 (b + c)2
2

c)2

2c
a+b

cyc

(a

b)2 + (b

c)2 ]

(a + b + c)2
(a + b)(b + c)

(a b)2
(b c)2
+
(a + c)(b + c) (a + b)(a + c)

b)2 (a + b) (b
+
a+c

Nu 2(b c) a b, khi ta c (a c)2 2(a


(a + b)(b + c), nn ta ch cn chng minh c
[2(a

c, khi bt

(a b)2
(a + c)(b + c)

1
(a + b)(b + c)

(a + b + c)2
(a + b)(b + c)

(a

b)2 + (b

c)2 (b + c)
a+c
c)2 v (a + b + c)2

b)2 (a + b) (b
+
a+c

c)2 (b + c)
a+c

294

CHNG 2. SNG TO BT NG THC


b)2

,(a

2(a + b + c)2
(a + b)(b + c)

2
c)2

+ (b
Ch rng

2(a+b+c)2
(a+b)(b+c)

a+b
a+c
(a + b + c)2
(a + b)(b + c)

2
a+b (a+b+c)
a+c ; (a+b)(b+c)

b+c
a+c

b+c
a+c

l cc hm ng bin theo a,

nn
2(a + b + c)2
(a + b)(b + c)

a+b
a+c

2(b + b + c)2
(b + b)(b + c)

b+b
c(2b + c)
=
b+c
b(b + c)

(a + b + c)2
(a + b)(b + c)

b+c
a+c

(b + b + c)2
(b + b)(b + c)

b+c
c2
=
b+c
2b(b + c)

Nu a b 2(b c), khi ta c (a c)2


nn ta ch cn chng minh c
[(a

b)2 + (b

c)2 ]

b)2

,(a

(a + b + c)2
(a + b)(b + c)

(a + b + c)2
(a + b)(b + c)
+ (b

(a+b+c)2
(a+b)(b+c)

a+b
a+c

(a + b + c)2
(a + b)(b + c)

(a b)2 +(b c)2 v (a+b+c)2

1
c)2

b)2 (a + b) (b
+
a+c

(a

a+b
a+c
(a + b + c)2
(a + b)(b + c)

b+c
a+c

(a+b)(b+c),
c)2 (b + c)
a+c

l mt hm ng bin theo a nn
a+b
a+c

[(3b 2c) + b + c]2


(3b
1
[(3b 2c) + b](b + c)
(3b
20b3 42b2 c + 31bc2 7c3
=
0
2(2b c)(3b c)(b + c)

Mt khc, t trng hp trn, ta c

(a+b+c)2
(a+b)(b+c)

b+c
a+c

2c) + b
2c) + c

0:

Bt ng thc c chng minh. ng thc xy ra khi v ch khi x = y = z = 1 hoc


x = y = 2; z = 0 hoc cc hon v tng ng.
Bi ton 2.80 Cho a; b; c l di 3 cnh ca mt tam gic. Chng minh rng
a2 + b2
b2 + c2
c2 + a2
+
+
a2 + c2
b2 + a2
c2 + b2

a+b
b+c
c+a
+
+
:
a+c b+a
c+b
(V Quc B Cn)

295
Li gii. Khng mt tnh tng qut, gi s c = minfa; b; cg, ch rng
X a2 + b2
(a2 b2 )2
(a2 c2 )(b2 c2 )
3
=
+
a2 + c2
(a2 + c2 )(b2 + c2 ) (a2 + b2 )(a2 + c2 )
cyc
X a+b
cyc

3=

a+c

(a b)2
(a c)(b c)
+
(a + c)(b + c) (a + b)(a + c)

Bt ng thc tng ng vi
(a

b)2

(a2

(a + b)2
+ c2 )(b2 + c2 )

+ (a

c)(b

1
(a + c)(b + c)
(a + c)(b + c)
c)
2
(a + b2 )(a2 + c2 )

1
(a + b)(a + c)

Ta c
(a + b)2
(a2 + c2 )(b2 + c2 )

1
(a + c)(b + c)

(a + b)2
1
2
2
(a + c) (b + c)
(a + c)(b + c)
(a + b)2 (a + c)(b + c)
=
0
(a + c)2 (b + c)2

Ta cn chng minh
(a + c)(b + c)
(a2 + b2 )(a2 + c2 )
,
Nu a

(a + c)2 (b + c)(a + b)
(a2 + b2 )(a2 + c2 )

c, th
(a + c)2 (b + c)(a + b)
(a2 + b2 )(a2 + c2 )

Nu b

1
(a + b)(a + c)

(b + c)(a + b)
a2 + b2

a(a + b)
a2 + b2

(b + c)(a + b)
a2 + b2

b(a + b)
a2 + b2

1:

c, th
(a + c)2 (b + c)(a + b)
(a2 + b2 )(a2 + c2 )

Bt ng thc c chng minh. ng thc xy ra khi v ch khi a = b = c hoc


a = b; c = 0 hoc cc hon v tng ng.
Bi ton 2.81 Cho cc s khng m a; b; c; khng c 2 s no ng thi bng 0:
Chng minh rng
s
a(b + c) b(c + a) c(a + b)
1
1 1 1
+ 2
+ 2
(a + b + c)
+ +
+ 27:
2
a + bc
b + ca
c + ab
2
a b
c
(Phm Hu c)

296

CHNG 2. SNG TO BT NG THC

Li gii. Bnh phng 2 v v nhn c 2 v cho 4, ta c th vit li bt ng thc


nh sau
!
!
X ab(a + c)(b + c)
X a2 (b + c)2
X
X1
+8
27 +
a
4
(a2 + bc)2
(a2 + bc)(b2 + ca)
a
cyc
cyc
cyc
cyc
,4
,

"

X a2 (b + c)2
cyc

(a2 + bc)2

X (b + c)2

bc

cyc

X ab(a + c)(b + c)
(a2 + bc)(b2 + ca)
cyc

+8

X a2 (b + c)2
cyc

(a2 + bc)2

"

24 +

X (b + c)2
bc

cyc

X ab(a + c)(b + c)
(a2 + bc)(b2 + ca)
cyc

+8 3

X c(a b)2 (a + b)2


bc)2
+
8
bc(a2 + bc)2
(a2 + bc)(b2 + ca)
cyc

X (b + c)2 (a2
cyc

0:

Bt ng thc cui hin nhin ng. Vy ta c pcm. ng thc xy ra khi v ch


khi a = b = c:
Bi ton 2.82 Cho cc s khng m a; b; c, khng c 2 s no ng thi bng 0:
Chng minh rng
a2 (b + c)
b2 (c + a)
c2 (a + b)
+
+
b2 + bc + c2
c2 + ca + a2
a2 + ab + b2

2(a2 + b2 + c2 )
:
a+b+c
(Dng c Lm)

Li gii. S dng bt ng thc Cauchy Schwarz, ta c


"

X a2 (b + c)
b2 + bc + c2
cyc

#"

X a2 (b2 + bc + c2 )
b+c

cyc

X
cyc

a2

!2

Ta cn chng minh
X

cyc

a2

cyc

X
cyc

a(a

cyc

cyc

b)(a

X a2 (b2 + bc + c2 )
b+c

cyc

a2 (b + c)

2abc

cyc

c) + abc 2

X
cyc

X
cyc

a
b+c

a
b+c
0

297
S dng bt ng thc Schur bc 3, ta c

a(a

b)(a

c)

0. Mt khc, bt ng

cyc

thc AM-GM cho ta


2

X
cyc

a
b+c

0:

Bt ng thc c chng minh xong. ng thc xy ra khi v ch khi a = b = c


hoc a = b; c = 0 hoc cc hon v tng ng.
Nhn xt 22 Chng ta c mt kt qu mnh hn l
a2 (b + c)
b2 (c + a)
c2 (a + b)
+ 2
+ 2
2
2
2
b + bc + c
c + ca + a
a + ab + b2

a3 + b3 + c3
a+b+c

(V Quc B Cn)
Ta c th chng minh bng cch tng t nh sau. S dng bt ng thc Cauchy
Schwarz, ta c
"
#"
#
!2
X a2 (b + c)
X a2 (b2 + bc + c2 )
X
a2
2 + bc + c2
b
b
+
c
cyc
cyc
cyc
Ta cn chng minh
X

a2

cyc

!2

vP
u a3 "
#
u
X a2 (b2 + bc + c2 )
u cyc
2t P
a cyc
b+c
cyc

Li theo bt ng thc Cauchy Schwarz, ta c


2

X a2 (b2 + bc + c2 )
b+c

cyc

= 2

a2 (b + c)

2abc

cyc

X
cyc

abc

cyc

a (b + c)

cyc

a
b+c
!2

ab

cyc

Ta cn chng minh

X
cyc

!2

vP
u a3
u
u cyc
tP
a
cyc

6
6 X
6
a2 (b + c)
62
6 cyc
4

abc

cyc

cyc

!2 3

ab

7
7
7
7
7
5

298

CHNG 2. SNG TO BT NG THC

Do tnh thun nht, ta c th chun ha a + b + c = n


1: t q = ab + bc +oca; r = abc;
q)
khi theo bt ng thc Schur bc 3, ta c r max 4q9 1 ; (4q 1)(1
; 0 . Bt ng
6
thc tr thnh

2q)2

, f (r) = q(1

Ta c

p
1

2q)2

(1

3q + 3r 2(q
p

3r)

3q + 3r[2q 2

r
q

(1 + 6q)r]

(6q + 1)(4q 1) + 2 + 6q
9(6q + 1)r + 2 + 6q 42q 2
p
p
2 1 3q + 3r
2 1 3q + 3r
(1 9q 2 ) + 4q(1 3q) + 3q 2
p
=
0
2 1 3q + 3r

f 0 (r) =

Nn f (r) ng bin, do
Nu 1 4q; ta c
h
f (r)
f (0) = q (1 2q)2
=
Nu 4q

1
q(1
2

1; ta c

f (r)

f
= q(1

q)(1

1)(1
6
2

2q)

q(1

2q)2

= q(1

2q)2

(1

4q)

(4q 1)(1 q)
6

(4q

q)(1

2q

q)

p
1

4q 1
9

= q(1

3q

q (1

2q)2

1
(1
2

42q 2

3q + 4q 2 )

0, nn
2

2q)

(1 + q

p
14q 2 + 24q 3 ) 2
12

2q

8q 2

p
14q 2 + 24q 3 ) 2 2q 8q 2 2(1 2q)
24(1 2q)
2
(1 + q 14q + 24q 3 )[2 2q 8q 2 + 4(1 2q)2 ]
48(1 2q)
2
(1 + q 14q + 24q 3 )(3 9q + 4q 2 )
24(1 2q)
3q)(4q 1)(3 6q 8q 2 )
0:
24(1 2q)
(1 + q

Bt ng thc c chng minh.

Bi ton 2.83 Cho cc s khng m a; b; c; khng c 2 s no ng thi bng 0:


Chng minh rng
a2 (b + c)
b2 (c + a)
c2 (a + b)
+
+
b3 + abc + c3
c3 + abc + a3
a3 + abc + b3

2:

299
(Dng c Lm)
Li gii. S dng bt ng thc Cauchy Schwarz, ta c
"

X
cyc

#"

a2 (b + c)
b3 + abc + c3

Ta cn chng minh
X

cyc

a + abc

a4 + abc

a4 + 2

cyc

X a2 (b3 + abc + c3 )

!2

a2 b2

cyc

cyc

Do

!2

b+c

a2 (b2

a b

a2 b2

2 2

cyc

a2

cyc

bc + c2 ) + 2abc

cyc

cyc

cyc

X a2 (b3 + abc + c3 )
cyc

cyc

b+c

cyc

X a2
b+c
cyc
X

X a2
abc 2
b+c
cyc
abc

cyc

X (a + b + c)(a

b)2
(a + c)(b + c)

cyc

(a + b + c)(a b)2
(a + c)(b + c)

cyc

b)2

(a
c

Nn ta ch cn chng minh c
X
cyc

a4 + abc

cyc

a2 b2

abc

cyc

a2 (a

b)(a

X (a
cyc

c)

b)2
c

0:

cyc

hin nhin ng do n chnh l bt ng thc Schur bc 4. ng thc xy ra khi v


ch khi a = b = c hoc a = b; c = 0 hoc cc hon v tng ng.
Bi ton 2.84 Cho m; n (3n2 > m2 ) l cc hng s cho trc v a; b; c l cc s
a+b+c=m
thc tha mn
: Tm gi tr ln nht v gi tr nh nht ca biu
a2 + b2 + c2 = n2
thc sau
P = a2 b + b2 c + c2 a:
(V Quc B Cn)

300

CHNG 2. SNG TO BT NG THC

Li gii. t a = x +

m
3 ;b

m
3 ;c

=y+

=z+

m
3,

x+y+z =0
x2 + y 2 + z 2 =

3n2 m2
3

3n2

) xy + yz + zx =
)

!2

khi t gi thit bi ton, ta c

m2
6

x=

P = x2 y + y 2 z + z 2 x +

m3
9

x2 y 2 =

cyc

xy

cyc

2xyz

(3n2

cyc

m2 )2
36

v ta c th vit li P nh sau

Ta c
X
cyc

3x

18

2
3n2

= 3+

= 12

m2

3n2

3=2

54

3n2
X

m2

2
9

) max P =

x2 y

3n2

m2

3=2

m2 )

cos 29

m2 )

cos 49

m2 )

cos 89

3n2

3=2

m2
2
+

m2

3=2

2
2
9

m2

cyc

p
8
2(3n2
>
x
=
>
<
p 32
2(3n
y=p 3
>
>
:
2(3n2
z=
3
P =

3n2

3n2

cyc

2
9

3=2

54

2
9

x2 y

)P

ta c

xy

X
324
x2 y 2
(3n2 m2 )2 cyc

Mt khc, cho

m2

!2

3n2

m2
2

m3
9

m3
9

3=2

m3
:
9

X
cyc

x2 y

301
Tng t, ch rng
( x) + ( y) + ( z) = 0
( x)2 + ( y)2 + ( z)2 =

3n2 m2
3

nn ta cng c
X
cyc

3n2

2
9

( x)2 ( y)

2
3n2

2
9

x2 y

cyc

ng thc xy ra khi

3n2

2
9

2(3n2 m2 )
3
2(3n2 m2 )
p 32 2
2(3n m )
3

y=
>
>
:
z=

Vy

min P =

m2

3=2

8
>
>
<x =

2
9

3=2

m2
2

v do
P

3=2

m2

3n2

m3
9

cos 29
cos 49
cos 89
3=2

m2

m3
:
9

Bi ton 2.85 Cho cc s dng a; b; c. Chng minh rng


c
a b
+ +
b
c a

3+

(a c)2
:
ab + bc + ca
(V Quc B Cn)

Li gii. Bt ng thc tng ng vi


(ab + bc + ca)
, (a2 + b2 + c2
,

1
[(a
2

b)2 + (b

ab

bc

c)2 + (a

a b
c
+ +
b
c a
ca) +
c)2 ] +

3(ab + bc + ca) + (a

ab2
bc2
ca2
+
+
c
a
b

ab

c)2

a)2

a(b
c

b(c
a

bc
+

c)2

ca
b)2

c(a
b

(a
(a

c)2
c)2

302

CHNG 2. SNG TO BT NG THC


,

c)2 (c + 2a)
c

b)2 (b + 2c) (b
+
b

(a

c)2 (a
b

(a

2b)

S dng bt ng thc Cauchy Schwarz, ta c


b)2 (b + 2c) (b
+
b

(a

c)2 (c + 2a)
c
=

[(a

b) + (b c)]2
b
c
b+2c + c+2a

(a

c)2 (b + 2c)(c + 2a)


2ab + 2bc + 2c2

Ta cn chng minh
a

(b + 2c)(c + 2a)
2ab + 2bc + 2c2
2

2b
a
2

, 4(ab + bc + ca

3abc) + 4b c + 11abc

0:

hin nhin ng.


ng thc xy ra khi v ch khi a = b = c:
Bi ton 2.86 Cho cc s khng m a; b; c; d, khng c 3 s no ng thi bng 0:
Chng minh rng
X

a
a+b+c

cyc

min 1;

4
1
;
2k 1 3k

(V Quc B Cn)
Li gii. Nu k
X
cyc

Nu 2

cyc

a
a+b+c

cyc

a
a+b+c

cyc

a
=1
a+b+c+d

1, khi theo bt ng thc Holder, ta c


"

1, khi ta c

X
cyc

ak
(a + b + c)k

ak
(a + b + c)k

cyc

"

cyc

#"

#k

a(a + b + c)

cyc

2k
k+1

!k+1

#k =

a(a + b + c)

2k

X
cyc

2k

2k
k+1

!k+1

2k

2k

k+1

a k+1 + b k+1 + c k+1 + d k+1


[(a + c)2 + (b + d)2 + (a + c)(b + d)]k

303
Do k

1)
a

2k
k+1

2k
k+1

X
cyc

1, nn
+c

2k
k+1

ak
(a + b + c)k

2k
k+1

a+c
2

;
h

a+c
2

cyc

2k
k+1

+d

+2

2k
k+1

b+d
2

b+d
2

2k
k+1

2k
k+1

ik+1

[(a + c)2 + (b + d)2 + (a + c)(b + d)]k


h
ik+1
2k
2k
k+1 + (b + d) k+1
(a
+
c)
1
= k 1
2
[(a + c)2 + (b + d)2 + (a + c)(b + d)]k

Khng mt tnh tng qut, gi s a + c


X

2k
k+1

ak
(a + b + c)k

1
2k

b + d, t t =

a+c
b+d

ktk (k + 1)(t2k + 1)k (t2k 2tk+1 + 2tk 1


(t2k+2 + tk+1 + 1)k+1
2
t=0
f 0 (t) = 0 , 4 t > 0
t2k 2tk+1 + 2tk 1 1 = 0
2
t=0
4
t>0
,
2k
k 1
1
g(t) = t +2t
2=0
tk+1
(k

1)t2k

4tk

+k+1

tk+2
h0k

1; ta c

(t2k + 1)k+1
1
= k 1 f (t)
2k+2
k+1
k
(t
+t
+ 1)
2

f 0 (t) =

g 0 (t) =

1
k+1

(ktk+1

2)

1)

h(t)
tk+2

Do h0 (t) 0, ta suy ra h(t) nghch bin, do g 0 (t) c ti a mt nghim thuc (0; 1],
v t y, ta suy ra g(t) c ti a 2 nghim thuc (0; 1], trong ta bit trc
mt nghim lun tha l 1. T y, ta c th d dng kim tra c
f (t)

min ff (0); f (1)g = min 1;


X
cyc

ak
(a + b + c)k

min

2k+1
3k

1
4
;
2k 1 3k

304

CHNG 2. SNG TO BT NG THC

V do

X
cyc

T y, trong trng hp k
P

cyc

a2
(a + b + c)2

4
9

2, s dng bt ng thc Holder, ta c


2P

ak
(a+b+c)k

6 cyc
4

X
cyc

a2
(a+b+c)2

ak
(a + b + c)k

3 k2
7
5

1
3k

4
:
3k

Bt ng thc c chng minh xong.


Bi ton 2.87 Cho a; b; c l di 3 cnh ca mt tam gic nhn. Chng minh rng
r
r
r
b+c a
c+a b
a+b c
+
+
3:
a
b
c
(Virgil Nicula)
Li gii. t

a(b + c
p
B = b + b(c + a
p
C = c + c(a + b
A = a+

y = 1
z = 1
Ch rng
B = (a
= (a

b) +
"

b)
c)

a
p
b) + c(a + b
c+a b
p
p
c(a + b c) + a(b + c
a+b c
p
p
a(b + c a) + b(c + a
p
b(a + c

x = 1

a)

hp

b) 1

a(b + c

p
a(b + c

b+c

a)

p
b(c + a

a+b

c
p
a) + b(c + a

c)
a)
b)
i
b)
b)

= z(a

b)

305
Bt ng thc tng ng vi
r

X
cyc

Xb+c

,
,

b+c
a

2a

(a

b)

cyc

(a

1
B

b)(A

cyc

1
A

B)C

cyc

b)2

zC(a

cyc

Do tnh i xng, ta c th gi s a

c)b

b2 +c2
2

pa
2

tam gic cho l tam gic nhn), khi ta c


p
p
p
c(a + b c)
b(a + c b)
a(b + c
p
p p
p
V b(a + c b)
bc; c(a + b c)
bc, nn ta c
b+c

x = 1

b(a + c
p
p
b
c
p
2 bc

a
=

a
p
b) + c(a + b

c)

(v theo gi thit,

a)

b+c a
p
2 bc

Ta s chng minh rng


y
Tht vy, ta c
)

0;

by + cz

p
p
c(a + b c)
a(b + c a)
p
p
c) + a(b + c a)
3a(b + c a) + c(a + b

p
c(a + b

Do , chng minh y

0, ta ch cn ch ra c

3a(b + c

a) + c(a + b

c)

(a + c

b)2

2c2 + (3b + 2a)c (a b)(4a b) 0


p
y l hm tng theo c v do c
a2 b2 , nn ta ch cn chng minh
p
2(a2 b2 ) + (2a + 3b) a2 b2 (a b)(4a b) 0
,

c)

306

CHNG 2. SNG TO BT NG THC

Do b

pa ,
2

p
, (2a + 3b) a2 b2 (a
p
p
, (2a + 3b) a + b
a
ta c

a+b

p
2 a

b)(6a + b)
b(6a + b)
b

v
2(2a + 3b)

6a

b = 5b

2a

Tip theo, ta s chng minh


by + cz
,b+c
V

b)
p
c) + a(b + c

c(a + b

b(c + a

a(b + c

a) +

p
a(b + c

a) +

Ta ch cn chng minh

b(c + a

b)

p
b(c + a

b)

b+c
, f (a) =

b(c + a
p
b(c + a

0
+p
a)
a(b + c

p
c(a + b

b) + c(a + b
b) + 3a(b + c
(b

c)
p
a) + b(c + a

p
a(b + c

a)

b) + 3a(b + c

a)

c) +

p
b(c + a

4(b + c)2 a2 + (b + c)(6b2 + 3bc + 5c2 )a

c(a + b

b)

c)
a)
c)(2b3

b2 c + 4bc2

c3 )

Do f (a) l hm lm, ta d thy


f (a)
Ta c

n
o
p
min f (b); f
b2 + c2

f (b) = c(4b3 b2 c + 10bc2 c3 ) 0


p
p
b2 + c2 = (b + c)(6b2 + 3bc + 5c2 ) b2 + c2 6b4 5b3 c
f

Do

13b2 c2

3bc3

p
b2 + c2
0
f
p
, (b + c)(6b2 + 3bc + 5c2 ) b2 + c2 6b4 + 5b3 c + 13b2 c2 + 3bc3 + 5c4
1
b2 + c2 + bc (6b2 + 3bc + 5c2 ) (6b4 + 5b3 c + 13b2 c2 + 3bc3 + 5c4 )
2
1
=
bc(2b2 bc + 5c2 ) 0
2

5c4

307
Ta ch cn chng minh
p
(b + c) b2 + c2

hin nhin ng v

1
b2 + c2 + bc
2

1
b2 + c2 + bc
2

(b + c)2 (b2 + c2 )

1
bc(4b2 + 4c2
4

bc)

T y, vi ch rng
b3 (a + c

) bB
v

c3 (a + b c) = (b c)[b2 (a b) + c2 (a c) + abc] 0
p
p
) b b(a + c b) c c(a + b c)
h p
i
p
cC = b2 c2 + b b(a + c b) c c(a + b c)
0

b)

(a

c)2

b2
(a
c2

b)2

Ta c
X

zC(a

b)2

c)2 + zC(a

yB(a

b)2

b)2

(a

cyc

(a

b)2

ybcC
+ Cz
c2

yBb2
+ Cz
c2

b)2 (yb + cz)


c

C(a

0:

Bt ng thc c chng minh. ng thc xy ra khi v ch khi a = b = c:


Bi ton 2.88 Cho cc s dng a; b; c. Chng minh rng
a2 + b2 + c2

ab3
bc3
ca3
+
+
3a2 + 2b2
3b2 + 2c2
3c2 + 2a2

(V Quc B Cn, Nguyn Hunh Bo Trung)


Li gii. Bt ng thc tng ng vi
X

11b2

a2

cyc

X
cyc

z(a

50ab3
3a2 + 2b2
b)2

308

CHNG 2. SNG TO BT NG THC


2

3b
v y; z tng t.
trong x = 22c3b26bc
+2c2
Trng hp 1. Nu a b c, khi ta c

y=

22a2 6ac 3c2


2a2 + 3c2

V
22a2

6ac

3c2

2a2

a
+ 3c2

22b2

6bc

2b2

b
+ 3c2

3c2

3b2

b
+ 2c2

a2 (22a2 6ac 3c2 ) 2b2 (22c2 6bc 3b2 )


+
2a2 + 3c2
3b2 + 2c2
2
2
2
2
b (22b
6bc 3c ) 2b (22c2 6bc 3b2 )
+
3b2 + 2c2
3b2 + 2c2
2
2
2
b (16b
18bc + 41c )
=
0
3b2 + 2c2

) a2 y + 2b2 x =

Tng t, ta c
22a2

6ac

2a2

1
+ 3c2

3c2
2a2

22a2
1
+ 3b2

3b2

6ab
3a2

1
+ 2b2

22a2 6ac 3c2


2(22b2 6ab 3a2 )
+
2
2
2a + 3c
3a2 + 2b2
2
2
2(22b2 6ab 3a2 )
22a
6ab 3b
+
3a2 + 2b2
3a2 + 2b2
2
2
16a
18ab + 41b
0
=
3a2 + 2b2
n 2
o
c)2 max ab2 (b c)2 ; (a b)2 , ta c

) y + 2z =

Khi , ch rng (a
2

z(a

b)2 = [y(a

c)2 + 2x(b

c)2 ] + [y(a

c)2 + 2z(a

b)2 ]

cyc

a2
(b c)2 + 2x(b c)2 + [y(a b)2 + 2z(a
b2
(b c)2 2
(a y + 2b2 x) + (a b)2 (y + 2z) 0
=
b2
y

Trng hp 2. Nu c

b
x=

a, ta c
22c2 6bc 3b2
3b2 + 2c2

13c2
3b2 + 2c2

13
5

b)2 ]

309
z=

13b2
3a2 + 2b2

22b2 6ab 3b2


3a2 + 2b2

13
5

Suy ra
x(b

c)2 + z(a

13
[(b
5

b)2

c)2 + (a

b)2 ]

13
(a
10

c)2

Nn ta ch cn chng minh c
13
+y
10

, 3c2 + 82a2

20ac

0:

hin nhin ng.


Bt ng thc c chng minh. ng thc xy ra khi v ch khi a = b = c:

Bi ton 2.89 Cho cc s khng m a; b; c, khng c 2 s no ng thi bng 0:


Chng minh rng
4

a2 + bc
b2 + ca
c2 + ab
+
+
2
2
(b + c)
(c + a)
(a + b)2

3+

3(a2 + b2 + c2 )
:
ab + bc + ca
(Ji Chen)

Li gii. S dng bt ng thc Cauchy Schwarz, ta c

X
cyc

2
)0

2
a
b+c

cyc

!2

ab

cyc

cyc

cyc

!2

ab

X
cyc

a
b+c

310

CHNG 2. SNG TO BT NG THC

Suy ra
2

X a2 + bc
4
+
(b + c)2
cyc

X a2 + bc
4
(b + c)2
cyc

= 4

X (a
cyc

= 4

X (a
cyc

(a

!2

cyc

ab

cyc

cyc

2
b)(a c)
+
(b + c)2

cyc

b)(a

4
(b + c)2

c)

cyc

!2

ab

cyc

ab
c) cyc
P
+

b)(a
(b + c)2

a
b+c

cyc

a2

cyc

ab

P
3 a2
cyc
+3+ P
ab
cyc

P
3 a2
1
cyc
+3+ P
ab
ab + bc + ca
cyc

Ta cn chng minh
X

(a

b)(a

c)

cyc

,4

,4

X a(a
cyc

b)(a

c)

X a(a

b)(a
b+c

c)

cyc

cyc

(a
b)(a
b+c

X a(a
cyc

b)(a
b+c

1
ab + bc + ca

4[bc + a(b + c)]


(b + c)2

(a

X (a
cyc

b) (b

c) (c

a)

b)(a c)(b
(b + c)2

c)

c)2

a +3

cyc

c)

c, khi

4(a

b)

a(a c)
b+c

4(a

b)2 (a2 + b2 + ab
(a + c)(b + c)

cyc

(a + b)2 (b + c)2 (c + a)2

Do tnh i xng, ta c th gi s a
4

4
(b + c)2

ab

b(b c)
b+c
c2 )

8ab(a b)2
(a + c)(b + c)

311
v

8
>
<(a
P

b)2 (bP c)2 (c a)2 a2 b2 (a b)2


a + 3 ab a2 + 2b2 + 3(2ab + b2 ) = (a + b)(a + 5b)
2

cyc
>
:cyc
(a + b)2 (b + c)2 (c + a)2

ab(a + c)(b + c)(a + b)2


!
P
P
a2 + 3 ab
b)2 (b c)2 (c a)2
cyc
cyc
ab(a b)2 (a + 5b)
(a + b)2 (b + c)2 (c + a)2
(a + b)(a + c)(b + c)

(a
)

Ta cn phi chng minh


8ab(a b)2
(a + c)(b + c)

ab(a b)2 (a + 5b)


(a + b)(a + c)(b + c)

, 8(a + b)

a + 5b

, 7a + 3b

0:

hin nhin ng.


Bt ng thc c chng minh. ng thc xy ra khi v ch khi a = b = c hoc
a = b; c = 0 hoc cc hon v tng ng.
Nhn xt 23 Ta c mt kt qu tng t l
a2 + bc b2 + ca c2 + ab
+ 2
+ 2
b2 + c2
c + a2
a + b2

a2 + b2 + c2
ab + bc + ca

7
(V Quc B Cn)

Tht vy, bt ng thc tng ng vi


X 2a2

cyc

(a2

b2 )

cyc

b2

c
(b
b2 + c2

a2
c)
cyc
P
+
2

cyc

ab

ab
0

cyc

1
+ c2

X (a b)2 X
1
(a b)2
+
2
2
2
+c
a +b
2(ab + bc + ca)
cyc
cyc
X
,
z(a b)2 0

a2

cyc

trong x =

(b+c)
(a2 +b2 )(a2 +c2 )

1
2(ab+bc+ca)

Do tnh i xng, ta c th gi s a
(a2

(a + b)2
+ c2 )(b2 + c2 )

(a2

1
b2 +c2

v y; z tng t.

c, khi ta c

(a + c)2
+ b2 )(b2 + c2 )

(a2

(b + c)2
+ b2 )(a2 + c2 )

312

CHNG 2. SNG TO BT NG THC

1
a2 + b2

1
a2 + c2
)z

1
b2 + c2
x

Ta s chng minh
a2 y + b2 x
,

a2 (a + c)2
b2 (b + c)2
a2 + b2
1
+
+
a2 + b2
b2 + c2
a2 + c2
2(ab + bc + ca)
,

a2

b2
a2
+
a2 + c2
b2 + c2

1
a2 (a2 + c2 ) b2 (b2 + c2 )
a3
2c
b3
+
+ 2
+ 2
2
2
2
2
2
2
2
2
+b
b +c
a +c
a +b
b +c
b + c2
a2 + b2
c2
c2
+
+ 2
+
2
2(ab + bc + ca) a + c2
b2 + c2

Ta c
a2 (a2 + c2 ) b2 (b2 + c2 )
+
b2 + c2
a2 + c2
)
v

a2

a2

b2 =

(a2

b2 )2 (a2 + b2 + c2 )
(a2 + c2 )(b2 + c2 )

a2 (a2 + c2 ) b2 (b2 + c2 )
1
+
2
+b
b2 + c2
a2 + c2

2a3
2b3
+
b2 + c2
b2 + c2

2(a2 + b2 )2
(a + b)(ab + c2 )

(a + b)(a2 + b2 )
ab + c2

Suy ra
a3
b3
a2 + b2
+
+
b2 + c2
b2 + c2
2(ab + bc + ca)
a2 + b2
c(a + b)
+
ab + c2
2(ab + bc + ca)
c(a + b) (a b)2 2c(a + b)
= 1+
+
ab + c2
2(ab + bc + ca)
(a b)2
c2 (a + b)(a + b c)
= 1+
+
2(ab + bc + ca) (ab + c2 )(ab + bc + ca)

2c
2
a + b2

) a2 y + b2 x
Khi t z

x, ta c z

y
(a

0. Tip theo, vi ch rng


c)2

a2
(b
b2

c)2

313
Ta c
X

b)2

z(a

x(b

c)2 + y

cyc

a2
(b
b2

c)2 =

(b

c)2
b2

(a2 y + b2 x)

0:

Bt ng thc c chng minh.

Bi ton 2.90 Cho cc s dng a; b; c; d tha mn a+b+c+d = abc+bcd+cda+dab.


Chng minh rng
a+b+c+d+

2a
2b
2c
2d
+
+
+
a+1 b+1 c+1 d+1

8:
(V Quc B Cn)

Li gii. t x = a + b + c + d = abc + bcd + cda + dab, khi t bt ng thc


AM-GM, ta c x 4. Ch rng
v
!
!
u
X
X
u X
t
2
ab 3
a
abc = 3x
sym

cyc

cyc

S dng bt ng thc Cauchy Schwarz, ta c


X 2a
a+1
cyc

2x2
2x2
P 2 =
P
2
a
ab
x+
x+x
2
cyc

x2

2x2
2x
=
+ x 3x
x 2

sym

Ta cn chng minh
2x

x+

, (x

2
4)2

8
0:

hin nhin ng. ng thc xy ra khi v ch khi a = b = c = d = 1:


Bi ton 2.91 Cho cc s dng a; b; c tha mn a + b + c = 3. Chng minh rng
8

1 1 1
+ +
a b
c

+9

10(a2 + b2 + c2 ):
(Vasile Cirtoaje)

314

CHNG 2. SNG TO BT NG THC

Li gii. Khng mt tnh tng qut, gi s a

)16

16
a

1; 23

c)a

16
b

20b2

69 + 84b =

16
b

5 (2b

1)2

16
c

20c2

69 + 84c =

16
c

5 (2c

1)2

1 1
+
b
c

b; 1

c. Ta c

20(b2 + c2 )

= 138
v

16
b

84(b + c) +

20a2 + 18 + 138
1
a
1
a

=
=

16
c

1)2 +

5 (2b

5 (2c

1)2

84(b + c)
1
a

5 (2a

2)2 =

5 [(2b

1)2 + (2c

5 [(2b

1) + (2c

1)2 + 2(2b

1)(2c

1)]2
1)]

Khi , ta c th vit li bt ng thc nh sau


16 1
+
b
a
Do a

16 1
+
c
a

1)2 +

10 (2b

c > 0, ta c a = 3

16 1
+
c
a

1)2

10 (2c
3
10

1
a

2 5

(2b

b v
16 1
+
b
a

10

Mt khc, ta c
16 1
+
b
a
Suy ra, nu (2b
Nu (2b 1)(2c

10

16
1
+
b
3 b

10 =

5(3

2b)(3 b) + 3
>0
b(3 b)

1)(2c 1) 0, th bt ng thc l hin nhin.


1) 0 v b 1, ta c

16 1
+
b
a
)

10
16 1
+
c
a

1
a

10

16 2
+
b
a

16 1
+
b
a

10

15 >

16
b

15 > 0

1
>0
a

1)(2c

1)

315
Do
16 1
+
b
a

1
a

5
Nu (2b

1)(2c

16 1
+
b
a

1)

1)2 + (2c

1)2 ]

0 v b

1, ta c c

1
2

1
a

=
=

Do b

16 1
+
b
a

1, ta ca = 3

16 1
+
c
a

7
6

1
a

16 1
+
c
a

10

5
2

100
7

4b)(56
7b(5 2b)

10

6
7

1
a

2 5

)a

16 13
+
b
7a
2(5

1)2

10 (2c

[(2b

6
7

10

16 1
+
c
a

1)2 +

10 (2b

(2b

b; b

5
4.

1)(2c

1)

Ta c
100
7

16
13
+
b
7 25 b
25b)
0
1
a

c, suy ra
16 13 95
16
13
+
+
c
6a
7
c
6(2 c)
(1 c)(178 95c) + 14
=
>0
6c(2 c)

10

7
6

96
6

1
a

Do
16 1
+
b
a

10 (2b
5

1
a

16 1
+
c
a
7
1)2 + (2c
6

1)2 +
6
(2b
7

10 (2c
1)2

1)2
2 5

1
a

(2b

1)(2c

1):

Bt ng thc c chng minh. ng thc xy ra khi v ch khi a = 2; b = c =


hoc cc hon v tng ng.

1
2

p
Bi ton 2.92 Cho a; b; c 2 0; 3 1 tha mn a2 + b2 + c2 + 2abc = 1: Chng
minh rng
3(a + b + c) 4(1 + abc):
(Jack Garfunkel)

316

CHNG 2. SNG TO BT NG THC

p
a
Li gii. Do tnh i xng, gi s a
b
c) 3 1
3(a + b + c) 4(1 + abc), ta s chng minh
!
r
r
1 a
1 a
f (a; b; c) f a;
;
2
2
h
,3 b+c
,

i
a) + 2a(1

2(1

3[(b + c)2 + 2a 2]
p
+ 2a(1
b + c + 2(1 a)

a
a

1
2.

2bc)

t f (a; b; c) =

2bc)

c)2 + 2(2bc + a 1)]


p
+ 2a(1 a 2bc) 0
b + c + 2(1 a)
"
#
3(b c)2
3
p
p
,
+ 2(1 a 2bc) a
b + c + 2(1 a)
b + c + 2(1 a)
,

3[(b

Ch rng (1 + a)(1
,

c)2 , bt ng thc tng ng


"
#
2(b c)2
3
p
+
a
0
1+a
a)
b + c + 2(1 a)

2bc) = (b

3(b c)2
p
b + c + 2(1
,

3(a + 1)
p
b + c + 2(1

+ 2a

a)

1 nn
a2 + (b + c)2

Suy ra
3(1 a)
p
b + c + 2(1

2a

a)

a)
p
a + 2(1

a;

a)

3(1

Tip theo, ta s chng minh rng


r

a)

a2 + b2 + c2 + 2abc = 1
p
p
1 a2
1 a

)b+c

2a

6
p
b + c + 2(1

3(1 a)
p
b + c + 2(1

, 2a

Do a

h
p
, 3 a + 2(1

i
a)

a
2

2a(1

a)

= 2a

a)

3 p
1
2+1

a>0

317
t t2 = 2(1

a) (t

0) ) 1

t2

1; a =

bt ng thc tr thnh

t2

t2

+t

2 t2
2 ,

1
(t 1)2 (t2 + 2t 2) 0:
2
hin nhin ng. ng thc xy ra khi v ch khi a = b = c =
p
p
3 1
3 1 hoc cc hon v tng ng.
2 ;a =
,

1
2

hoc b = c =

Nhn xt 24 t a = sin A2 ; b = sin B2 ; c = sin C2 th A; B; C l 3 gc ca mt tam


p
gic v A; B; C
2 arcsin 3 1 , ta c bt ng thc Jack Garfunkel vi mt
gi thit "lng" hn l
sin

A
B
C
+ sin + sin
2
2
2

4
3

1 + sin

A
B
C
sin sin
2
2
2

Bi ton 2.93 Cho cc s dng a; b; c tha mn abc 1. Chng minh rng vi mi


k 1, ta c
a+k b+k
c+k
a+b+c
+
+
:
b+k
c+k a+k
(Phm Kim Hng)
Li gii. Bt ng thc tng ng vi
,

k
b+k

(a + k) 1

cyc

X b(a + k)
b+k

cyc

+ (k

1)

3k

cyc

+ (k

1)

3k

cyc

S dng bt ng thc AM-GM, ta c


X b(a + k)
cyc

b+k

p
3
3 abc

p
3
3 abc

cyc

Do

X b(a + k)
cyc

b+k

+ (k

1)

3 + 3(k

1) = 3k:

cyc

Bt ng thc c chng minh. ng thc xy ra khi v ch khi a = b = c = 1:

318

CHNG 2. SNG TO BT NG THC

Bi ton 2.94 Cho cc s khng m a; b; c; khng c 2 s no ng thi bng 0:


Chng minh rng
p
p
p
p
2
2
2
2
(a + b + c)2 :
a a + bc + b b + ca + c c + ab
3
(V Quc B Cn)
Li gii. Trc ht, ta s chng minh rng
X p
X p
a 2(a2 + bc)
a (a + b)(a + c)
cyc

cyc

X hp
a
2(a2 + bc)
cyc

X
cyc

i
p
(a + b)(a + c)

a(a b)(a c)
p
p
2
2(a + bc) + (a + b)(a + c)

Do tnh i xng, ta c th gi s a b c, khi ta c


p
p
p
a (b + c)(b + a) b (a + b)(a + c); a 2(b2 + ca)
T

a
p
p
2
2(a + bc) + (a + b)(a + c)

Nn
X
cyc

a(a b)(a c)
p
p
2
2(a + bc) + (a + b)(a + c)
"
a
p
(a b) p
2
2(a + bc) + (a + b)(a + c)

Tip theo, ta s chng minh rng

X p
a (a + b)(a + c)
cyc

X
cyc

a (a + b)(a + c) + 2

X
cyc

2(b2

2(a2 + bc)

b
p
+ ca) + (b + c)(b + a)

b
p
p
2
2(b + ca) + (b + c)(b + a)
2
3

X
cyc

!2

p
ab (a + b)2 (a + c)(b + c)

4
9

X
cyc

!4

319
S dng bt ng thc Cauchy Schwarz, ta c
(a + b)2 (a + c)(b + c) = (a2 + ab + bc + ca)(b2 + ab + bc + ca)

(ab + ab + bc + ca)2

Ta cn chng minh
X

a (a + b)(a + c) + 2

cyc

4
9

ab(2ab + ac + bc)

cyc

!4

cyc

Do tnh thun nht, ta chun ha cho a + b + c = 1. t q = ab + bc + ca; r = abc,


khi ta c
X
X
a2 (a + b)(a + c) =
a2 (a + bc) = 1 3q + 4r
cyc

cyc

ab(2ab + ac + bc) = 2

cyc

!2

ab

cyc

a2 bc = 2q 2

2r

cyc

Bt ng thc tr thnh
1

3q + 4r + 2(2q 2
,

1
(1
9

3q)(5

4
9

2r)

12q)

0:

hin nhin ng do q 31 :
Bt ng thc c chng minh xong. ng thc xy ra khi v ch khi a = b = c:
Bi ton 2.95 Cho cc s dng a; b; c tha mn abc = 1. Chng minh rng
a+3
b+3
c+3
+
+
2
2
(a + 1)
(b + 1)
(c + 1)2

3:
(Vasile Cirtoaje)

Li gii. Ta c mt ch rng lun c t nht 2 trong 3 s a; b; c, chng hn a; b sao


cho (a 1)(b 1) 0 ) 1 + ab a + b. Ta c
1
1
+
2
(a + 1)
(b + 1)2
)
v

1
ab(a b)2 + (ab 1)2
=
1 + ab
(1 + ab)(1 + a)2 (1 + b)2

1
1
+
(a + 1)2
(b + 1)2

1
1
+
a+1 b+1

4
a+b+2

1
c
=
1 + ab
c+1
4
4c
=
ab + 3
1 + 3c

320

CHNG 2. SNG TO BT NG THC

Do
b+3
a+3
+
2
(a + 1)
(b + 1)2

1
1
1
1
+
+
+
2
2
(a + 1)
(b + 1)
a+1 b+1
2c
4c
+
c + 1 1 + 3c

= 2

Ta cn chng minh
2c
4c
c+3
+
+
c + 1 1 + 3c (c + 1)2
,

4c
1 + 3c

, 4(c + 1)2

c(c + 3)
(c + 1)2
(c + 3)(3c + 1)
1)2

, (c

0:

hin nhin ng. ng thc xy ra khi v ch khi a = b = c = 1 hoc a ! +1; b !


+1; c ! 0 hoc cc hon v tng ng.
Bi ton 2.96 Cho cc s khng m a; b; c; d; k: Chng minh rng
1+

ka
b+c

1+

kb
c+d

1+

kc
d+a

1+

kd
a+b

(1 + k)2 :
(Vasile Cirtoaje)

Li gii. Khng mt tnh tng qut, ta c th gi s (a c)(d


nu (a c)(d b) 0, ta ly (a0 ; b0 ; c0 ; d0 ) = (b; c; d; a) khi
P (a0 ; b0 ; c0 ; d0 ) =

1+

ka
b+c

1+

kb
c+d

1+

kc
d+a

c)

b)

1+

0. Tht vy,
kd
a+b

= P (a; b; c; d)
v
(a0

c0 )(d0

b0 ) = (b

d)(a

T y, ta c
1+

ka
b+c

1+

kb
c+d

1+k

a
b
+
b+c c+d

1+

k(a + b)2
ab + bc + ac + bd

1+

kc
d+a

1+

kd
a+b

1+k

c
d
+
d+a a+b

1+

k(c + d)2
cd + da + ac + bd

321
Suy ra
P (a; b; c; d)

1+
"

Mt khc, ta c

k(a + b)2
ab + bc + ac + bd

k(c + d)2
cd + da + ac + bd
#2
k(a + b)(c + d)
1+

1+ p
(ab + bc + ac + bd)(cd + da + ac + bd)

(a + b)(c + d)
p
(ab + bc + ac + bd)(cd + da + ac + bd)
2(a + b)(c + d)

2(a + b)(c + d)
ab + bc + cd + da + 2ac + 2bd

(ab + bc + cd + da + 2ac + 2bd) = (a

c)(d

b)

0:

Do bt ng thc cn chng minh ng. ng thc xy ra khi v ch khi a =


c; b = d = 0 hoc a = c = 0; b = d.
Bi ton 2.97 Cho cc s khng m a; b; c; khng c 2 s no ng thi bng 0:
Chng minh rng
a
a+b

b
b+c

c
c+a

3 a2 + b2 + c2
:
4 ab + bc + ca
(Nguyn Vn Thch)

Li gii. Bt ng thc tng ng vi


4

X a2 [c(a + b) + ab]
(a + b)2

cyc

,4

X ca2
X a3 b
+4
a+b
(a + b)2
cyc
cyc

S dng bt ng thc AM-GM, ta c


4

X
cyc

Ta cn chng minh
2

a3 b
(a + b)2

X ca2
a+b
cyc

a2

cyc

a2

cyc

X
cyc

a2

X
cyc

a2

322

CHNG 2. SNG TO BT NG THC


,

a2

cyc

ab + 2abc

cyc

X
cyc

1
a+b

S dng bt ng thc Cauchy Schwarz, ta c


2abc

X
cyc

9abc
P
a

1
a+b

cyc

Nn ta ch cn chng minh c
X

a2

cyc

X
cyc

9abc
ab + P
a

a3 + 3abc

cyc

cyc

ab(a + b):

cyc

y chnh l bt ng thc Schur bc 3. Vy ta c pcm. ng thc xy ra khi v


ch khi a = b = c:
Bi ton 2.98 Cho cc s khng m a; b; c; khng c 2 s no ng thi bng 0:
Chng minh rng
a
b
c
3abc
+
+
+
b + c c + a a + b 2(a2 b + b2 c + c2 a)

2:

(Bch Ngc Thnh Cng)


Li gii. Trc ht, ta s chng minh kt qu sau
2(a2 + b2 + c2 )
abc
+ 2
(a + b + c)2
a b + b2 c + c2 a
,

a2 b

abc
+ b2 c + c2 a

, abc(a + b + c)2
,

2(ab + bc + ca) a2 b2
(a + b + c)2

[2(ab + bc + ca)

a2 b(a

b)2

abc

cyc

a2
X

b2
a2

cyc

c2 ](a2 b + b2 c + c2 a)
!
X
ab
cyc

T y, gi s c = min fa; b; cg v t a = c + x; b = c + y (x; y


dng bin i bt ng thc v dng sau
(x2

xy + y 2 )c3 + (2x3

3x2 y + 2y 3 )c2 + (x2

y 2 )(x2

c2

xy

0), ta c th d

y 2 )c + x2 y(x

y)2

323
Nu y x hoc x
chng minh
f (c) = (2x3

3y th bt ng thc trn l hin nhin. Nu 3y


3x2 y + 2y 3 )c2 + (x2

y 2 )(x2

y 2 )c + x2 y(x

xy

x
y)2

y; ta s
0

Ta c
f

= (x2 y 2 )2 (x2 xy y 2 )2 4x2 y(x y)2 (2x3 3x2 y + 2y 3 )


= (x y)2 [(x + y)2 (x2 xy y 2 )2 4x2 y(2x3 3x2 y + 2y 3 )]
(x y)2 [(x + x)2 (x2 xy y 2 )2 4x2 y(2x3 3x2 y + 2y 3 )]
= 4x2 (x y)2 [(x2 xy y 2 )2 y(2x3 3x2 y + 2y 3 )]
= 4x2 (x y)3 [x2 (x 3y) + y 2 (y x)] 0

Vy nn bt ng thc trn ng.


Tr li bi ton ca ta, s dng bt ng thc trn, ta ch cn chng minh c
P
3 a2
X a
1
cyc
!2
b
+
c
2
P
cyc
a
cyc

,
,

1X
2

(a

cyc

(a b)
(a + c)(b + c)

(a

b)2
0

(a + b + c)2

b)(a2

b2 )[(a + b + c)2

b)(a2

2(a + c)(b + c)]

(a

c2

a2 + c2

b)(a2

b2 )(a2 + b2

c2 )

cyc

Khng mt tnh tng qut, gi s a

(a

cyc

cyc

a2 + b2

b2

b2 )(a2 + b2

b
0;

c; khi ta c
(a

c)(a2

c2 )

(b

c)(b2

c2 )

c2 )

cyc

(a c)(a2 c2 )(a2 + c2
(b c)(b2 c2 )(a2 + c2
= 2c2 (b c)(b2 c2 ) 0:

b2 ) + (b
b2 ) + (b

c)(b2
c)(b2

c2 )(b2 + c2
c2 )(b2 + c2

a2 )
a2 )

Bt ng thc c chng minh. ng thc xy ra khi v ch khi a = b = c hoc


a = b; c = 0 hoc cc hon v.

324

CHNG 2. SNG TO BT NG THC

Bi ton 2.99 Cho cc s dng x; y; z tha mn xyz = 1: Chng minh rng


1
1
1
2
+
+
+
2
2
2
(1 + x)
(1 + y)
(1 + z)
(1 + x)(1 + y)(1 + z)

1:

(Phm Vn Thun)
Li gii. Do x; y; z > 0; xyz = 1 nn tn ti cc s dng a; b; c sao cho x = ab ; y =
a
c
b ; z = c : Khi bt ng thc tr thnh
X
cyc

a2
2abc
+
(a + b)2
(a + b)(b + c)(c + a)

S dng bt ng thc Cauchy Schwarz, ta c


#"
#
"
X
X
a2
(a + b)2 (a + c)2
(a + b)2
cyc
cyc

a2 +

cyc

!2

ab

cyc

Nn ta ch cn chng minh c
P

cyc

a2 +

cyc

!2

ab

(a + b)2 (a + c)2

2abc
(a + b)(b + c)(c + a)

cyc

2abc
,
(a + b)(b + c)(c + a)

a2 +

cyc

!2

ab

cyc

(a + b)2 (a + c)2

cyc

2abc
,
(a + b)(b + c)(c + a)

4abc

(a +

cyc
b)2 (a

+ c)2

cyc

(a + b)2 (a + c)2

2(a + b)(b + c)(c + a)

cyc

cyc

t m = a + b; n = b + c; p = c + a th bt ng thc tr thnh
m2 + n2 + n2 p2 + p2 m2

mnp(m + n + p):

Bt ng thc ny hin nhin ng theo bt ng thc AM-GM. Vy ta c pcm.


ng thc xy ra khi v ch khi x = y = z = 1 hoc x ! 0; y ! +1; z ! +1 hoc
cc hon v tng ng.

325
Nhn xt 25 Chng ta cng c 2 li gii khc cho bi ton ny, xem [2]. C 2 li
gii , chng u s dng nhng cng c c bit v kh c sc nhng theo quan
nim bn thn, chng ti thy rng nhng li gii s dng bt ng thc kinh in
gii bt ng thc lun l nhng li gii c sc v hp dn nht, chng lun d
hiu v khng i hi chng ta phi c mt kin thc g cao xa c.
Chng ta cng c mt kt qu tng t l
b2
c2
ab + bc + ca
a2
+
+
+
2
2
2
(a + b)
(b + c)
(c + a)
4(a2 + b2 + c2 )

1
(V Quc B Cn)

chng minh kt qu ny, chng ta cn c b sau


a4 b2 + b4 c2 + c4 a2 + 6a2 b2 c2

3
abc(a2 b + b2 c + c2 a + ab2 + bc2 + ca2 )
2

t x = a2 b; y = b2 c; z = c2 a th bt ng thc trn tr thnh


p
3 p
x2 + y 2 + z 2 + 6 3 x2 y 2 z 2
[ 3 xyz (x + y + z) + xy + yz + zx]
2
Chun ha cho xyz = 1 th bt ng thc tr thnh
f (x; y; z) = x2 + y 2 + z 2 + 6
Gi s x = min fx; y; zg ) t =
f (x; y; z)

f (x;

p
yz; yz)

yz
=

3
2

x+y+z+

1
1 1
+ +
x y z

1; ta c
1 p
y
2
1 p
y
2
1 p
y
2

y+

p
8 yz
(8

p
3

3)

1
; t; t
t2
1)2 + t2 + 1]

3
yz

3
yz
0

p p
f (x; yz; yz) = f

) f (x; y; z)
=

(t

1)2 [(t2

2t
2t4

Bt ng thc va pht biu trn c chng minh. Tr li vi bt ng thc ban


u, bng bin i tng ng, ta thy bt ng thc tng ng vi
P
P 4 2
P
ab
a b
2abc ab(a + b) 7a2 b2 c2
cyc
cyc
cyc
P
+
0
4 a2
(a + b)2 (b + c)2 (c + a)2
cyc

326

CHNG 2. SNG TO BT NG THC

Theo kt trn, ta ch cn chng minh c


P
P
abc ab(a + b) + 26a2 b2 c2
ab
cyc
cyc
P
4 a2
2(a + b)2 (b + c)2 (c + a)2

cyc

Chun ha cho a + b + c = 1; t q = ab + bc + ca; r = abc; khi bt ng thc tng


ng
q
r(q 3r) + 26r2
2(1 2q)
(q r)2
, q(q
, (93q

r)2

2r(q + 23r)(1
2

46)r + 2q(q

1)r + q

2q)
3

y l mt hm lm theo r; li c
f (0) = q 3

v khi b = c = 1 th bt ng thc tr thnh


2a + 1
a2 + 2
,
,

2a(2a2 + 2a + 2 + 26a)
4(a + 1)4

2a + 1
a2 + 2

(a3

a(a2 + 14a + 1)
(a + 1)4

3a + 6a + 1)(a 1)2
(a2 + 2)(a + 1)4

0:

Bt ng thc cui hin nhin ng nn ta c pcm.

Bi ton 2.100 Cho cc s khng m x; y; z; khng c 2 s no ng thi bng 0:


Chng minh rng
s
s
r
p
8x2 + yz
8y 2 + zx
8z 2 + xy
9 2
+
+
:
y2 + z2
z 2 + x2
x2 + y 2
2
(V Quc B Cn)
Li gii. S dng bt ng thc GM-HM, ta c
s
s
p
2 2
X 8x2 + yz X 8x2 + y2y2 +zz 2
p X 4x2 (y 2 + z 2 ) + y 2 z 2
=
2
y2 + z2
y2 + z2
y2 + z2
cyc
cyc
cyc

327
t a = x2 ; b = y 2 ; c = z 2 th ta cn chng minh
p
X bc + 4ab + 4ac
b+c
cyc

9
2

Bnh phng 2 v, ta c bt ng thc tng ng


p
X a
X [4ab + c(4a + b)][4ab + c(a + 4b)]
X ab + bc + ca
+3
+2
(b + c)2
b+c
(a + c)(b + c)
cyc
cyc
cyc
Vi mi x; y

81
4

0; ta c
4(x2 + 3xy + y 2 )2
xy(x y)2
=
(x + y)2
(x + y)2

(4x + y)(x + 4y)

p
(4x + y)(x + 4y)

2(x2 + 3xy + y 2 )
x+y

T y, s dng bt ng thc Cauchy Schwarz, ta c


p
p
X [4ab + c(4a + b)][4ab + c(a + 4b)]
X 4ab + c (4a + b)(a + 4b)
(a + c)(b + c)
(a + c)(b + c)
cyc
cyc
X 4ab +

(a + c)(b + c)

cyc

2c(a2 +3ab+b2 )
a+b

X 2ab(a + b) + c(a2 + 3ab + b2 )


(a + b)(b + c)(c + a)
!
!
P
P
a
ab

cyc

6
=

cyc

cyc

(a + b)(b + c)(c + a)

Ta cn chng minh

X ab + bc + ca
cyc

(b + c)2

+3

X
cyc

12
a
ab
cyc
cyc
a
+
b + c (a + b)(b + c)(c + a)

Trong bi vit CYH technique, chng ta chng minh


X ab + bc + ca
cyc

(b + c)2

9
4

81
4

328

CHNG 2. SNG TO BT NG THC

Do ta ch cn chng minh

X
cyc

a
ab
12
cyc
cyc
a
+
b + c (a + b)(b + c)(c + a)

a3 + 3abc

cyc

18

ab(a + b):

cyc

Bt ng thc cui chnh l bt ng thc Schur bc 3. Vy ta c pcm. ng thc


xy ra khi x = y = z hoc x = y; c = 0 v cc hon v tng ng.
Nhn xt 26 Ngoi ra, chng ta cn c mt kt qu kh kinh ngc l
s
47x2 y 2 y 2 + 2xy + 2xz + 14yz
2(8x2 + yz)
y2 + z2
5(x2 + y 2 + z 2 ) + 2(xy + yz + zx)
(V Quc B Cn)
Vi ng thc xy ra khi (x; y; z) (1; 1; 1); (0; 1; 1); (1; 1; 0):
Chng ta c th d dng kim tra c
X 47x2
cyc

5(x2

y 2 y 2 + 2xy + 2xz + 14yz


=9
+ y 2 + z 2 ) + 2(xy + yz + zx)

Nn bt ng thc ny cht hn bt ng thc bi ton ban u rt nhiu. Hin nay


vn cha c mt li gii n gin no cho bt ng thc trn.

Bi ton 2.101 Cho cc s khng m a; b; c; d; khng c 3 s no ng thi bng 0:


Chng minh rng
a(2a + b) b(2b + c)
c(2c + d) d(2d + a)
+
+
+
a+b+c
b+c+d c+d+a
d+a+b

a + b + c + d:
(Park Doo Sung)

Li gii. Bt ng thc tng ng vi


X a(2a + b)
cyc

a+b+c

X a(a c)
a+b+c
cyc

cyc

329
, (a
,

(a

c)

a
a+b+c

c
+ (b
a+c+d

c)2 (a + c) + (a c)(ad
(a + b + c)(a + c + d)

bc)

(b

d)

b
b+c+d

d
a+b+d

d)2 (b + d) + (b d)(ab
(a + b + d)(b + c + d)

0
cd)

Ch l
2(ad
2(ab

bc) = (a
cd) = (b

c)(b + d)
d)(a + c)

(b
(a

d)(a + c)
c)(b + d)

T y, ta c th bin i bt ng thc v
(a

c)2 (2a + 2c + b + d) (a2 c2 )(b d)


(a + b + c)(a + c + d)
(b d)2 (2b + 2d + a + c) (a c)(b2
+
(a + b + d)(b + c + d)

d2 )

(a c)2 (2a + 2c + b + d) (b d)2 (2b + 2d + a + c)


+
(a + b + c)(a + c + d)
(a + b + d)(b + c + d)
a+c
b+d
(a c)(b d)
+
(a + b + c)(a + c + d) (a + b + d)(b + c + d)

Nu (a c)(b d) 0 th bt ng thc hin nhin. Nu (a


bt ng thc AM-GM

c)(b

d)

0; s dng

(a c)2 (2a + 2c + b + d) (b d)2 (2b + 2d + a + c)


+
(a + b + c)(a + c + d)
(a + b + d)(b + c + d)
s
(2a + 2c + b + d)(2b + 2d + a + c)
2(a c)(b d)
(a + b + c)(a + c + d)(a + b + d)(b + c + d)
Do , chng minh bt ng thc cho, ta ch cn chng minh c 2 bt ng
thc sau
s
(2a + 2c + b + d)(2b + 2d + a + c)
a+c
(a + b + c)(a + c + d)(a + b + d)(b + c + d)
(a + b + c)(a + c + d)
s

(2a + 2c + b + d)(2b + 2d + a + c)
(a + b + c)(a + c + d)(a + b + d)(b + c + d)

b+d
(a + b + d)(b + c + d)

Chng hn, ta s chng minh bt ng thc th nht, bt ng thc tng ng


(2a + 2c + b + d)(2b + 2d + a + c)
(a + b + d)(b + c + d)

(a + c)2
(a + b + c)(a + c + d)

330

CHNG 2. SNG TO BT NG THC

t x = a + c; y = b + d; ta c
(2a + 2c + b + d)(2b + 2d + a + c)
(a + b + d)(b + c + d)
=
(a + c)2
(a + b + c)(a + c + d)
V
4(2x + y)
x + 2y

4(2a + 2c + b + d)(2b + 2d + a + c)
(a + c + 2b + 2d)2
4(2x + y)
x + 2y

(a + c)2
x
=
(a + c)(a + c + b + d)
x+y

x
7x2 + 10xy + 4y 2
=
>0
x+y
(x + y)(x + 2y)

nn bt ng thc ng. Tng t, ta cng c


s
(2a + 2c + b + d)(2b + 2d + a + c)
(a + b + c)(a + c + d)(a + b + d)(b + c + d)

b+d
:
(a + b + d)(b + c + d)

Vy ta c pcm. ng thc xy ra khi a = c; b = d:


Bi ton 2.102 Cho cc s dng a; b; c: Chng minh rng
s
r
r
r
p
p
p
a(b + c)
b(c + a)
c(a + b)
a+ b+ c
+
+
2
2
2
a + bc
b + ca
c + ab

1
1
1
p +p +p :
a
c
b
(Phm Hu c)

Li gii. Bt ng thc tng ng


s
X a(b + c)
X
ab(a + c)(b + c)
+2
2 + bc
2 + bc)(b2 + ca)
a
(a
cyc
cyc

Xp

cyc

X 1
p
a
cyc

Ch l
(a2 + bc)(b2 + ca)

ab(a + c)(b + c) = c(a + b)(a


s
X
ab(a + c)(b + c)
)
3
2 + bc)(b2 + ca)
(a
cyc

Do , ta ch cn chng minh
X a(b + c)
cyc

a2 + bc

+6

Xp
cyc

X 1
p
a
cyc

b)2

331
,
,

Xb+c
p
bc
cyc
X

cyc

b+c
p
bc

cyc

2 p
p
b
c
X6
p
,
4
2 bc
cyc

X a(b + c)

a2 + bc

a(b + c)
a2 + bc

p
(b + c) a
bc
p
+
2 bc(a2 + bc)

0
0
2

3
7
5

0:

Bt ng thc c chng minh. ng thc xy ra khi a = b = c:


Bi ton 2.103 Cho cc s dng a; b; c: Chng minh rng
r
p r
r
r
b
c
a
3 3
(a + b)(b + c)(c + a)
a
+b
+c
:
c+a
a+b
b+c
4
a+b+c
(Mathnfriends contest)
Li gii. t x = bc; y = ca; z = ab; bin i, ta c bt ng thc tng ng
p s
y
z
3 3
(x + y)(y + z)(z + x)
x
p
+p
+p
x+y
y+z
4
xy + yz + zx
z+x
p
X
x
1
3 3
p
p
p
,
4 xy + yz + zx
(x + y)(x + z)
(x + y)(y + z)
cyc

S dng bt ng thc sp xp li (vi gi thit y l s hng nm gia), ta thy


x

1
y
1
p
p
+p
(x + y)(x + z)
(y + z)(z + x)
(y + z)(y + x)
(y + z)(y + x)
1
y
z
1
p
=
+p
+p
(y + z)(y + x)
(z + x)(z + y)
(z + x)(x + y)
(y + z)(y + x)
"
#
r
1
y
xy + yz + zx
= p
1+ p
xy + yz + zx
(y + z)(y + x)
(y + z)(y + x)
q
p
xy+yz+zx
t u = p y
1 ) (y+z)(y+x)
= 1 u2 . Theo bt ng thc AM-GM,
VT

(y+z)(y+x)

xy + yz + zx
(y + z)(y + x)

1+ p
(y + z)(y + x)

=
=

p
(1 + u) 1

u2 =

p
(1 + u)3 (1

(1 + u) (1 + u) (1 + u) 3(1
3
s
p
4
(3/2)
3 3
=
:
3
4

u)
u)

332

CHNG 2. SNG TO BT NG THC

Bt ng thc c chng minh xong. ng thc xy ra khi v ch khi a = b = c:


3) tha mn a21 + a22 +

Bi ton 2.104 Cho cc s khng m a1 ; a2 ; :::; an (n


a2n = 1: Chng minh rng
1
p (a1 + a2 +
3

+ an )

a1 a2 + a2 a3 +

+ an a1 :

Li gii. t
1
fn (a1 ; a2 ; :::; an ) = p (a1 + a2 +
3

+ an )

a1 a2

a2 a3

an a1

Khng mt tnh tng qut, gi s a1 = max fa1 ; a2 ; :::; an g :


p1 th
Nu an
3
fn (a1 ; a2 ; :::; an )
1
= p
3
an

an

fn

1 (an 2

a1 ; a2 ; :::; an

+ an

+ an )

an a1

a2n

) fn (a1 ; a2 ; :::; an )
Nu an

p1
3

p1
3

) a1

fn (a1 ; a2 ; :::; an )
1
= p
3
+an
1
p
3

an
q

a2n

an

) an

fn

+ a2n

2 + an

q
a2n
an

1
p
3

) fn (a1 ; a2 ; :::; an )

fn

2 an 1

= an

an

3;

2
2 + an

an
2
2 + an

an

+ a2n

q
+ a1 ) a2n

an

2;

Ta c

q
a2n

a2n

a1 ; a2 ; :::; an

p1 :
3

an

1
p
3
fn

+ (an

a1 ; a2 ; :::; an

2 + an

+ a2n

2;

q
a2n
1

q
a2n

+ a2n

+ a2n

1 ; an

q
a2n

+ a2n

q
a2n

+ an

+ a2n

+ an

+ a2n

an

2 an 1

q
a2n

1
+p
3

+ a2n

an

a1 ; a2 ; :::; an

3;

a2n

+ a2n

1 ; an

T y, bng php quy np theo n; ta d dng chng minh c bt ng thc


cho. ng thc xy ra khi v ch khi n = 3 v a1 = a2 = a3 = p13 :

333
Bi ton 2.105 Cho cc s a; b; c 2 R: Chng minh rng vi mi p > 0; ta c
b2 ca
c2 ab
a2 bc
+
+
2pa2 + p2 b2 + c2
2pb2 + p2 c2 + a2
2pc2 + p2 a2 + b2

0:
(Vasile Cirtoaje)

Li gii. Bt ng thc tng ng


X
cyc

2p(a2 bc)
2pa2 + p2 b2 + c2

,3

cyc

,3

cyc

2p(a2 bc)
2pa2 + p2 b2 + c2
(pb + c)2
2pa2 + p2 b2 + c2

S dng bt ng thc Cauchy Schwarz, ta c


3

X
cyc

X
cyc

a2

pb2
c2
+ 2
2
+ pb
c + pa2
2

(pb + c)
:
2pa2 + p2 b2 + c2

X
cyc

p(a2

(pb + c)2
+ pb2 ) + c2 + pa2

Nn ta c pcm. ng thc xy ra khi v ch khi a(b2 +pc2 ) = b(c2 +pa2 ) = c(a2 +pb2 ):

Bi ton 2.106 Cho cc s khng m x1 ; x2 ; :::; xn tha mn x1 + x2 +


Chng minh rng
!
! n
n
X
X
p
1
n2
p
p
xi
:
1 + xj
n+1
i=1
i=1

+ xn = 1:

(China TST 2006)

8
< yi 1 (i = 1; 2; :::; n)
n
P
Li gii. t y1 = x1 + 1 )
: Bt ng thc tng ng
y1 = n + 1
:
i=1

vi

n
X
p

yi

i=1

n
X
1
p
yi
i=1

n2
n+1

334

CHNG 2. SNG TO BT NG THC

n
X
i=1

n p
P

yj

j=1

1
p

yi

n2
n+1

S dng bt ng thc Cauchy Schwarz, ta c


n p
P

i=1

yi
p
y1

p1
n

1
n

y1

1+

+ (y2

y2

1) +

ny1 + 2(n + 1)

1 p1n +
p
y1

yn

+ (yn 1) (y1
p
y1

p1
n

1) +

1
n

1
n

2n + 1
ny1

Tng t, ta c
n p
P
yj

j=1

n
X
i=1

yi

n p
P

yj

j=1

nyi + 2(n + 1)

n
X

yi

i=1

v
u n
u X
tn

2n + 1
8i = 1; 2; :::; n
nyi

nyi + 2(n + 1)

2n + 1
nyi
2n + 1
nyi

nyi + 2(n + 1)

i=1

v
u
u
= tn n(n + 1)

v 0
u
u
u B
u B
un @n(n + 1)
t

n n(n + 1)

2n + 1 X 1
n i=1 yi
2n + 1
n

!
1

n2 C
C
n
A
P
yi

i=1

2n + 1
n2
n
n+1

=p

n2
:
n+1

Bt ng thc c chng minh. ng thc xy ra khi v ch khi x1 = x2 =


xn = n1 :

335
Bi ton 2.107 Cho cc s dng a; b; c: Chng minh rng
a2
b2
c2
+
+
2a + b + c 2b + c + a 2c + a + b

3
4

a2 + b2 + c2
:
3
(Michael Rozenberg)

Li gii. S dng bt ng thc AM-GM, ta c


r

a + b + c a2 + b2 + c2
+
3
a+b+c

a2 + b2 + c2
3

Nn ta ch cn chng minh c
X

a2
2a + b + c
cyc

a + b + c a2 + b2 + c2
+
3
a+b+c

3
8

Chun ha cho a + b + c = 1 th bt ng thc tr thnh


X
cyc

X
cyc

a2
a+1
a2
a+1
,

vi x =

1 2a
1+a

3 2
a
8
3 2
a
8

x(3a

1
24

3
1
a+
16
48
1)2

cyc

v y; z tng t. Ta c
X

x(3a

cyc

1)2 =

(4x + y + z)(a

b)(a

c)

cyc

v
4x + y + z

=
=

4(1 2a) 1 2b 1 2c
+
+
1+a
1+b
1+c
1
1
4(1 2a)
+3
+
4
1+a
1+b 1+c
4(1 2a)
12
12(1 a)2
+
4=
1+a
3 a
(1 + a)(3 a)

336

CHNG 2. SNG TO BT NG THC

Ngoi ra, do hm f (t) = 11+t2t nghch bin nn nu ta gi s a


T , ta c
X
(4x + y + z)(a b)(a c)

c th x

z:

cyc

(4y + z + x)(b c)(b


(4y + z + x)(b c)(b
= (z y)(a b)(b c)

a) + (4z + x + y)(a
a) + (4z + x + y)(a
0:

c)(b
b)(b

c)
c)

Bt ng thc c chng minh xong. ng thc xy ra khi v ch khi a = b = c:


V d 2.1 Cho cc s khng m x1 ; x2 ; :::; xn (n
Tm gi tr ln nht ca biu thc
P (x1 ; x2 ; :::; xn ) = x31 x22 + x32 x23 +

3) tha mn x1 +x2 +

+ x3n x21 + n2(n

1) 3 3
x1 x2

+xn = 1:

x3n :

(V Quc B Cn)
Li gii. Gi s x1 = max fx1 ; x2 ; :::; xn g : Ta chng minh
P (x1 ; x2 ; :::; xn )

P (x1 ; x2 + x3 +

+ xn ; 0; :::; 0)

Tht vy,
P (x1 ; x2 + x3 +
+ xn ; 0; :::; 0)
3
= x1 (x2 + x3 +
+ xn )2
3
3
2(x1 x2 x3 + x1 x3 x4 +
+ x31 xn 1 xn ) + x31 x22 + x31 x2n
(x31 x2 x3 + x31 x3 x4 +
+ x31 xn 1 xn ) + (x32 x23 + x33 x24
+
+x3n 1 x2n ) + x31 x22 + x31 x2n
x31 x22 + x32 x23 +
+ x3n x21 + x31 x2 x3
Ta chng minh
x31 x2 x3

n2(n

, x22 x23 x34

1) 3 3
x1 x2

x3n

x3n
1

n2(n 1)

Ta c
x22 x23 x34

x3n

(x2 x3

xn )2

x1 + x2 + x3 +
n

x2 + x3 +
+ xn
n 1
+ xn

2(n 1)

2(n 1)

1
n2(n

1)

337
Do
P (x1 ; x2 ; :::; xn )

P (x1 ; x2 + x3 +

+ xn ; 0; :::; 0)

Li c
P (x1 ; x2 + x3 +

= x31 (x2 + x3 +

+ xn ; 0; :::; 0)

x1
3

108

108
:
3125

ng thc xy ra chng hn khi x1 =


108
max P = 3125
:

3
5 ; x2

+ xn )2 = x31 (1

x1

108

2
5 ; x3

x1 )2

x1 + 1
5

x1

= xn = 0: Vy nn

Bi ton 2.108 Cho cc s khng m a; b; c; khng c 2 s no


Chng minh rng
r
r
r
r
a
b
c
3(ab + bc + ca)
+
+
+3
b+c
c+a
a+b
a2 + b2 + c2

ng thi bng 0:
p
7 2
:
2

(V Quc B Cn)
Li gii. Gi s a = max fa; b; cg : Ta chng minh
r
r
r
b
c
b+c
+
c+a
a+b
a
s
b
c
bc
b+c
,
+
+2
c+a a+b
(a + b)(a + c)
a
s
bc
bc
bc
,2
+
(a + b)(a + c)
a(a + b) a(a + c)
2a
,p
bc

2a

2a

b c
p
+
bc
2

b
p

2a + b + c
p
(a + b)(a + c)

(b c)2
p
p
p
(a + b)(a + c) a + b + a + c

bc

1
b c
6
+(b c)2 4 p
p
p
bc
bc
b+ c

(a + b)(a + c)

1
p

a+b+

a+c

3
7

25

338

CHNG 2. SNG TO BT NG THC

Ta c
1
p

bc

b+

c
1

bc

b+

1
p
p
p
(a + b)(a + c) a + b + a + c
p

(b + b)(c + c)

3
p
p
p
4 bc
b+ c

1
p

b+b+

c+c

>0

Li c
ab + bc + ca
a2 + b2 + c2
)
Suy ra
VT
vi x =

a
b+c

a
+
b+c

b+c
a

a(b + c)
a2 + (b + c)2
a(b + c)
a2 + (b + c)2

ab + bc + ca
a2 + b2 + c2
r

s
p
b+c
3a(b + c)
3 3
p
=x+
+3
a
a2 + (b + c)2
x2 2

2:Mt khc ta d thy x +

p
p3 3
x2 2

p
7 2
2

ng thc cho ng. ng thc xy ra khi v ch khi (a; b; c)

8x

2 nn bt

3 + 2 2; 1; 0 :

Bi ton 2.109 Cho cc s dng a; b; c; d tha mn a2 + b2 + c2 + d2 = 4: Chng


minh rng
s
1 1 1 1
1
3
(abcd + 1)
(a + b + c + d)
+ + +
:
2
a b
c d
(Phm Hu c)
Li gii. Trc ht, ta s chng minh rng
9(a + b + c + d)
Tht vy, gi s d = min fa; b; c; dg ) 1

4abcd + 32

d > 0; t

P (a; b; c; d) = 9(a + b + c + d)
v

4abcd

a2 + b2 + c2
; p=a+b+c
3
p
p
) 2 3 3x p x 3; x 1

x=

32

339
Ta chng minh
P (a; b; c; d)
, 9(3x

P (x; x; x; d)
4d(x3

p)

abc)

T bt ng thc Schur bc 4
X

a2 (a

b)(a

c)

cyc

(p2

) abc

6x2 )(p2 + 3x2 )


12p

Ta cn chng minh
p(3x

, (3x
Do 3x

p
81

(p2

4d x3

p)

6x2 )(p2 + 3x2 )


12p

d(p3 + 3p2 x + 6px2 + 6x3 )


p

p) 27

p
x 3 nn
3d(p3 + 3p2 x + 6px2 + 6x3 )
p

78x2 d = 81

81
=

3 + 26(1

d)(3

26d(4
d

d2 )

Ta cn phi chng minh


P (x; x; x; d)
, 9(3x + d)

4x3 d

32

4x3 )d 32 27x
p
4x3 ) 4 3x2 32 27x

, (9
, (9

, f (x) =
f 0 (x) =

12(x

(9

32 27x
p
4x3 ) 4 3x2

1)(81x4 47x3
(9 4x3 )2 (4
) f (x)

119x2 + 9x + 81)
3x2 )3=2

f (1) = 1

Tr li bi ton, s dung bt ng thc AM-GM, ta c


x4 = abcd

1;

1 1 1 1
+ + +
a b
c d

p
4

4
4
=
x
abcd

d2 )
0

340

CHNG 2. SNG TO BT NG THC

Do , kt hp vi bt ng thc trn, ta ch cn chng minh c


r
4
3 4(x + 1)
4
2(x + 8) 9
x
, g(x) =

x(x4 + 8)3
x4 + 1

729
2

Ta c

(x4 + 8)2 (9x8 11x4 + 8)


>0
(x4 + 1)2
729
) g(x) g(1) =
:
2
Bt ng thc c chng minh. ng thc xy ra khi a = b = c = d = 1:
g 0 (x) =

Bi ton 2.110 Cho cc s khng m a; b; c; khng c 2 s no ng thi bng 0:


Chng minh rng
1
1
1
+
+
(a + 2b)2
(b + 2c)2
(c + 2a)2

1
:
ab + bc + ca
(Phm Kim Hng)

Li gii 1. Khng mt tnh tng qut gi s a = max fa; b; cg : S dng bt ng


thc AM-GM, ta c
1
1
+
(a + 2b)2
(c + 2a)2

2
(a + 2b)(c + 2a)

1
1
+
(a + 2b)2
(b + 2c)2

2
(a + 2b)(b + 2c)

Nn ta ch cn chng minh c
1
2
+
(b + 2c)2
(a + 2b)(c + 2a)
, f (a) = 2(b + c)a3 + (2b2

bc

1
ab + bc + ca

7c2 )a2 + (4c3

b2 c

bc2

2b3 )a + 2b2 c2

hoc
1
2
+
(c + 2a)2
(a + 2b)(b + 2c)
, g(a) = 4a3 b + (2c2

bc

7b2 )a2 + (2b3

1
ab + bc + ca

b2 c

bc2 )a + 2b3 c + 2b2 c2

2bc3

Ta s chng minh rng trong 2 bt ng thc trn, c t nht 1 bt ng thc ng.


lm c iu ny, ta ch cn chng minh c
f (a) + g(a)

341
, h(a) = 2(3b+c)a3 (5b2 +2bc+5c2 )a2 +2(2c3 b2 c bc2 )a+2b3 c+4b2 c2 2bc3

Ta c
h0 (a)

= 6(3b + c)a2 2(5b2 + 2bc + 5c2 )a + 4c3 2b2 c 2bc2


= 10ab(a b) + 4ab(a c) + 2c(a c)(3a 2c) + 2b(a2 bc) + 2b(a2
= 10ab(a b) + 2b(a2 bc) + 2(a c)(3ab + 3ac + bc 2c2 ) 0

Nn h(a) ng bin. Do
Nu b c th
h(a)
Nu c

h(b) = b(b + 2c)(b

c)2

h(c) = c(2b + c)(b

c)2

0:

c2 )

b th
h(a)

Bt ng thc c chng minh xong. ng thc xy ra khi v ch khi a = b = c:


Li gii 2. Gi s a = max fa; b; cg v xt 2 trng hp
Trng hp 1. Nu a 3b + c: t a + 2b = x + y; b + 2c = y + z; c + 2a = z + x; ta
c
3a + b c
3b + c a
3c + a b
x=
0; y =
0; z =
0
2
2
2
v
a=

5x

y + 2z
;
9

b=

5y

z + 2x
;
9

c=

5z

x + 2y
9

Khi , bt ng thc c vit li l


1
1
1
+
+
(x + y)2
(y + z)2
(z + x)2

27
x2 + y 2 + z 2 + 11(xy + yz + zx)

S dng bt ng thc Iran 1996, ta c


1
1
1
+
+
2
2
(x + y)
(y + z)
(z + x)2

Trng hp 2. Nu a

3b + c; do c + 2a
1
(c + 2a)2

9
4(xy + yz + zx)
27
x2 + y 2 + z 2 + 11(xy + yz + zx)
3a

3(a + 2b) nn

1
9(a + 2b)2

Do , ta ch cn chng minh c
1
10
+
(b + 2c)2
9(a + 2b)2

1
ab + bc + ca

342

CHNG 2. SNG TO BT NG THC

S dng bt ng thc AM-GM, ta c


10
1
+
(b + 2c)2
9(a + 2b)2

p
2 10
2:1
>
3(a + 2b)(b + 2c)
(a + 2b)(b + 2c)

Ta cn chng minh
2:1
(a + 2b)(b + 2c)
, 2:1(ab + bc + ca)
, a(1:1b + 0:1c)

1
ab + bc + ca
(a + 2b)(b + 2c)
2b2

1:9bc

Ta c
a(1:1b + 0:1c)

2b2

1:9bc

Bt ng thc c chng minh.

a(1:1b + 0:1c) 2b2 2bc


(3b + c)(1:1b + 0:1c) 2b2
1
[(3b c)2 + 4b2 ] 0:
=
10

2bc

Ph lc A

Mt s bt ng thc thng
dng
A.1

Bt ng thc trung bnh cng-trung bnh nhntrung bnh iu ha (AM-GM-HM)

Vi mi s dng a1 ; a2 ; :::; an ; ta c
p
a1 + a2 +
+ an
n
a1 a2
n

an

ng thc xy ra khi v ch khi a1 = a2 =

A.2

1
a2

n
+

1
an

= an :

Bt ng thc AM-GM suy rng

Cho cc s dng 1 ; 2 ; :::;


khng m a1 ; a2 ; :::; an ; ta c
1 a1

tha mn

2 a2

n an

ng thc xy ra khi v ch khi a1 = a2 =

A.3

1
a1

a1 1 a2 2

= 1: Khi vi mi s

ann :

= an :

Bt ng thc trung bnh ly tha

Cho cc s dng a1 ; a2 ; :::; an : Vi mi s thc r; t


8
< ar1 +ar2 + +arn r1
; r 6= 0
n
Mr =
: p
n
a1 a2
an
;r = 0
343

344

PH LC A. MT S BT NG THC THNG DNG

Khi Mr l hm tng theo r vi mi r: Chng hn M2 M1 M0 ; tc l


r
p
+ a2n
a21 + a22 +
a1 + a2 +
+ an
n
a1 a2
an :
n
n

A.4

Bt ng thc trung bnh ly tha suy rng

Cho cc s dng p1 ; p2 ; :::; pn tha mn p1 + p2 +


a1 ; a2 ; :::; an : Vi mi s thc r; t
Mr =

(p1 ar1 + p2 ar2 +


ap11 ap22

+ pn = 1 v cc s dng
1

+ pn arn ) r

apnn

;r =
6 0
;r = 0

Khi Mr l hm tng theo r vi mi r:

A.5

Bt ng thc Bernoulli

Vi mi s thc x

1; ta c
(1 + x)r
(1 + x)r

1 + rx ; r
1 + rx ; 0

1_r
r 1

Ngoi ra, vi mi s thc a1 ; a2 ; :::; an tha mn a1 ; a2 ; :::; an 0 hoc 1


0 th ta c
(1 + a1 )(1 + a2 )
(1 + an ) 1 + a1 + a2 +
+ an :

A.6

a1 ; a2 ; :::; an

Bt ng thc Cauchy Schwarz

Vi mi s thc (a1 ; a2 ; :::; an ) v (b1 ; b2 ; :::; bn ); ta c


(a1 b1 + a2 b2 +

+ an bn )2

(a21 + a22 +

+ a2n )(b21 + b22 +

+ b2n ):

ng thc xy ra khi v ch khi ai : aj = bi : bj 8i; j 2 f1; 2; :::; ng :

A.7

Bt ng thc Holder

Cho cc s dng xij (i = 1; m; j = 1; n): Khi vi mi ! 1 ; :::; ! n


+ ! n = 1; ta c
0
1! j
!
n
m
m
n
Y
X
X
Y
!
@
xij A
xijj :
i=1

j=1

j=1

i=1

0 tha ! 1 +

A.8. BT NG THC MINKOWSKI

A.8

Bt ng thc Minkowski

Vi mi s thc r

1 v vi mi s dng a1 ; a2 ; :::; an ; b1 ; b2 ; :::; bn ; ta c


"

A.9

n
X

(ai + bi )

i=1

# r1

n
X

ari

i=1

! r1

n
X

bri

i=1

! r1

Bt ng thc Chebyshev

Cho cc s thc a1
i) Nu b1 b2

a2
bn th
n

an : Khi

n
X

n
X

ai bi

i=1

ii) Nu b1

b2

ai

i=1

n
X

bi

bi

i=1

bn th
n

n
X

n
X

ai bi

i=1

A.10

345

ai

i=1

n
X
i=1

Khai trin Abel

Gi s x1 ; x2 ; :::; xn ; y1 ; y2 ; :::; yn l cc s thc ty . t


ck = y1 + y2 +

+ yk

8k = 1; 2; :::; n

Khi , ta c
n
X
i=1

A.11

xi yi =

n
X1

(xi

xi+1 )ci + xn cn :

i=1

Bt ng thc Maclaurin

Vi mi s khng m a1 ; a2 ; :::; an ; ta c
S1

S2

Sk =

sP

trong

Sn

a1 a2
n
k

ak

346

A.12

PH LC A. MT S BT NG THC THNG DNG

Bt ng thc Schur

Cho cc s khng m a; b; c: Khi , vi mi r > 0; ta c bt ng thc sau


ar (a

b)(a

c) + br (b

c)(b

a) + cr (c

a)(c

b)

ng thc xy ra khi v ch khi a = b = c hoc a = b; c = 0 hoc cc hon v tng


ng.

A.13

Hm li, hm lm

i) Mt hm s f c gi l li trn khong I nu v ch vi mi x; y 2 I v vi mi
s khng m ; tha mn + = 1; ta c
f ( x + y)

f (x) + f (y):

ii) Mt hm s f c gi l lm trn khong I nu v ch vi mi x; y 2 I v vi


mi s khng m ; tha mn + = 1; ta c
f ( x + y)

f (x) + f (y):

Trong trng hp f (x) kh vi cp 2 trn [a; b] th f (x) li trn [a; b] nu v ch nu


f 00 (x) 0 8x 2 [a; b] v f (x) lm trn [a; b] nu v ch nu f 00 (x) 0 8x 2 [a; b]:

A.14

Bt ng thc Jensen

Cho p1 ; p2 ; :::; pn l cc s dng.


i) Nu f l mt hm li trn khong I; khi vi mi a1 ; a2 ; :::; an 2 I; ta c
p1 f (a1 ) + p2 f (a2 ) +
+ pn f (an )
p 1 + p2 +
+ pn

p1 a1 + p2 a2 +
p 1 + p2 +

+ pn an
+ pn

ii) Nu f l mt hm lm trn khong I; khi vi mi a1 ; a2 ; :::; an 2 I; ta c


p1 f (a1 ) + p2 f (a2 ) +
+ pn f (an )
p 1 + p2 +
+ pn

A.15

p1 a1 + p2 a2 +
p 1 + p2 +

Tng, tch hon v-i xng

Ta
P k hiu
- Tng hon v. Chng hn
cyc

X
cyc

a2 b = a2 b + b2 c + c2 a

+ pn an
+ pn

A.15. TNG, TCH HON V-I XNG


a;b;c;d
X

347

a2 bc = a2 bc + b2 cd + c2 da + d2 ab

cyc

- Tng i xng. Chng hn

sym

a2 b = a2 b + b2 c + c2 a + ab2 + bc2 + ca2

cyc

a;b;c;d
X

ab = ab + ac + ad + bc + bd + cd

sym

- Tch hon v. Chng hn

cyc

Y
(a

b) = (a

b)(b

c)(c

a)

cyc

a;b;c;d
Y

(a

b) = (a

b)(b

c)(c

d)(d

a)

cyc

- Tch i xng (dnh cho 4 bin). Chng hn

cyc
a;b;c;d
Y
cyc

(a

b)2 = (a

b)2 (a

c)2 (a

d)2 (b

c)2 (b

d)2 (c

d)2 :

348

PH LC A. MT S BT NG THC THNG DNG

Ti liu tham kho


[1] Andreescu T., Cirtoaje V., Dospinescu G., Lascu M., Old and New Inequalities,
GIL Publishing House, 2004
[2] Cirtoaje V., Algebraic Inequalities, GIL Publishing House, 2006
[3] Phm Kim Hng, Secret in Inequality, nh xut bn Tri Thc, 2006
[4] Hojoo Lee, Topic in Inequalities, online electronic book, 2006
[5] Kim-Yin Li, Using tangent lines to prove inequalities, Mathematical Excalibur,
volume 10, number 5, 2005

349

You might also like